0% found this document useful (0 votes)
1K views468 pages

Analytical Solid Geometry - 3d

This document describes coordinate geometry in three dimensions. It defines key terms like the origin, axes, and coordinate planes used to locate points in three-dimensional space. A point in space is located by its coordinates (a, b, c), which give the distances to the point from the x-axis, y-axis, and z-axis, respectively. Together, the three coordinates uniquely specify the position of the point as a vector from the origin.

Uploaded by

Janusz Sioda
Copyright
© © All Rights Reserved
We take content rights seriously. If you suspect this is your content, claim it here.
Available Formats
Download as PDF, TXT or read online on Scribd
0% found this document useful (0 votes)
1K views468 pages

Analytical Solid Geometry - 3d

This document describes coordinate geometry in three dimensions. It defines key terms like the origin, axes, and coordinate planes used to locate points in three-dimensional space. A point in space is located by its coordinates (a, b, c), which give the distances to the point from the x-axis, y-axis, and z-axis, respectively. Together, the three coordinates uniquely specify the position of the point as a vector from the origin.

Uploaded by

Janusz Sioda
Copyright
© © All Rights Reserved
We take content rights seriously. If you suspect this is your content, claim it here.
Available Formats
Download as PDF, TXT or read online on Scribd
You are on page 1/ 468

Κλλ4&4*Α*4 $€/Ιλ£4

Analytical Geometry

(Co-ordinate Solid CJeometryji

(For Degree and Honours Students of Indian Universities & for Various
Competitive Examinations like P.C.S.Sl JLA.S. etc.)

By

A .R . Vasishtha D .C . Agarwal
Retired Head, M.Sc., Ph.D
Department of Mathematics Retired Principal & Head,
Meerut College, Meerut. Department of Mathematics
S.S.V. (P.G.) College, Hapur.

&
A.K . Vasishtha
M.Sc., Ph.D
C.C.S. University, Meerut.

KRISHNA Prakashan Media (P) Ltd.


KRISHNA HOUSE, 11, Shivaji Road, Mccrut-250 OOl (U.P.), India
B .Sc. M ade Easy Series (Fully Solved)
AU Indian Universities
Trigonometry A. R. Vasishtha & Others
Matrices A. R. Vasishtha & Others
Algebra A. R. Vasishtha & Others
Differential Calculus A. R. Vasishtha & Others
Integral Calculus A. R. Vasishtha Sc Others
Differential Equations A. R. Vasishtha Sc Others
Geometry of 2 Dimensions A. R. Vasishtha & Others
Geometry of 3 Dimensions A. R. Vasishtha & Others
Modem Algebra A. R. Vasishtha Sc Others
Vector Calculus A. R. Vasishtha Sc Others
Ordinary Sc Partial Differrential Equations A. R. Vasishtha & Others
Partial Differrential Equations A. R. Vasishtha Sc Others
Statics A. R. Vasishtha Sc Others
Dynamics A. R. Vasishtha Sc Others
Real Analysis A. R. Vasishtha Sc Others
Numerical Analysis A. R. Vasishtha & Others
Hydrostatics A. R. Vasishtha Sc Others

Book Code : 450-17(B)


Seventeenth Edition : 2015
Edition : 2016
P rice : T 270.00 O nly
Name, styk or anypart of this book thereofmaynot be reproducedInanyfoimor by anymeans without the writtenpermission
from the publishers andthe authors. EveiyoffcirthasbeenmadetoavokierrorsaromlsslaMkithhpubIkatkmtosptteofthIs,
some errors might have crept to. Anymistake,erroror discrepancynotedmay bebrought to ournotice whichshallbe tokencare
ofin the next edition, it Is nottfledthat neither thepublisher northe authororseUerwiHbe responsiblefor anydamage or toss of
actionto anyone, ofany kind, to anymanner, therefrom. Forblndtog mistakes, mtsprtotsorfor missingpages, etc. thepublisher's
IlobllttytsHmltedtoreplacementwlthlnonemonthafpurchasebysknilaredltlanANevensestnthlsconnectIonaretobebotne
by the purchaser.

Published by : Satyendra Rastogi nMitran


for KRISHNA Prakashan Media (P) Ltd.
11, Shivaji Road, Meerut - 250 001 (U.P) India.
Phones : 91.121.2644766,2642946,4026111, 4026112 Fax: 91.121.2645855
Website : www.krishnaprakashan.com
E-mail: [email protected]
Printed at Brahma Printers, Meerut
PREFACE TO THE REVISED EDITION

In this edition the book has been thoroughly revised.


Many more questions asked in various university
examinations have been added to enhance the utility of
the book. Suggestions for the further improvement of the
book will be gratefully received.
— The Authors

PREFACE TO THE FIRST EDITION

This book has been w ritten to meet the requirements


of the B.A. and B.Sc. students of Indian Universities, and
is so written as to cover the courses commonly prescribed
for these examinations.
The book has been w ritten in a very simple and lucid
style so that both students and teachers will find it an
interesting study. Care has been taken to explain the
fundam ental principles fully and rigorously. Each article
is illustrated by means of typical examples fully solved,
so as to give the students a clear idea of the types of
examples he has to solve.
Any suggestions for the improvement of the book
will be gratefully accepted.
Finally, our thanks a fe due to the publishers and the
printers who have taken great pains in producing the
book in the present nice form.
— The Authors
CONTENTS

Chapters Pages

I. System of Co-ordinates 1 — 14
2. Direction Cosines and Projections 15 — 45
3. The Plane 46 — 87
4. The Straight Line 88 — 173
5. Shortest Distance 174 — 200
6. Volmne of Tetrahedron 201— 216
7. Skew Lines 217 — 233
8. Change of Axes 234 — 241
9. The Sphere 242 — 309
10. The Cylinder 310 — 324
11. The Cone ; 325 — 393
12. Central Ccohcoids 394 — 464
Systems of Co-ordinates

§1. Introdoction. Studentsknow well that in co-ordinate


geometry o f two dimensions (i.e. plane analytical geometry) the
position of a point in a plane is referred to two intersecting lines
(in the plane of the point) called the axes of reference and their
point of intersection called the origion of co-ordinates. The axes
are called rectangular axes if they are at right angles, otherwise
they are called oblique axes. Whatever the axes may be, they
divide the plane into four quadrants called the first, second, third
and fourth quadrants respectively.
But it is not always possible to determine the positions of all
the points we can imagine with reference to above co-ordinate
axes. For example, codsider the five corners of a rectangular
parallelopiped, they do not lie in one plane. Such points are
called points In space. A point in space can be demonstrated as
follows :
Consider your study room and let dimensions of the room be
that o f a rectangular parallelopiped. Now consider any particle in
air, then this particle in air is a point in space.
The geometry of such points in space is discussed in
“Analytical geometry of three dimensions” also called “ Solid
geometry.”
§ 2. Definitions. Origin, axes and co-ordinate planes.
Draw two mutually perpendicular lines X OX and Z rOZ in
the plane of the paper. Let these lines intersect at O9 then
through O imagine a third line Y OY perpendicular to both of
the above lines, so that OY is perpendicular to the plane of the
paper and is directed upwards.
2 Analytical Geometry 3-D

The point O is called the origin. The three mutually per­


pendicular lines namely Χ Ό Χ , T O Y and Z'OZ are called the
axe* of reference (rectangular), and are said to be x-axis, y-axis
and z-axis respectively. OX is taken to be positive direction of
x-axis whereas OX' as negative direction of x-axis. In a similar
way OY and OZ are taken to be the +ve directions and OY' and
O T as —ve directions of y and z-axes respectively.

When these three axes are taken in pairs, they give us three
planes YOZ, ZO X and XOY.- T hest three planes are called yz* zx
and xy-planes respectively. The set of these three planes is called
the set of co-ordinate planes (rectangular).
The axes are oblique axes, if they are not rectangular.
Note. In the rest of the book the axes will be assumed to be
rectangular unless otherwise stated.
§ 3. Co-ordinates of a point in space. Consider a point P in
space. Through P draw a plane PNAM parallel to TOZ plane
l.e. perpendicular to x-axis meeting it in the point A ; if OA=*a,
then a is called the x-co-ordinate of P. Similarly through P
draw planes PNBL parallel to the plane ZOX and PMCL parallel
to the plane XOY meeting y and z axes in the points B and C
respectively ; if 0 5 = b and OC=c then b is called y-co-ordinate
of P and c is called z-co-ordinate of P. The three numbers a, b, c
are called the co-ordinates of the point P and are written by
Systems o f Go-ordinates 3

ordered triads of the form (a, b, c). These co-ordinates are


measured positive or negative in the sense explained in § 2.
Let i, j, k denote the unit vectors along OX, OYt OZ respect­
ively. Let r be the position vector of the point P whose co-ordi­
nates are (a, b, c). Then we have
OP·= O N + NP
= O A+ AN + N P ^O A + O B + O C
l.e., r= o l+ h j+ c k .
The vector a i+ 6 j+ ck is more conveniently written as (a, b, c).
Hence we may write
r=(fl, b, c).
Thus (a, b, c) are the co-ordinates o f a point P i f and only if
the position vector of the point P is the vector a\+ b\+ ck which is
simply written as the vector {a, b, c).
Remark I. If (a, b, c) are the co-ordinates of a point P in
space, then it is usually written as the point P (a, b, c).
Remark 2. The co-ordinates (a, b, c) of the point P deter­
mined as above are called the cartesian co-ordinates of the point Pt
but for convenience we shall simply say that (a, b, c) are the
co-ordinates of the point P.
Thns we see that the distances with proper signs of the origin
from the points on the axes in which the planes through the given
point P drawn parallel to the co-ordinate planes meet, are called
the co-ordinates of the point P.
4 Aualyttcal Geometry 3-D

§4. Properties of co-ordinates of a point P.


(A) The co-ordinates of a point P are respectively the distances
with proper signs o f the point P from the three co-ordinate planes.
See figure of § 3. Through the point P draw planes parallel
to the co-ordinate planes cutting the axes in the points A9 B and
C respectively. These three planes together with the co-ordinate
planes form a rectangular pafallelopiped. We have,
the perpendicular distance of the point P from the yz-plane
= LP = C M = O A = a
=x-co-ordinate of the point P9
the perpendicular distance of the point P from the zX'plane
= M P = C L = OB=b
^-co -o rd in ate of the point Pt
and the perpendicular distance of the point P from the xy-plane
= N P= AM = OC=c
=»z-co-ordinate of the point P.

(B) The co-ordinates o f a point P are the distances from the


origin O of the feet of the perpendiculars from the point P to the
co-ordinates axes.
See figure of § 3. Since the plane PNAM is perpendicular
to OX9 PA (a line in this plane and cutting OX) is perpendicular
to OX, Similarly PB and PC are perpendiculars to OY and OZ
respectively. Thus
the x-co-ordinate of the point P=OA9A being the foot of
the perpendicular from the point P on the x-axis.
Similarly the .y-co-ordinate of P=OB9 the z-co-ordinate of
P=OC,wherethepointsJ? and Care the feet of the perpendiculars
from P on the y and z-axes respectively.

§ 5. Octants. The three co-ordinate planes namely yz-plane,


zx-plane and xy-plane divide the space into eight parts called the
octants, and to which octant the point P belongs is determined by
the signs of the co-ordinates of the point P. The following table
determines the signs in eight octants :
Systems o f Co-ordinates 5

.
OXYZ

O X Y 'Z '
,ZA ,XO
Octant

O XY'Z'
O XYZ

O X Y 'Z
6* *
O O

I
I
X + + + + — — — —

y + — I _ ' + + I _ + —
I i
Z + + — . — + + — —

Ex. I. What are the positions of the following points ?


(0 (1 ,2 ,3 ), (Ii) ( 1 ,- 2 , 3 ) , (Hi) ( 0 ,0 ,- 3 ) ,
(/v) ( - 1 , - 2 , 0 ) , (V) (2 ,0 ,0 ), (vi) ( - 1 ,- 2 ,- 3 ) .
Sol. (i) (I, 2, 3) is a point in the octant OXYZ and its dis­
tances from the co-ordinate planes yz, zx and xy are I, 2 and 3
respectively.
(ii) (I, —2,3) is a point in the octant OXY9Z and its distances
from the co-ordinate planes yz, zx and xy are 1,2 and 3 res·
pectively.
(iii) (0, 0, —3) is a point on OZt t.e. on the —ve side of
the z-axis situated at a distance 3 from the Origin O.
(iv) ( - 1 , - 2 , 0 ) is a point in the co-ordinate plane xy
since its z-co-ordinate is zero. It lies in the octant OX9Y 9Z and
its distances from the co-ordinate planes yz and zx are I and 2
respectively.
(v) (2, 0, 0) is a point on the positive side of the *-axis
situated at a distance 2 from the origin O.
(vi) (—I, —2, —3) is a point in the octant OX9Y 9Z ' and
its distances from the co-ordinate planes .yz, zx and xy are I, 2
and 3 respectively. ;
§ 6. Change of origin. Let OX, OY9 OZ be a rectangular
set of axes. Referredto these axes letthe co ordinates of two
points P and Q be (Xi, yu Zi) and (X2, y*% *a) respectively; Sup­
pose we want to shift the origin from O to the point P 9 i,e, we
want to find the co-ordinates of Q referred to P as origin.
Analytical Geometry 3-2)

Draw the new axis PX1, PYi and PZi parallel to the original
axes OX, OY and OZ respec­
tively.
The position vectors of
the points P and Q with res­
pect to O as origin are given
by
OP^Xi l +y, J+zJs,
O Q = xti+ yti+ zJt.
Also the position vector
of the point Q with respect to
P as origin is PQ. Now we have
P Q ^ O Q -O P = (x,i + y J + Zjk) - (X1I + J 1J + Z1It)
=(X1- X 1) i+ ( y ,- y ,) J-Hr1- Z 1) k
=(*i-*i, y » -y u z»-zih
Therefore the co-ordinates of the point Q with respect to the
new origin P are (X1-X 1, y t —yu Zt - Z i).
§ 7. The distance between two given points.
Let P and Q be two given
points in space.
Let the co-ordinates of
the points P and Q be (X1, _ylt
Z1) and (X1, y2. zt) with res­
pect to a set OX, OY, OZ of
rectangular axes. The posi­
tion vectors of the points P
and Q are given by
O ^=X 1I+ ^ j+ z jk .
and O g —Xii+J'tj+Ztk
Now we have P Q = O Q -O P .
“ (•’f J + J 's H - i^ - iX ii+ ^ j+ z .k )
=(X1-X 1) Ι-ΚΛ-Λ) J-Hs1-Z 1) k.
Λ T Q = I T Q I—λ/ίί**- ·—^1) *Hz1-Z 1)1).
Thus the distance PQ between two points P (X1, yu Z1) and
Q (JU y*. 2») is given by
Systems o f Co-ordinates 7

PQ = V i ( X i - x i )*+ ( y t - y i Y + (*2- * ι) ’}·


Corollary. The distaoce between the points (0, 0,0) and
0 *1. V u Zl) I S - v ' W + J ’^ + Z i » ) .
§ 8. Division of a line. To determine the co-ordinates of a
point R which divides the join of the line joining the two points
P (xt, yu Zi) and Q (x2, y t, z2) Internally in the ratio m i: m,.
Let OX, OY, OZ be a set of rectangular axes.
The position vectors of the two
given points P (Xi, y u zx) and
Q (x2, y t. z,) are given by
Ο Ρ - χ ,Ι + ^ Λ + ζ ^ -(1)
and Ο β = x 2i -KVij+ z ak -(2)
Also if the co-ordinates of the
point R are (x, y, z), then
OR=> x i+ y j+ zk . ...(3)
Now the point Λ divides the
join of P and Q in the ratio W1 : m8,
so that
mI PP ,
m = R Q or ,Wl (r Q)=M* (PP)·
—>■ —►
Hence mtPR=mxRQ
or m8 (O R -O P )= m 1{OQ~OR)
^ ^ V
or (Wj + Wa) O R= m x OQ+ma OP

or o k = V ± ° Q + m* OJ?_
OT1+W2
nr -. ■ - ■ - ,. (OT,Xa+WaX,) * +(OT1J^2 + OT2Vi) j + (W 1X ^ m 2Z1) k
(OT1+ OT,)
(using (I), (2) and (3)]
Comparing the coefficients of i, j, k, we get
S q OT1X 2 +O T8X 1 OTi y 2 + W 2-V1 , OT1Z8 + OT8Zi
“ OT1-J-OT2 ’ y OT1+ OT2 * OT1+ OT8
Cor. I. The middle point of the segment PQ is obtained by
PUttingm1-OT2. Hence the co-ordinates of the middle point of
PQ are (} (X |+ x2), i Cv1+.V2), $ (Z1+ z 2)).
Cor. 2. If mt μ : I, then the co-ordinates of the point
P flfc ( X i + μ χ * >'i + μ}'I Z\ + μζ2\
* a te [ - j + r · μ + l ' μ + ί J
8 Analytical Geometry 3-2)

These are called the general co-ordinates of a point on the


Une PQ.
Cor. 3. If the ratio (m,/mt) is +ve then the point R divides
PQ internally and if it is —ve then externally.
For direct applications, the co-ordinates of the point R which
divides externally the join of P and Q in the ratio m ,: m% are
—max, m,.ya.—mty, OiiZ1--O l1Zi \
I r n ,-■mt ’ mx—mt Oil -O i1 I
§ 9. (A) Centroid of a triangle. Let ABC be a triangle.
Let the co-ordinates of the vertices A, B and C be (Xt, y it r,),
(xt. yt , zt) and (x3, y%, za) respectively. Let AD be a median of the
&ABC. Thus D is the mid. point of BC.
.·. The co-ordinates of D are
/**+*$ y*+y* z%+z»\
i 2 ' I * 2 r
Now if G is the centroid
(l.e., centre of gravity) of AABC,
then G divides AD in the ratio
2 : 1 . Let the co-ordinates of G
be (X, y, 2). Then
2. ( * + » ) + , . „
o r X = * 2 + * » .+ * « .
2+1
Similarly J>=i (Ti+Ta+T#)* 2= 3J (¾+¾+¾)*
(B) Centroid of a tetrahedron.
Let ABCD be a tetrahedron,
the co-ordinates of whose verti­
ces are (xr, jv, z,)%r = I, 2, 3, 4.
Let Gi be the centroid of the
face ABC of the tetrahedron. Then
the co-ordinates of Gi are

The fourth vertex D of the tetrahedron does not lie in the


plane of Λ ABC, We know from statics that the centroid of the
tetrahedron divides the line DGi in the ratio 3 : 1 . Let G be the
centroid of the tetrahedron and if (£, j>, z) are its co-ordinates,
then
Systems o f Co-ordinates 9

3 *H -X s±*, + hX|
Xi+*»+** -I-Xt
3c or X= 4
3+1
Similarly y = \ (^l+ ^ i+ y * + ^ )* (*ι+**+*·Ψί»)·
10. (A) Spherical polar co-ordinates.
Let X'OX, T O Y and Z OZ
be the set of rectangular axes.
Let P be a point in space.
Draw PN perpendicular from
P to the xy-plane. The posi­
tion of P is determined if the
length OP, angles ZOP and
XON are known. Suppose
OP=r, Ζ.ΖΟΡ=θ and L X O N
measured positively in the
directions shown by arrows
in the figure. The quantities
r, Θ, <f>, defined as above, are
called the spherical polar co-ordinates of P and are written as
(r, $, φ).
Now we shall find relations between these co-ordinates and
cartesian co-ordinates. Let (x , y, z) be the cartesian co-ordinates
of P. Hence we have
Z=PAT=OP cos ( L O P N )= r cos (LZO P)= T cos 0. ...(I)
Also ON=OP sin LOPN=T sin 0 [ 7 LO N P = 90°]
Λ X=O N cos φ = τ cos <f>sin 0, ...(2)
and y = ON sin Φ= r sin φ sin 0. ---(3)
Thus relations (2), (3) and (I) give the relations between
x, y, z and r, 0, φ.
Now squaring the relations (2) and (3) and adding, we get
x'+j)*=aOA/*, or u*=x2+y* where u=ON
ot tf( x t + y*)=u=r sin 0. ...(4)
Dividing (4) by (I), we get tan 0=V'(**+X2)/z·
Dividing (3) by (2), we get tan φ= γ/χ.
Squaring (I) and (4) and adding, we get x i + y, + zi =r*.
Thus the relations between spherical polar co-ordinates and
cartesian co-ordinates are
x = r cos φ sin 0, y = r sin φ sin 0, z= r cos 0
xt + yi + 2t = ri, tan 0=-^(**+.?2)/*. tan Φ=υ Ιχ ·
10 Analytical Geometry 3-D

(B) Cylindrical co-ordinates. See figure of § 10 (A). Let P


be a point in space. The position of P can also be determined if
the measure of ON, IIXON and NP are known. Suppose ON=u,
£Χ Ο Ν = φ , NP=z. The quantities u, φ, z are called the cyclindri-
cal co-ordinates of P and are written as (u, φ, z).
Let (x, y, z) be the cartesian co-ordinates of P, then N has
the co-ordinates (x, y, 0). Hence, we have
X=O N cos φ= ϋ cos φ, y= u sin φ, z= z.
Also u*=x%+ yi, tan <f>=yjx.
Solved Examples
Ex. I. P is a variable point and the co-ordinates o f two points
A andB are ( —2 ,2 ,3 ) and (1 3 ,- 3 ,1 3 ) respectively. Find the
locus o f P i f IPA= IPB .
Sol. Let the co-ordinates of P be (x, y, z).
.\ PA=s/{(x-\-2)i -\-(y—2)i + (z—‘i) i}, ...(I)
and Λ Β = ν{ (* -1 3 )* + (7 + 3 )* + (ζ-1 3 )2}. ...( 2)
Now it is given that 3PA=2PB i.e., 9PA*=4PB*. ...(3)
Putting the values of PA and PB from (I) and (2) in (3), we
get
9 ( ( ^ + 2^ + ( 7 - 2^ + ( 2 - 3)^ = 4 {(x—13)*+ -1 -3 )* + ( z —13)*}
or 9 (x*+7, +2*+ 4x—47 —6z+17}=4 (x2+ 7 2+ za—26X+ 6 7
—26z+ 347}
or 5x*+57 2+5z* + 140x—6O7 + 5O Z -1235=0
or x*+ 7 *+z, + 28x —127 + IO z -24=0.
This is the required locus of P.
Ex. 2. At Bt Care three points on the axes o f x ,y a n d z
respectively at distances a, b, c from the origin O', find the co-ordi­
nates of the point which ts equidistant from A, B, C and O.
Sol. Let the required point be P (x, y, z).
The co-ordinates of the points A, Bt C and O are (a, 0, 0),
(0, b, 0), (0, 0 , c) and (0, 0 , 0) respectively.
It is given that PA=PB=PC=PO .
Taking PA=PO, or PAi = PO tt we have
(x—fl)*+7*+z*=x 2+ 7 2+ z 2
or —2 ax + aa= 0 , or x = a/2. ...(I)
Now taking PB=POt or PBi = PO it we have
xi + ( y - b ) i + zi = x i + yi + zi
or —2by+b*=0, or y=b/2 ...( 2)
Againtaking PC=POt or PCi= PO i, we get z=c/2. ...( 3)
System o f Co-ordinates 11

Hence the co-ordinates of the required point P are (a/2, 6/2,


c/2).
' Ex. 3. Find the centre o f the sphere which passes through the
points O (0, 0, 0), A (a, 0, 0), B (0, b, 0) and C (0, Oj c).
Sol. Let the centre of the sphere be P (x, y, z).
Since the sphere passes through A, B, C and O, so P (x, y, z)
is equidistant from A, Bt C and Ο. Λ PA = PB= PC’=·PO.
Now proceeding as in Ex. 2 above, the centre P of the sphere
is given by (}a, \b, Jc).
Ex. 4. Show that (0, 7, 10), (—1,6, 6), (—4, 9, 6) 'form an
Isosceles right angled triangle.
Sol. Let ABC be a given triangle and let the coordinates
of the vertices A, B and C be (0, 7, 10), ( —1, 6, 6) and ( —4, 9, 6)
respectively. .·. ^ £ = ^ ( ( 0 + 1 ^ + ( 7 - 6 ^ + ( 1 0 - 6 ) * } = VO*)
BC= V ( ( - 1+4)*+(6 -9)*+(6 —6)*}=\/(18)
C ^ = V { (-4 -0 )* + (9 -7 )* + (6 -1 0 )* } = v '(3 6 ).
We have AB=BCt hence A ABC is an isosceles triangle.
Again Λ£*+£(7*=18+18=36=<;Λ*. Λ /.A B C = 90°.
Hence A ABC is also right angled triangle. Therefore, the
given triangle is an isosceles right angled triangle.
Ex. 5. Find the co-ordinates of the point which divides the join
o f (2, 3, 4) and (3, —4, 7) In the ratio 2 : —4.
Sol. Let the co-ordinates of the required point be (x, y, z),
then by § 8, we have
„ _ 2 (3 )-4 (2 )_ 6 —8 , . __ 2 ( - 4 ) - 4 ( 3 ) _ - 2 0 _ „ w
2 -4 = 2 ~ l >y~ JZ i -2 ’
2 (7 )-4 (4) 14—16_ —2
2 -4 -2 —2
Hence the required point is (I, 10, I).
Ex. 6. A point P lies on the tine whose end points are
A (I, 2, 3) and B (2, 10, I). I f z co-ordinate o f P Is 7, find Its other
co-ordinates.
Sol. Let the co-ordinates of the point P be (x, y, z) and let
it divide the join of A (I, 2, 3) and B (2, 10, I) in the ratio μ : I.
Then [by cor. 2, § 8]
μ+ 1
But it is given that the z-co-ordinate of the point P is 7.
Λ 7 = ^ ~ or 7 ^ + 7 = μ + 3 or 6μ=>—4 or μ = —2/3.
Mt ^
12 Analytical Geometry 3-D

M(2)+1 (I) (-4 /3 ) + 1


—1
* μ+ 1 (-2 /3 )+ 1
μ (10)+1 ( 2 ) _ -(2 0 /3 )+ 2
and 14.
μ+l -(2 /3 )+ 1
Ex. 7. Fwnf /As re//o in which the xy-plane divides the join
o f ( —3, 4, —8) and(5, —6, 4). Also find the point of intersection
o f the line with the plane.
Sol. Let the xy-plane (/.<?., z= 0 plane) divide the line
joining the points ( —3, 4, —8) and (5, —6, 4) in the ratio μ : I,
in the point R. Therefore, the co-ordinates of the point R are
(5μ—3 —6μ+4 4μ —8)
[See cor. 2, § 8]
μ + T ’ μ + 1 'J T F T ) ' -(Ο
But on xy-plane, the z co-ordinate of R is zero.
Λ (4 μ -8 )/(μ + 1)=0, or μ = 2 .
Hence μ : 1 = 2 : I. Thus the required ratio is 2 : I.
Again putting μ —2 in (I), the co-ordinates of the point R
become (7/3, —8/3, 0).
Ex. 8. Findtheratios in which the sphere x8+y*+z*=504,
divides the line joining the points (12, —4, 8) and (27, —9, 18).
Sol. Let the sphere x, + y 8+z*=504 meet the line joining the
given points in the point (xi, y u z,).
Then x1a+y,*+z,»=504. ..(I)
N ow supposethatthe point (xi,yi, zt) divides the join of
the points (12, --4, 8) and (27, —9, 18) in the ratio μ : I.
27μ+12 „ _ - 9 μ —4 _ ! 8 μ + 8
Then X1«
“μ + Γ , Λ ” ' μ +1 ' ^ “ μ+Ι
Putting the values Ofx1, yu zx in (I), we get
(27μ+ 12)* , ( —9μ—4)8 (18μ + 8)8
( μ + 1)* (μ+!)* (μ + υ * β
or 9 (9μ+4)» + (9μ + 4)*+4 (9μ+4)*»504 (μ+1)8
or 14 (9μ+4)*=504 (μ+1)3, ΟΓ (9μ+4)8- 3 6 ( μ + ί) 3.
Taking the square root, we get 9μ + 4 = ± 6 (μ+1).
Taking +ve sign, 3μ=2, or μ /1—2/3, or μ : 1=»2 : 3.
Again taking ^ v e sign, 15μ= —10, or μ/1=>—2/3,
or μ : 1= 2 : - 3 .
Ex. 9. Fromthepoint (I, —2,3) lines are drawn to meet
the sphere x*+y8+ z 8= 4 and they are divided in the ratio 2 : 3.
Prove that the points of section lie on the sphere
5 (x*+y3+ z 3)—6 (x -2 y + 3 z )+ 2 2 = 0 .
Systems o f Co-ordinates 13

Sol. Suppose any line through the given point ( 1 , - 2 , 3 )


meets the sphere xi + y, +z*=4 in the point (xlt y u Z1). Then
*i*+yi’ +Zi*“ 4. ...(I)
Now let the co-ordinates of the point which divides the join
of (I, —2, 3) and (xlt y it Z1) in the ratio 2 : 3 be (x»t y t, zt). Then
we have
2 . Xj - f 3 . 1 nr 5 x a- 3 1
χ% VI *1
2+3 2 I
2 .^ , + 3 . ( - 2 ) 5y8+ 6 *
y 2: or ^ l1
2+3 " 2 f
2 . z, + 3 . 3 5za— 9 i
Z2 - or Zl
- 2+3 2 J
Putting the values of r lt y u zu from (2) in (I), we have
(5xa-3 )* + (5 y a+6)»+(5za-9 ) * = 4 x 4
or 25 (xa*+.ya*+ z2*) —30xa+60y2—90z2+ l 10=0
or 5 (Xti -^ y f Zti) - 6 (xa—2 /a+ ?za)+ 2 2 = 0 .
.·. the locus of (x2, yif zt) is
5 (x*+.y*+ Zi) - 6 (x —2>>+3z)+22=0,
which is the equation of a sphere.
Ex. 10. Prove that the three points A, B and C whose co-ordi­
nates are ( 3 ,- 2 ,4 ) ,( 1 ,1 ,1 ) and ( —1,4, —2) respectively, are
colllnear.
Solution. The general co-ordinates of a point R which divides
the line joining A (3, —2, 4) and B (I, I, I) in the ratio μ : I are
IfiJri μ_^2 μ + 4 \
\/* + 1* Μ+'ϊ* μ + l / ···(!)
If 0 ( —1, 4, —2) also lies on the line AB, then for some value
of μ the co-ordinates of the point R will be the same as those
of C.
A let the x-co-ordinate of the point R —the x-co-ordinate
of the point C.
Then (μ+3)/(μ + 1) = —I, or μ = —2.
Putting μ = - 2 in (I), the co-ordinates of R are (—1,4, —2)
which are also the co-ordinates of C. Hence the points A, B and
C are collinear.
Also we note that C divides AB in the ratio μ ; !,/> ., —2 : I.
14 Analytical Geometry 3-D

Exercises
1. Find the locus of a point P which moves in such a way that
its distance from the point A («, v, tv) is always equal to a.
Ans. Jc*+?*+**-lu x —2vy—2ivz-fu, + v1+ h’' —a*=0.
2. The axes are rectangular and At B are the points (3 ,4 ,5 ),
( —1, 3, —7). A variable point P moves such that (i) PA=PB
and (ii) PA*—PBt = Ik i. Find th e lo cu so fP in each of the
above cases.
Ans. (i) 8x+2.y+24z+9=0. (ii) 8 ^ + 2 ^ + 2 4 7 + 9 + 2 ^ = 0 .
3. Show that the points (1,2, 3), (2, 3, I) and (3, I, 2) form an
equilateral triangle.
Hint. Show that the length of each side of the triangle is V 6:
4. Prove that the three points A, B and C whose coordinates are
( 3 ,2 , - 4 ) , ( 5 , 4 , - 6 ) and ( 9 ,8 ,- 1 0 ) respectively are
collinear.
2
Direction Cosines and Projections
§ I. Angle between two non-coplanar (i.e. non-intersecting lines).
Let PQ and M N be two non-coplanar lines. The angle
between two non-coplanar lines PQ and M N is equal to the angle
between two straight lines OA and OB drawn from any point O
parallel to PQ and MN respectively. Thus the angle between

Λ*

the lines PQ and M N is equal to the angle AOB.


§ 2. Direction cosines of a line.
Definition. I f *, β, γ are the angles which a given directed line
yfnakM with the positive directions o f the axes o f x , y and z respec·
tM fp, then cos a, cos β. cos γ are called the direction cosines
(briefly written as d.c.'s) o f the line. These d.c.'s are usually denoted
by /, m, η.
Let AB be a given line. Draw a line OP parallel to the line
AB and passing through the
origin 0. Measure angles a,j3, y
as shown by arrows in the
figure, then cos a, cos β, cos γ
are the d.c.’s of the line AB. It
can be easily seen that I, m, n
are the direction cosines o f a
line if and only If /i-f w j+nk Is
a unit vector in the direction of
that line.
Clearly OP’ (i.e., the line
through O and parallel to BA) makes angles 180°—a, 180*—β,
16 Analytical Geometry 3-D

180°—y with OX, OY and OZ respectively. Hence d.c.’s of the


line BA are cos (180° - a ) , cos (180°—β), cos (180°- y ) Le., are
—cos a, —cos β, - cos y.
Remark. Since the angles α, β, y are not coplanar,
.·. a+ /3 + y /3 6 0 °.
D.c.’s of the coordinate axes.
Since the axis of x makes angles O01 90°, 90° with the axes of
x, y, z respectively, therefore by definition, its d.c.’s are cos 0°,
cos 90°, cos 90° l.e., 1 ,0, 0.
Hence the d.c.’s of the x-axis are I, 0, 0.
Similarly the d.c.’s of the y-axis are 0 , 1, 0
and the d.c.’s of the z-axis are 0, 0, I.
§ 3. I f the length o f a line OP through the origin O be r, then
the co-ordinates of P ore (Ir, mr, nr) where I, m,n, are the dc.'s
of OP.
Draw PM perpendicular
from P to OX meeting it at M.
Let (x, y, z) be the co-ordi­
nates of Ρ, then OM =x. From
the right angled &OMP, we
PM
have >cos a = I.
OP =
or xlr= l or x = /r.
Similarly y=mr, z=nr.
A P is the point (Ir, mr, nr).
§ 4. I f l,m ,n are direc­
tion cosines o f any line AB, then
to prove that I*+ m* + n '= I. (Kanpur 1983)
Through the origin O draw a line OP parallel to the given
line AB so that the d.c.’s of OP are I, m, n. Suppose OP is of
length r. If the co-ordinates of P are (x, y, z), then we have
(see § 3) x=!r, y=mr, z=nr. ...(I)
Now Tt =OP2 or r* = (x -0 )* + O '-O )a-1-(2-0)*
or ra=>x2+y*-|-z2 or Γ2= Ι2τ2+ η 2τ2+η*τ*, using (I),
or l24-m2+n*e=l.
Remark. We have O P = xi+ yj+ zk= lri+ m ri+ nrk.
A a unit vector in the direction of OP
OP ... .
«k Ir io p i = r i - ( 2)
Direction Cosines and Projections 17

The relation (2) shows that the direction cosines o f the tine OP
are the coefficients o f I, j, k in the rectangular resolution o f the unit
vector in the direction o f OP.
Thus if I, m, n are the d.c.’s of a line, then a unit vector
along that line is /i+ m j+ nk.
Λ I /i+ m j+ n k 1=1
or V ift + « * + n*) = I, or •/•+ « •+ a * —I.
§ 5. Direction ratios.
Definition. I f the direction cosines I, m, n of a given line be
proportional to any three numbers a, b, c respectively, then the
numbers a, b, c are catted direction ratios (briefly written as d.r .’s)
o f the given line.
Relation between direction cosines and direction ratios.
' Let a, b, c be the direction ratios of a line whose d.c.’s are
/, m, n. From the definition of d.r.’s, we have
l/a=m /b—nlc=k (say). Then I=Icai m=*kb, n—kc.
But /«+m*+»*=!·
Λ k* (a*+b*+c*)=l, or k*= l/(a*+b*+c*)
or k ^ ± l / y / ( a t +b, +c*).
Taking the positive value of k, we get
. a b c
" V io '+ b '+ c * )' '” = V (a*+ b*+ cV
Again taking the negative value of k, we get
—a -b —c
v V + ^ + c * ) ’ m ~ y(a * + b '+ c* y n=V ( a * + b * + o '
Remark. Direction cosines of a line are unique. But the
direction ratios of a line are by no means unique. If a, b, c are
direction ratios of a line, then ka9 kb, kc are also direction ratios
of that line where k is any non-zero real number. Moreover if
a9 b, c are direction ratios of a line, then a i+ ij + ck is a vector
parallel to that line.
Rule. Let a, b, c be the d r.'s of a given Iine9 then, to find
actual direction cosines o f this line, divide each o f a, b, c by
V X a'+ ^ + c2).
SOLVKD EXAMPLES 2 (A)
Ex. I. Find the d.c.’s o f a line whose direction ratios are
2, 3, - 6 .
Solution. We have Vt(Z)*4-(3/ + ( - 6 ) 1}= vM -t-9+36)-*7.
2 3 _^
Hence (by § 5) the d.c.’s of the given line are ψ , ψ , - y ·
18 Analytical Geometry 3-D

Ex. 2. Prove that Slnt a + sin2 β + sln1 y= 2, where a, £, γ are


the angles which the given line makes with positive directions o f the
axes. (Agra 1978)
Sol. We have (See § 4) cos2 α + cos2 £+cos2 y=a j.
Changing cosines into sines, we get
( I—sin2 a) + (1 —sin2 £) + (1 —sin2 y)«= i
or sin2 a+ sin 2 £ + sin2 y = 2.
Ex. 3. Find the direction cosines o f the line which is equally
inclined to the axes. (Agra 1979)
Sol. Let the direction cosines of the line be /, m, n.
Since the line is equally inclined to the co-ordinate axes, therefore
/ae=/rt2e=*/I2.
But /*+m2+ n 2~ I. Λ 3/2= I or /2= I /3 or / « ± 1 / ^ 3 .
Similarly m = ± l/ \/3 , n ^ + 1/%/3.
Hence the direction cosines of the line are ± 1 /^ 3 , ± l / \ / 3 ,
± l / \ / 3 . There will be eight such lines, one lying in each octant.
Ex. 4. Find the direction cosines /, m, n of two lines which are
connected by the relations / —5m + 3n = 0 and 7/2+ 5m2—3n2«=0.
Sol. The given relations are
/—5m +3/i=0 or /«=5m—3/j ...(I)
and 7/2+5m 2- 3 n 2= 0. ...(2)
Putting the value of / from (I) in (2), we get
7 (5m -3n)2+ 5m2- 3 /i2- 0
or 180m2—210m«+60n2= 0 or 6m2-T m n + 2n1= 0
or 6m2—4m/i—3m/i+2/i2=»0 or (2m—n) (3m—2/i)=0.
Λ m//?=»J, and m//*=»2/3.
Now when m//i«= 1/2 i.e. n«=2m, we have from the relation (I)
l*=>5m—6m or
/=. —m or — a=—
m I
m I , / I I m n
Thm Γ *ι*ιοί - ι ~ Τ , = 2
l m n y/(lt +mt +nt) I
or - I - T = T = ν { (-1 )* + 1 Α+2*} “ V6
Λ The d.c.’s of one line are — \l\/6 , 2/i/6.
Again when - = Λ - i.e. n = ^ , we have from (I)
n j 2
Dlreeihm Catbies and Projections 19

—. m 2 - I m . .
Thus — = 3- γ α ~2 £lVM18
I m n \/{1*+ιη*-\-η%) I
Τ β Τ β Τ β ν ΐ ΐ β+ 2 · + > ί = V (M /
Λ The d.c.*s of the other fine are l/\/(14). 2 / ^ ( 1 4 ) , 3/vO 4)·
Ex. 5. Find the direction cosines It m, n o f the two lines which
are connected by the relations /+ιπ+#ι=»0 and mn—2nl—21m=0.
(Meerut 1985)
SeL The given relations are
/+#n+«=»0 or / = —iff—λ ---(I)
and mn—2nl —2l m=0. ---(2}
Puttingthevalue of I from (I) in the relation (2)# we get
mu—2ι» (—jw—it)—2 (—m —n) m = 0 or 2^ + 5^ 111+ 21^-=0
or (2jr+ ji) (m +2n)=0.
in I
— =- a nd—2.
n 2
From (I), we have — «=>————= ——— I . /TI
v' n n « —P )
Now when m /n=—+ (3) gives //n = J —1 = —
Λ #π /1= π /-2 and //I = 11/—2
. I m n \/(J1+ lll¾+ ,1,) *
- I ° r = - 2 = y r{ l '+ i , +(-2)*>
The d.c.*s of one line are 1/^6» I/V r6, —2/\f6.
Again when m /2 = —2, (3) gives //a=»2 - 1 —1.
I m n y/(P+nP+iP) I
" I = - 2 = I = ^ (^ + (-2 )-+ 1 * }= ^
The d.c.*s of the other line are 1/ λ/6> -2 /-^ 6 , 1 /^ 6 .
§ ¢. Prejection of a point on a given line.
Let P be a given point and AB the ^
given straight line. Draw perpendicular
P M fto m P to ABt meeting AB in M.
Then the foot of the perpendicular M is
called the projection of the given point
P on the given line AB. e ~A
The point M is the point in which the
plane through P and perpendicular to
AB meets the line AB.
§ 7 . - —j—**— « r - —c----it y fa liar ta ansthrr Mae (in the

Suppose we are to find the projection of a segment CD of a


20 Analytical Geometry 3-D

line on another given line AB. Let the points C , D' be the
projections of the points C, D on the line AB, then the segment

C D ' is the required projection of the segment CD. The projec­


tion of the segment CD on the line AB may also be defined as the
intercept C D ' made on the line AB by the planes through the
points C and D each perpendicular to AB.
To Bni the length of the projection C D '. Through D draw a
lineDP parallel to U C and meeting the plane through C per­
pendicular to AB in P. Thus DP=DrC'. .:-(1)
Let 9 be the angle between the lines AB and CD. Also AB
is parallel to PD and hence l_CDP=e. Again the line CP lies
in the plane which is perpendicular to AB and hence CP is per­
pendicular to DP. Z.DPC=90°.
Therefore DP=DC cos 9. ...(2)
From (I) and (2), we have DrC = D C cos 0
or C D '=CD cos 9.
Remark'. Let DC= a and let b be a unit vector along BA.
Then
a*b=| a 11 h I cos 9, by definition of dot product of two vectors
=DC cos 9 [V J b J = I and | a |*=ZX7J
= projection of DC on BA.
Thus to get the projection of DC on a line BA, we take the
dot product of the vector DC with a unit vector along BA.
§ 8. Projection of a broken line an a given line, Or Given n
points Ctl Ct,..., Ca (say) in space, to Iindthe projec­
tion of CtCa on a given line.
Suppose itil is a given line. Let the projections of the n
points Cit Ct , ...»Cm on the given line AB be the points
Direction Cosines and Projections 21

Mu 4*8. » Afn respectively. Tbustbe points Afle Mi9...9 Mm lie


on the same straight line AB. Hence in view of § 7, we have
projection of C1C1=M iMu
projection of C1C1= M 1M1,...,
and projection of Cm^i Cm= M n^1 Mn.
Λ sum of the projections of CiCiv C1Cl9...9 C„_i Cm on the
Woe AB=M iM1-)^M1M1-^ · Ί* Mm
= M iMm= projection of C1Cn on AB.
Hence we conclude that :
Projection of CiCn on a line >4/?=Sum of the projections of
CiCiv ClCl r - A . ! Cn on the line AB.
§ 9. Direction cosines of a line joining two points P (xlv yu Z1)
and Q (x2v yt9 z*).
Let the line PQ make angles
α. β9 γ with x, y, z axes respec­
tively. If I9 m, n be the d c.’s
of this line, then
I= cos e v Wi-=COS β9 n =cos y.

Now if L and M are the


projections of the points P and
Q respectively on the x-axis,
then we have
OL=Xu OM=Xu so that
L M = O M -O L = X 1- X 1.
But LM is the projection of PQ on the axis of x. Hence by
§ 7Vwe have LM =PQ cos a or X 1 —Xi= l.PQ
or (X1- X 1)II=PQ. ...(I)
In a similar way by projecting PQ on y and z axes, we have
Iy1^yiV m = PQ and (Z1^ z 1)In=PQ. ...(2)
So from (I) and (2),-we have
x2~ * i_y t—yt ^jz1- Z 1_^pQ

Hence we conclude that the actual direction cosines I9 m9n of


the line joining two points P ( x u yu Zi ) and Q (xSv y l9 Z1) are
x* -x i y* yi Z2- Z 1
PQ ’ PQ ’ PQ
respectively9 where PQ = λ/ { ( χ 2— * i ) 2+ 0 ¾ - J i ) 1·+(Z1- Z 1)1).
Again the direction cosines o f the line PQ are proportional to
22 Analytical Geometry 3-D

X1—Xi, j i - j i , X*—Xi which are therefore the C itdiM rtti·* of the


line PQ.
Vector o d h t t Since the co-ordinates of the points P and Q
are (xtl yu Z1) and (X1, J1, zt) respectively, therefore
OP*=*the position vector of P = i|i+ y ij+ Z ik
and 0f?«»x*i+.y*i+z*k.
Λ P Q = O Q -O P -(X 9- X 1) !+Cf1- J i) !+(Zt-Z 1) k.
Also I PQ \= P Q = V U x t-X iV + iy t-y iF + fa -Z ifl··
Λ a unit vector along i+ ^ = ^ I + ^ 1k·
Hence the dx.’s of the line PQ are
Xt -X l J1- J 1 Zt -Z 1
PQ * PQ * P Q '
Also the d.r.’s of the line PQ are X1-X lt J1- J lt Z1-Z 1.
$ 1 ·. If O and P are two points (·, 0, ·) and (x„ y„ S1), then to
prove that the projectio· of OP on a line whose direction
cosiaes are I. ■ . a is Ιχ,+ n jt+ M ,.
Construct a rectangular parallelepiped otth diagonal as OP
pnd faces parallel to the co-ordinate planes. [See figure of § 3.
chapter I, page 3]. Then clearly we have
OA=X1, O B =yt =AN, CO=Z1=NP.
Now considering OP as a broken line consisting of the parts
OA, AN and NP, ore have
the projection of OP on a line with dx.*s I, m, n
-»sum.of the projections of OA, AN and NP on the line with
d.c.*s I, m, n
« ^ !+ to y i+ a z i· P>y § 7]
Vector Method. We have Ofi=X1I+ jj + z ,k .
Abo a unit vector along the line whose dx.*s are I, m, m
= R + m j+nk.
A projection of OP on the line whose d.c.’s are I, m,n
=(*i*+Jij+Zik)*(/i+i»ij+Bk)=ix,+»nj1+*z1.
§ 11. Ta find Ae prajectian af the line Joiaiac two points P(XltJiaS1)
and QCxlt Jlt **) ·■ another fine whose d.c.’s are It ■ , a.
Let O be the origin. Then
OP^Xti+JU+Zik OO=X1I+ j j + Z1I .
Direction Cosines and Projections 23

P Q = O Q -O P = (X 3-X i) 1+ 0 ¾ -*) J+(*s-*i) k·


Now the unit vector along the line whose d.c.’s are i, m, h
= /i+m j+nk.
Λ projection of PQ on the line whose d.c.’s are I, m, n
*1) •+O'*-;'.) k].(H+mj+nk)
= I (x 3- x t)+ m (ya- y i) + n (za- z ,) .
Remark. InthearticleslO and 11, i,m ,n are the actual
direction cosines and not direction ratios.
SOLVED EXAMPLES 2 (B)
Ex. I. I f P (2, 3, —6) and Q (3, —4,-5) are two points, find
the d.c.'s of OP, PO, OQ and PQ where O is the origin.
Sol. By § 9, the direction ratios of OP are 2—0, 3—0, —6—0,
i.e., are 2, 3, —6.
Also O P = + 3 2+ ( —6)®}= y /(49)«=»7.
Hence d.c.’s of OP are
2_ J_ - 6 2 3 -6
OP’ OP’ O P t e ' are 7' 7’ 7 ' A ds.
The d.r.’s of PO are 0—2, 0—3, 0 - ( - 6 ) i.e., —2, —3, 6.
Also P O = \/{ ( - 2)2+ ( —3)*+6‘}=7.
.·. d.c.’s of PO are —2/7, —3/7, 6/7. Ans.
The d.r.’s of OQ are 3—0, —4—0, 5—0 i.e., 3, —4, 5.
Also 0 2 = ^ ( ( 3 ^ + ( - 4 ^ + ( 5 ) ^ = ^(50)= 5-^2.
Λ d.c.’s of OQ are 3/(5-y/2), —4/(5\/2), 5j(Sy/l). Ans.
The d.r.’s of PO are 3 - 2 , - 4 - 3 , 5 - ( - 6 ) i.e., I, - 7 , 11.
Also Ρ β = ·\/{(3 —2)2+ ( —4 - 3)**+(5+6)*} = \/(17l).
Λ d.c.’s of PQ are I M l ? ! ) , - 7 M 1 7 1 ) , 11/^/(171). Ans.
Ex. 2. Find the length of a segment of a line whose projections
on the axes are 2, 3, 6.
Sol. Let CD be a segment of a line whose direction cosines
are /, m, n. Then, we have
2 = the projection of CD on the x-axis= /.CD ,,,(I)
3= the projection of CD on the y-&x\s=m,CD -(2 )
6= the projection of CD on the z-axis=«.CO. ...(3)
Squaring (I), (2), (3) and adding, we have
4 + 9 + 3 6 = (i2+ m , + n 2) CD*
or 49=1 .CO* [ v l*+m*+n*= 1]
or C D = 7. Ans.
Ex. 3. I f P, Q, R, S are four points with co-ordinates (3, 4, 5),
U Analytical Geometry 3-D

(4, 6, 3), (—1, 2, 4), (I, 0, 5) respectively, then find the projection
o f PQ on RS. Also find the projection o f RS on PQ.
Sol. To find the projection of PQ on RS, we should find the
d.c.’s of RS.
The direction ratios of RS are
1 - ( - 1 ) , 0 - 2 , 5 - 4 Ie., 2, - 2 , I.
Also /{ 5 = - ^ ( ( 1 - ( - 1 ) ^ + ( 0 -2 ) ^ + ( 5 - 4 ) ^ = - ^ ( 4 + 4 + 1 ) = 3 .
d.c.’s of RS are 2/3, -2 /3 , f/3.
Hence the projection of PQ on RS i$ (See § 11)
= / (Xi-X,) + W (>*-3’i) + n (z»-2|)
= } (4 -3 )-1 (6 -4 )+ } ( 3 - 5 ) = f - |- i = - f Ans.
Again to find the projection of RS on PQ, we should find the
d.c.’s of PQ.
The direction ratios of PQ are
4 - 3 , 6 - 4 , 3 - 5 i.e., I, 2, - 2 ,
Also PQ = v/{(4 - 3)*+(6 T4)2+ (3 - 5)2}= V ( I + 4+ 4 )= 3 .
Λ the d.c.’s of PQ are 1/3, 2/3, -2 /3 .
Λ projection of RS on PQ is (See§ 11)
\ = I 0 - ( - 1 ) } H ( 0 - 2 ) - 1 ( 5 - 4 ) = } - $ - } = - }· Ans.
Ex. 4. I f P, Q, R, S are four points with co-ordinates
(2, 3 , - 1 ) , (3, 5 , - 3 ) , (I, 2, 3), (3, 5, 7) respectively, prove by
projections that PQ is at right angles to RS.
Sol. In order that PQ is at right angles to RS, the projec­
tion of PQ on RS should be zero.
The direction ratios of RS are 3—1, 5 —2, 7—3, i.e., 2, 3, 4.
Also RS= V ( O - I ) ' +(5 -2 ,2+ (7 —3)2}
—^ (4 + 9 + 1 6 ) = ^(29).
.·. d.c.’s of RS are 2/^(29), 3,V(29), 4/y(29).
Hence the projection of PQ on R S is (See § 11)

V (2 9 ) (3 2 ) + V(29)‘(5 3*+ V(29)* 3 *


2+ 6 - 8 §
V<29) °-
Therefore PQ is at right angles to RS.
§ 12· Angle between two IineaS.
To show that the angle Θbetween any two lines whose direction
cosines are I19 mu nx and /2, mi9 n2 is given by
cos 0= IiI2+ Di1Pi2-ΜιΟ*.
(Meerut 1980, 85S: Kanpur 82)
Direction Cosines and Projections 25

Lel AB and CD be two given lines whose d.c.’e are Iu i»i* *1!
and It, m2,n 2 respectively. Through the origin O draw lines OP
and OQ parallel to AB and CD
respectively so that direction
cosines of OP and OQ are
Iu mu n,; and /*, m2t H2 respec­
tively. Take a point P(xu yu zt )
on OP such that OP=Ti. Since
the d.c.’s of OP are Iu mit nu
therefore the co-ordinates of P
may be written as
Viru HJiTit H1T1).
Λ X1- I 1r „ Xi-Tn1Tit Zi = HlTi. -(1 )
Similarly ir we take a point Q (x2, y t, z%) on OQ such that
OQ=T2tIhea
Xi^U ru Λ = IH2Tt , Zt=H2T2. —(2)
Let Θ be the angle between the lines AB and CD, then 9 is the
angle between OP and OQ.
Now the projection of OQ on OP
—Ii (Jfa- O)-Hiw1 ()'i - 0 ) +W1 (Z| 0)
IiX2+ M 1X2 + I i i Z2 — ( 3)
But the projection of OQ on OP=OQ cos θ= τ2 cos Θ. ...(4)
From (3) and (4), wc have r2 cos $—I1X2+/w ^»+H1Z2
r2 cos Θ= I1I2T2+TniIn2T2-^-HlTt2T2 using (2)
or cos Θ- I 1I1+In1In., -I niD2. -(5 )
Remark If Θ is the acute angle between the two lines, then
cos Θis + ive and so we have
cos Θ==| IiI2-^miHi21 Hin2
Corresponding formula when direction ratios of the iines are
given. Let O1, bit C1 and a2, b2, c2 be the direction ratios of the
two given lines . Then their actual direction cosines are given by
Oi bj _ _ __ Cl _____
and
V i tOi 2 HV + c, V V W + ^ + C i 2') · V iO i'+ bS+ cS)
_____ Q2 __ b2 ______ c*______
v W + V r c r V V io 2*+ b j+ c 2·) * *
Using these values of d.c.’s in (5), the augle 0 between these
two Iines is given by
a,a,-H b,b* r C1C1
cos Θ«
+ ΒΛΗ^ι *) V +Ba2-HC1*) ...(6)
26 Analytical Geometry 3-/)

Cor. I. To find sin 9 and tan 9 in terms of d.c's and d.r's


of the two given lines.
F irstof all we state the Lagrange's identity which is as
follows :
If /,, Wi, W1 and I2y w2, w2 are two sets of real numbers, then
( / l 8 + W ,a + W ia) (/2 2 + W 2a + W22) - ( / 1/2 + W iW 2 + WlW2)2
= (WiW2-W 2Wi)2+ ^ ! /2 - ^ / , ) 2 + ( / ^ 2- ^ 1)*. ...(7)
[Remember]
Now, we have
sin2 9 = 1—cos* 0 - 1 - ( / ^ + W1W2+ W1W2;2
= ( / l 2 + W i 2 + / Z 12) (/22 + W 22 + W 22) — (/1/2 + W 1W 2 + W1W2) 2
= (w1w2- w 2w1)a+ (w1/2+ w2/1)2+ (/1/w2- / 2w1)a, using (7).
Λ sin 9= \/{Σ (w,w2—W Z2W1)*). ...(8)
The value of sin* 9 may be conveniently remembered in terms
of determinants as follows :
We write the corresponding direction cosines of the two lines
above and below in two rows as follows ;
/, w, w, I
/2 W2 W2 j .. (A)
Then sin* 9 = m1 "l I- /l Al Wi
m z w2 | + I Z2 w2 + W2
To get the first determinant we suppress the first column in
(A), to get the second determinant we suppress the second column
and to get the third determinant we suppress the third column.
In terms of direction ratios,‘the formula (8) is given by
(6,C2 - b2cx)2+ (C1W2— CiQl )2+ (Q1b2 - α φ \)a
sin2 9=
+ bS + C1*)V ( 02a+ V + c2*) . (9 )
Again, tan COS0
I " " '" '
ΛZ2+ WZ1W2+ W1W2 . ( 10)
in terms of direction cosines, while in terms of direction ratios,
\/{Σ (b\C2 b 2C\ )*}
we have tan 0 =
bxb 2 + C1C2 ...(H )
Note that the formula for tan 0 is the same whether we are
given direction cosines or direction ratios.
Cor. 2. Condition for perpendicularity. If the lines are per­
pendicular to each other, then 0= 90° i.e. cos 0=cos 90° = 0 and
therefore from (5), the required condition is
I1I2+ Dl1ID2+ D1H2= O* ...(12)
Direction Cosines and Projections 27

In terms of direction ratios the required condition of perpen­


dicularity [from (6)] is
M a f + ...(1 3 )
Note that the condition for perpendicularity is +e same
whether we have direction cosines or direction ratios.
Cor. 3. Condition for parallelism : If the lines are parallel to
each other then 0 = 0 t.e. sin 0=0 and from (8), we have
(/It1H2 - Hi2Zi1) 2 + (/I1 12 - n 2/ i ) 2 + ( Z1Hi2 - ZaHi1) 2 “ 0 .

The L.H.S. being sum of three squares of real quantities will


be zero if each of them is separately zero and hence we have
Ht1H2 -Hl2ZIi = O, H1Za-ZI2Z1- 0 , Z1HI2-Z 2ZZI1= O
or I1 __m i ni V(Zi 2+ ZZi18+ /I12) V ( I ) a i
Z2 ZH2 H2 V ( ^ 22 + /H22 + w22) V (I)
Λ I1= I2, Hl1= Dl2, D1= O2. (14)
This shows that the two lines will be parallel if their direction
cosines are the same.
In terms of direction ratios the required condition of para-
at b, Ci
Ilelism is
H2^ b 2- C2
This shows that the two lines are parallel if their direction
ratios are proportional.
Vector approach for § 12. (i) To find the angle between two
lines whose d c ’s are I19 Hi1, nx and I2t zn2, zi2.
/

We have a= th e unit vector along the line whose d.c.’s are


Zi , /W1, H1=Z1I+ HlJ+ H1It,
and b= the unit vector along the line whose d.c.’s are
Z2, ZH2, Zl2=Z2I + ZH2J+ n2k.
If 0 be the angle between the two lines, then
a*b=| a I | b | cos 0, by definition of dot product of two vectors
=> (Zii+ ZHJ+ Hjk) · (/2i+ Hi2j + w2k)
=(I) (I) cos 0 [ v I a |«=1H b | ]
^ COS 0 = Z1Z2+ ZH1ZH2+ H1W2.
Also by definition of cross product of two vectors, we have
a x b = j a I | b I sin 0 N, where N is a unit vector perpendi­
cular to both a and b
^ sin 0 N, because | a | = 1=| b |.
■a x b)2= S in 2 0. [V N1= N - N = I )
28 Analytical Geometry 3-Z)

i \ k

/. n1

It m% W2
C-(W Iifia -W a W i) i+ (V 2 -w2/|) j + (ZifW2 - Z 2W1) k.
sin* 0 = (a x b )* - (axb )* (ax b )
= (fWiWa—fWaWi^+ifii/a-Wa/i^+C/ifWa- Um i ) 1.
Condition for perpendicularity. Thetwo lines are perpendi­
cular if and only if a«b=0
i.e., iff (/ii+w ij + wik)«(/ii + msj+ *sk)= 0
i.e. 9 iff /i/2^fw1W2H-Wjff2=O.
Condition for parallelism. The two lines are parallel if and
only if the vectors a and b arc collinear
i.e.y iff a = Ab, where A is some scalar i.e . 9 real number
i e . 9 iff /!! + WjjH-W1Ii = A (/2i + w2j + w2k)
Le ., iff Zi=AZ2, /Wi= Aw2t Wi = Aw2
W|
i e . 9 H r i- = S . 1W2*
U W2
(ii) To find the angle between two lines whose dr.'s are a1$ bl9
Ci and a%9 Zf2, c2.
We have
A = a vector along the line whose d.r.’s are al 9 Dl 9 C1
—^ii + Z/ij + Cik,
B = a vector along the line whose d.r.’s are a2t b2%C2
= u2i + Zf2j + c2k.
Now proceed as in case (i). Here | A 1=^/(^1*+¾1+ ^ 1) and
I B !---v W + W + r?2)·
§ 3. To find the perpendicular distance of a point P (x\ y'f z ’)
from a line through A (a, b, c) and whose direction cosines
are I, m, n
Let AB be a line through A (a9 b9 c) and whose d.c.’s are
I9 m9 n. Let IjN be the perpendicular from P to AB.
Now AN-- projection of the
line segment joining A (at b9 c)
Direction Cosines and Projections 29

We have, PN9= A P t - A N t
or /W*={(x'-o)*+(/-Z>)*-f(2'-c)*}
- { ( x ' - o ) l- H y '- b ) m + iz’- c ) »}*
or FAZ*={(*'—«)*+(/—6)·+ (z’—c)1) {/*+ ot*+ n*} -
—((Xi- a) / + ( / —6) m+(2’—c) n}*
= Σ { ( / —b) n—(zr—c) ot}* [by Lagrange's identity]
Λ JW =Vt? {(/-6) O -(Z t-C ) ot}*].
Aliter. Let /,P A N = » .
We have, P N t = A P t sin* 9.
Now 6 is the angle between the lines AP and AB. Here
the d.c.’s of AP are (x’-a )IA P , (y ’-b )/A P , (Zr-C )IA P
and the d.c.’s of AB are /, ot, n
I (y r-b )IA P (Zr-C )IA P t
Λ sin* tf=|·
: OT /I

Remark. IntheformuIa(I)f Zt OT1Harethe d.c.’s of the line


AB. If however, a ,/3, y are the d r.’s of the IineAf, then to get
PNt we should divide the R.H.S. of (I) by a*+/3*+y*.
SOLVED EXAMPLES 2 (C)
Ex. I. Ifpoints P, Q are (2, 3., —6), (3, —4, 5), then find the
angle between OP and OQ where O is origin.
Sol. The direction ratios of OP are 2—0, 3—0, —6—0 i.e.t
are 2, 3, —6.
Also O P - V{(2—Oj*+(3—0)*+(—6—0)*]= y / (49)=7.
Λ the d.c.'s of OP are 2/7, 3/7, - 6/7.
Thedirection ratios of OQ are 3—0, —4—0, 5—0, i.e., 3,
- 4 , 5.
Also OQ= V'{(3-0)*+(-4-O)*+(5—0)^=^(50)=5^2.
Λ the d.c.’s of OQ are 3/(5 y/2), - 4/(5y/1), 5/(5y/2).
30 Analytical Geometry 3-D
If Θis the angle between OP and OQy then
cos 0 = /1/2 + m]m2+ TtlH2. [See eq n . ( 5 ), § 12]
-3 6
cos 35y/2
-1 8 χ /2 . Λ ___ , | -1 8 y /2 \
or cos 0 — +=CO S' Aqs
35 35 ;
Ex. 2 (a). I f points P, Q are (2, 3, 4), (I, —2, I). then prove
that OP is perpendicular to OQ where O is (0, 0, 0). [MagaA 68]
Sol. T hedirectionratiosof OP are 2 - 0 , 3 - 0 , 4 - 0 , Le.
2, 3, 4.
The direction ratios of OQ are 1—0, - 2 - 0 , 1 - 0 l.e. I,
- 2, 1.
If OP is perpendicular to OQ, then we must have
alai +b1b2+c1c2=0. [See eqn. ( 13), § 12]
Now ^2+^1^2+01(-2=(2).(1)+(3).(-2)+(4).(1)
= 2 -6 + 4 = 0 ,
which shows that OP is perpendicular to OQ.
Ex. 2 (b). Show (hat the line joining the points (Ot I9 2) and
(3 ,4 ,6 ) is parallel to the line joining the points (—4 ,3 , —6)
(5. 12, 6).
Sol. Thedirectionratios of the line joining (09.1,2 ) and
(3, 4, 6) are 3—0, 4—1, 6—2 Le., 3, 3, 4.
The direction ratios of the line joining ( —4, 3, —6) and
(5, 12, 6) are 5 - ( - 4 ) , 12 —3, 6 - ( - 6 ) Le , 9, 9, 12.
We see that the direction ratios of the two lines are propor­
tional because we have 3/9=3/9=4/12.
Hence the two lines are parallel.
Ex. 3. I f the vertices P9 Q and R of a triangle have coordinates
(2, 3, 5), (—1, 3, 2) and (3, 5, —2) respectively, find the angles o f
the triangle PQR.
Sol. Thedirectionratiosof-Pg are —1—2, 3 —3* 2 - 5 i e 9
- 3 , 0, - 3 .
We have ^ { ( - 3)8+ 0 * + (—3>2}= λ/(Ι8 )= 3 λ/2.
.·. lb . i 0 .·. of PQ a t. j= 2 ,
Similarly d.c.’s of QR are

and d.c.’s of PR are


Direction Cosines and Projections M

Now using the formula [eqn. 5, § 12] cos Θ=I1I2^n iim 2+ ^ n i9


we shall find the angles P9 Q9 R of &PQR as follows*:
We have, cos P —cos of angle between PQ and PR
=_J_ _L+o
V 2 '3 V 6 T 3V6
LWjzZV___L
V V V V V (> /
-J L
3 V ( 12) 6\/3
= l / v /3 . Λ z L ^ c o s " 1 (IZVrS).
Again cos g«=cos of angle between QP and QR
I
' V i 3+ ° + V 2 HH-
z e = 9 o ·.

[ Note that d.c.?s of β Ρ are


Finally cos R = cos of angle between PP and Ρ β
V 2 ’ °* V 2]

- ( - l ) ( - ^ ) +( - l ) ( - ^ H l ) ( i b )
R=Cos"1^(2/3).
Ex. 4. Prove that the straight lines whose direction cosines are
given by the relations al+bm+cn=Q and fmn+gnl + hlm=0 are
perpendicular If fla+ glb + h/c=0
and parallel if V (af ) ± V ( b g ) ± \/( ch) = Q-
(Meerut 1984, 85 P, 87, 89)
Sol. From the first relation, we have n= —(al+bm)/c.
Putting this value of n in the second relation, we have
al+bm\
C
or afml + bfm2+ agl2+ bglm - chlm = O
or ag —a + — (af+ bg-ch) + bf= 0.
ml m ...(D
Now if Iu mu W1 and I29 w2. n2 are the direction cosines of the
two lines, then the roots of (I) are Iijm1 and /2/w2.
. Λ Γα. ' h h bf Λ/a WiiW2
product of the roots= —. — = — or -?- = — rr
nil wa ag fja gb
/|/2 W1W2
' m - by ^mmetry.
( f!°) (gib)
We know that the lines are perpendicular if
/1/2+ WjW2+ WiW2=O
i e if f/a+glb + hlc=09 which proves the first part.
If the lines are parallel then the direction cosines are the
same. This shows that the roots of (I) are equal, for which the
condition is tB7=AAC9
i.e. (af +bg - ch)2=4ag.bf.
32 Analytical Geometry 3-D

Taking square root, af\-bg - ch— ±2y/(afbg)


or a f± 2 y /(a f bg)+bg=ch
or W i af ) ± y/{bg))%={ch).
Taking square root, y/(af)±.\/{bg)*= ± Vr(CA)
Ot y/(of)±y/ibg)±y/(ch)---0,
which proves the second result. Proved.
Es. 5. Show that the lines whose direction cosines are given by
the equations 2/ + 2m -n«= 0, and m n+ nl+ lm = 0 are at right angles.
(Mctiiit 1986 S)
Sol. From 2/+2m —n=0, we have n=2l+2m. --Ό)
Putting this value of n in m n+ nl+ lm - 0, we get
m (21+2m)+(21+2m) l+ lm ^ 0
or 2fi+Slm+2mi = 0 or (l+2m) ( 2/+ m ) = 0.
Λ l——2m and 21+m —0.
When / = —2m, from (I) n-——2m.
• ^ ffi _ 2 _ y/jP + m * + ^) _,!■
2 ™ -l + 3
Λ The d.c.’s of one line are 2/3, —1/3, 2/3.
Again when 2/= - m , from (I), n=m .
. I _ _ m _ n _ y/iP+m'+ri*) I
'· - 1= 2 = 2 V{( —I) * ( 2)»} 3
A The d.c.’s of other line are —1/3, 2/3, 2/3.
The lines will be at right angles if IlIi + Mim2-I n(n2= 0.
We have
/i/*+m,w 2+n,n2= ^.
Hence the lines are at right angles.
Ex. 6 . Show that the straight lines whose direction cosines are
given by the relations a i+ b m + cn -0 and ui*+vm*+wn%—0 are
perpendicular or parallel according as
a* (v+ivj+fr2 (M-fw) + ca (w+ v)= 0
or a*Iu+ViIv+Ci Iw=0. (Meerut 1984, 86, 89S)
Sol. From the first relation, we have n=~~(al+bm)ic.
Putting this value of n in the second relation, we have
ul'+ vm '+ w { -< * / 4 ^ } * = o
or (CtU-^a2W) l2+ 2abwlm + (b2w + c2v) m2- 0
or (c2u + a2w) (i/m)2 Ή2abw (llm) + (btw-rciv) —0- ...(I)
Let Iu mu rii and /*, mu n%be the d.c.’s 01 iLc two lines.
Directions Cosines and Projections 33

Tbeo the roots of (I) are Iijmx and Itjmt .


b9w+c9v
product of the roots= — . — =
mx m% 'c*u+a*w
ixl% mxmt ηλη% , by symmetry.
b2w+c*u c2u + q 2w a*v +b*u .. ( 2)
The Iioes will bs perpendicular if /|/g+tfiima+/iif?i=0
i.e. (b2w+c9v)+(c9u+a2w)+(a9v + b2u)<=»0
or a2 (v+w)+b2 (m+ w) + c* (v + n) = 0. Proved I.
The lines will be parallel if the d.c.’s of the lines are the same
i.e. if the roots of ( I) are equal, for which the condition is
B9~4AC t
Le. 4a2b%w%=4 (c2u+a2w) (6*w-|-c*v)
or Q9C2WV+ b9c9uw+ c9a9vu= 0 ,
or a9ju+b9jv+c9lw=>0. Proved II.
Ex. 7. Show that the lines whose d.c.’s are given by t+ m + n =*0
and 2mn+3ln—5/m=0 are at right angles. (Meerot 1983)
Sol. From the first relation, we have /= i—m—n. ...(I)
Putting this value of / in the second relation, we have
2mn + 3 ( —m—n) n—5 (—m—n) m = 0
or Sm2+ 2mn—3n2= 0 or 5 (m/π )1+ 2 (m/π)—3 = 0 . ...( 2)
Let Iu mu /I1 and /a, mtt n% be the d.c.’s of the two lines.
Then the roots of (2) are m jni and mtjnt.
mxmt nxnt
product of the roots = ™1.. — = - rj u r ------ *
nx -5
...(3)
Againfrom (I), /i = - / - m and putting this value of n in the
second given relation, we have
2m ( —/—m) + 3/ (—/—m)—5/m=0,
or 3 (Ijm)9+ 10 (//m) + 2=0.
• (l_ ? or il,% m' m2
mx' m2= 3 2^" 3 ...(4)
From (3) and (4), we have (say).
/i/a + miw*+ nxn2= (2 + 3 —5) / : = 0 ./: = 0 .
The lines are at right angles. Proved.
Ex. 8, Lines OAt OB are drawn from O with direction cosines
proportional to I, —2, —I; 3, —2, 3. Find the direction cosines of
the normaI to the plane AOB.
Sol. Since OA and OB lie in the plane AOBt therefore
these lines are perpendicular to any normal to the plane AOB.
34 Analytical Geometry 3-D

Let o,, at, O8 be the direction ratios of such a normal, U siogthe


condition of perpendicularity 0161+ 0363+ 0363= 0,
we have ο» ( 1)+03 ( —2) + as ( - 1)= 0 - (1)
and Oi (3 )+ 0» ( —2) +o, (3)=0. (2)
Solviog (I) OOd ( 2),
Aa
~ 1 .( - 2 ) - ( - 2 ) 3
Q\ O2 Os 01 02 Os
—8 ~ - 6 ~ 4 or 4 “ 3 " - 2
Therefore the d.c.’s of the normal are
4 3 - 2 ___ ___
\/{(4), +(3)»+(-2)*}’ v W + ( 3 ) ‘ + ( - 2 ) 2} ’ V [(4)1 + (3)*+ ( - 2)11}
4 3 -2
ΟΓ V(29>* V(29)' V(29)’
Ex. 9. I f l u mu «» and lt, mt, n3 are direction cosines o f the
two lines show that the direction cosines of the line perpendicular to
both are proportional to /W1W2-ZWoWm /IiZa-W 2Z1. Unis-Umi.
Prove further if the given lines are at right angles to each other
then these direction ratios are the actual direction cosines.
Sol. Suppose.that the required direction cosines of the line
are It mt n. Since the line is perpendicular to the given lines, we
have
//, + /Wm1 H-Zizii=O ...(I) and //a+zwiw2+ Zizi2=O. (2)
Solving (I) and (2), we have
I zw w
ZWiZi2- Hiinl^ n lI2- w2/, e IlHi2- Umi . (3)
This shows that the required d r.’s are /W1ZIvj-IW2Wi , W1Z2- w2/,.
Iimt -U m i.
Now suppose Θ is the angle between ifre two given lines
whose d.c.’s are Iit mlt Wi and /2, _/#*, w2.
Then sin 0 = y/{E (ZWiW2-IW2W1)2) [see cor. I, § 12] ...(4)
If 0=,90° i.e.t the lines are perpendicular, then (4) pives
\/{Σ (zw, W2- zw2Wi)*}«=!. - *··<5)
Λ in this case from (3 ) the d.c.’s I9 zw, w of the line are given
by i
I m n y/i /2H- zwa4- w8)
ZW1W1-Z W tWl - Z / , — ZI2Z1 " I J H 2 - I2W 1 ^ — Hl2Hl ) 2)

1” [Using (5) and l*+m*H-Wa==I]


Direction Cosines and Projections 35

-I.
Hence in this case the actual direction cosines I9 w, n are
/W1W2-W 2W1, Wj/2—w2/lf /JW2-Z 2Wi. Proved.
Ex. 10. Show that the direction cosines o f a line perpendicular
to a pair of mutually perpendicular lines with direction cosines as
Iu mu W1 and /2, w 2, W2 respectively are
WiW2 -W2W1, Wj/2—w2/i, Z1W2-Z2Wi. (Bhagalpur 1966)
Sol. See example 9. This is other way of stating the same
problem.
Ex. 11. Prove that three concurrent lines with direction cosines
Iu Ήι, Z2, w2, w2; and /3, w3, w8 are coplanar if
A W i Wi

/* W 2 W2 = 0 .

/. W 3 W3 ( M e e r a t 1983 S )

Sol. Let Z, w, w be the d.c.’s of the normal to the plane in


which the two concurrent lines with d.c.’s Iu mu nt and /2, w2, W2
lie. Then the line whose d.c.’s are /, w, w is perpendicular to the
lines whose d.c.’s are Il9 w t , W1 and Z2, w 2, w2. Therefore
/!/+WiW-HWiW=O ...(I) and /2/+ w 2w+w 2w= 0 . . (2)
Again if the third concurrent line whose d.c.’s are /3, w2, W2
also lies in this plane, then it is also perpendicular to the normal
to this plane.
Λ /3/+ w 3w+waw«=0 . ...(3)
Eliminating I9w, w between (I), (2) and (3), we get the
required condition as I1 Wi Wi
Ia w2 W2 = 0.
/s W3 W2

Ex. 12. Prove that the three lines drawn from a point with
direction cosines proportional to I, —I, I; 2, —3, 0, and I, 0, 3 are
coplanar.
Sol. Let a9 b9 c be the direction ratios of the normal to the
plane in which the two concurrent lines with direction ratios
I, —1 , I and 2, —3, 0 lie.
Clearly these lines will be perpendicular to this normal. Hence
applying the condition for perpendicularity of two lines, we have
1.0 + ( - 1 ) .6 + 1 .C=Of ’ ...(I)
36 Analytical Geometry 3-D

and 2 . e + ( - 3) .h + 0 .c = 0. ...( 2)
„ , . .. a b c a b c
Solvngtbese. ¢ + 3 - 5^ - - 3+2 » ' 3 - f ^ l
Again the third concurrent line with d.r.’s I, Ot 3 will lie in
this plane if the normal with d.r.’s 3, 2, —1 is also perpendicular
to this third line.
We have (3) (1)+(2) (0 ) + ( - 1 ) ( 3) = 0, showing that the lines
with d.r.’s 3, 2, - I and 1 ,0 ,3 are perpendicular. Hence the
three given lines are coplanar.
Ex. 13. The d.c.’s of two intersecting lines are Iu f?i and
Uf Ff1 Show that all lines through the intersection o f these two
whose d c ’s are proportional to I1+ kl3, mi + km%%nx+ kn% are cop­
lanar with them.
Sol. Let /, FFf, π be the d.c.’s of the normal to the plane in
which two intersecting lines whose d c.’s are I1^mu nx and
U$ Fff1, π* lie. Clearly these lines will be perpendicr.hr to this nor­
mal. Therefore /iZ+Fn^+FiiFf —O, ···(*)
and V+Fff1FH+ Ff2Fi=O. ·*·(2)
Now any line through the point of intersection of these lines
with d.c.’s proportional to h + ktu mt + kmt, nx+kn2 will lie in
this plane if the normal to this plane is also perpendicular to this
line.
We have l(li+ klt)+ m (m i+ km 2)+n(nl +kni)
s = lli+ m m i+ nnl -\-k{IU + mmt +nni)
= 0 +A:.0, using (I) and ( 2)
= 0.
This shows that the line whose d.c.’s are I, m, n is perpendi­
cular to a line whose d.r.’s are l\+ klit m i+ km 2, nx+kn2.
Hence all lines through the intersection of the given Iinesand
with d.c.’s proportional to lt+ klt, m ,+ km a, nx+ kn2 are coplanar
with them.
Ex. 14. Ifa variable line in two adjacent positions has direction
cosines I, m, n and 1+81, m+8m, n+8n, show that the small angle
89 between the two positions is given by (80)*=»(8/)*+(8m)*+(8n)*.
• (Meerut 1984 P, 87 P, 88)
Sol. Since l,m ,n and (1+81), (m + 8m), (η+δ/ι) are the
actual direction cosines, we have /*+m *+n*=l ·- (I)
and (l+8l)*+(m+8m)*+(n+8nj*=* I
or (/*+ m*+ π»)+ 2 /8 /+ 2m8m+ 2πδπ+ (81)*+ (8m)* + (Sn)*= I
Direction Cosines and Projections 37

or \+2(ΙδΙ+η&η + η&η) + (81)2+ (δηι)'+(δη)ι= \ fUsing (1)]


or 2(/8/+ // 18//1 + /18/1)= —{(8/)*+|8#»i)* + (8/i)*}. ...(2)
Now it is given that 80 is the angle between two adjacent
positions ot the line. Therefore
cos 80 —/.( /+ 8/) + m.(//i + 8//i)+/i.(/i+ 8/i). ...(3)

Now cos S ffc = I-^ y + ^ y + ···


m*
.·. if 8Θ is small, we have cos Sd= I -
2! '
Then Irom (3), we have

1~ ('2 'T = (/2+ ro 2+ "*)+ ( /s/+ mSm+nSn)

or I - l J l = I - I {(δ/)*+(δ/Μ)Η(δ«*)} (using (I) and (2)]


or (Se)*=(SI)*+(Sm)*+(8n)K Proved.
Ex. 15. Prove that the acute angle between the lines whose
direction cosines are given by the relations
I -\-m+ n = 0 and l2+ m %—n2=0 is π/3. (Meerut 1986 P)
Sol. The relations giving the d.c.’s ot the two lines are *
/+/H-M=O, ..( I ) and / 8+ m 8—n2« 0. ...( 2)
From (I), / i = ~ (/+m). Putting this value o f/ι in (2), we get
l2+ m *~ (/+/w)8= 0 , or Hm=O9 or Im=O.
.*. /= 0 or m= 0.
When /«=0, wc have from (I), m = —it.
I m n
Λ F esT e = I- Γ.73)
Again when m = 0f we have from (I), /=3- / 1.
I m n
Λ ...(4}
From (3) and (4), we observe that the d.r.’s of the two lines
are 0, I, —1; and I, 0, —I.
.·. their d.c.’s are 0, l/\/2 , —I jy/2\ and l/y/2 9 0, —1/λ/ 2.
If 0 is the acute angle between the two lines, we have

:. 9—π IS.
Ex. 16. Show that the area of a triangle whose vertices are
the origin and the points (xlt y t, zt) and (xt, >’*, z*) is
3S Analytical Geometry 3-D

Sol. The direction ratios of OA are x i9 y i9 z x and those of


OB are x %
9J 2, z2.
Also OA = >/{(*i “ 0)s+ (Ji - O)2+ (Z1- 0 ) 2}= V(Xi2+ J i 1 + Zi1)
and OB = y/[{xt - 0)a+ (y 2- O)2+ (z2- 0)2}- v W +Ja 2+ Z21).
the d c.’s of OA are
*! Jl
V W + J i2 + ZiV V (* i2+ J i a-hZi2) ' + J i* + Zi2)
and the d.c.’s of OB are
*2 J2 z2______
V(* 2a+ J 2a+ z22) ’ V i^ 22+Ja2+ Z32) · V (^ 2a+ J 22+ z22)
Hence if 0 is the angle between the lines OA and OB9 then
i _________ λ/{Σ ( J i Z2- J 2Zi )a}______ λ/{Σ ( J i Z2- J » * i )2}
sin 0=
V l* i 2+ J ia+Zia) v W + J 22+ V ) β OA.OB
Hence the area of A OAB=^.OA OB sin θ [ 7 /_ΑΟΒ=Θ]
- J OAOB ^ 1*2" w * )9}
• - * · υ Α ·υ β · OA.OB
= 1\/{Σ (JiZ2- J a r 1)2). Proved.
Ex. 17. I f Ii9 mi9 H1 and I29 m2%n2 are the d.c.'s o f two con­
current Iines9 show that the d.c 's o f two lines bisecting the angles
between them are proportional to lx± l 29 Wi±m2, nx± n 2.
Sol Let O be the origin.
Draw OA and OB parallel to
the given concurrent lines. Let
I19 mi9 /I1 be the d.c.’s of OA
and I29 w2, n2 the d.c.’s of OB.
Cut off OA = OB = r. Then the
co-ordinates of A and B are
Oir9 mir9nxr) and (/2r, m2r9 n2r)
respectively.
Again take a point C on BO produced such that OB=OC r.
Thus the co-ordinates of C are ( —/2r, —m2r, —n2r).
Suppose M and N are the middle points of AB and CA. Then
the co-ordinates of M and N are
/ / i M - / 2r m , r 4 m 2r nxr + n2r\
, 2 ’ 2
flir —l2r mxr - m2r ηγτ —η2τ
and \ 2 ’ 2 2 )
respectively. Clearly OM and CW are the internal and external
bisectors of the angle AOB.
Direction Cosines and Projections 39

Hence the direction ratios of OM and ON are


£ (/1 + /2) r9 £ (/Wi + /w2) r, £ (W1+ wa) r
and $ (/1 - / 2) r, £ (/W1-Wi2) r, £ (W1-W2) r respectively,
/.e , the d.c.’s of OM and QN are proportional to Ii + I2, /Wi+/w2,
tfi+wa and /1 —/2, /Wi~ma, Wl-Wa respectively. Proved.
Ex. 18. F/W /Ae direction cosines of the lines bisecting the
angles between the lines whose direction cosines are Iu mu Wi and
/*, /w2t w2 nwd /Ac angle between these lines Is 0.
Sol. Proceeding as in Ex. 17 above the direction ratios of
the internal bisector OM of the angle AOB are
/i+/g. /wi+/wa> W1 +wa, ...(I)
Since 0 is the angle between the lines whose d c.’s are
Iu mu Wi and /a> /w2. w2,
Λ cos 0 = hh+ ntinh+ W1W2= Z 1 Z1Za.
Also JS7 /1*=/i*+rw 1*H-w1*= I and 27 Z21 W a1 +Wi21 + waa=» I.
We have
V { ( / i + / 2 )a+ ( / W i+ / w a)2+ ( w 1+ w a)a}«=» χ / {-27 Ιι2+ Σ /28+ 227 Zi Z2J
= V O + 1 + 2 cos 0)=3 V {2 ( I +cos 0))=2 cos £0 .
Dividing the direction ratios (I) by
V{(/1+ / 2)2+ (/Wi+ /W2)2+ (Wi+ W2)2)
i.e. by 2 cos 0/2, the d.c.’s of the internal bisector OM are
/1+/2_ /Wj+ /Wa Wi-f W2
2 cos 0/2 * 2 cos 0/2 * ^ cos 0/2 Ans.
Similarly the d.c.*s of the external bisector ON are
/l —12 /Wi -ZW2 W1 -W 2
2 sin 0/ 2’ 2 sin 0/2 ’ 2 sin 0/2 Ans.
Note that in this case, we have
V((/i -' /a)2+(/Wi - /w2)a+ (w1- W2)2)= V{27 /ia+27 /**-227 ZiZ2)
= V O + 1—2 cos 0)=2 sin $0.
Ex. 19. The vertices of a triangle PQR are the points
( —It 2, —3), (5, 0, —6) and (0, 4, —1 ) in order. Find the direction
ratios of the bisectors of the angle QPRt
Sol. Suppose the internal bisector of the LQPR meets the
side QR in L. Then we know that
QL : LR=PQ : PR. ...(I)
Now F 0=V {(5 + l) 2+ (O -2 )2+ ( - 6 + 3)2) = V(49) = 7,
and P R = V ii 0 + l ) 2+ ( 4 - 2 )2+ ( - l + 3j2}= V W - 3 .
Putting the values of PQ and PR in (I), we have
QL : L R = 7 : 3.
40 Analytical Geometry 3-D

This shows that L divides QR internally in the ratio 7 : 3 and


hence the co-ordinates of L are
/7 0 + 3.5 7.4 + 3 0 7 .(-1 ) + 3 .(-6 ) \
7+3 * 7+3 ’ 7+3
Le., /L Ii
*2*5’ 2 /
the direction ratios of the interna! bisector PL are

( 5 + 1 . ^ - 2. - ^ + 3 )/.*., are *, I Le., are 25, 8, 5.


Again the external bisector of the /_QPR will meet the side
QR in M, where Hf divides QR externally in the ratio 7 : 3 and
hence the co-ordinates of M are
/7
(7.0
.0 --3 .4 -33.0
3 5 77.4 .0 7.( - I) - 3 .( - 6 ) j
\ 7_3 * ‘ 7 -3 * 7-3

Le., ( - r - ’ r)·
Λ the direction ratios of the external bisector PM are
2, i i . 5.
Le., are —11. 20, 23.
Ex. 20. I f the edges o f a rectangular parallelopiped be a, b, c
show that the angles between the four diagonals are given by
-1 / ± β * + 6*±ο »1
'+ b ' + c*)/
Ita*+
(Kanpnr 1982, Meerut 84 S)
Sol. Let O, one of the vertices of the rectangular parallel­
opiped, be takeu as origin and the three coterminous edges OA,
OB and OC as the co-ordinate axes.
Direction Cosines and Projections 41

Let the edges O A= a, OB=b, OC=>c. The four-diagonals are


OF, AG, BE and DC.
The co-ordinates of the vertices of the parallelepiped are
given by O iO, 0, 0) ; A (a, 0 ,0 ); B (0, b ,0 ) ; C (0; 0. e ) ;
F (a, b, c) ; D (a, b, 0 ); E (a, 0, c ) ; G (0, b, c).
Note that the vertex D lies in the xy-plane, the vertex E in
the zx-plane and the vertex G in the ,yz-plane.
The d.r.’s of the diagonal OF are a —0, b —0, e—0 i.e.,
a, b, c.
the d.c.’s of the diagonal OF are
_____ o_ b _ __ c______
V tfli Vb1 Vc1/ V tfliVb 1 Vc*/ V ifli VbaVca)
Similarly the d.c.’s of the diagonal AG are
__—a b c_____ .
V tflaV b 1 Vcs/ V tfllVb 1 Vc1/ V ffli-Hb1+ c*) '
the d.c.’s of the diagonal BE are
a —b c_____ .
V tfla-Hb1 -He1/ V(fl, + b*'-He*)1 V (fliVb 1 -J-Ca) *
and the d.c.’s of the diagonal CD are
a b —c
V t A 1 -Hf l l - M 1/ V t fl!i-Hb *+ Ci )* V t fli H b* V caj
the angle 0 between the diagonals OF and AG is given by
a x ( —o) -Hbxb-l-cXe
cos 0 = -
V tfl*+ba-Hc*)V(fli+b*+ ca)
[using cos e= lil2+mimtA-Htni}
-A a-Hba-HcaI
i.e., Θ=COS'r* l - “ -
1 flMT b iT C ' f
The total number of pairs of the diagonals are *e2 i.e., 6 . In
a similar way the angles between the rest five pairs of the diago­
nals are determined and all of these six angles are given by
, r±fla±b*±<*[
a± b a± r sl
cos'
lVb1Vte1 I
I flaVb1V
The above expression will give only six valid values because
the ambiguous signs cannot be either all Vive or —ive for in that
case 0= cos-1 I or cos-1 ( —1) i e., 0=0 or 180° which is impossi­
ble as no two of the diagonals are parallel.
Ex. 21. A line makes angles x, 8, γ, 8 with the four diagonals
of a cube. Prove that cos* α-ί cos 2 fi + cos* y V cojs 8=4/3.
(Gorakhpur 1982; Meerut 81, 83, 85; Lucknow 81 ; Kanpur 83)
42 Analytical Geometry 3-D

Sol. Io a cube all the edges are equal. Proceediogasabove


io Ex. 20 the d.c.’s of the diagooal OF (puttiog b=c=a) are
____ a__ ____ a___ _ _ a
. /(β*+β*+β*)’ /(e* + e* + a* )’ /(α*+ο*+α*)
U ., 1/λ/3 , 1 //3 , 1 //3 .
. Similarly the d.c.’s of the diagooal AG are
- 1 / / 3 . l/v/3, l/v/3,
the d.c.’s of the diagooal BE are l/v/3, —l/v/3, l/v/3,
and the d.c.’s of the diagooal CD are l/v/3, l/v/3, - 1 /- /3 .
Now let /, 01, n be the d.c.’s of the line which makes aogles
a, β, y, 8 with the four diagonals of the cube. Theo we have
cos a = 7 .( l/ /3 ) + o i .( I/ / 3 ) + 0 .( 1 //3 ) =-(/+/ 0 + 0 )// 3
or cos* a= (/+oi+o)*/3, -(1)
cos )8 = /.( 'l / / 3 ) + oi.( 1 //3 ) + 0 .( i / / 3 ) = ( - / + o i + o ) //3 ,
or cos* β - ( —/+οι+ο)*/3. ...U)
Similarly cos* γ - ( I - οι+ο)*/3 *··(3)
aod cos* 8=(1+m —o)*/3. - (4)
Adding the relations (I), (2), (3) and (4), we get
cps* a H cos*/3+cos* y +cos* δ
— 1 { (/+ o i+ o )* + (-/+ o i+ o )* + (i—w +o)*+(/+oi-o)*}
= i {4 (i*+/o»+o*)}=.f. [V i*+oi*+o*=l].
Ex. 22. Find the angle between two diagonals of a cube.
Sol. Proceeding as io Ex. 21 above, the d.c.’s of the two
diagonals OF and AG of the cube are 1 //3 , 1 //3 , 1 //3 and
—1 //3 , 1 //3 , 1 //3 respectively. Thus if 0 is the acute angle
between the diagonals OF and AG, then we have
cos ¢-1 ( 1 / / 3 ) . ( - 1 / / 3 ) + ( 1 //3 ) .( 1 //3 ) + ( 1 //3 ) .( 1 , / 3 ) |= J
i.e., 0=cos -1 (J). .
This is the required angle.
Ex. 23. I f Oi, oij, oi), (/2, οι», o2), (/,, οι3, ο») are the direction
cosines of three mutually perpendicular lines, then find the direction
cosines of a line whose direction cosines are proportional to
li+ h+ l», 011+ 012+ 013, «l+Oj+Oa, and prove that this line Is
equally inclined to the given lines.
Sol. Since/1, ot], O1 ; /2, Ot2, O1 ; /3|nig, Ot are the d.c.’s of
three mutually perpendicular lines, we have
/l/i+OIjOIg + OiOg-O, /3/3+ OI2Ol3+ O2Ogs=O,)
Vi+Oi 3IOi+O3O1 =O. X ---(I)
Also ‘/1* + oii*+ Oi*= I2*+ oi2*+ o2*= /3*+ oi3*+ O3*= I .)
Direction Cosines and Projections 43

Now we have vW i + /2+ /3)* 4 (nt, + m24-m8)*4- (n, 4 B8+ ««)*]


*= 4-mi* 4-«i*) 4- U**+ »»**4-«**) 4- (V 4- m* 4- »**)
4-2 {(/,/24-WI1W24- BiW2) 4-(lth+ m tm ,+ ntn9)
4-( I9Il 4" BI8OTi 4" B8Bj))]
e V il 4-14-14-2 (04-04-0)], [using the relations (1)]
“ V3.
Thus the required d.c.’s of the line are given by
Λ4-/24-/8 bii 4-ot84 w8 Bi 4-b84-b3
VJ ' V3 ’ “ V% ‘ -(2)
Let 0 be the angle between the lines whose d.c.’s are 1%, tnu n,
and those given by (2). Then we have
cos 0= / i .(/ i + /8+ /8)/\/3+OT i . (mi 4-O t 84-ot3)/V3
4-«ι·(ηι4-«*+Λ»)/·\/3
= ( IIV 3) ((A*4-B»i*+«i*)+(Ms 4- OT1Bi24- «i«2)
4- (/*/l 4-OT8OTi4- B3Bl)]
= (I/V 3 ) [ 1 + 04- 0], [using the relations (I)].
Λ 0 = cos- 1 (1/V3).
Similarly the angle between each of the lines with d.c.’s
U, O
t 8, b8; I9, Bt8l n3 and the line with d c.’s given by (2) is
cos->(l/V3).
Ex. 24. Iftwopalrs ofopposite edges o f a tetrahedron are
perpendicular, then prove that the third pair is also perpendicular.
Sol. Let OABC be a tetrahedron. Let O be chosen as origin.
Let the co-ordinates of the vertices A, B and C be (Xi, ^it Zi),
(x3>y9, z8) and (x3, ^3, z9) respectively.
The d.r.’s of the edges OA, OB and OC are x8, y u Z1 ;
X3, y%. Z8 ; and x3, y9, z9 respectively. Also the d.r.’s of the edges
BC, CA and AB are X3- X 8, y9—y 9, Z9- Z 9 ; X i-x 8, y \—y 9, Zi-Z8
and X8—Xi, y 9—yi, Z9- Z i respectively.
Now suppose the edge OA is perpendicular to the opposite
edge BC. Then using the condition for the perpendicularity of
two lines, we have
Xi (X s-x8)+ fi ( J ' s - J ’a l + Z , (Z3- T 2)=O. ...(I)
Also if the edge OB is perpendicular to the opposite edge
CA, then we have
X2 (Xi —x3) + . ' ' 2 0 'i_ v3)+ z 8 (Zi—z3) =0. ...(2)
Adding (I) and (2), we have
x3 (Xi—x2)+J'3 Iyi - -^!+ Z 3 (Zj —z»)e=o. ...(3)
The relation (3) shows that the third pair of opposite edges
OC and AB is also perpendicular.
44 Analytical Geometry 3-D

Ex. 25. I f a pair o f opposite edges o f a tetrahedron be perpen­


dicular, then show that the distances between the middle points of
the other two pairs of opposite edges are equal.
Sol. Let OABC be a tetrahedron. Proceeding as in Ex. 24
above, if the pair OA aod SC of opposite edges be perpendicular,
then we have
Xi (x8- x2)+.Hi (y»-}'i)+zi (z3 - z 2) = 0 . ...(I)
Let Mi and M t be the middle points of OB and CA, so that
their co-ordinates are given by M1(Jx2. $>'2. iz2)and Mi ( J(X1+ x3),
J 0'i-H.Ha)» i (Zi+z3)).
Now
MlM f = U (Xi-Hi8- xa)}s+{i 0 Ί -Kv3- M 1 Hi U-r+ ζ,-ζ*)}*
= 1 {(Χι + Λ ί - ^ + Ο 'ι+ Λ -.Ρ ί^ Η ζ ,-Ι Z3- Z 2)*). ...(2)
Similarly if Mt and Mt are the middle points of OC and AB,
we have
Mt Mt* = J {(*1 + .X2- X j)*+(.V1H 12-.V3)* + (Zi -I-Z2- Z3)*). ...(3)
Now we want to prove that MtMt -- AI9Ma.
Subtracting (3) from (2), we get
MlMt* -M 9Ma1
= i [{(λ 1 + χ3- χ 8)’ - ( Λ ι + χ:8- Χϊ Η + Κλ + Λ - ^ ) * - ^ ! + ^ - ^ ) * }
+ { ( z i + z 9-■ z2)*—(ri -(- Z2- Z 3)2)]
=*έ [2χ, (2χ9 -2xt)+2yt (2y3- 2 y t)+2zl (2ζ 8- 2ζ2)],
using the formula a2—b*=(a+b) (a—b)
= X 1 (X8 - X 2) - H ^ i ( Λ - Λ ) + Ζ ι (Z8 - Z 2)
=0, using (I).
.·. MlMf-=M9Mi2 or MlM9= M 9Mi.
Ex. 26. I f in a tetrahedron OABC, OA2+BC2= OB2+CA*
=OC2+AB2, then show that its pairs o f opposite edges are at right
angles.
Sol. OABC is a tetrahedron. Let O be choseu as origin
and let the co-ordinates of the vertices A, B and C be (x,, y u z,),
(x2, .Hs. Z2) and (x3, y9, Z9) respectively.
Now OA2+ BC2=· {(xi—O)2-H(j>i—O)2+ (zi—0)2}
+ {(**—J 2)*+ O a - y *)2-H(z3—z2)2)
= (^ 12+ ^ * +Zi*)-H (x2*+.H22-Hz22)
• + (Xs* +>’32+ Z3*)—2 (X2X3 +yty 9+ Z2Z8)
= Σ x f + Σ x f + Σ χ92—2Σ XtX3. ...(I)
Proceeding similarly, we have
OB2+CA2= Σ χι2+ Σ χ 92+ Σ X32- 2Σ x 3.v , ...(2)
and OC2+ ΑΒ*=Σ x f + Σ x f + Σ χ / - 2 Σ χ,χ2. ...(3)
Direction Cosines and Projections 45

Now OA*+BCt =OBt +CA* gives


Σ Xit + Σ Xt*+ Σ Xit - 2 Σ XtX,-.*Σ χ χ*+Σ χ<?+Σ X3* - 227 X3X1
or Σ XtX3--Σ XtXi=-O
or (X3X3+y3y3+ Z3Z3) - (X3Jf1+ J’3>’i + T3Z1) - 0
or x3 (X3- X i) + ^ (y3- y i ) + z 3 (r»-zi)=0.
This shows that the edge OC is perpendicular to the opposite
edge AB L e ., an opposite pair of edges of the tetrahedron is
perpendicular.
Similarly by taking OBt +CAt =OCt +ABt and OAP+BC*
^=aOCi +AB*, the other two opposite pairs of edges of the tetra­
hedron will be at right angles.
EXERCISES
1. The direction cosines of t wo straight lines, inclined at an angle
Θ are Iu mu /I1 and I3, m3, n3. Show that direction cosines of
the bisector of the angle between them are
Ii + 13 m ,+ m 3 W1+ n 3
2 cos 0/ 2’ 2 cos 0/2* 2 cos 0/2 (Meerut 1984S, 85S)
2. Prove that the line joining the points (I, 2, 3) and ( —1, —2,
3) is perpendicular to the line joining the points ( —2, I, 5)
and (3, 3, 2). (Meerut 1985 S)
3
The Plane

§ I. Plane.
Definition. A plane is a surface such that all the points of a
straight line joining any two points on the surface lie on It. Or in
other words if we take any two points on the Surface9 the straight
line joining these two points wholly lies on the surface.
§ 2. (A) The equation of a plane. (Normal form).
Tofind the equation of a plane in terms of p l.e.9 the length of
the perpendicular from the origin to the plane9 and direction cosines
I9 m9 n of this perpendicular.
Let OX9 OYt OZ be a set
of rectangular axes with O as
origin. Let p be the length
of the perpendicular ON
from the origin O to the given
plane ABC. We shall take
p always positive. The d.c.’s
of the perpendicular ON are
I9 m9n. The direction of
AX

the perpendicular ON is from


origin towards the plane.
A line perpendicular to the
plane is called a normal to the
plane. Thus It m. n are the
d.c.’s of the normal to the plane, the direction of the norma!
being that from the origin to the plane. Ifn is a unit vector
along the perpendicular ON9 then
n=/i+tfij + wk and hence O N ^ p n ^ p (/i + mj + nk) ...(I)
The Plane 47

Let P with co-ordinates (x, y, z) be any point on the plane,


so that 0 P=»xi+.yj+zk. ...( 2)
We have, NP= O P—ON=»(x—pl) i+ (y—pm) j4-(z—pn) k.
So long as P lies on the plane, NP is always parallel to the
—►
plane and consequently perpendicular to ON and so also perpen-
A -►
dicular to n which is a unit vector in the direction of ON.
NP* n=0. ...(3)
Putting the values of NP and ή in (3), we get
{(x - p l ) i+ (y -p m ) j + (z—pn) k}*(/i-f m j+nk)—0
or (x —pl) l+ (y —pm) m + (z—pn) « = 0,
or Ix+my-t-nz—p (Z1 -Pmt -M 1)e O
or lx+ m y+ nz= p ...(4)
[V Z1-Pmt -Pnt - I ]
The equation (4) is satisfied by the co-ordinates of every point
on the plane, but by no point off the plane. Hence this is the
equation of the plane and is known as the normal form of the
equation of the plane.
Remark. In the equation (4), p is positive and Zt -Pmt -M 1 C*I.
Only then p is the length of the .perpendicular from origin to the
plane and /, m, n are the d.c.’s of the normal to the plane, the
direction of the normal being from origin towards the plane.
Cor. If the perpendicular ON makes angles α, β, y with the
co-ordinate axes then clearly we have /=cos a, m=cos β, n =>cos γ,
and the equation (4) becomes
x cos cos β + z cos ...(5)
(B) The general equation of a plane.
The equation (4) may be written as
(J tiM j+ 2k)*(^+m j+»k)e P, - ( 6)
where I, m, n are the d c.’s of the perpendicular ON.
Now suppose a, b, c are the d.r.’s of ON, so that /, m, n in
terms of a, b, c are given by
(/, in, n)«=(a, b, c)/V(e*+ 6t + c t).
Putting these values of /, m, n in the equation (6), we get
(xi+jj+zkM <ji-M j+ck)=»pv'(e*+ 6t + c t)
or (xi-Mj+zk).(ei+Z>j+ck)«= 0, —(7)
48 Analytical Geometry 3-D

where ς=ρχί(α*+&+ϋ*) and a i+ 6j+ric is a vector perpendicular


to the plane. Hence the equation o f the plane can be written in the
form (7).
Taking q*=>—d, the equation (7) of the plane may be written
as
a x+ b y+ cz+ d —0. ...( 8)
The equation ( 8) is the general equation of a plane, where
the numbers a, b, c arc the d r.’s of the normal to the plane i.e , a
line perpendicular to the plane and the length of the perpendicular
from origin to the plane is —<//\/(e*+ft*+c*). the number d being
negative.
§ 3. To prove that the general equation o f the first degree in
x, y, z namely ax+ by+ cz+ d ^ 0 represents a plane and that the
coefficients a, b, c o f x, y, z in this equation are d r.’s o f the normal
to this plane.
The general equation of the first degree in x, y, z is given by
ax+ by+ cz+ d—0. (I)
Let A (j»i , y u z,) and B (xu y u za) be any two points on the
surface (I), so that we have
axi+by\+czi+d=*0 ·· ( 2)
and AX2+ by*+ cz2+ d= 0. .(3 )
Multiplying the relation (3) by ·μ’ and addiug to (2), we get
O (Xt + pX-J+b ()’ι+μγ»)+0 (Z1+ μΐ,) \-d ( ’ + μ )—0.
Dividing both the sides by (I + μ), we get
a (xt+μχ*) ;b O'!+/*?*) . c (ζ,+μζ.?) , , Λ
1+0 + " I +,- + 1+7. + , U 0 · ...(4)
The relation (4) shows that for every value of μ ψ - I, the
point lies on the surface (I). But
these are the general co-ordinates of a point which divides the
join of A(xi, yu Zi) and B(x2, y2f za) in the ratio μ : I. Since μ may
take any real value other than —I, every point of the straight line
AB lies on the surface (I). Hence the equation (I) represents a
plane.
Subtracting (2) from (3), we have
a (X t-X i) + b (yt - y ,) + c (Z2- Z 1) = 0. ...(5)
The relation (5) shows that .the two lines whose d.r.’s arc
a, b, c and xt —xi, y-i—y u 2%—Zi are perpendicular. But Xi - X u
The Plane 49

y t - y u Zt - Z l are d.r.’s of the line AB which is any Iinein the


plane (I). Therefore a line whose d.r.’s are 0, b,c is perpendicular
to every line lying in the plane (I) and so it is perpendicular to
the plane (I). Hence a, b, c are d.r.’s of the normal to the
plane (I).
Remark. In the general co-ordinates of a point on the line
AB9 we cannot have /Li=- I because there can be no point on the
line AB which divides it in the ratio —I : I i.e.9 which divides it
externally in the ratio I : I.
Note. The number of arbitrary constants in the general equa­
tion of the plane.
The general equation of the plane is
ax+ by+ cz+ d= 0 or (a/d) x+(b/d) y+(c/d) z= —I.
This equation shows that there are three arbitrary constants
namely a/d, b/d9 c/d in the equation of a plane. Therefore the
equation of a plane can be determined to satisfy the three condi­
tions, each condition giving us the value of a constant.
§4. To reduce the general equation of the plane to the normal
form.
The general equation of the plane is
ax-f by+ cz+ d~ 0. .-(I)
If I9 m9 n are the d.c.'s of the normal to the plane, then the
equation of the plane in the normal form is
lx+ m y+ nz= p. - ( 2)
If (I) and (2) represent the same plane, then
l___m π V(/»-f _____ I_____
a b ~~ c —i/”’-*-y/{a*+bl +c2) V (a l+ ^ * + cl) '
where the same sign either -five or —ive is to be chosen through­
out.
Λ l= ±a!y/(o*+ b*+ c% )9 m = ± 6/ Vfat + ^ + <?*),
n = ± clxl(a t +b*+c*)9 and /> = ± ψ ν/(β ί + ί 1 + ^ ).
Substituting these values in (2), the normal form of the plane
(I) is given by
ax by cz
V i^ + ^ + c*) ± ~ Vi**+**-«-*eJ V(fla-f b*+c*)
d
*= ± ■V iβ· + δ* t c1) ...(3)
The sign in equation (3) is so chosen that p f;e.,
±rf/V (aa + ft*+^a) is always positive.
50 Analytical Geometry 3-D

Workiog role to reduce the equation of a plane in normal form.


Transpose the constant term in the equation of the plane to
the R.H.S. and adjust the equation in such a way that this const­
ant term on the R.H.S. is positive. Now divide the equation by
)9 where a 9 b 9 c are the coefficients of X 9y f z in the
y / i a %+ b %+ c %
equation of the plane. The resulting equation will be the equation
of the plane in the normal form
lx + m y + n z= p .
Here p will be the length of the perpendicular from the origin
to the plane and I9m9 n will be the d.c/s of the normal to the
plane.
§ 5. , Intercepts form :
To find the equation to the ρΐαηέ in terms of the intercepts
a9 b9 c which the plane cuts on the coordinates axes. (Meerut 1985)
Let the general equation of the plane be
A x+ B y+ C z+ D = 0. ...(I)
Since the plane (I) makes an intercept a on the x-axis, the
point (a, Ov 0) lies on the plane (I)v so that we have
Aa+ D = 0 i e.9 A = - D fa .
Similarly the points (Ot b 9 0) and (0, 0, c) lie on the plane (I),
so that
Bb+D = 0 and C e+ D=O9 giving B= - D fb 9 C = - Dfc.
Putting the values of A9 B9 C in (I)v we get
( —Dfa) x + i —D/b) y + (-D /c ) z+ D = 0
or xla+ yfb+ zfc= I. ..( 2)
The equation (2) is the required equation of the plane in
terms of the intercepts a9b and c made by the plane On the axes
of xvy and z respectively.
§ 6 . Plane through a given point and perpendicular to a given line.
To find the equation of a plane through a given point A(xl9 y X9 zx)
and perpendicular to a line whose direction ratios are a9 b9 c.
Let (xv y 9 z) be the coordinates of any current point P on the
plane. Since the plane passes through the point A (Xi, y X9 z x)9 the
line AP lies in the plane.
The d.r.’s of the line AP are x —x l9 y —yXf z—zx. Also the
d.r.’s of the normal to the plane Le.9 of a line perpendicular to
the plane are a9b9 c.
Now the normal to the plane is perpendicular to every line
lying in the plane and therefore the lines (whose d r.’s are a9 b9 c
and x —X\9y —yu z—zx are perpendicular.
51

Λ β (χ—χ ,)+ 6 [ y -y t)+ c (X-Zi)=O,


which is the equation of the required plane.
Remark. The equation of any plane passing through the
point (X1, y u Z1) is
a (x—x,)+ * ( y —y t)+c ( z - z t)=0.
In this equation a, *, c are d.r.’s of normal to the plane.
As a particular case, the equation of any plane passing
through the origin is ax+ by+ cz= 0,
in which the coefficients of x, y, z i.e., a, *, c are d.r.’s of the nor·
mal to the plane.
§ 7. Equation of a plane through three points.
To find the equation o f a plane which passes through three points
whose co-ordinates are (Xli yu zt), (xt, y%, zt) and (x,. y%, z»).
(Kanpur 1983)
Let the general equation of the plane be
' ax+ *y+ ez+ d = 0. -(1 )
If the equation ( I) of the plane passes through the given
points (X1, y lt zt), (xt, y t, zt) and (x«, y%, z»), the coordinates of
these points will satisfy the equation (I), so that we have
OX1+ * ^ + C Z 1 + */=!), —(2)
aXt+byt+czt+d= 0, ...(3)
and ex*+ *y,+ cz,+ d= 0. ...(4)
Eliminating a, b, c and d from the above equation (1),(2),
(3) and (4), the equation of the required plane is given by
X y Z I

Xi y\ Zl I

Xt y* Zi I

Xi y* Zl I
Cor. Conditionforfour points (xlt y u Z1), (x„ y t , zt), (x*. y», z*)
and (x<t ylt Z4) to be coplanar.
The equation of the plane passing through first three points
is given by equation (5). If the fourth point namely (x4. yt, z4)
also lies on this plane, then the co-ordinates of this point will
satisfy the equation (5), so that we have
Xt y* Zt I Xi y\ Z1 I

Xi y\ Zl I Xi y* Zi I
=0, l.e..
Xt y% Zl I X% y* Z9 I

Xi yi Zl I Xt y« Zt I
- ( 6)
52 Analytical Geometry 3-D

The condition (6) is the required condition for four given


points to be coplanar.
Note. Forsolvingnumerical cxamplesan easier method can
however be followed as explained in Ex. 7 and Ex. 8 below.
Solved Examples 3(A)
Ex. I. Reduce the equation of the plane x+ 2 y—2z—9 = 0 to
the normal form and hence find the length o f the perpendicular
drawn form the origin to the given plane.
Sol. The equation of the given plane is
x + 2y—2z - 9 - 0 .
Bringing the constant term to the R.H.S., the equation
becomes x + 2 y —2z=*9. (I)
[Note that in the equation (I) the constant term 9 is positive.
If it were negative, we would have changed the sign throughout
to moke it positive.]
Now the square root of the sum of the squares of the coeffi­
cients of x 9y 9z in ( 1 ) = ^ ( ( 1 ^ + ( 2 ^ + ( - 2 ) ^ - ^ = 3 .
Dividing both sides of (I) by 3, we have
I 2 2
T x + 7 y ~ T z=3, .»(2)
The equation (2) of the plane is in the normal form
/x+my+nz=/*.
12 2
Hence the d c *s I9 m9n of the normal to the plane are y “
and the length p of the perpendicular from the origin to the plane
is 3.
Ex. 2. The co-ordinates o f a point A are (2, 3, —5). Determine
the equation to the plane through A at right angles to the line OA9
where O is the origin. (Meerut I986S)
Sol. Here the plane passes through the point A (2, 39 —5)
and is perpendicular to the line OA i e , the line OA is normal to
the plane. *
The d.r?’s of OA are 2—0, 3—0, —5—0 i.e., 2, 3, —5.
Thus the plane passes through the point (2, 3, —5) and 2, 3,
—5 are d.r.’s of normal to the plane. Therefore the equation of
the plane is
2(x—2)+3(jp—3 ) - 5(z—( - 5 ) ) = 0 [Refer § (6 )]
or 2 (x -2 )+ 3 O > -3 )-5 (z + 5 )= 0
The Plane 33

or 2x+ 3 y —5z—38=»0
or 2 r+ 3 p —5z=38.
Ex. 3. O fstheoriginand A is the point (a9 b9 c). Pindtbe
i.c's of the join OA and deduce the equation o f the plane through A
at right angles to OA.
Sol. The co-ordinates of the points O and A are (O9 O9 0) and
(o9b9 c) respectively. Thus the direction ratios of the join OA are
a—O9 b —O9 c —0 i.e.9a, b9 c.
Hence the d.c.’s of the join OA are
alV (a'+ b'+ c*)9 b/V<d*+b*+c«), dV (a*+ P+ c*).
Now we are to find the equation of the plane passing through
the point A (a, b9 c) and perpendicular to the line OA. Here the
line OA is normal to the plane and its d.r.’s are at b9 c. Therefore
the equation of the required plane is
a(x—a)+ b(y—b)+ c(z—c}= 0. or ax+by f cz=a%+b%+<*.
Ex. 4. Find the equation of the plane perpendicular to the line
segment from (—3, 3, 2) to (9, 5, 4) at the middle point o f the
segment.
Sol. The end points of the given line segment are ( —39 3, 2)
and (9t 5t 4). The d.r.’s of the line segment are 9—(—3), 5—3,
4 -2 i.c., 12, 2, 2. The co-ordinates of the middle point of the
line segment are ($(9-3), $(5 + 3), $(4 + 2)) i.e.9 (3, 4, 3).
Thus the plane is to pass through the point (3, 4, 3) and d.r.’s
of normal to the plane are 12, 2, 2. Therefore the required equa­
tion of the plane is
12 (jc—3 ) + 2 ( ^ - 4)+2fz—3)= 0
or 6(x —3 )+ 0 > -4 )+ (z —3)=0, or 6x+.y+z=25.
Ex. 5. Find the intercepts made on the co-ordinate axes by the
plane x —3y+2z=9.
Sol. The equation of the given plane is
*-3>>+2z=9. ...ti)
Dividing both sides;by 9· the equation (I) may be written as
x_ Z - z »1 .
o_ + -3 + ( 9 /2 ) ' ..(2)
Comparing the equation (2) with the equation
x/a+y/b+z/c=-I 9
he intercepts on the co-ordinate axes are given by
a= th e intercept on the x-axis=9
b = the intercept on the j-axis®»—3
nd c—the intercept on the z-axis·*9/2.
54 Analytical Geometry 3-D
Ex. 6 . A plane meets the eo-ordinate axes in A, B. C such that
the centroid o f the triangle ABC is the point (p, q, r)i show that the
equation o f the plane is x/p+ ylq+ z/r= 3 .
(Ka spar 1983: Mecrat 85S}
Sol. Let the equation of the plane be
x/a+ylb-\-z/c= l. —(I)
The plane (I) meets the x-axis in the point A, so putting y = 0
and z = 0 in (I) we get x= a. Thustbe co-ordinates of the point
A are (a, 0, 0). Similarly the plane ( I) meets y and z axes in the
points B and C whose co-ordinates are given by B (0, b, 0) and
C (0, 0, c).
Thus the co-ordinates of the centroid of the triangle ABC are
given by (J(e+ 0 + 0), J(0 + i + 0), |( 0 + 0 +c)) i.e., (Jo. Jb, Jc).
But it is given that the co-ordinates of the centroid of the
triangle ABC are (p, q, r), so that we have
P = Jo, q= {b, r = Jc or o=3p, b=3q, c=3r.
Substituting these values of o, b, c in the equation ( I ), the
equation of the required plane is given by
xH3p)+yH3q)+zl(3r)=‘l or x/p+ylq+z/r = $·
Ex. 7. Find the equation to the plane through the three points
(0. - I. - I ) , (4, 5, I) and (3, 9. 4).
Sol. The equation of any plane passing through the point
(0, —I. —I) is given by
o(x—0) + H p - ( - 1)}+C{ z - (—I )}=0
or ox+*(^ + l)+ c(z-|-l)= 0 . ...(I)
If the plane (I) passes through the point (4, 5, I), we have
4a+ 6b+ 2c= 0. ...( 2)
If the plane (I) passes through the point (3, 9, 4), we have
3o+10b+5c=0. ...(3)
Now solving the equations (2) and (3), we have
o ___ b c _ 4
3 0 -2 0 “ 6^ -20 “ 4 0 - I 8 =A I8ay*
Λ o=10A, b = —14A, c=22A.
Putting these values of a ,b ,c in (I), the equation of the
required plane is given by
A[10x—l4(_y+l)+22(z-!-l)]=0
or 10x-14O> + l)+ 2 2 < z+ l)= 0
or 5x—7 y + llz + 4 = 0 .
Ex. 8. Show that the four points (0, —1, —1), (4, 5, I), (3,
9 ,4) and (—4, 4, 4) are coplanar. (Meerut 1982)
The Plane 55

Sol. Proceediog as in Ex. 7 above the equation of the plane


passing through the three points (0, —1, -1 ),(4 , 5, I) and ( 3, 9,4)
is given by 5 ;c -7 .y + llz + 4 = 0 . ...(I)
If the fourth point ( —4, 4, 4) also lies on the plane (I) then
the co-ordinates of this point must satisfy the equation (I).
Putting Jt= —4, y = 4, z= 4, the L.H.S. of (I)
= 5 .( - 4 ) - 7 .4 + 11.4+ 4 = 0 = the R.H.S. of (I).
Hence the equation (I) is satisfied by the point (—4, 4, 4).
Therefore the given four points are coplanar.
§ 8 . Equations of the co-ordinate planes.
(i) The equation to yz-plane. The ^-coordinate of each point
lying on the .yz-plane is zero, and hence the equation to .yz-plane
is given by Jt=O.
(ii) The equation to zx-plane. It is gixen by y ^ 0.
(iii) The equation to xy-plane. It is given by z= 0 .
§ 9. (A) The equations to the planes parallel to the co-ordinate
planes.
The equation of the plane parallel to the yz plane and at a dls-
tauce ‘a* from it. The jc-coordinate of every point on this plane
is equal to *a\ Hence the equation of the required plane is given
by jc=a.
Similarly, the equation o f the plane parallel to the xz-plane and
at a distance ‘6’ from it is given by y ^ b .
Also the equation of the plane parallel to the jcy-plane and at
a distance *c* from it is given by z=c.
(B) The equation of the planes perpendicular to the co-ordinate
axes.
The equation of the plane perpendicular to the x-axis. This
plane is obviously parallel to the .yz-plane and hence its equation
is given by jc=a. [See § 9(A) above].
Similarly the equations of the planes perpendicular to y and
z axis are respectively given by y=*b and z=c.
§ 10. Angle between two planes.
Definition. The angle between two planes is defined as the angle
between their normols drawn from any point to the planes.
Let the equations to the two planes be
aiX + hy+ C iZ+ d^O . ...(I)
and + + ^ ie 0. ...( 2)
56 Analytical Geometry 3-D

The d r.’s of the normal to the plane (I) are au bu C1 and the
d.r.’s of the normal to the plane ( 2) are ut, b2t c%.
~ If 0 is the angle between the planes (I) and (2), then 0 is the
angle between the lines whose d r.’s are ult bit Ci and a%, b2fc2·
. _ QlQ t+M a+ CiCt________
V (ei* + V + Ci*> V (ei* + V + Cie) -(3)
For the acute angle between the two planes, cos 0 is positive
and for the obtuse angle it is negative. The numerical value of cos 0
in both these cases is the same because cos (π—0; = —cos 0.
Condition of perpendicularity of two planes.
Two planes are perpendicular if their normals are perpendi­
cular. Therefore the planes (I) and (2) are perpendicular if the
lines whose d.r.’s are U1, A1, C1 and u8, bit c8 are perpendicular the
condition for Which is
Uiat + btb2H- C1C2« 0. · · -I*)
Condition of parallelism of two planes.
Two planes are parallel if their normals are parallel. There­
fore the planes (I) and ( 2 ) are parallel if the lines whose d.r.’s are
u,, hi, Ci and a%9 ha, c2 are parallel the condition for which is
ClIci ..<(5)
l.e., the coefficients of x, y, z in the equations of the two-planes
should be proportional.
Remember. The equation of any plane parallel to the plane
ax+ by+ cz+ d^Q
is ax+ b y+ cz+ X ^ 0.
$ 11. The two sides of a plane.
Two points P (xlt y u Z1) and Q (x2, yt , zt) lie on the same or
opposite sides of the plane ax + by+ cz+ d—U according as
axi+ byi+ czi+ d and axt -{-byt +czt + d
are o f the same or opposite signs.
T h eeq u atio o o fth ep lan eis ax+by+cz+d=Q .,.(I)
Suppose the line PQ meets the given plane (I) at the point R
such that PR : RQ = Zzi1 : mt. Then the co-ordinates of the point
R afe Zm1Xt + mtXx m,yt + mty, Qt1Zfhm tZ1 V
\ mx+mt ' AZi1 + mt ’ /Ziii-AZit /
Since the point R lies on the plane (I), therefore
Q ZmlXt +ZZttXi \ - b / z/ι, Vt +ZZit y,\ c / zziizt + ZZitZi \ .
\ /zii+zzit Z*1" \ zzii r zzit I zzti + /zit . / +
OV ml(axi +byt + czt +d) + mt(axx+ byx+czt +d)----0
The Plane 57

/Mt α χ ,+byy+czi+d
m% Qxi ^ b y i + cz*-t-d ...( 2)
Now the ratio Otxjmi is positive or negative according as PQ
is divided at R internally or externally Le., the points P, Q lie on
the opposite or same side of the plane (I).
Hence from (2) if axx+ by\+czx+d and axt+ byt+ czt+ d are
of the same sign, then r»i/ma is negative Le., the points P and Q
Iieon th e sam esid eo f the plane (I) If αχχ+byx+CZl -^d and
axt+ byt+ czt+ d are of the opposite signs, milmt is positive Le.,
the.points P and Q lie on the opposite sides of the plane (I).
§ 12. To find' the length of the perpendicular from the point
(Xi, yi, Zr) to a given plane.
Let the equation of the given plane be
a x+ b y+ cz= d = 0. ...(I)
To find the length of the perpendicular from the point
(*i, yi, zi) to the plane (I).
Shifting the origin to the point (JC1, y x, Z1), the equation (I)
becomes a (x+xjJ + fi (y+ yi) + c (z + r,)+ d = 0
or ax+by+cz+axx+byx+czx+d=!). . . ( 2)
Dividing both sides of ( 2) by y/(a*+b*+c*), we get
__ a . b c
V(o*+b*+c2) X+ y / i ^ +b' +c *) y + 2
, Ox1^ b y l +Cz1+ d
V l* + * * + * 1) " * ...(3)
The equation (3) of the plane is in the normal form with a
proper adjustment of sign throughout the equation.
The length of the perpendicular from the new origin to

Hence the length of the perpendicular from the point


QXi+byx+czx+d
(Xi,yu z,) to the plane (I) ■±- y/(a*+b*+cx)
Since the perpendicular distance of a point from the plane is
always positive, therefore a positive or negative sign is to be
attached before the radical according as axi+byx-$-czx+d
is positive or negative Le., according as (x,, y lf z,) lies on the
same side or On opposite side of the plane as the origin, provided
d is positive.
Working rule. To find the length p of the perpendicular
58 Analytical Geometry 3-D

from the Point(JcliiVl f Zi) to the plane ax+by+cz+d=*09 we


substitute the co-ordinates of the given point in the left hand side
of the equation of the plane and then divide this expression by
\/[(coeff. of *)2+(coeff of ^)a+ (coeflf. of z)2].
Thus » x ,+ b y ,+ c z ,+ d

If the value ofp obtained from this formula is negative, we


can ignore the sign and give the positive value in^ answer, unless
there is some special reason.
If the equation of the plane is in the normal form
/jc+m y+uz—p = 0, the length px of the perpendicular from the
point (jclf yl9 Z1) to the plane is given by
P\ = lx\ + my\ + nzi—p, for in this case VO 2+ m%+ n a) cssI*
§ 13. To find the distance between two parallel planes.
Find the lengths of perpendicular distances of each plane
from the origin and retain their signs. The algebraic difference
of these two perpendicular distances is the distance between the
given parallel planes. But while applying this meihod we should
be careful that the coefficients of x in the two equations of the
planes are of the same sign.
Alternative method. Take a point on one of the two given
planes, then the required distance is the length of the perpendicu­
lar drawn from this point to the other plane.
SOLVED EXAMPLES 3 (B)
Ex. I. Write the equations of the planes in the following cases :
(i) parallel to the xy-plane and 5 units below I/,
(Ii) parallel to the yz plane and having x intercept 3,
(ii/) perpendicular to the z-axis at the point (0, 0, 4),
and (iv). parallel to the zx-plane and 6 units behind it.
Sol. In view of § 8, the equations of the planes in the diffe­
rent cases are as given below ;
(i) z= - 5 , (H) * = 3 , (Hi) z= 4,
and (iv) p·» —6, or p + 6 = 0.
Ex. 2. Fmd the equation o f the plane which is horizontal and
passes through the point ( 1 , - 2, —5).
Sol. Let us choose the co-ordinate axes so that the axes
of JCand y lie in a horizontal plane (the plane of the paper, say)
and the axis of z is perpendicular to this plane. Hence the
required plane is perpendicular to the z-axis and passes through
The Plane 59

the point (I, —2. —5), therefore, the equation of the required
plane is given by z = —5 or z+ 5 = 0 .
Remark. If the axes of x and z be taken in a horizontal plane
(the plane of the paper, say) then .y-axis is perpendicular to this
plane and hence the equation of the required horizontal plane will
be y= —2 .
Ex. 3. Find the equation of the plane through the point
(—I, 2, 4) and parallel to the plane 2x+3.y—5 z+ 6 = 0 .
Sol. The equation of the given plane is
2jc+3.y—5z+ 6= 0. --(I)
Since the equations of the parallel planes differ only in the
constant term, therefore the equation of any plane parallel to the
plane (I) is given by
2x+3y—5z+ J r= O . --(2)
If the plane (2) passes through the point (—1,2, 4), we have
2 ( - 1 ) + 3 (2 )-5 (4)+fc=0 or * = 16.
Substituting this value of k in the equation (2), the equation
of the required plane is 2x + 3.y—5z+16=0.
Ex. 4. Find the equations of the planes paraliel to the plane
3x—fry—2z 4 = 0 at a distance 3 from the origin.
Sol The equation of any plane parallel to the plane
3x—6y—2z—4 = 0 is 3 x -6 y —2z+fc=0. ---(I)
Let p be the length of the perpendicular from the origin to
the plane (I). Then
[See § 12]
^ ^ { ( 3 / + ( - 6 / + ( - 2/}
or p— ±*/7.
But according to the given condition p is 3. Hence
*/7 = ± 3 => * = ± 21 .
Patting the values of k in (I), the equations of the required
planes are given by 3x - 6jc- 2z ± .11 =0.
Ex. 5. Find the equation of the plane parallel to the plane
2 x -3 y —52+ 1= 0 and distant 5 units from the point ( —1, 3, I).
Sel. The equation of any plane parallel to the plane
2x—3 y^5z + 1 =-0 is 2 x—2 y — 5z+ * = 0 . ...(I)
L et^ b e the length of the perpendicular from the point
(—1» 3, I) to the plane (I), then [see § 12]
__ , 2 ( - 1 ) - 3 ( 3 ) - 5 ( 1 ) + * , -1 6 + * 7
ic VU 2I H t - 3 / + ( - 5 / ) β ± ν(3»)
60 Analytical Geometry 3-D

But according to the question, q=5.


Λ 5 = ±(-l6+Jfe)/V (38) or Jfc=16±5V(38).
Substituting the values of k in succession in (I), the equations
of the required planes are given by
2* - 3y - 52+ 16 ± 5V( 38)= 0.
Ex. 6 . Find the equation o f the plane through the point (α, β, y)
and parallel to the plane ax+ by+ cz—0.
Sol. ' The equation of any plane parallel to the plane
ax+ by+ cz= 0 is ax+ by+ cz+ k= 0. —(I)
If(I) passes through the point (α, β, γ), we have
a*+ bβ+ cγ+ k= 0
or k = —aa—bfi—cy.
Putting the value of k in the equation (I), the equation of the
required plane is \
ax+ by+ cz—act—bfi—cy = 0
or ax+by+cz=a»+bfi+cy.
Ex. 7. Find the equation' o f the plane through (I, 0, —2) and
perpendicular to each of the planes
2 x + y —z —2=0 and x —y —z—3=0.
(Gorakhpur 1978; Meerut 85S)
Sol. The equation of any plane through the point .(1 , 0, —2)
is a (jc—1)+ 6 ( y - 0)+ c (z + 2 ) = 0. .. (I)
Ifthe plane (I) is perpendicular to the planes2 x+ y—z —2 = 0
and J t - y —z —3=0, we have (See § IOJ
a (2) + 6 ( l ) + c ( - 1 ) = 0 /.*., 2a + 6 - c = 0, ...( 2)
and a (1)+ 6 (—l ) + c ( —1)=0 /:*., a 6 —c—0. ...(3)
Adding the equations (2) and (3), we have c= fa.
Subtracting (3) from (2), we have b = —\a.
Putting the values of b and c in (I), the equation of the
required plane is given by
a ( x - l ) - J a y + fa ( z + 2)= 0
or 2x—2—y + 3 r+ 6 = 0 , or 2x—y + 3 z + 4 = 0 .
Ex. 8. Find the equation o f the plane through the points
(I, —2, 2 ), (—3, I, —2) and perpendicular to the plane
x+ 2 y—3z=5. (Gorakhpur 1980)
Sol. Theequationofanyplanepassing through the point
(I, - 2, 2) is
a(x-l)+ b(y+ 2)+ c(z-2)= 0. . . (I)
If the plane (I) passes through the point ( —3, I, —2), we have
The Plane 61

a ( - 3 - 1 ) + 6 ( l+ 2 ) + c f - 2 - 2 ) = 0
or —4e+ 3 b—4c= 0, or 4 a -3 6 + 4 c = 0 . .:.(2)
If the plane (I) is perpendicular to the plane x + 2 y —S r=5,
we have a. I + b .2 + c .( - 3)=0, or a + 2 b -3 c = 0 . ...(3)
Solving the equations (2) and (3) for a, b, c, we have
a b c
( - 3 ) ( - 3 ) - 2 .(4) “ 1 x 4 —4 .( - 3 ) = 4 x 2 - 1 .(- 3 )
or y -= jg —-^-=A, (say)· .'· α=λ, 6 = 16A, C=IlA.
Potting the values of a ,b ,c in (I), the equation of the
required plane is
λ [ ( * - 1) + 16 0-+2)+ 11 (2-2)1 = 0,
or x+I6>’+ l l r + 9 = 0 .
Ex. 9. Find the. angle between the planes 2x - y + z = 6
and x + y + 2 z= 7 .
SoK The d,r.’s of the normal to the plane 2 x—y+z**6
are 2, —1, I.
The d r.’s of the normal fo the plane x \ y - \- 2 z —l are I, 1,2.
Now the angle between two planes is equal to the angle
between their normals. Therefore if Θ be the angle between the
given planes, we have
cos β = - 2· + ( - 1 ) .1 + 1.2__________
λ/{(2~)*+'(- i J H (D i)VU υ ’+ιυ'-Η -ο*}
= 3 /6 = J . Λ O=J tt.
Εχ. 10. Find whether the two points (2, 0, I) and (3, —3, 4)
lie on the same side or opposite sides o/ the plane x - 2^+2=6.
Sol. Talcing all terms to the left hand side, the equation of
the plane may be written as x —2,y+z —6 = 0 ..(I)
Substituting the co-ordinates of the point (2, 0, 1 \ the value
of the left band side of the equation (I) of the plane
=-2—0 + 1 - 6 = - 3 .
Again substituting the co-ordinates of the point (3, —3, 4),
the value of the left hand side of the equation (I) of the plane
= 3 + 6 + 4 -6 = 7 .
Since the values —3 and 7 are of opposite signs, the given
two points lie on the opposite sides of the given plane. [See § 11]
.
Ex. 11 Findthe equation of the locus o f a point P whose
distance from theplane bx —2y+ 3z+ 4 = 0 is equal to its distance
from the point ( —1 ,1 ,2 ).
62 Analytical Geometry 3-D

Sol. Let the co-ordinates of a point P be (xlt * , *)· It is


required to find the locus of the point P.
Let p be the perpendicular distance of the point P (xlf yi, Zi)
from the plane 6x—2 )'+ 3 z+ 4 = 0 .
6x, —2* + 3z,+4 6 x ,- 2 * + 3 z ,+ 4
TheD P= ν ΐ 3 ο + 4 + 9 Γ “ --------7----------- -(I)
Again let d be the distance of the point P (X1, * , * ) from
the point (—1, I, 2).
Then d= V{tx,+ IJt - H * - IJt -K * - 2)·}. -(2 )
According to the given condition, we have p = d t.e., p*=d*
i.e. (6x,—2 * + 3zi+ 4)*/49=(x, + 1)*+(*—I »*+(*—2)·
or 36xt*+ 4**-I-Pz1* + 16—24xx *+36ζιΧ1+48χ1—12*Z|
—16*+24zt =49 Jx1*+ 2xx+ 1+ * * —2 * + 1+ Zi*—4z^ + 4}
or 13xt*+ 4 5**+ 40zi*+ 24xty t —36Z|Xx+ 12y,zl -I-50xi—82*
—220Z!+278=0.
Hence the locus of the point P (xlt * , Z1) is
13x*+45)>, +40z, +24x)'—36zx+12yz+· 50x—S2y—220z+278=0.
Ex l 2. Findtheperpendicular distance between the parallel
planes 2x—3y—6z—21 = 0 and 2 x—3y—6 z + l4 - 0 .
Sol. The equations of the given planes are
2x—3j>—6z—21=0, .. (I) and 2 x - 3 ^ - 6 z + l4 = 0 . ...(2)
[Note that the coefficients of x in the equation (I) ond 12)
are of the same sign).
Let P1 and pt be the lengths of the perpendicular distances of
the planes (I) and (2) from the origin. Then we have
2 . 0 - 3 . 0 - 6 .0 - 2 1 -2 1
px^ V (4 + 9 + 36) - 7 - 3»
2 . 0 - 3 .0 - 6 .0 + 1 4 14 ,
and
Pt~ VC4+9+36) “ 7 ~ 2'
Hence the perpendicular distance between the given planes
- Λ - Λ - 2 -(-3 )= 5 .
Alternative method. The co-ordinates of a point on the first
plane 2x—3 y - 6 z —21 = 0 are (0, —7, 0).
The perpendicular distance between the given planes
= the perpendicular distance of the point ( 0 , - 7 , 0) from
the second plane 2x—3y—6z+ 1 4 = 0
0 + 2 1 - 0 + 14 35 , .
The Plane 63
Ex. 13. Find the locus o f a point, the sum o f the squares o f
whose distances from the planes
x + y + z= 0 , x —_y=0, x+ y~-2z= 0 is 7.
Sol. Let P(x i, yi, Zj) be a point whose locus is required.
Let P1=Sthe distance of P from the plane x + y + z = 0
= (*i +.V1+Z1)/ν ' (3),
P1S^the distance of P from the plane At-^=O
~ ( x i- y i) I V ( V ,
hnd P3= the distance of P from the plane x + y —2z=*0
=»(*ι+Λ ~ 2zi)IV(6).
Now according to the given condition, we have
A*+P**+P»‘ = 7 . ...(I)
Substituting the values of pi, p*, p3 in (I), we get
i (■*■) +^i + «i)*+J f X i- P i^ + i (ΛΓι+Λ'ι—2zj)*=-7
or 2 ( V + V + Z 1*+ 2 ^ + 2 ^ , + 2 ^ , 2 , ) + 3 ( X i* - y * - 2 x 1yi)
+ (-«ι’ +^ι*+ 4z,*+ 2x,p,—4z,X,—4p,z,)—42
or 6Ar,, 4-6y,, + 6z,*=42, or ACii -Ky1*-|-z,*—7.
the required locus of P (aT„ yu z,) is jc2+>>, + z, = 7 .
§ 14. A plane through the intersection of two given planes.
Tofind the equation of any plane through the line of intersection
o f the two given planes.
Let the equations of the two given planes be
P s a ix + biy + ciz+ dl = 0, ...(I)
and Q ^ a i X+ b iy+ CiZ-^di =O. ...(2)
Then the equation P+X Q = 0, i.e., the equation
(fl,x+h,p+CiZ+di)+A (atx+ bty+ctZ+dt)= 0 ...(3)
is the required equation of any plane through the line of inter·
section of the planes ( I) and (2).
F irstw eobservethat the equation (I) is of first degree in
x, y and z and so it represents a plane. Again whatever Λ may be
if a point satisfies both the equations (I) and (2), it definitely
satisfies the equation (3). Thus all the points of the line of inter­
section of the planes (I) and (2) also lie on the plane (3). Hence
(3) is the equation of any plane passing through the line of inter­
section of the planes (I) and (2).
§ 15. To find the condition that a line whose d.r.'s are I, m, n
may be parallel or be perpendicular to a given plane.
Let the equation of the given plane be
ax+ by+ cz+ d= 0. —(1)
Thus the d.r.’s of the normal to the plane (I) are a, b, c. The
d.r.’s of the given line are I, m, n.
64 Analytical Geometry 3-Z)

Tbe line is parallel to the plane. If the given line is parallel


to the line (I), it is perpendicular to the normal to the plane (I ',
the condition for which is
al + bm + c n - 0. ...(2)
The Iioe is perpendicular to the plane. If the given line is
perpendicular to the plane (I), it is parallel to the normal to the
plane (I), the condition for which is a/l= b /m -c/n
§ 16. The angle between a line and a plane.
Definition. The angle between a line and a plane is defined to
be the complement of the angle between the line and the normal to
the plane.
Clearly this angle can be determined by the methods explained
earlier.
Solved Examples 3(C)
Ex. I. Find the equation o f the plane passing through the line
o f intersection of the planes 2x—7y+4z=3, 3x —5 y + 4 z + ll = 0,
and the point (-*2, I, 3),
Sol. The equation of anv plane through the iine of inter­
section of the given planes is [See § 14]
(2 x -7 y + 4 z -3 )+ A (3 x -5 y + 4 z + 11)=0. . (1)
Ifth e p la n e (I) passes through the point ( —2 ,1 ,3 ), then
substituting the co-ordinates of this point in the equation (I),
we have
{2 ( - 2 ) - 7 ( 1 ) + 4(3)-3}+ * {3 ( - 2 ) - 5 (1 )+ 4 (3 )+ 1 )= 0
or ( —2) +A (12)=0, or A= l/6.
Putting this value of A in (I), the equation of the required
plane is (2x—7;»+4z—3) + (1/6) (3x—5 y + 4 z + 11)=0
or 15x —47>>+ 28z=7. '
Ex. 2. Find the equation of the plane through the line o f inter-
section o f the planes x + 2y —3z—6 = 0 and 4x+ 3y—2 z + 2 = 0 and
passing through the origin.
Sol. The equation of any plane through the line of inter­
section of the given planes is
(x+2>’—3z—6)+A (4x+3>> —2z+2) = 0. ...(I)
If the plane ( I) passes through the origin i.e.9 through the
point (Ot 0, 0), then we have
(0+0 —0—6)+A (0 + 0 - 0 + 2 ) = 0 , or - 6 + 2A=0, or A=3.
Substituting this value of A in the equation (I), the* equation
of the required plane is
(* + 2jj—3 z -6 ) + 3 (4x + 3y—2 z+ 2)=0,
or Ity ^ r = O . :
The Plane 65

Ex. 3. Find the equation of the plane through the line o f inter-
section o f the planes x + 2 y + 3 z ~ 4 = 0 and 2 .v + j~ z + 5 = 0 and
perpendicular to the plane 5 x + 3 y + 6 z+ 8 = 0 .
(Lockflow 1982, Meerat 84S)
Sol. Theequation of any plane through the line of inter­
section of the planes x + 2 y + 3z—4 = 0 and 2x+ y —z + 5 = 0 is
< x+ 2y + 3 z--4)+2 (2 x + v —z+ 5 )--0
or x (1+22) + v (2 + 2 )+ z ( 3 - - 2 ) - ( 4 - 5 2 )- 0 . ..( I )
If the plane (T) is perpendicular to the plane 5jc-f- 3_v+ 6z-f 8=0,
we have (1 + 2 2 ).5 + (2 + 2 ).3 + (3 -2 ).6 = 0
or 5 + 1 0 2 + 6 + 3 2 + 1 8 -6 2 = 0 , or 72 + 29 = 0, or 2 = 29/7.
Substituting the value of 2 in (I), the equation of the required
plane is
■* (I - 58/7)++(2 -29/7)+z (3+ 29/7) - (4+ 145/7)=0
or —51x - 15y + 5 0 z - 173=0,
or 5lx+15y--50z+173=0.
Ex. 4. Find the equation of the plane through the line o f inter­
section o f the planes a x+ by+ cz+ d= 0 and ax+ β γ + y z + ^ —O and
perpendicular to the xy-plane.
Sol. The equation of any plane through the line of inter­
section of the planes a x+ by+ cz+ d= 0 and ax + fty + y z + 8 = 0 is
(ax+ by+ cz+ d)+ \ (ax+ /Jy+ yz+ 8)= 0 · ...(.I)
or x (a + a 2 )fy (6 +/? 2)+ z (c+y2)+(if+82)=0. - (2)
Now the equation of the .rj'-plane is given by z = 0
or 0 .x + 0 .y + l.r- 0 ; -.(3)
If the planes (2) and (3) are perpendicular to each ■other,
we have
0.(a -f- a2) + 0.(6+ β\) + I / c -f y2) = 0, or 2^ -cl·/.
Putting this value of 2 in the equation ( ! ), the equation of the
required plane is
(ax+ by+ cz+ d )-(c!y) («x+ ftv+ yz+ S )= 0
or γ (ax+ by+ cz+ d)—c («x+ i3>>+yz+8)=0
or (ay—ra) x + (6 y —<*0) y+ (cy cy) z+ (dy—cb)—0
or far—cot) x+ (by—cfi) y+ (d y cS)=0.
Ex. 5. Find the equation o f the plane through the line o f inter­
section o f the planes ax+by+cz + d —0 and <χχ+βν+γζ+8—0 and
parallel'to x-axis.
Sol. Theequation of any plane through the line of inter­
section of the given planes is
66 Analytical Geometry 3-D

ax+ by+ cz+d+ X (ax+/?;y+yz+8)=0 ...(I)


o r" x (a+ort)+;y (6+j?A)+z(c+yA)+(</+SA)=0. ...(2)
Nowthe d.c.’s of the x-axis are 1,0,0 and the d.r.’s of the
normal·to the plane (2) are α+αλ, 6+0A,r+yA. The plane (2) is
parallel to the x-axis if the normal to the plane (2) is perpendicu -
Iar to the x-axis, the condition for which is.
I .(α+ αλ+Ο.(6+0λ)+ 0.(c+ yA)= 0,
giving λ = —a/a.
Putting this value of λ in the equation (I), the required equa­
tion of the plane is given by
a(ax+ by+ cz+ d)—a (ax+0A +yz+8)=O
or (ba—αβ) y+(cx —ay) z+ (d a —αδ)=0.
Ex. 6. Find the equation o f the plane through the point
(I, —2, 0) and normal to the line joining the points (2, 3 ,-2 ) and
( 1 ,- 2 , 4 ) .
Sol. The d.r.’s of the line joining the points (2, 3, —2)
and (I, —2, 4) are I —2, —2 —3, 4 - ( - 2 ) Le., —I, —5, ". These
are the d.r.’s of the normal'to the plane. Sincetheplane isto
pass through the point (I, —2, 0), its equation is
—l(x - 1 ) - 5 (^ + 2 )+ 6 ( z -0 )= 0 ,
or —x —5λ+6ζ—9 = 0 or x+5;y—6 z+ 9 = 0 ,
or x+5.y—6 z = —9.
Ex. 7. Find the equation o f the plane through the points
(I. —2, 4) and (3, —4, 5) andparallal to the x-axis (i.e. perpedi-
cular to the yz-pldne).
Sol. The equation of any plane through the point (I, - 2 , 4)
is a(x—l)+ 6 (j> + 2 )+ 2 )+ c(z—4)=0. ...(I)
If the plane (I) also passes through the point (3, —4, 5),
we have
a (3 -1 )+ 6 ( —4 + 2 )+ c ( 5 -4 )= 0 ,
or 2ai-2b+ c= 0. --(2)
Now the plane (I) is to be parallel to the x-axis i.e. perpen­
dicular to the xy-plane whose equation is
x = 0 i.e., l.x + 0 .v + 0 .z= 0 .
Hence we have
a.l + 6.0+ c.0 = 0 , or a= 0 . - -(3)
Putting the value o f a from (3) in (2), we get c=2b.
Substituting the values of a and c in the equation (I), the
equation of the required palane is
0 + 6 (y + 2)+26 (z—4)=0, or j;+ 2 z - 6 = 0 .
The Plane 67

Some Important Solved Ezamp Ies


Ex. I. A variable plane at a constant distance p from the origin
meets the axes in A, B and C. Through A, Bt C planes are drawn
parallel to the co-ordinate planes. Show that the locus o f their point
of intersection Is x~2-J-y*+ z“*= p~*. (Meerut 1984 P)
Sol. Let the equation of the variable plane be
xlo+ylb+zlc**\, ···(»)
where a, b, c are variables.
The plane (I) meets the co-ordinate axes in the points A, B
and C whose co-ordinates are (a, 0 , 0), (0, b, 0) and (0, 0 , c)
respectively.
It is given that the length of the perpendicular from (0, 0, 0)
to the plane (I) is p.
I I 1 , 1 , 1
P v/{(l/fl)*+(l/h)*-Hl/c)«} ΟΓ p* ~<P + b* Ttfi '
...(2)
Now we shall find the equation of the plane through the point
A (a, 0, 0), and parallel to the yz-plane.
The equation of the yz-plane is jr=0.
Any plane parallel to the plane x = 0 is given by x=X.
If it passes through the point A(a, 0. 0), we have α=λ.
Hence the equation of the plane through A and parallel to the
yz-plane is x —a. ■...(3)
Similarly the equations of the planes through the points B
and C and parallel respectively to the co-ordinate planes y = 0 and
Z=Oare y= b .,..(4) and z= c. —(5)
The locus of the point of intersection of the planes (3),.(4)
and (S) is obtained by eliminating a, b, c between the equations
(2), (3), (4) and (5).
Putting the values of a, b, c from (3), (4) and (5) in (2), the
required locus is given by
l//»*=l/jr*+l/y*+l/z* or p~t= x~*A-y~t +z~*.
Ex. 2. A variable plane passes through a fixed point (α, β, γ)
and meets the axes o f reference in A, Bt C. Show that the locus o f
the point o f intersection o f the planes through At Bt C parallel to
the co-ordinate planes is a*-1+ P y 1-^yz-1==I . (Meerut 1984)
Sol. Let the equation of the variable plane be
x/a+ ylb+ z/c= ], ...(I)
where a, b, c are parameters i.e.t Variablesv
68 Analytical Geometry 3-D

The plane'(I) passes through the point (α, β9 y).


Λ afa+P/b+y/c= I. ...(2)
The plane (I) meets the co-ordinate axes in the points Ay B
and C whose co-ordinates are respectively given by (a, 0, 0),
(0, 6, 0) and (0, 0, c)m The equation of the planes through A9 3
and C and parallel to the co-ordinate planes are
y = b 9 z= c respectively. ....(3)
[See Ex. I above]
The locus of the point of intersection of these planes [given
by the equations (3)] is obtained by eliminating the parameters
a9 by c between the equations (2) and (3). Putting the values of
a9 b9c from (3) in (2), the required locus is given by
alx+ P iy+ y!z= \ or ιοΓι +β)Γ1+ γ ζ -1= I.
Ex. 3. A point P moves on the plane x/a+y.lb+z/c= I which
is fixed. The plane through P perpendicular to OP meets the co­
ordinate axes in A9 B and C. The planes through A9 B and C parallel
to the yz9 zx and xy-planes intersect in Q. Prove that i f the axes
be rectangular9 the locus o f O is
± . ± 2 __L L,_L.
x* ~*~y* ~~ax r b y ^ cz (Kanpur 1983)
SoL .The equation of the plane is
xfa+ ylb+ zlc= I. (I)
Let the co-ordinates of the point P be (α, β, γ). Since the
point P (a, β9 y) lies on the plane (!), we have
afa+P/b+ylc= I. ...(2)
The direction ratios of O Pare α—0, β —0, γ -O ie .9 a, β, γ.
Heoce the equation of the plane passing through the point
P (α, β 9 y) and peipendicular to OP is '
a (x - * ) + P ( y - Ρ Η γ ( ζ - γ ) = 0 9
or ax+fiy+yz=a*+p2-f y*. .. (3)
The plane (3) meets the axes in the points A9 B and C whose
co-ordinates are respectively given by
({α*+β*+Λ/α, 0, 0), (0, {α*+β*+γ*}Ιβ90}
and (0 ,0 9{**+β*+?}Ιγ).
Again the equation of the plane through A and parallel to the
yz-plane i.e.9the plane .x=0 is
x = (« 2+ 3 2+y*)/cL ...(4)
Similarly the equations of the other two planes are
\Κ=(«2+ ^ + Λ / β (5) and ζ=(α*+β*+γ*)Ιγ. ...(6)
The Plane 69

Now Q is the point of intersection of the planes (4), (S) and


(6). The locus of the paint Q is obtained by eliminating a, β, y
between the equations (2), (4), (5) and (6).
From (4), (S) and (6), wc have
I I I β*
v« +
x* T «/ · ‘•'r* ( a 1+
r z* ~ (α , + ρ0 2V
ί + 7r2!)*+
)*τ '(* Η /Ρ + 7 ί)*τ (»*+β*+Λ*
**+β*+γ*
(a*+p*+y*)* «*+//* r Jt (7)
.1 .1 - I >.la+$:b\-Yic i___
ind ax ^ Tf' cbL· [using (2)]
y c+z^ — (β*+//*+γ*)* a*-t-β*+γ*' - ( 8)
From (7) and (8), the required locus of Q is given by
I/**+ 1/y»+ Hzt= 1/(βχ)+1/(δκ) + l/(cz).
| x . 4. A variable plane is at a constant distance 3p from the
origin and meets the axes in A, B and C. Prove that the locus o f the
centroid of the triangle ABC is xri -r jr 9+z~i =p~a.
(Meerut 1983, 85, S9S ; Luckauw 81)
SoL Let the equation of the variable plane be
x/a+y/6+z/c=l. ...(I)
It is given that the length of the perpendicular from the origin
to the plane (I) is 3p.

*** 3P~~ V (UaiA-1Ibi+ 17c*) ΟΓ V +b* + e» ‘ ...(2)


T hepIane(I) meets the coordinate axes in the points A, B
and C whose co-ordinates are respectively given by (a, 0,0),
(0, b, 0) and (0, 0, c). Let (x, y. z) be the co-ordinates of the
centroid of the triangle ABC. Then
x=(a+(>+0)/3, y = (0 + 6 + 0 )/3 , z= (0 + 0 + c)/3
Len . x= $e, y = \b , z= \c .
Λ a= 3x, b=3y, c=3z. „.(3)
The locus of the centroid of the triangle ABC is obtained by
eliminating a, b, e between the equations (2) and (3). Putting the
values of a, b, c from (3) in (2), the required locus is given by

sp-SS+SS+SS· “ *-’+ ^ + * -* = 0-*.


Ex. 5 (a). A variable plane is at a constant distance p from the%
origin and meets the axes in A, B and C. Show that the locus o f the
centroid o f the triangle ABC is
or* + y * + a r * = 9 jr * . (Lucknow 1981)
Sol. Proceed as in Ex. 4 above.
70 Analytical Geometry I-D

Ex. 5 (b). A plane meets the co-ordinate axes at A, B, C such


that the ,centroid o f the triangle ABC is the point (a, b, c). Find the
locus to the plane ABC. (Meerut 1989)
Sol. Let the equation of the plane be je /e '+ y /i'+ z /c '= l...(l)
the plane (I) meets the co-ordinates axes, in A, B, C. Hence we
have A {a', 0, 0), B (0, b', 0) and C (0, 0, c’).
Since centroid of A ABC is (a, b, c)
Λ e = i (a' + 0 + 0 ), b = } (O+b'+O), c = i (0+0 -He')
or a '= 3 e , b'=3b, c' = 3c.
Substituting values in (I), the equation of the required plane
is xla+y/b+ zlc= 3.
Ex. 6. Available plane is at a constant distance p from the
origin O and meets the axes in A, B and C. Show that the locus o f
the centroid o f the tetrahedron OABC is
x~*+y~*+z~, =l6p~*. (Lucknow 1982; Meerut 83)
Sol. Let the equation of the variable plane be
xla+ ylb+ z/c= l. ...(I)
It is given that the length of the perpendicular from the origin
O to the plane (I) is p.
I 1 1 1 1
·· P ~ V O /a*+1/**+!/«*) ΟΓ Pt " a * + i* ...(2)'
The plane (I) meets the axes in the points A, B and C whose
co-ordinates are (a, 0, 0), (0, b, 0) and (0, 0, c) respectively. Let
(x, y, z) be the co-ordinates of the centroid of the tetrahedron
OABC. Then
* = (0 + e + 0 + 0 )/4 , ^ = (0 + 0 + 6 + 0 )/4 , z = (0 + 0 + 0 + c)/4 . .
Λ a= 4 x , b=4yt c=4z. ...(3)
The locus of the centroid of the tetrahedron OABC is obtained
by eliminating a, b, c between the equations (2) and (3). Putting
the values of a, b, c from (3) in (2), the equation of the required
locus is given by
ΟΓ ^ + r ’+ r - ’- l i / r * .
Ex. 7. Two systems o f rectangular axes have the same origin.
Ifaplanecutsthem at distances a, b, c and a', b', c', respectively
from the origin, show that
-Lj--Lj--L--L -L L
a* + b2 + c* ^ a '2 + 6'2+c'!'(Lucknow 1978; Kanpur 82)
Spl. Let O be the origin. Let OX, OY, OZ be one set of
The Plane 71

rectangular axes.and let the equation of the plane with respect to


this set of axes bie x/a+ylb-i-z/c=l. ...(i)
Let the second set of rectangular axes be chosen as 0 (, OyI, Οζ
and let the equation of the same plane with respect to this set of
axes be f/o '+ ^ /h '-K /c '= I. .,.(2)
Now we know that from a point outside the plane only one
perpendicular can be drawn to the plane. The origin being the
same for both the systems, the length of the perpendicular from
the origin to the plane in both the cases will be the same. Hence
we have
I I
V{(l/«)2+ (Im +(I/C*)} “ V {(V a y + ( W + ( IIey y
ot I/ο*+1/A*+ l/c2= l/a'*+l/6'*+l/c'*.
Ex. 8. A plane meets a set o f three mutually perpendicular
planes in the sides o f a triangle whose angles are A, B and C respecti­
vely. Saow that the first plane makes with the other planes angles,
the squares of whose cosines are
cot B cot C, cot C cot A, cot A cot B.
Sol. Let the three mutually perpendicular planes be chosen
as the co-ordinate planes, and let
x /a + j'/h + z /c = ! ...(I)
be the equation of any palne which meets the axes of x, y and z
at the points A, B and C respectively.
Clearly the co-ordinates of the vertices A, B, C of the Δ ABC
are (a, 0, 0), (0, b, 0), (0, 0, c) respectively.
The direction ratios of the side AB are a, —b, 0 and the direc­
tion ratios of AC are a, 0, —c. Hence the angle A between the
sides AB and AC is given by
tan
O1Ot + ^ b t A-C1Ci · [See § 12, chapter 2]
nr tl_ V K ^ -O )* + (0+ac)»+(0+ah)*]
or tan A S r+ o + 0 ----------- *
Λ cot A =OiI i / ( b V + O2Ci + aW).
Similarly cot B= b2/y/(b2c2+ c2a2+ a2bP)
and cot C = c2/ v (a2b2+b2c2+c2a2).
Now suppose a is the angle between the plaqe (I) and one of
the co-ordinate planes say the plane x = 0 . The d.r.’s of the nor­
mals to these planes are l/a, I/b, l/c and I, 0 ,0 respectively.
.. rn- ff- (l/fl).l + (l/6)-0+(l/c).0
• co VU/«2+ l/^ + r /c * ) V ( l2+ 0 a+ 0 2)
72 AnalyticalGeometry ό-D

IPct
or cos· * « : =cot B cot C
(W + c V + a V )
Similarly if β and 7 are the angles which the plane (I) makes
with the coordinate planes 7 = 0 and z = 0 respectively, then we
have cos* 3 =cot C cot A and cos* y=cot A cot B.
Ex. 9. Find the equation o f the plane which bisects the Join o f
F (*ι. Λ· *i) and Q (*ι. Λ» Zt) perpendicularly.
Sol. The required plane passes through the middle point of
the segment PQ and is perpendicular to PQ. The co-ordinates of
the middle point of PQ are
(I (*1+*.), i (Λ+Λ)> i (Zi+Z*)·
Also the direction ratios of PQ are (JC1 -X 1, yx—yx, Zx- Z i). Thus
the direction ratios of the normal to the plane are Xx- X i, yi~y*,
zx- Zi and hence the equation of the required plane is given by
( * i - ^ ) {*—i (*ι+*ί)} + ( Λ - Λ ) { y - i 0 ’i+>'*)}
+ (Zi—Zi) {Z—J (Zl + Z*)}=0
or x (x i- x ,) + y (yx -y t)+ z (zx- z t)
-= Η(*Λ- x»*)+iy * - λ *Η-Ui*-·z**)}·
Ex 10. From any point P are drown PM and PN perpendiculars
to zx and xy-planes. O is the origin and «, β, y and S are the angles
which OP makes with the co-ordinate planes and with the plane
OMN. Prove that i f the coordinates of the point P are (a, b, c),
then.
(0 the equation o f the plane OMN is xla—y/b-~zlc=0
abc
(«) S=sin~l V<e*+b*+c*) ν(δν*+<Αι»+<Λ>*)
and (sis) cosec* S=Cosect a+cosec* β+cosec2)'.
Sol. The co-ordinates of the point P are given to be (a, b,c).
The equation of the zx-plane is 7 = 0. Now since PM is drawn
perpendicular from P to the zx- plane, therefore M is the foot of
jthe perpendicular from P {a h c) to the zx-plane and hence the
co-ordinates of the point M are (a, 0, c). Similarly the co-ordin­
ates of the foot N (lying in the xy. plane) are (a, b, 0).
Now we shail find the equation of the plane OMN.
The equation of any plane through the origin O (0, 0. 0) is
Ax+ By+ C z= 0. ...(I)
Ifthe plane (I) passes through the points M (a, 0, c) and
N (a, b, 0), we have A.a+B.0+C.c=0 a.nd A.a+ B.b+C.0—0.
Solving these for A, B, C, we get —— = JL = SL .
—be ca ab
The Plane n

PutUng the values ot A, B, C in (I) the equation of the plane


OMN is given by
- bcx+cay+abz—O or x/α - y jb - z /c —O. ...(2)
This proves the result (i).
To find the angle S. Since S is the angle between the line
OP and the pilane OMN, therefore 90° - S is the angle between the
line OP and the normal to the plane OMN.
The directionratiosofthe line OP are a —0, b - 0 , c - 0 l.e.
a, b, c. The direction ratios of the normal to the plane OMN are
I la, - \{b, - 1/c. [See equation (2)]
Hence.cos (90®-S)* a ( l/a ) + b ( ^ llb ) + c ( - llc )
V (e*+**+c*)V (I/e*+ l/h*+l/c·)
_ abc
V(e*+**+<·*) V(****+<a »*+W
neglecting the —ive sign because 90° - S is the acute angle between
the line OP and the normal to the plane OMN
abc
or
\ S,n δ _ y/i<P+iP+c*) V ( f iV + W + Λ » ) ...(3)
abc
or i-l
Os=Sin y/(<P+b*+c*)
This proves the result (ii).
Again let « be the angle between the line OP and the co­
ordinate plane x = 0 , so that 90®—a is the angle between the line
OP and the normal to the plane X=O whose d.c.’sare 1,0, 0.
Hencewehave
α.Ι-1-h.O+c.O
cos (90®-«)=
V (e * + ^ + c 4)
u * e*+6*4-:t’*
or Sin as= °r COSeC -------d is­
similarly, we have
cosec*\ a/?*» β\+
—— ft* TC'
- anda cosec2 2 y «*+£*+c*

Λ cosec4 y.-f cosec2 /1-fcoscc* j (Q2 - / ’2-


(hV -K '/2Aa}-(Ciii-J- A2-4-fa>
tro-c*
=Coseci <i. lusing the relation (3)]
This proves the result (iii).
§ 17. Equations of the planes bisecting the angles between
two given planes.
Let the equations of the two j iven planes be
74 Analytical Geometry 3-Z)

aiX+biy'+ctf+dimmO, ..(I)
and V + ^ + v H = 0 · --'-(2)
The equations (I) and (2) should be so written that the cons­
tant terms dx and d2 are both positive. However, we can also
write the equations (I) and (2) in such a way that dx and dt are
bofh negative.
If (X1, y l9 Z1) be the coordinates of any point on the plane
bisecting the angle between the given planes, thpn the perpendi­
cular distances of this point from both the planes should be equal
numerically. Since we consider perpendicular distance as positive
* when measured in the direction from the origin to the plane, for
points on the plane bisecting the angle in which the origin lies,
the perpendicular distances will have the same sign, and for points
on the other bisector, opposite signs. Therefore if the point
(X1, y X9 Z1) lies on the bisector of the angle in which the origin Iies9
we have
a^ i + bxyx+ C1Z1+ dx α2χ Λ+ h2yx+ c2zt +dz
V W + V + Ci*) ~ V W + W + c f)
Since thisrelation is satisfied by every Point(Vl i ^l i Z1)On
this bisector, the equation of this bisector plane is
Q\X+ bxy + c\z -M 1 a2x + b2y + c 2z+ d2
V W + b i 2 + C i 2) ~ V W + V + C *)1 ' -(3 )
Similarly if (xx, y l9 Z1) lies on the bisector of the other angle
between the two planes, we have
a xx x+ b ± y i + c xz x+ d x O2Xx+ b2y x4 - C2Z1+ d2
VW +bf+cf). " V W + b f +cf)
The equation of this bisector plane will be
axx + b iy + c xz + d x a2x + b 2y + c 2z+ d 2
V W + b f + c f ) ~ ~ V W + b * + c 2*) ...(4)
Hence the equations (3) and (4) are the required equations of
the planes bisecting the angles between the given planes. The
plane (3) bisects the angle in which the origin lies and the plane
(4) bisects the angle in which the origin does not lie. But we
should not forget to write the equations (I) and (2) in such a way
that dx and d2 are of the same sign.
To distinguish between the two bisecting planes as regards the
bisector of the acute or obtuse angle between the given planes.
If we are required to find which of the two bisecting planes
given by (3) and (4) represents the plane bisecting the acute or
obtuse angle between the given planes (I) and (2), we find thfc
The Planes 75

value of cos Q9 where Θ is the acute angle between the bisecting


plane and any of the given planes. From this value of cos Q9 we
find the value of tan Θ by using the formula tan 0 = ^ (s e c 2 Q— I)
(or otherwise). In case the value of tan Q < I then Q < and
hence this bisecting plane bisects the actue angle between the
given planes. Again in case the value of tan B > I then Q > \n
and hence this bisecting plane bisects the obtuse angle between
the given planes.
An important remark. To find whether origin lies in the acute
or obtuse angle between the given planes (I) and (2).
The costant terms dx and dt in the equations (I) and (2)
should be of the same sign.
(i) If the origin lies in the acute angle between the given
planes (I) and (2), then the angle Θ(say) between the normals to
these planes is obtuse and therefore the value of cos Q is negative
i:e. U1S2+ ^ b 1+C1C1= negative is the condition for the origin to lie
in the acute angle between the planes.
(ii) Iftheorigin liesin the obtuse angle between the give ή
planes (I) and (2), then the angle Q(say) between the normals to
these planes is acute and therefore the value of cos Q is positive
Le99 a ^ + ^ b j + C 1C1 = positive is the condition for the origin to lie
in the obtuse angle between the planes.
Thus to distinguish between the two bisecting planes, we
should first find whether the origin lies in the acute or obtuse
angle by the above method. Then we should find the plane bisec­
ting the angle which contains the origin and the plane bisecting the
angle which does not contain the origin. In this way we can dis­
tinguish between the two bisecting planes with respect to the posi­
tion of origin and the acute and obtuse angles.
Solved Examples (3) D
Ex. I. Find the equations o f the bisectors of the angles between
the planes lx - y -2z - 6= 0 and 3x+ 2y —6z -1 2 = 0 and distinguish
them.
Sol. Writing the given equations in such a way that the
constant terms are both positive, the equations of the given planes
are —2 x + j+ 2 z + 6 = 0 , ...(I)
and —3x—2 y + 6 z + 12=0. ...(2)
The equation of the bisector plane of the angle between the
planes (I) and (2) which contains the origin is given by
76 Analytical Geometry 3-D

—2x+ y+ 2z+ 6 —3x—2 v + 6 z + l I


V (4 + l+ 4 ) ~ V (9+ 4+ 36)
or 7 (~ 2 x + ,y + 2 z+ 6 )= 3 ( —3x—2.y+6z+12)
or 5 x~ l3 y+ 4 z - 6=0. ...(3,'
The equation of the other bisector plane is
—2x+.y+2z+6 —3 x -2 .y + 6 z+ l2
'✓ < 4 + 1 + 4 ) = ~ V 19+ 4+ 36)
or 7 ( - 2 x T j>+2z+6't= - 3 ( - 3 jc- 2 ^ + 6 z+12)
or 2 3 z '-j'-3 2 z —78=0 --(4)
Now let Θ be the acute angle between the plane (4) and the
bisecting plane (3)· Then
I -2..S+1.( -13)+2.4 I 15 /5
COS 9 I V(4-h i+ 4 )V (2 5 + 169+16) | _ 3V2I0 V 42
Λ tan 0=v/(sec* l)= V [(42/5*) - IJ ^ V (3 7 /5 » 1 , so that 0>45°.
Hence the plane 5jc- 13>'+4r - 6 = 0 bisects the obtuse angle
between the given planes (I) and (2) so that the other plane
23x—y - 32r—7 8 = 0 bisects the acute angle. ,
We also note from the given planes (1+ and 12), that
+ ^1^+^iCj - (—2) ( —3 ) + 1 (^-2 )+ 2 (6)
x - 6 —2 + 12 - - 1 6 = positive.
H e n c e th e o rig in lic sin th e o b tu se angle between the given
planes. [See. remark to § 17].
This confirms tha* the plane (3) is the bisector of the obtuse
angle. Hence the equation (4) is the bisector of the acute angle.
Ex. 2. Show that the origin lies in the acute angle between the
planes x + 2 y + 2 z - 9 --0 and 4x—3y + 12 r + 13-=0. Find the planes
bisecting the angles between them and point out the one which bisects
the acute angle.
Sol. In order that the constant terms are positive, the
equations of the given planes may be written as
\ x —2)'—2z+ 9 --0 ...(I)
and 4 x -3 > :+ 1 2 z+ 13=0.
Wc have
^ 1 ^ + 6 ^ + ^ = (- 1 ) .4 + ( - 2 ).( -3 ) + (- 2 ) .(1 2 )
= —4 + 6 —2 4 = -22«= negative.
Hence the oiigin lies in the acute angle between the planes
(I) and (2) {See remark to § 17 ·.
The equation of the plane bisecting IiitT Snglc between the
given planes (I) and (2) wlueh contains origin is
The Plane 77

x -2 y -2 z + 9 4 x -3 H -1 2 z + 1 3
V O + 4 + 4 ) ^ V U 6 + 9 + I4 4 )
or 13 ( —jc—2v—2z+9) = 3 (4jc—3 ^ + 12z+13)
or 25jc+ I ly+ 62z —78=0. .(3)
We have pioved above that origin lies in the acute angle bet­
ween the planes and so the equation (3) is the equation of the
bisector plane which bisects the acute angle between the given
planes.
The equation of the other bisector plane (/.*., the plane bisec­
ting the obtuse angle) is
- jc 2)> 2z+ 9 4jt S y + 12 z + 13
V U +4+4) “ V U 6+*+144)
or JC+ 35y Oz 156—0. .(4 )
.. the equations (3) and (4) give the planes bisecting the
angles between the given planes and the equation (3) is the bisec­
tor of the acute angled
Ex. 3 Find the bisector of the acute angle between the planes
2jc—>>+2z+3=0 and 3x - 2 y + 6 z + 8 - 0.
Sol. Proced as above in Ex. I or Ex 2. The required bisector
plane is 23jc—13^+ 32z+ 45= 0.
Ex. 4. Find the equation of the plane that bisects the angle
between the planes 3 jc— 6j >+2z+ 5 = 0 and Ax — I2.y+3z=3 which
contains the origin. Is this the plane that bisects the obtuse angle ?
Sol. Proceed as above in Ex. I or Ex. 2. The required
bisecting plane is 67jc - 162^+47z+44=0, and this is the bisector
of the acute angle.
§ 18. Combined equation of a Pair of planes.
To findthe condition that the general homogeneous equation
of second degree in jc, y and z namely
ajp+byt+cz*+ 2fyz+2gzx+2hxy=*Q.
may reprsent a pair o f planes and to find the angle between them
Also to find the condition o f perpendicularity o f these planes.
The general homogeneous equation of second degree is
ax*+by*+cz*+2fyz+2gzx+2hxy=0 ...(I)
Let the equations of the two planes represented by (I) be
Z1JC+JW1J+W1Z = 0 and ZaJc+/iijjH* JifZ = 0.
78 Analytical Geometry 3-D

These equations will not contain the constant terms for other­
wise their product will not be homogeneous. Thus we have
a x *+by*+ C z t A 2fyz+ Igzx+ 2hxy
= (IlXAm tyAn1Z) (ItX+miy+riiz).
Comparing the coefficients of like terms on either side, we
have
IlIt=O, m1mt =b, /I1Zj2=C, /M1Zi2-I-ZniZi1= I / ,!
H1ItA n J 1=Tg, Ifnt A lfn 1=Th J‘ ...(2)
The required condition is obtained by eliminating I1, /M1, M1
and It, mt, nt from the relations (2). This is conveniently done by
considering the following product of two zero-valued determi­
nants :
/i k 0 z, k 0
W1 MI2 0 X ZZI4 ZMi 0 =0.

*1 »2 0 ”% ni 0 [Remember]
Multiplying the two determinants by row-by-row multiplica­
tion rule, we have
V ik IiftttAliftt1 Iftt A ltFt1
M1I1A m tIi 2ZM1ZM1 Zn1Ms -I-ZMt M1 =0.
H1It A t l t II M1ZMt -I-Mt ZM1 Tn1Ot
On putting the values of I1It, Z1Znt +ZtMi1 etc. from (2), we have
Ta Th 2g a h g
Th Tb 2/ = 0 or h b f

2g 2 / . 2c g f C
or a b c A V g h -a p -h g * eh*=0. .. (3)
This is the required condition that the equation (I) represents
a pair of planes passing through the origin.
To find the angle between the planes. L e tg b e th e angle bet­
ween the two planes represented by the equation (I).
Then Θis the angle between the planes I1X A mXyAn1Z=O and
Zt*+ niJy+Mtz= 0 and so is given by
tan 0 = (IM1)It -/MtB1)*]*/*
IikA tttftitAnint ’
where Z1Z1+ /M1ZM1 + M1M2 = a A b + c
and Σ (m1nt —m ft1)*=E [(/M 1MiH-MisM1) * - 4znIzntzi,zit]
The Plane 79

=Σ (AP-Abc)
=A (/* —ftc)+4 (g*—ca)+4 (A*—eft),
so that [^(Ot 1W2- 01,/1, ) ^ = 2 ^ ( /* + £ * + * * -* £ -« * -« * )·
• a _ 2 V iP + g * + h * -l» c -c a -a b )
* * -------------- a + b + c -.(4 )
or , _ ta„ - . [ W + £ ± ! ^ | £ ^ r £ « l
Condition of perpendicularity. The two planes given by (I)
will be perpendicular if 0=£π i.e. tan 0= tan J tt= oo. Hence the
relation (4) gives a + b+c=0.
Thus two planes given by (I) will be perpendicular if
the coefficient of Xa+ the coefficient Of y2+ the coefficient of za=0.
Solved Examples 3 (E)
Ex: I. Prove that the equation x2+4y2 —z2+ 4xy= 0 represents
a pair o f planes and find the angle between them.
Sol. Comparing the given equation with the homogeneous
equation of second degree in X9y , z [See equation (I). § 18], we
have a * I, b = 4, ¢ = - 1 , / = 0 , g = 0 , A==2:
Λ abc+ Ifgh—a /2—bg*—cA8
—1 .4 .( - 1 ) + 2 .0 .0 .2 - 0 - 0 - ( - 1 ) (2)2= - 4 + 4 - 0 .
Hence the given equation represents a pair of planes.
If Θ is the angle between the planes, then
tan f *+g*+A» - be - ca-ab) [See resu,t (4)> § 18]
fl+A+c
2 ^ /(0 + 0 + 4 + 4 + 1 -4 )
putting for a, ft, c etc.
1+ 4 - 1
= W 5. .·. tf=tan-1 (iV5)·
Alternative method. The given equation may be written as
(jc*+4jcj'+4iy *)-zi =0, or (x+2y)t —zt = 0
or (x+ 2y+ z) (x+ 2y—z)=0.
Λ * + 2 ^ + z = 0 , x + 2 y —z=0.
These being linear equations in x, y, z represent two planes.
If 0 is the angle between these planes, then
COS e ______ 1.1+2.24-1.(-1) 4 2
V ( l+ 4 + I ) V ( l + 4 + l ) ~ 6 “ 3
Λ tan 0 = W 5 .
Ex. 2, Prove that the equation
2x*—6ya—12z*+1Syz+ 2zx+ xy= 0
represent^ a pair o f planes and find the angle between them.
80 Analytical Geometry 3-D

Sol. Comparing the given equation with the equation (I) of


§ 18, we get
* = 2 , b - --6. c·---- 1 2 , / - f . l 8 - 9 , g = $ .2 « I , Λ'=|.
Λ abc+ 2fgh - cif- hg* ch*
—2 (-6 ).( —12) f 2.9.1.^-2.81 +6.1 + 12.1 \
—14419 —162+6 ).-3= 162—162=0.
Hence the given equation represents a pair of planes.
If 9 be the angle between the planes, then
t^ 2 V i n ± gU_ h ^ b c ^ - a » ee (4) of § i8 |
O + P l C. . '·
2V(8l + l + f - 7 2 + 2 4 + 12) 2Vii-< 185)1
“ “ ". ' 2 - 6 - 1 2 ~ -' >6
- VO 85)
16
sec* ¢= I + tan* # - 1 + ^ = ¾
/> 21 „16 ■ , IU \
sec 9= ,-; or cos
IO • - π or *-cos (π)
Alternatiye method. Arranging the given equation as a quad­
ratic in X9 we have
2x2+ x (2z+y) —(6v* + 12z2 - 18^ 2) = 0.
( 2 z ^ y ) ± V [ ( 2 z *- v)2+ 4 . 2 . ( 6 ^ 4 . U z 2- W y z ) ]
X=
2\2
or 4x= . 2z v± \/[4z2+4zv + v2+ 48v2+ 96z2—144yz]
= - 2 z - v ± V(49j2- 140rz+ IOOz2)
= —22—y ± \/( 7 y ~ W z ) '1
— —2z —y±{7y~}Qz).
Λ 4x= -2z—y + lv -IOz nnd 4 x ^ —2 z+ v —7y+10z
or 4x—6j; + l?z= O1and 4λ*+8^—8z=-0
or 2.Y—3v+ 6z~0 and x + 2 v —2z= 0.
These being linear equations in x, y and z represent the two
planes. If Θ is th«e angle between these planes then using
cos it = αχα%+ IhbjjTCxC1
V U shj. h+ *-t-Crj
■, ··, V ti (h~
t1- h V*n Tt
r rg ;>; 've ^ave
2. i + ( —3)42)16 .(-2 ) _ —Hi
cos#=
V H + 9 + 36) Vi.' + 4 + 4 )' 2 1 ’
giving the obtuse angle between the planes.
If 9 is the acute angle between the pliines, then cos ¢=16/21.
Λ ¢=005-/(16/21).
The Plane 81

Ex. 3. Show that the equation —


y —z z —x x —y
represents a pair o f planes.
So). Multiplying the given equation by
(y —z) (z—x) (x—y), we have
a ( z - x ) ( x - y ) + b ( y - z ) ( x - y ) + c ( y -z ) (z -x )= 0
or a (zx—y z —x*+ xy)+ b (xy - y * - z x + y z )
+ c (yz - xy -- 2*+r.v)—0
or ax^+by^+cz*—( b + c - a ) yz (c+a b) zx
—( a + b - c ) xy= 0. :. (1)
Comparing the equation (I) with the general homogeneous
equation of second degree in x, y, z i.e., the equation
Ax*+By*+Cz*+2Fyz+2Gzx+2Hxy=0, [Refer eqn. (I) of § 18]
we have A—a, B = b , C = c , F——\ (ό+ c -a), (7= -- \ (c+a—b),
H ---- 1 ( a+ b- c).
[Note that we have used capital letters A, B, B etc. because
small letters a, b, c etc. are used in the question.]
The equation (I) will represent a pair of planes if
A H G i
I I
H B F i-=0. [Refer §18]
J
G F C I
Putting for A, B, C etc. we have
A H G
H B F
G F C
β —1 (o+b—c) —J (c+a—b)
—\ (a+b -c ) b - i( * > + c a)
—i (c+a—b) —J (b+c—a) c
ο ο o i
- i (a+b-c) b -i(b+ c-a) |,
—i (c+a—b) —i ( b + c - a ) c I
adding the second and third rows to the first row
= 0.
Hence the given equation represents a pair of planes.
82 Analytical Geometry 3-D

Ex. 4. I f the equation


Φ (x , y, z)= axl +byi +cz%+2fyz+2gzx+2hxy=Q
represents a pair o f planes, then show that the products o f the distan­
ces o f the two planes, from {a, β , γ ) is
Φ Ο*, β, 7)
V[Da2+4D /2-2DAc)
Sol. Let the equation
Φ (x> y, z)= axl -\-byt -\-czi -\-2fyz+ Igzx-|-2hxy= 0 ...(I)
represent two planes given by
liX+m1y-\-niz= 0 ...(2) and /,χ+ η/& + η2ζ=0, ...(3)
so that we have
Φ (x> y> z)=(/jjr f W|>’+ n 7z) {ltx+ m ty+ n tz), ...(4)
where φ (x , y, z) is given by (I).
Comparing the coefficients of like terms on either side of (4),
we have
IJ t= a, /M1W4==&, HlUa=C I
/«,η,+/Mjn1= 2/, H1It A-UtIi^ g , l1mt + ltm1=2h I ...(5)
Let P1 and pt be the perpendicular distances of the point
(®. P> y) from the planes (2) and (3) respectively.
Then we have
_ Λα+ζη,/ί 4-w,y _ lt*+mtP+nty
P1 v 'ft'+ M i'+ n i’)’ H ~ V W + w*2+ «**)'
Multiplying, we get
(lia+ m fi+ η,γ) (/2a+OTs/?4 nty)
PiPt V d S + m S + n S W U S + m f + n f ) ...( 6)
Now substituting a, β, γ for x, y, z respectively in (4), we get
(/jot+wi^+Wjy) (/4a + mtp+ nty)= φ (a, β, γ). .. (7)
Also ( I f + m S + n f ) Of+ m^+ n./ )
= UtIii + Wl2OTj2+ H12Ht* + (Z12OTj2+ I32Hl12)
+ (OT12W22+ tOj 2W12) + (H12/*2+ Wj2Zl2)
= W2+ A2+ C2+ D {(/jOTj +ZjOT,)2 - 2/j/jOTjOTj}
= e 2+A*+c*+D (4A2—2aA), using the relations (5)
=W2+ A2+ c2+ (4A2- lab) + (4/2- 2A-')+ (4g2- 2ca)
= (a 2+A2+ c 2)+ 4 (/*+g*+A 2) - 2 (Ac+ce+aA)
=Da2+ 4D/2—2DAc.
VVxt + ’"i*+Wi2) -/(/ j2+ μ,2+ w,2)= V[Da2+ 4D/2- 2DAc], ...(8)
Substituting the values from the relations (7) and (8) in (6),
we get „0= Φ («. fit Ύ)____
V[^e*+4D/2-2DAc]
The Plane 83

§ 19. Projection on a plane.


Recall the definitions of the projection of a point and the
projection of the segment of a line on a plane (see § 6, § 7 of
chapter 2).
Similarly the projection of an area A on a given plane is
defined. Let A be an area enclosed by the curve PQR.... Let
P'yQ'yR'y··· be the feet of the perpendiculars drawn from
Py Qy Ry ··· to the given plane. Then the projection pf the area A
enclosed by the curve PQR ... on the given plane is the area A'
enclosed by the curve P'Q'R' .. If 9 is the angle between the
plane of the area A and the plane of projection, then A' —A cos 9.
Now we shall discuss two theorems on the projections.
Theorem I. Let the projections o f an area A on the co-ordinate
planes yz, zx and xy be A9, A, and At respectively, then
A -A f+ A S+ A f.
Proof. Let the direction cosines of the normal to the plane
of area A be /, m, n. Also the normal to the ,yz-plane is x-axis
whose d.c.’s are I, 0, 0. If a be the angle between the plane of
area A and the yz-plane, then a is the angle between the normals
to these planes and so
cos a*=/.l+m .0+n.0=/.
Now the projection A9 of the area A on the .yz-plane is given
by A9= A cos a = Al.
Similarly we have Ar =Am, At=An.
Squaring and adding, we have

Theorem 2. The projection of a given plane area A on a given


plane ξ is equal to the sum o f the projections o f A9, Ar and Az on the
given plane ξ, where A9, Ar and At are the projections o f the area A
on the co-ordinate planes viz. yz, zx and xy-planes respectively.
Proof. Let /, m, n be the d.c.’s of the normal to the plane A,
and let /', m', it' be the d.c.’s of the normal to the plane ξ. Now if
9 is the angle between these two planes, then
cos 9=ll'+ m m '+ nn'. ...(I)
Now let the projection of the area A on the plane ξ be A' ;
then we have
A '= A cos 0 or A'=A (IT+mm'+nn'). .. (2)
Also by definition and in view of theorem I , we have
Ax—>Al, Ar =Am, At=An. ...(3)
84 Analytical Geometry 3-D

From (2), we have


A9=^(AI) Γ + (Am) m'+(An) //'
= A J ’+A9m’-l-Arf [using the relations (3))
= (the projection of the area Ax on the plane £)
+ (the projection of the area A9 on the plane £)
+(the projection of the area Az on the plane £).
Proved.
§ 20. Area of a triangle.
To find the area of a triangle ABC the co-ordinates o f whose
vertices are A (xl9 y l9 Z1) , B (x2, zt) and C (x3, V3 , z3).
Let /, m, n be the d.c.’s of the normal to the plane of the
triangle ABC and let Λ denote the area of this triangle.
Let Ax Bte and Cx be the projections of the three vertices A9
B and C respectively on the ^z-plane. Clearly the co-ordinates of
these points are given by Ax (0, y l9 Z1 ) , Bx(0, y t9 zt\ Cx (0, y3, z3).
Let Δ* denote the area of the triangle AxBxCx i.e. A* is t^e
area of projection of the area A on the .yz-ptane, so that we have
Δ*=Δ·/. ...(Π
Also by the co-ordinate geometry of two dimensions,
Λ *1 I
Δ*=έ y* *2 I ...(2)

>» z Z I
area A on zx and xy-planes, then
& y= A .m (3) and Λ .= Δ.π
Z1 I y\ I
where Δ»=ί xI Zi I . Δ .- i X2 y» I

*3 Zz I *8 y» I
Squaring (I), (3) and (4) and adding, we get
Δ*2+ Δ /2+ Δ*2= A2 (/2+m2+/i*)=A2.l
or Aa= Δ*2+ Δ /2+ Δ·2· ...(5)
This gives area A of the triangle 'ABC.
Solved Examples 3 (F)
Ex. I. Find the area o f the triangle whose vertices arc A(I. 2, 3),
B (2, - I , I) a n d C ( I, 2, - 4 ) . (A ga 1979)
Sol. Let A*> Δν, Λ* be the areas of the projections of the
The Plane 85

area Δ of triangle ABC on the yz. zx and xy-planes respectively.


We have
Λ Zi I ! 2 3 I

y* Zi . i I= i -I I I 21
'2 ’
y a Z9 1 1I 2 -4 I
Zl I I 3 I
h
_I I Zt I =i 2 I I T_
I ** 2
I Xi Z9 I I -4 I
(numerically),
\ * y i I I 2 I
and A = ~ i X2 y2 I =i 2 -I I =0.

Xa I I 2 I
the required area A =V TA a2 *-· A /2+A»*]
441 , 49
-Ji ■0^=-iv/(490) square units.
Ex. 2. Aplanemakes intercepts OA=a, OB—band OC=c
respectively on the co-ordinate axes. Show that the area o f the tri­
angle ABC is (Z>zt,2-|- c2a2+ ο2Λ2).
Sol. The points At B and C lie on the axes of x, y and z
respectively, so that their co-ordinates are A{a, 0, 0) B(0, b, 0),
C(0, 0, c).
Let Adenote the area of the triangle ABC. The projection
of triangle ABC on the plane is the triangle OBC and if A*
denotes its area then
A m=\.OB.OC=\bc. . ...(I)
The projection of the triangle ABC on the zx-plane is the
triangle OCA and its area A* <s given by
A r =i-OC.OA=ica.
Also the projection of the triangle ABC on the xy-plane is the
triangle OAB and its area A# is given by
lS t—\.OA.OB==\ab. ...(3)
the area A of the triangle ABC is given by
A*+ A*8+ A**+ A*8= i (b'c'+c'a'+a'b*)
or A = i \/(h 8c8+ c 8a8+a*0*).
Ex. 3 Find the area o f the triangle included between the plane
3x —4_y-f z - 12 and the co-ordinate planes.
86 Analytical Geometry 3-2)

Sol, The equation of the given plane is


3 * -4 y + r= 1 2 or x/4-j>/3+ z/12= l ...(I)
The plane (I) meets the co-ordinate axes in the points
A(4, 0, 0), 21(0, - 3 , 0) C(0, 0, 12).
Using the notations of Ex. 2 above, we have
A » —\-OB.OC—\,.( -3).12=18 (numerically),
A ,= lO C .O A = l.\2 A =24,
&t=\.OA.OB·=* |.4.( —3) =6(numerically).
:. area Δ of the triangle A B C = ^{A » tJc Α **+Δ·2)
= | V (l^*+24a+6*)=3v'(26) square units.
Ex. 4. From a point P(x', y', z') a plane is drawn at right
angles to OP to meet the co-ordinate axes at A, B and C. Prove that
the area o f the triangle ABC is r*/(2x'}»'z'), where r is the measure
o f OP. (Kanpur 1983, 84 ; M.U. 1990)
Sol. The d.r.’s of the line joining O (0, 0, 0) and P(x', y'<z')
are x '- 0 , y '—0 , z '—0 i.e., x', y \ z' .
the equation of the plane through P(x', y', z') and per­
pendicular to OP is given by
x ’ (x - x ’y+ y' (y - y ') + z ' ( z - z ') = o ,
or xx' + yy’+ zz '= x 'a+.y, -hz'J '
or x x '+ y y '+ zz'= r* [V r= O P = y/(x’a+ y,?+z'*)]
f t __ , _y_L z ,
or
The plane (I) meets the co-ordinate axes in the points
AiriIx', 0. 0), B(0, r*ly\ 0) and C(0, 0, r a/z').
Let A be the area of the triangle ABC. Also let Δ» be the
area of projection OBC on the jz-plane of the area of triangle
ABC. We have
A ,~ \ O B .O C = l.y .rl
...(2)
Now d.r.’s of the normal to the plane of AABC i.e., d.r.’s of
the line OP are x', y', z', so that the d.c.’s of this normal are
x'/r, y ’fr, z'/r. [ 7 T = V i* '2+ / 8+ * '2)]
Also d.c.’s of the normal to the· plane of triangle OBC i.e.,
d.c.’s of x-axis are I, 0, 0.
If a be the angle between the planes of tringles ABC and
OBC, we have
cos « = l.(x 7 r)+ 0 .(//r)-f 0.(z'r)= x'r.
Now Δ * = Δ cos a.
The Plane 87
«4 yf r5
Λ 2 / ? = Δ · Τ ’ ΟΓ Δ "^Λ > ν·
Exercises
I. A plane makes intercepts 9, 9/2, —9/2, upon the coordi­
nate axes. F indthe lehgth of the perpendicular from the origin
on it. (Ans. 3)
-2. Show that the four points (0 ,-1 , 0), (2, I, —1), (I, I, I)
and (3, 3, 0) are coplanar and hence show that the equation of
the plane passing through these points is 4 x —3y+2z=3.
(Agra 1978)
3. Show that the four points (0, 4, 3), ( - 1 , - 5 , —3),
( —2, —2, I) and (I, I, —1) are coplanar.
4. Find the equation of the plane through the origin and
parallel to the plane 3 x + 9 ^ —7 z+ 5 = 0 . (Ans. 3x+9y —7z=0)
5. Show that the planes 3x+4y —5z=9 and 2 x + 6 y + 6 z= 7
are at right angles.
Hint Two planes are at right angles if their normals are at
right angles. The d.r.’s of the normals to the two given planes
are 3, 4 —5 and 2, 6, 6 respectively and we see that these lines
are perpendicular.
6. Find the equation of the plane which contains the line of
intersection of the planes x + y + z - 6 = 0 and 2x-J-3jy+4z-F5e»0
and is perpendicular to the plane 4x+5y —3z—8= 0.
(Ans. x+7y+13z+96==0)
The Straight Line

§ I. Geaeral equations of a straight line.


We know that every equation of first degree in x, y and z
always represents a plane (see chapter 3).
Now let us take two equations of first degree together
i.e. + Ciz+ di =■0, a**+ ijj»+ c,r+ dj= 0. ...(I)
Any point which simultaneously satisfies both the equations
given by ( I), will lie on the curve of intersection of the planes
given by (I). Since the two planes intersect in a straight line,
therefore the equations (I) represent the equations of a straight
line. The equations (I) are called the general equations of a
straight line. Therefore any two equations o f first degree in x, y
and z taken together always represent a straight line.
§ 2. Symmetrical form of the equations of a straight line.
(A) To find the equations o f a straight line which passes through
,
a given point (Jct, y,, z , ) and whose direction cosines are I m, n.
Let OX, OY, OZ be chosen
as rectangular co-ordinate axes.
Let A be the given point
(X1, y„ Z1) on the line. Choose
a general point P with co-ordi­
nates (x, y, :) on the line at a
distance r (say) from the given
point A. Draw perpendiculars
AM and PN on the jc-axis from V
the points A and P respectively,
so that A/Wis the projection of the segment AP on the .v-axis
and is given by
M N - A P - I or .v—.Vi- rl.
Similarly projecting the segment AP on the y-axis and z -a x is .
The Straight Line 9

wc get v —Vi —rnt and Z -Z 1=TM.


Therefore the co-ordinates (x, j , z) of any point P on the line
satisfy the equations
X -X 1 y J 1^ z - Z 1_
I m n -(0
It should be noted here that V is the actual distance of any
point P (x9y %z) on the line from the given point (x1$ J1, Z1).
X - X 1 y h jL -Z i
Hence —— =
m n
are the equations (symmetrical form) o f the straight line.
Vector method. We have
AP= position vector of P - position vector of A
~ W + y i + z k ) -(xii+yii+zl·)
= (* -* i) i + ( j -J 1) j+ (z ZlJk.
Since I9 m> n are the direction cosines of the straight line AP,
therefore a unit vector along the line AP is /i+w j+wk.
Now the vector AP iscollinear with the unit vector /i+m j+nk.
Λ AP=r ( / i + / « j + w k ) , where the scalarr is the distance AP.
Λ (X -X 1) i + ( j - J j ) j + ( Z - Z1) k=rli+rm i+rnk.
Equating the coefficients of.i, j , k on both sides, we get
X - X 1=T /, J - J 1=HM, Z -Z 1=Tfl.
• * y~y\ 2
I m ” μ
are the required equations of the given straight line.
(B) Symmetrical form in terms o f direction ratios.
Let the direction ratios of the required line be a, b, c. Hence

V(a*+6*+ca)’ ’ -,/(a'+b'+c*) are the d,rect,on


cosines of the line. Thus the equations (2) of the line become
J L iii_________ JL -J'i __ ________
a 6 c
V(«*+ b*+c*) V{a*+fr+c*) V(u*+h*+c*T
x xi y - y i Z —Zt
or = _ _ ! = t (say).
a 6 ...(3)
The equations (3) arc the required equations of the straight
line. It should be noted here that ‘r ’ is not the actnal distance
of any point P(x, y, z) on the line from the given point (xlt yt, z,).
90 Analytical Geometry 3-Z)

From equations (2) and (3) we observe that the form of equa­
tions of the straight line remains unaltered if we use direction
ratios instead of direction cosines.
Corollary. From equations (I) and (3), the general co-ordi­
nates of a point on a line are given by
( X i + l r - i . X x + m r , Zi + n r ) or ( x t + a r 9 y i + b r 9 Z 1Jr Cr).
(c) The parametric form.
In the equations (I) and (3), r represents a real number which
changes a the position of the point P o n i h t line changes, so
that r is a parameter. For convenience let this parameter r be
denoted by Ί*. Hence parametric equations of the straight line
are given by
X--X1+ It, y= y i+ m t, z ^ + n t . (4)
or \= X !+ a t, y = y i+ b t, Z=Z1+ ^ ...(5)
where I9m, n and a, b, c are direction cosines and direction ratios
of the line respectively.
Note. If we use.the actual direction cosines then the co­
ordinates of a point (on the line) distant r from the given point
(xl9 J 1, Z1) and (xx+ l r 9Ji+ m r, zx+n r). From this point of view
the equations/l) or (2) are also called distance form of the equa­
tions of the straight line.
§ 3. Line through two points.
To f i n d th e e q u a tio n s of a s t r a i g h t lin e p a s s in g th r o u g h tw o
p o i n t s w h o s e c o - o r d in a te s a r e (.V1, y l9 Z1) a n d ( jc2> } r2t -2)·
Let A (vx, J 1, Z1) and B (x29yt9 z2) be the given points through
which the line passes. The direction ratios of the line passing
through the points-A and B are X9^ x l9 z2—*1-
Hence the required equations of the straight line are given
by [from equations (3), § 2 (B)]
x *1 _ y -y i Z-Z1
Xr-X1. y * - y * >·(!)
. ' . SOLVED EXAMPLES (A)
Ex I. Find the point in which the line —— —Z~y£
meets the plane x —2y+z=?20.
Sol. The equations of the line are
_ £ l~ 2= r {sav)
3~ 4 12 r y;* ...(I)
The equation of the plane is x —2^+ z= 20. ---(2)
The Straight Line 91

The co-ordinates of any point Q on the line (I) are


( 2 + 3 r , - l + 4r, 2+12r). # ...(3)
Suppose the line (I) meets the plane (2) in this point Q, hence
we have
( 2 + 3 r ) - 2 ( ~ l+ 4 r)+ (2 + 1 2 r)= 2 0 , or r= 2 .
Putting this value of r in the co-ordinates of Q given by (3)
the co-ordinates of the required point are given by
(2+3.2, - 1 + 4 .2 , 2+12.2) or (8 , 7, 26).
Ex. 2. Find the co-ordinates of the point where the line joining
the points (2, —3, I) and (3, —4, —5) cuts the plane 2 x + ^ + z = 7 .
Sol. The direction ratios of the line joining the points
(2, -3,-1) and (3, -4,. - 5 ) are 3 - 2 , - 4 - ( - 3 ) , - 5 -1 i.e.
I, - I , --6.
Hence the equations of the line joining the given points are
x —2__y+3_ z - 1 _ ,__ x ...(I)
■r, (say).
3 -I ~ -6
The co-ordinates of any point on this line are
( r + 2 , —r —3, - 6r + l) . - (2)
It this point lies on the given plane 2x-\-y+z—l . we have
2(/ + 2 ) + ( - r - 3 ) + ( - 6r + 1 )= 7 , or r = - I .
Putting this value of r in the co-ordinates of the point given
by (2), the co-ordinates of the required point are given by
(1 ,-2 ,7 ).
Ex. 3. Show that the distance, o f the point o f intersection o f
the line *3 ondthe plane x —j + z = 5 from thepoint
(--1, - 5 , - 1 0 ) is 13.
Sol. The equations of the given line ar
x —2 y + I z - 2 .
TT - - - W - r ^ yh ...(I)
The co-ordinates of any point on the line (I) are (3r+2,
4 r—I, 12r+2). If this point lies on the plane x - y + z = 5 , we
have 3 r+ 2 —(4/-,-1)+12/-+2=5, or Ilr= O , or r= 0 .
Putting this value of r, the co-ordinates of the point of inter­
section of the line (I) and the given plane are (2 , - I, 2).
.·. The required distance=distance between the points
(2, - I, 2) and ( - 1 , ^ 5 , —10)
= V { (2 + l)2+ ( —l + 5)H-(2+10)2}
” V (9+16 + 144)=y(169) = 13.
92 Analytical Geometry 3-D

Ex. 4. Find the points in which t ie line


=? ~ - cuts the surface I.Ix*-- 5y*4-z*,=0.
Soh The equations of the given line are
.v+l y - 12 z —7
= V - —r (say).
I “ 5 2 - ( 1)
The co-ordinates of any point on the line (I) arc
( r —I, 5r+12, 2r-f-7). .(2)
If this point lies on the given surface
I Ix2 -5 y s4-22= 0 , \ve have
II ( - r -I)* 5 (5r4-12)*+(2r+7)*=.0
or 11 (r*+ 2r4-1)-5 (25r*4- l20r+M 4)4-(4ra4-28r4-49)=0
or —I IOr*- 550r - 660=0, or r 2-f5 r4-6=0
or (r 4-2) (M 3 ) - 0 or 1 - - - 2 , ?.
Putting these values of r in (2), the required points of inter­
section are (I, 2, 3) and (2, —3, I).
Ex. 5. Find the equations o f the straight lines through the point
(a, b, c) which arc Ά) parallel to z-axis (i.e. perpendicular to the
xy-plane) and (H) perpendicular to the z-axis (i.e. parallel to the
xy-plane).
Sol. Let the equations of any line through the point (a, b, c)
, x —a y -b z - c
I m n --(I)
where /, m, n are the d.c.’s of the line.
(i) The d.c.’s of the z-axis (i.e. of the line perpendicular to
the .xp-plane are 0, 0, I. Hence if the line (I) is parallel to z-axis
-or perpendicular to the xy-plane then /, m, n arc proportional to
0, 0, I. Hence the equations of the required line are
x - a y - b z —c
H r-a sI T " I - *
(ii) If the line (I) is perpendicular to the z-axis or parallel
to the xy-plane then we have
/.0-f»».0-|-w.l=0 or n=0.
'I herefore in this case the equations (I). of the required line
are given by
x —o y —b z —c
x ~ m ~ 0
Ex. 6. Find the distance o f the point (I, —2, 3) from the plane
x —y-f z - 5 measured parallel to the line ~*τ=ζ-= — .·
2 3 —o
The Straight Line 93

Sol. Note here that we are not required to find the perpen­
dicular distance of the point (I, -r-2, 3) from the given plane, but
we are required to evaluate the distance of the point (I, —2, 3)
from the given plane measured parallel to a line whose direction
cosines are proportional to 2, 3, —6. For this we proceed as
follows :
The equations of the line through the point (I, —2, 3) and
parallel to the fine whose direction cosines are proportional to
2, 3, —6 are given by
x - 1 y + 2 _z —3
r (say).
T “ 3 “ —6
The co-ordinates of any point on it are (2 r+ l, 3 r - 2, 6/-+3).
If this point lies on the given plane x —y -\-z= 5, we have
2 r+ l -(3 r- 2 ) + ( - 6/-+3) = 5, or -7 r= —I, or r= l/7 .
\ The point of intersection is ~γ j
.·. The required distance=The distance between the points
(I, - 2, 3) and (9/7, —11/7, 15/7)

= 1/7 V (4 + 9 + 36) = l/7 .7 = l.


Ex. 6. Find the equations o f the straight lines which bisect the
angles between the lines x lli= ylm 1= z/nu xjl2= ylm t = zln2.
Sol. Proceeding as in Ex. 17 page 38 chapter 2, the direction
cosines of the bisectors are proportional to /i± /2, rn1± m 2^ nx± n t.
Clearly the given lines pass through the: point (0, 0, 0) and hence
their bisectors also pass through the point (0, 0, 0) and so the
required equations of the bisectors are
x y ______z
U±U ==in1± m 2:=nl± n 2 *
Ex. 8. Find the equations of the line through the point
(* 1, V1, Z1) at right angles το the lines1

I1 Iiil H1 It m. nt
Sol. Let the equations of the required line through the
point (x„ ylt r,) be
x -Χ ι v >i z - z \
I / ' τη ~ ~ n ’ -( 1 )
94 Analytical Geometry 3-D

where the d.r.’s Z, m, n of this line are to be determined. Since


line (I) is perpendicular to the given lines, hence we have
Ili +Wim1+ ...(2) and Il2+ mm2+ ηιι2—0. ...(3)
Solving the equations (9) and (3), we have
____I________ m n
Wi1Wg—Iii2Hlz^ n lL —/I8Z1“ IlOi2- I2Oi1
Putting these proportionate values of /, m, n in (I), the equa­
tions of the required line are given by
X -X i _ y - y i _ Z -Z i___
/W1ZI2 —/W1W1seW1Z1—W1Z1 Z1Wig—Z3Wi1
Ex. 9. Find the equation o f the plane through the point
(«, β , y) and (Z) perpendicular to the straight line
(*- X 1VZ=0 ' - .yO/iw= (z - Z1)//!,
(H) parallel to the lines x lh = y Im1= z/nt and x//g=v/w?j=z/W|.
Sol. Let the equation of any plane through the point
(«, β, γ ) be A (x-v.) + B (y —fi)+C ( z - y ) = 0 . ...(I)
(i) The equations of the given line are
(x —Xi)!l=(y—yi)lm—(z -Zi)/w. .. .(2)
If the plane (I) is perpendicular to the line (2), then the
normal to the plane (I) is parallel to the line (2); and hence Ai
B9 C9 the d.r.’s of the normal to the plane (I), are proportional
to I9 m9 w. So the equation of the required plane is given by
I (x -a)+ w i (y—β)+ η (z—y)=0.
(ii) The equations of the two lines are given as
x/li=*yImi=sZlni and XjU^yImi -=Zjnt .
Theplane (I) will be parallel to these lines, if its normal
whose d.r.’s are A9 B9 C is perpendicular to both the given lines.
Hence we have Al1+Bm1+Cn1= 0 and AU+Bmt +Cnt = 0.
Solving these equations, we have
A_________B________ B
/W1Wjl - WfW1“ W1Za-WJ lT U m t -Umi
Putting these proportionate values of Ai B9C in (I), the
equation of the required plane is given by
(W1Wa -WiaW1) ( x - a ) + (W1Zj -W2Z1) ( y - - β ) + (1ρη2 - Ipn1) (z - y ) = 0 .
Some Examples on the foot of perpendicular from a point to a
plane.
Ex. 10. Find the co-ordinates o f the foot o f the perpendicular
drawn from the origin to the plane 3 x+ 4 y—6 z+ 1=0. Find also
the co-ordinates of the point on the line which is at the same distance
from the foot o f the perpendicular as the origin is.
The Straight Line 95

Sol. The equation of the plane is 3 x 4 - 4 ^ —6 2 4 - 1 = 0 ; ;..(!)


The direction ratios of the normal to 'he plane (I) are
3,4, —6. Hence the line normal to the plane (I) has d r .’s
3. 4, —6, so that the equations of the line through (0, 0, 0) and
perpendicular to the plane ( I) are
Xft=JrH=Zl- 6 = r (say). ...(2)
The co-ordinates of any point P on (2) are
( 3 r ,4 r ,- 6 r ) . -(3 )
If this point lies on the plane (I), then
3 (3/-)4-4 (4/-)-6 ( —6r)+l*=0, or r — —1/61.
Putting the value of r in (3), the co-ordinates of the foot of
the perpendicular P are (—3/61, - 4/61, 6/61).
Now let Q be the point on the line JQ
which is at the same distance from the
foot of the perpendicular as the origin.
Let (*,, y u Z1) be the co-ordinates of the
point Q. Clearly P is the middle point p I
of OQ. Hence we haVe
* i+ 0 3 ^,4-0 4 Zi 4-0 6 O
2 “ '6 1 ’ 2 — 6 1’ 2 “ 61
or JC1= —6/61, ^1= —8/61, 2 ,= 12/61.
.·. The co-ordinates of Q are ( —6/61, 8/61, 12/61).
Ex. 11. Show thatif the axes are rectangular, the equations
to the perpendicular from the point (*, β, ;>) to the plane
ax+ by+ cz+ d= 0 are ( x - u )/a= (v - 3 ) /6 = ( 2 - y)/c.
Deduce the perpendicular distance of the point (a, β, γ) from the
plane. Find also the co-ordinates of the foo t o f the perpendicular.
Sol. The equation of the plane is
ax 4 -6 r4 -« + i/~ 0 . -(1)
The direction ratios of the normal to the plane (I) are a, h, c.
Ifthelineisperpend icu larto the plane (I), then it is parallel to
the normal to the plane, Hence the d.r.’s of the line perpendi­
cular to the plane (I) are a, b, c. Also it passes through the
point (ot, β, y), therefore the required equations of the perpendi­
cular afe
x —x y —β z —y
~ r (say).
a - 6 c - ( 2)
The co-ordinates of any point on the line (2) are
(hr4-a, br+β, cr+y). -(3 )
96 Analytical Geometry 3-D

If this point lies on the plane (I), we have


a (ar+a)+/> (6r+/?)+c (cr+y)-|-</=0
or r (ai + bi A-ci)=~(a*+b$+<ry+d)
or r = —(av.-\-a/i-\-cy - r/)/(a2+6*+c*). - -(4)
Putting this value of r from (4) in (3), we get the co-ordinates
of the foot of the perpendicular.
Now the perpendicular distance of the point (α, β, γ ) from
the plane (l)= th e distance between the points (a, β, y) and
(ar+a, br+β, cr+ γ )
= V f ier + * ' a ) * + { b r + f i -/?)*+(cr+)—}’)*]
^ry/W + b t+ c* )
(ax+bfi+cy+d) .« .. __ax+bfi+cy+d_
-- («·+*»+*·) v ( a + 0 + c ^“ V(e*+6*+c»)
Ex 12. Findtheequations of the line through (I, —I, 2) per­
pendicular to the plane 3.v+5_y—Az= 5 and deduce the length o f the
perpendicular from (1, 1,2) upon the plane and also the co-ordi­
nated o f the foot of the perpendicular.
Sol. Proceeding as in Ex. 11 above, we get
(i) the equations of the required line as
_—_I
X =y T
+ 1 = z_ _2 =r(say)
. .
(I)
(ii) The co-ordinates of any point on the line (I) are
( 3 r + l,5 r —I , —4r+2). --r(2)
If this point lies on the plane 3x+5j» -Az=S, we have
5 (3 r+ l)+ 5 (5r—1 ) - 4 ( -4 r+ 2 )= 5 , or r=3/10.
Putting the. value of r in (2), the co-ordinates of the foot of
the perpendicular are (19/10, I /2, 4/5).
(iii) The required distance

=V[9/10)a+(3/2)2+(6/5)2J=3V-/2.
Ex. 13. Find the incentre o f the tetrahedron formed by the
planes Jt=Oi v=0, z= 0 and xA -v+ z= a.
Sol. The three planes a*=0, v=0 and z= 0 meet in the
point (0,.0, 0). Hence the incentre of the tetrahedron lies on the
perpendicular from (0, 0, 0) to the plane .v+^rj-z—r?.
The d.r.’s of the normal to the plane a* + > 4 z - a are I, I, I.
Hence the equations of the perpendicular from (0, 0, 0) to the
plane x + y + z = a aie
The Straight Line 97

f-f-f-rM r). ...(I)


The co-ordinates of any point on (I) are (r, rf r).
Ifthis point lies on the plane x+ y+ z= a9 we have
r+ r+ r= a 9 or r ^ { a .
Hence the foot of the perpendicular is [or the perpendicular
from (0,0, 0) meets the plane x + y + z ^ a in] (Jfl, Jo, Jo).
Let the incentre of the tetrahedron be (JC1, y l9 Zj). Now we
know that the incentre divides the jGin of (0, O90) and ( K Ia9 la)
in the ratio 3 : I [3 on vertex side and I on plane side]. Hence we
3. J n + 1 .0
have
3+1 = Λ β *i or xI ^ y i z=zI=
Λ The co-ordinates of required incentre are (Jo, Ja, Jo).
Ex. 14. A variable plane makes intercepts on the co-ordinate
axes the sum o f whose squares is constant and equal to k2. Show
that the locus o f the foot o f the perpendicular from the origin to the
plane is {x~2+y~2+z~2) (X2-LjpHz2)= * 2.
Sol. Let the equation of the variable plane be
x Ia+y Ib+ z/c= I --0 )
where a9 b9 c are its intercepts on the co-ordinate axes, so that
fl*+**+c*=Jk*. -(2 )
The d.r.’s of the normal to the plane (I) are Ifa9 l/be 1/c.
Hence the equations of the line perpendicular to the plane (I)
■[i.e. parallel'to the normal to the plane] and passing through the
x - 0 y —0 z - 0 _
origin are -r (say).
Ma - l / h ~ I /e .(3 )
Any point on line (3V is (r/tf, r/fc, r;c). ...(4)
If this point lies on the plane (I), we have
r/o*+r/4*+r/c*=l or r ~ - s+ f r i + c _,·
Let (x, y 9 z) be the co-ordinates of the foot of the perpendi­
cular, then putting the value of r in (4), we get
X=
I
a (a-^+br2+ ^* ) ΟΓ X <γ Ή £ ”2+ ^ “2
Sinr11Jarly and
e -*+ 6 -*4 er* I
Now jr*+y*+z*=
(o-*+Ar-*+c-*)*— a-*+A-*+c~* ..(5)
and jr*+.v-*+r-*=(a-*+ir*+c-*)* (o*+fc*+c*)
= (e-* -J.^ + ( T s)tJfc*, using (2). ...(6)
98 Analytical Geometry 3-D

Multiplying (S) and (6), we have


( * * + * * + 2 *)* ( jc- * + y - * + z - * ) = * *
Iliis is the equation of the required locus.
Ex. 15. Fbtd the equations o f the line through the points
(a, h, c) and (a \ I ', o') and prove that it passes through the origin if
aaf -\-bbf -\-ccf= rr’, where r and r’ are the distances c f the point
from the origin.
Sal. The equations of the line through the points (a, b, c)
and (e', V , c') are
x —a y —a z —c
a '- a ~ b f - b ~ c ' - c ••(I)
The line (I) passes through the origin, hence we have
Ο- a 0 —b 0—c d -a V -b d -c
af—a ~ b f—b’~ c, —c ΟΓ -b —c
or e' , , b’ , . Cf , , o' bf cf
a b c a b c
From these relations, we-immediately get
ab' a'b= 0, be’- V c = 0, ca’-c 'a = 0 . ••(21
By Lagrange’s indentity, we have
(e*+h*+c*) (a'»+h'»+c',)-(ao'+A&'+cc')*
= (Obf-O fb)*+(bc' Vc)*+ (COf -C fO f
= 0+ 0+0
or (tP+V+c*) (a'i + b'i +c'r)=(aaf+bb’+cc’Y
Now r= th e distance of (a, by c) from the origin
= V K e - 0 ^ + ( ^ - 0 ) 2+ ^ - 0 ) ^ = ^ ( ° * + ^ + 45*)-
Similarly r'«V (e* * + h '* + c'i).
Putting these values in (3), we have
(ae'+AA'+cc'), =(rr')* or aV+bbf+ccf= rrf.
This is the required result.
Ex. 16. Find the htioge o f the point (I, 3, 4) indie plane
2x y + z + 3= 0.
Sul. The equation of the plane is
2*—j + r + 3 = 0 . ...(I)
Let (I, 3,4) be the point P. Draw a perpendicular PN from
the point P to the plane (I). T akea point Q on this perpendi­
cular on the otherside of the plane (I). T h en Q is called the
image of P if AT, the foot of the perpendicular, is the middle point
o f JfQ. Let the co-ordinates of Q be ( jc, , j , . r,).
The Straight Line 99

The d.r.’s of the normal to the plane (I) are 2, —I, I. Hence
the equations of the line through (I, 3, 4) and perpendicular to
the plane (I) are
jc-1 y - 3 2 —4
= r (say).
2 “ -1 ~T~
Any point on (2) is (2 r+ l, —r+ 3 , r+ 4). If this point is the
foot of the perpendicular i.e. the point N, then it will lie on the
plane (I) and we have
2 ( 2 r + l ) - ( —r + 3 ) + (r+ 4 )+ 3 = 0 ,
or 6 r+ 6 = 0 , or r— — I.
Putting this value of r, the co-ordinates of N are (—1,4, 3).
Now as explained above, N is the middle point of PQ. Hence
we have - I . .1+*! Λ_3+Λ 3 4+2i
2 ’ 2
or X i - —3, = 5 , Z1=2·
The image of P (I, 3, 4) is the point Q ( —3, 5, 2).
§ 4. To transform the general form of the equations of a straight
line to symmetrical form.
Let the general form of the equations of the straight line be
given by the equations
« iJ + ^ + C iZ + ^ O l
, eiX+A*y+CjZ+</t =0) .---(I)
Now we are required to write down the symmetrical form of
the straight line given by equations (I). For this we must know
(i) the direction cosines or direction ratios of the line and (ii) the
co-ordinates of a point on the line. To find these two we proceed
as follows :
Step I. To find direction cosines or the direction ratios o f the
line given by equations (I). Let /, m, n be the direction cosines or
direction ratios of the line. Since the line is common to both the
planes and therefore it is perpendicular to the normals of both the
planes. The direction ratios of the normals to the planes given
by equations (I) are au blt ct and at,'b2, c, respectively. Hence
we have
lax+m bi+nc^O and lat+mbt+nct = 0.
Solving these equations for /, m, n, we have
I _____m________ n
b\Cj b^ci C1U1—Cjflj a%bf—ttfii
100 Analytical Geometry 3-D

Λ the direction ratios of the line are


C1Q2—CeCij, a A —<ιΦι· ·*·(2)
Step 2. To find the co-ordinates of a point on the line given by
equations (I). The co-ordinates of a point cn a line can be chosen
in many ways. One of these ways is that*we choose the point as
the one where the line cuts the xy-plane (i.e. z= 0 plane), provi­
ded the line is not parallel to the plane z= 0 i.e. provided
Uib2-O fil^zO.
Putting z= 0 in both the equations given by (I), we get
01^ + ^ + ^ 1= 0* a2x+ b 9y+ d 2=^0.
Solving these equations for x , y , we get
x________ y_________I
b\d2—b\dy d^a2—d2a^ afb2-~ ajbi
Hence the co-ordinates of a point on the line (I), where it
cuts the z= 0 plane are
Ibid2 b2di dia2—d2Oi
O ib2 - O j b l ’V -(3)
Hence the equations of the line in symmetrical form are
-Ύ ( bjd%—b2di\ / dIa2-^d2Pi \
\ α Α —αφι / J \α φ 2—αφι ) z -0
bie2- ~b2Ci ^iO2 ^2Oi οφ2 o2bi
Note. If o A - a A —Q, then instead of taking z= 0 we should
take the point where the line cuts x= 0 plane or y —Q plane.
SOLVED EXAMPLES (B)
Ex. I. Find in symmetrical f o r m the equations of the line
3 x+ 2 y- z —4 = 0 = 4 x + y —2z+3
andfind its direction cosines.
Sol. The equations of the given line in general form are
3x+ 2y—z - 4 = 0, 4 x + y -2 z -f 3 = 0. ...(I)
Let I9 m, n be the d.c.’s of the line. Since the line is common
to both the planes, it is perpendicular to the normals to both the
planes.
Hence we have
3 /+ 2 m —w= 0, 4 l + m ~ 2 λ= 0.
Solving th.se, we get
I m n I
- 3 _ 2 ~ - 5 _ V (9+ 4+25) _ V W
Λ the d.c.’s of the tine are -3/v/(38), 2/V(38), -5 /^ (3 8 ).
Now to find the co-ordinates of a point on the line given by
The Straight Line IOl

(I), let us find the point where it meets the plane 2 = 0. Putting
2=0 in the equations given by (I), we have
3x+ 2y—4=0, 4x-f_v+3=0.
Solving these, we get
x v I - -
6-f 4 ~ —16—9 ~ 3 —8’ ° Γ 2 ,y '~S·
Λ The line meets the plane z= 0 in the point (--2, 5. 0).
Therefore the equations of the given line in symmetrical form
are
jc+2 y - 5 z - 0
--3 “ 2 “ - 5 *
Ex. 2. Find the equations to the line through the point (I, 2, 3)
parallel to the line x —y + 2 z —5=0; 3jc+ ^ + z —6=0.
[Jodhpur 1967]
Sol. The equations of the given line in general form are
λ - y + 2 z --5 = 0 , 3 x+ y+ z -^==0. ...(I)
Let /, m, n be the d.c.’s of this line. Then we have
I -m+2n=Q, 3/+ m + /i= 0.
Solving these, we have
I m n I m n
I + ΟΓ ~ 3 ==5~= ~4‘
Since the required line is parallel to the line (I), the d.c.’s of
the required line are proportional to /, m, n i.e. —3, 5, 4.
Hence the equations of required line are given by
( x - 1 ) / -3 = 0 --2 )/5 = (2 -3 )/4 .
Ex. 3. Find the equations. o f the line through the point
(Xi, y lt Zi) and parallel to the line
OiX+hj^+C^+rf!=0, <7ax--f 6tv+c2z+</*—0.
Sol. Let /, m, n be the d.c.’s of the given line, then procee­
ding as in § 4, we get
I _____ m n
Vl ClOt —Cj0| ...(I)
The required line passes through (X1, ylt Z1) and its d.c.’s are
proportional to /, m, n given by (I). Hence the equations of the
required line are given by
X-X1 _ y -y t, _ Z -Z 1
A1Ca- A aC1 C1Aj -Cjfl1 O1Aa- U2U1
Ex. 4. Find in symmetrical form the equations o f the line
x= ay+ b, z=cy+ d.
102 Analytical Geometry 3-D

Sol. The equations of the given line may be written as


x —ay+ 0.z—b—0, 0.x-\-cy—z+ d = 0 . ...(I)
Let I, m, n be the d.c.’s of the line. Then we have
1.1 - a .m + 0./1= 0, O.Z+c.m—l.n = 0 .
. . , 1 m n
.. I= an, cm*=n, so that·—= — = — . ,- ,
a .l c ...(2)
Now let us find the point where the line (I) meets the plane
y = 0 . Puttingy=O in the equations (I), we get x= b and z= d.
.*. The line meets the plane y = 0 in the point (b, 0, d).
Therefore the equations of the given line in symmetrical form are
( x -b )la = (y —0 )/l= (z —d)/c.
Alternative method. The given equations of the line may be
written as
x —b=ay, z —d=cy
x —b y z —d y
or
a . I ’ c I '
x —b _ y _ z —d.
•· a I c '
These are the required equations of the line in symmetrical
form.
Ex. 5. Show that the lines
x= ay+ b, z=cy-\-d and x= a'y+ b', z= c'y+ d '
are perpendicular if acf+cc’+ 1=0.
Sol. Proceeding asin Ex. 4 above, if the d.c.’s of the line
x= ay+ b, z= cy+ d are I1, mlt nly we have
IJat= m j I =U1Ic'. ...(2)'
Similarly if lt, mt, n2 are the d.c.’s of the line x= a ’y+ b ',
z= c'y+ d', then we have
lt/a'=mt/l= n ale.'. ...(2)
The given lines will be perpendicular, if
Z1Z1-I-Wi1Wt +/J1Zt4=O
i.e. a a '+ l.l+ c c '= 0 , or aa'+cc’+ 1=0.
§ 5. The plane and the straight line :
To find the co-ordinate o f the point o f intersection o f a given
line and a given plane and to deduce the conditions th a t:
(Z) the line may be parallel to the plane,
(U) the line may be perpendicular to the plane, and
(Hi) the line may be lying in the plane
The Straight Line UB
Let the equations o f the given Hne in symmetrical form be
X-X1 y - y x
Z
- ~ = r (say)
I m a -(»)
and the equation of the given plane be
a x + b y + e z + d = 0. ...(?)
The co-ordinates of any point P on the line (<) are
(xi+lr, mr, zx+mr).
If this point P lies on the plane (2), we have
<4xi+lr)+Hyi+i"r)+c (*ι+ «τ)+ < /= 0
or r(al+b.H-\en)=—(ax1+by1+ c zt-i-d)
or r= - (Ox1-L by^ cz1+ d)j(al+ bm+cn). ...(3)
Thus the co-ordinates of the point of intersection P of the
line (I) and the plane (2) are (X1+ /r, A1+mr, Zt +nr) where r is
givn by (3).
(i) Theeaaiitieas of paralelism. If the line (I) is parallel
to the plane (2), then this line must be perpendicular to the nor­
mal to the plane (2) and hence we have a + b m + e * = 0.
Again the point (X 1,^ , Z1) should not lie on the plane i.e. we
must have Ox1-^byl + czt -\-dy^0, for otherwise the line (I) will not
be simply parallel to the plane (2) but it will lie in the plane (2).
Hence the required conditions that the line (I) is parallel to
the plane (2) are
al+ b ai+ ca= 0, aX i+by^czt+d^O .
(Ii) The condition of perpeodicalarity. If the tine (I) is
perpendicular to the plane (2), then the line (I) must be parallel
to the normal of the plane (2) and hence the required condition
of perpendicularity is given by
a b c
I ■ a'
(ili) The conditions that the line a t ; lie ia the plane. If the
line (I) lies in the plane (2), then for all values of r the point
P(Xi+/r, y t +m r, z,+ «r) will lie on the plane (2) ijt.
a(x1+ /r )+ i(y 1+m r)+c(z1+iir)<f=0
or r(ai+ftm +cn)+(ex1+by1+cr1+-d)=*0
should be true for all values of r, and hence we must have
al+bm +cit= 0 and a x i+ b y ^ c z ^ d —O.
Hence the required conditions that the line (I) lies in the
plane (2) are
104 Analytical Geometry 3-D

al+ b m + cn = 0 , ax1-?-by1+ cz14 4 = 0


These conditions can also he deduced as follows :
Ifth e lin e (I) liesin the plane (2). then the line is perpen­
dicular to the normal to the plane (2), for which we have
al+bm +cn=0.
Again the point Z1) must lie on the plane (2) [since the
line (I) is passing through the point (jr„ yt , Z1,)]. hence we have
Oxl-^b yl +Czl+ (/=-0.
SOLVED EXAMPLES (C)
Ex. I. Find the equation cf the plane through the line
P = ax+ by+ c:-\ d = 0, Q= a‘x + b ’y+ c’z + d '= 0
and parallel to the line xjl= y-m = zin.
Sol. The equation of any plane through the line F= 0, Q=O
i.t . through the line of inlerycotion of the planes F= 0 and Q=O
is P=XQ=O ...(I)
or (o x + b y + cr+ J)+ A (:/.r+ b 'j+ c 'z + < /')= 0
or [a+Xa') x + (b + W ) y+ic+Xc’)z+ d+ X d'= 0. ...(2)
The d.c.’s of the normal to the plane (2) are proportional
to a+Xa’,b+Xb',c+Xc'. The plane (2) [or (1)] wilt be parallel to
the line x/Z=y/m=z/« if the normal is perpendicular to the line
xjl= ylm = zjn, hence we have
(α+Αβ') l+(b+Xb’) m+ic+Xc'j n= 0
or X{a’l+ b'm + c'n)— (al+bni+cn)
or X= —(al+bm+cn)/(a'l+b’m+c'n).
Putting this value of A in (I), the required equation of the
plane is given by
P —{(al+bm+cn.i:a'l+b'm+c'n)} Q= 0
or P (a'l+b'm+c'r.) - Q (al+bm+cn).
Ex. 2. Find the equation i / the plane through the line
3x—4y+ 5z«10, 2x+ 2y—3z=4
and parallel to the line x= 2y= 3z.
Sol. The equations of the given line are
3jc—4y+5z=10, 2 x + 2 y -3 z= 4 . ...(I)
The equation of any plane through the line (I) is
(3 x -4 y + 5 z -lU )+ A (2*+2y 3 ;- 4 ) = 0
or (3+11) x + ( —4+2A) y i-(5-3A) z - 1 0 - 4A=0. ..-(2)
The plane (I) will be parallel to the line
x= 2y= 3z i.e. if
(3+ 2λ ).6 + 1 -4 τ2 λ ).3 + (5 -3 Α ).2 = 0
The Stndght Line 105

or A(1 2 + 6 6)+18 12+10=0, or A= - *.


Pultingthisvalue of A in (2), the required equation of the
plane is giveu by
(3 H ) x + ( —4 -4) y-1-(5+4) z - 1 0 + V = 0,
or x —20y+27z=14.
Ex. 3. Find the direction cosines o f the line whose equations are
x + y = 3 ami x + y + z = 0 and show that it makes an angle o f "Hf
with the plane y —z + 2=0.
Sol. The equations of the line are
x + y =3, x-+y+::-=0. ...(I)
Let /, m, n be the d.c.’s of the line (I), then we have
/+ m + 0 .n = 0 , /+ m + n = 0 .
Solving, we get
I m n y/(l*+m*i-n2) I
T “ ^ r =17=V(UTTF^Toi)=V2
·. l= lJy/2, W= -1 /V 2 , n=0.
The equation of the plane is y —z + 2=0. ... (2)
The d.r.’s of the normal to the plane (2) are 0, I, —1, i.e.
direction cosines are 0, l/y/2, -1/-^2.
Now suppose Θ is the acute angle between the line (I) and the
normal to the plane (2).
Then using the formula cos O=IJi + WJ1W,+/I1M1, we have
I \_
cos 0=
V 2*
.*. 0=60°.
N ow thcanglebctw ccnthc Iinc(I) and the plane (2) is the
complement of the angle $. So the required angle between the
line (I) and the plane (2) is 90°—0 i.e., 90° —60° i.e., 30°.
Ex. 4. Find the equation of the plane through the points
(2, - 1 ,0 ) ,( 3 , - 4 ,5 ) ;
and parallel to the line 3x--2y= z.
SoL The equation of any plane through the point (2, —1, 0)
is a ( x — 2)+ 6 0 ' + l)+ c ( z — 0)=0. ...(I)
if the plane (I) passes through the point (3, —4, 5), we get
a (3—2 )+ 6 ( 4 + 1)+ c (5)=0, or a ~ 36+5c=0. ...(2)
X y
The equations of the line are 3 x = 2 y = r, or = -j-
6‘
-(3 )
Thc plane (I) will be parallel to the line (3), if
2o+36+6c=0. -(4 )
106 Analytical Geometry 3-D

b
Solving (2) and (4), we get
- 3 3 —4 ~ 9 '
Putting these proportionate values of a, b, c in (I), the requi­
red equation of the plane is given by
—33 (x—2)+ 4 O '+ 1)+ 9 (z—0)= 0, or 33x+4y—9z -70= 0.
Ex. 5. Find the equation o f the plane through (2, I, 4) perpen­
dicular to the line o f intersection o f the planes
3x+4y+Tz+4=Q and x —^ + 2 z + 3 = 0
Sol. Let /, m, n be the d.c.’s of the line of intersection of the
two planes 3 x + 4 y + 7 z+ 4 = 0 , x —^+2z+3*»0.
Then we have 3/+ 4 ot+ 7 b = 0, /- m + 2 n = 0 .
m I m n
Solving,
8+ 7 “ 7 - 6 ” - 3 - 4 ’ ΟΓ 15~ I ~ —T
Thus d.r.’s of the normal to the required plane are 15, I, - 7 .
Alsothe required plane is to pass through the point (2 ,1 ,4 ).
Hence its equation is
1 5 ( * - 2 ) + l O'—1 )-7 (z—4)=0
or 1 5 ^ + ^ -7 2 - 3 = 0 .
Ex. 6. Prove that the lines 3jc+2y+z-5=0«=»x+<y—2z—3
and 2 x - y —z —0 = 7 x + IOjp - 8z—15 are perpendicular.
Sol. Let I19 nii* be the d.c.’s of the first line. Then
3 ^+ 2 //^+ /^= 0 , Ιχ+ηΐι- 2/^=0. Solving, we get
I1 OT1 W1 _ I1 OT1 B1
- 4 - l “ l + 6 — 3 —2 - 5 " 7 “ I '
Again let /„ mt, nt be the d.c.'s of the secon I line, then
2/*—ot»—b*= 0, 7/t + IOot1- 8nt =0.
C l- h OT2 B2 . Z1 Bl1 B1
Solving, 8+ 10- - _ 7 + l6 - 2o + 7 ΟΓ 2 “ 1 “ 3 ’
Hence the d.c.’s of the two given lines are proportional to
—5, 7, I and 2, I, 3. We have
-5 .2 + 7 ,1 + 1.3=0
.'. the given lines are perpendicular.
§6. Tofind the equation o f the plane through a given Une
whose equations are given in (i) general form and (ii) symmetrical
form.
(i) Let the equations of the line in general form be given by
P s a 1x+ b1y+ c1z+ d1=0, Q =a2x+ bty+ ctz+ dt =0: ...(I)
Then P+XQ=Q Le.,
a ^+ b rf+ C iZ + d ^X (atx+ b iy+ c1z+ di)=Q
is an equation of first degree in x, y and z and hence represents a
The Straight Line 107

plane. Also P+XQ=0 is satisfied by all those points which satisfy


(I). Hence the equation of the required plane is given by
P-I-AQ=O.
(ii) Let the equations of the line in symmetrical form be
X -X 1 y-yx Z-Z1
I m n ...(2)
The required plane passes through the line (2) and hence it
passes through the point (X1, y lt Z1) which is a point on the
line (2).
The equation of any plane through the point (X1, yx, Z1) is
given by
a (x -x O + b -iy —^ + C ( Z - Z 1)=O. ...(3)
Again, the required plane passes through the line (2), hence
the normal to the plane (3) is perpendicular to the line (2).
Λ al+bm+cn=0.
Therefore, the required equation of the plane is
a (X -X 1)-Hb (y —yO+c (Z-Z1)=O, where al+bm +cn=0.
§ 7. Tofind the equation o f the plane through a given line and
parallel to another line.
Let the equations of the given line be
( x - X i) / li= ( j> - JP1Vwi1= ( Z - Z 1)In v .. ( I )
1S
Proceeding as in § (ii), the equation of any plane through
the line (I) is
a (X-X1)+ b ( y - y j + c (Z-Z1)=O, ...(2)
where al1+bm1+cnl ^=0. ...(3)
Again let the plane (2) be parallel to another given line whose
equations are given by
(x - x»)//*= (y - J’i)/m ,=(z - z2)/wt. ... (4)
Since the plane (2) is parallel to the line (4), the normal to
the plane (2) will be perpendicular to the line (4) and so we have
aJt +bmt+cnt = 0. ...(5)
Eliminating a, b, c between the equations (2), (3), (5), the
required equation of the plane is given by
X -X 1 y - y t Z -Z 1i
i
I1 M1 J=O
W1
I
It mt wt I
or (X-X1) (/W1Wj-WJtW1)+ (y .P1) (W1Z1-WtZ1)
+ (Z -Z 1) (Z1Wit -Z 1Wi1)=O.
108 Analytical Geometry 3-D

SOLVED EXAMPLES (D)


Ex. I. Find the equations of the planes through the line
(x—2)/2“ O'- 3)/3=(z—4)/5, are parallel to the co-ordinate axes.
Sol. The equations of the given line are
(* - 2 ) /2 = O'—3)/3= (z - 4)/ 5 ...(I)
The.equation of any plane through the line (I) [see § 6] is
a (x --2)+* (y -3 )+ c (z -4 )= 0 .. (2)
where 2a+36 K v = O . ...(3)
We are required to find the equations of the planes parallel to
x-axis, j’-axis and z-axis respectively.
(i) The d.c.’s of x-axis are I, Ov0. If the plane (2) is parallel
to the x-axis, then the normal to the plane (2) is perpendicular to
the x-axis i.e., we have
a. 1 + 6 .0 + c.0=0. ...(4)-
Solving (3) and (4), we have
a b e _a b c
H K - O = 0 —3 or O' = T = ~ 3 '
Putting these proportionate values ol a, b, c in (2). the requi­
red equation oF the plane is given by
0.(.*-2)+ 5 (.V—3 ) - 3 .(z—4)=0,
or S y - 3z—3= 0.
(ii) The d.c.’s of the ^-axis are 0, 1,0, if the plane (2) is
parallel to the .y-axis, we have
a.Q+b.l +C.0e=r0. ...(5)
Solving (3) and (5).‘we get
a b c ^ u b c
0 - 5 = 0" = 2 - 0 ΟΓ - K = O ^' 2
Putting these proportionate values in (2), the required equa­
tion of the plane is given by
- 5 (x 2 )+ 0 .(^ -3 )+ 2 (z—4)=0, or 5 x - 2 z - 2 = 0 .
(iii) The d.c.’s of the z-axis are 0, 0, !. If the plane (2) is
parallel to the z-axis, we have
a .0 + 6 .0 + c.l= 0 . ...(6)
Solving (3) and (6), we get ~
Putting these proportionate values of a, 6, c in (2), the requi­
red equation of the plane is given by
• 3 (x—2) - 2 O ' — 3)=0, or 3x—2y=0.
Ex. 2. Prove that the equation o f the plane through the line
(x—l)/3«=OH-t>)/4=(z+l)/2 and parallel to (x—2)/2=(y—1 )/-3
The Straight Line 109

= (z 4-4)/5 is 25jc-—I \ y ~ \ l z ~ 109=0 and show that the point


(2, I, —4) lies on it.
Sol. The equations of the given line are
(* -1> /3 = 0 4 -6 )/4 = ( z 4-1)/2. ...(I)
The equation of any plane through the line (I) is
a (jc- 1)4-6 Cv4-6)4-c (^4-1)=0 .v(2)
where 3a4-46+2c=0. ...(3)
The plane (2) is to be parallel to the line
(jc-2 )/ 2 = (. v- 1)/(- 3 ) = (z 4-4)/5. . ...(4)
Hence the normal to the plane (2) is perpendicular to the line
(4), so that we have
2a—3/>4-5c= 0. ...(5)
Solving (3) and (5), we get
a b c a b c
204-6 “ 4 15 = 9 8 ° Γ ^ ^ Ί Τ ^ - Π
Putting these proportionate values of a, b, c in the equation
(2), the required equation of the plane is
26 ( x - 1 ) - 1 1 (^ 4-6) -17 (z4 -l)= 0
or 26* —I l y - 17z —119—0.
Substituting the point (2, I, —4) in the equation (6) of the
plane, we get 2 6 x 2 —Il x l - 1 7 x —4—119=0, or 0 = 0 Le. the
point (2, I, —4) satisfies the equation (6) of the plane.
Remark. In the above Ex. 2, the point (2, 1 , - 4 ) lies on the
line (4) and also on the plane (6). Hence the line (4) will wholly
lie on the plane (6).
Therefore both the lines(l) and (4) are coplanar anu me
equation (6) gives the plane containing both of them.
Ex. 3. Find the equation of the plane which contains the two
parallel lines
* 4-.1 .V-2 z , Jf-3 jM-4 z—l
3 “ Ύ ~T 3 “ 2 ” I ·
Sol. The equations of the two parallel lines are
(jc4-1)/3=(.y—2)/2=(z—0)/1 ...Cl)
and (jc— 3 )/3 = (^ 4 -4 )/2 = 0 - 1)/1. ·*·(2)
The equation of any plane through the line (I) is
a ( x + l ) + b \ y ~ 2 )+ c r= 0 , ...(3)
where 3a+2b+c—0. *··(4)
The line (2) will also lie on the plan? (3) if the point (3, —4,
I) lying on the line (2) also lies on the plane (3), and for this we
HO Analytical Geometry 3-D

have a (3+l)+/> (—4 —2 )+ c .l= 0 or 4o—66+c« »0. .(5 )


Solving (4) and (5), we get y = - y = ^ g - .
Putting these proportionate values of a, b, c in (3), the requi­
red equation of the plane is
8 (x + 1 )+ 1 .(y -2 )-2 6 ::= 0 , or 8 x + .y -2 6 z+ 6 = 0 .
Ex. 4. Find the equation o f the plane which contains the line
(y—3 ) = | (z—5) and which is perpendicular to the plane
2je+7.y—3z—I .
Sol. The equations of the given line are
(* -0 )/l= 0 > —3 )/2 ~ (z-5 )/3 . ...(I)
The equation Of any plane through the line (I) is
a (x)+ b (}>- 3 )+ c (z—5)= 0 ...(2)
where I e+2.fc+ 3.c=U. ...(3)
-Also the plane (2) will be perpendicular to the plane
2x+ 7y—3 z = l
if 2 .fl+ 7 .t-3 .c = 0 . ...(4)
Solving (3) and (4), we get
a b c _a^ - J f JL
6^T 2“ 6+3™ 7--4 ΟΓ ~ 9 ^ T = T -
Putting thesr- proportionate values of a, b, c in (2), the requi­
red equation of the plane is
—9x+3 O'—3)+ (z—5)=0, or 9x—3;y -z+ 1 4 = 0 .
Ex. 5. Showthntthe equation tolhe plane containing the line
=2—^ and the point ,0, 7, --7) is x + y + z = 0. Hence

show that the line a^so ^ ex in t^ie same P^ane·


Soi. The equations of the given line are
( x + 1 ) / - 3 = 0 1 3)/2= 0 + 2 )/1 . ...(I)
The equation of any plane through the line (I) is
a (x + l)+ * i^ -3 )+ c (z + 2 )= 0 ..(2 )
where —3 a + 2 £ + l.c = 0 . ...(3)
The plane (2) is also to pass through the point (0, 7, 7).
Λ α+4* 5r=0. ...(4)
Solving (3) and (4), we get
a b c a b c
, - 1 0 - 4 “ 1—15e —12—2 ° r T = T = T-
Putting these proportionate values of a, b, c in (2), the requi­
red equation of the plane is
The Straighi Lhe 111
l.(jc + l) + l.( y - 3 ) + l.( z + 2 ) = 0 or x + y + r = 0 . ...(5)
The equations of the second line are given to be
JC y —7 z + 7
...(6)
The line (6) passes through the point (0, 7, —7) and this
point also lies on the plane (5). Now the line (6) will lie in the
plane (5) if the normal to the plane (5) [whose d.r.’s are I, I, 1]
is perpendicular to the line (6), the condition for which is
<βΑ+^ι^*+£ι£*, = 0» i·6· 1 .1 + 1 .2 + 1 .(-3 )= 0 ,
which holds good. Hence the line (6) also lies in the plane (5).
This proves the required statement.
Ex. 6. Find the equation to the plane through the point
(* \ β', γ ’) and the tine ( x - a )jl= (y-fi)lm = (z-y )ln .
(Gorakhpur 1981 ; Ranchi 68)
Sol. The equation of any plane through the given line is
a(x—* )+ b (y -fi)+ c (z -y )= 0 ...(I)
where a.l+b.m+c.n=0. ..,(2)
The plane (I) will pass through the point (*', β’, γ ') if
e(*'—/.)i 4 ( j i '- P ) + i ( / - y ) = 0 . ...(?)
The equation of the required plane will be obtained by elimi­
nating a, b, c between the equations (I), (3) and (2). Hence elimi­
nating the constants a, b, c between the above equations, the equ­
ation of the required plane is given by
I oc—a ■ .y - β ζ-γ

* β ’—β y '- y =0

I m n
or ( x - a) {η(β’- β ) -m (y '--y ))= 0 .
Ex /. Show that the plane through the point (a, β, γ) and the
liner . -\-q=rz+s is given by
I.χ py+q rz+ s
« ρβ +q ry + s = 0.
I I I (Meerut 1983 S '
Sol. The equations of the given line are
x= py+ q= rz+ s,
x - 0 _y+ (qlp) z+(s,tr)
I VP ~ ' Vr · .,.(i;
The equation of any plane through the line (I) is
e(x 0)+b(y+qlp)+c(z+s!r)=0 (2)
where l.a + ( l/p ) .i+ ( l/r ) .c = 0 . .1)
112 Analytical Geometry 3-D

The plane (2) will also pass through the point (a. 3, γ ) if
aa-f b(fl+q!p)+c(y+slr)=0. ...(4)
The equation of the required plane is obtained by eliminating
the constants a, b, c between the equations (2), (4) and (3) and
hence it is given, by
y-\q\p z+ sir

β + q!p y+ slr

Vp 1/r
Multiplying second and third columns by p and r respectively,
we get the required equation as
p y+ q rz+ s
a ρβ+ q ry+ s 0.
I I I
Ex. 8. Show that the equation of any plane through the line
X-K
I z yH T - zjI T is (* " a) T + (y P) m + (z - y) T = °
where λ+ μ + ν = 0 .
Sol. The equations of the line are
( x - * ) ll= ( y —p)/m=--(z-y)!n. .··(!)
The equation of any plane through the line (I) is
a[x -α )+ δ (ν —fi)+c(z - y ) = 0 —(2)
where al+bm+cn=Q. —(3)
Now choosing the values O=X11I9 6—μ/m, c=v//i,
the equation (2) of the plane is given by
( x - & ) (XIl)+ ( y - β ) (μ!*η)+(ζ—γ ) ( v /ji) — 0 '

where -m + — -n= 0, .o f λ + μ + ν = 0 .
I m n
This proves the required result.
Ex. 9. Fintf the ajurfion o f the plane through the line
ax+ by+ cz—1 ^ x + b 'y + ^ z
and (xx+fiy-ryz - i'.v-f/^v+y'r.
Sol. The equations of the first Mne are
ax+ by+ cz=0-~c x+ b 'y c'z.
This line clearly passes through the orii *. : /. #;.\ n the
d.c.’s of this line then we have
tf/4· Am+c7z = 0 and a7+ft'm +c'/i=0.
The Straight Line 113

Solving these,
I _ m — n
bc'—b'c ca'—da ab’—a'b
Hence the symmetrical form of the first line is
__x___ y z
bc'—b’c ca'—da aV —o b ···(!)
The equations of the second fine are
ax+ + yz= 0 = a'* + β 'γ+ y z .
This line also passes through the origin. Hence its symmctri-
cal form is * — - , y . = .,, Z—,-0 ■ n\
βγ —β y yOL—ya ap —a p
The equation of any plane through the line (I) is
A x+ B y+ C z= 0 ..-(3)
where A(bc'—b'c)+B(ca’—c'a)+C(efr'—a’b)=0. ···(*)
Also if the line (2) lies in the plane (3) then the following two
conditions must hold :
I. The line (2) passes through the point (0, 0, 0) and so the
plane (3) should also pass through (0, 0, 0) and clearly it is true.
II. The normal to the plane (3) should be perpendicular to
the line (2). the condition for which is
Α(βγ'—β'γ) + Β(γα'—y'ot)+ α’β) = 0. ...(5)
The equation of the required plane is obtained by eliminatiog
A, B, C between the equations (3), (4) and (S). Hence the equation
of the required plane is
x y z
bc'—b'c ca'—c’a ab’—a'b =0.
β γ'—β'γ γχ'ι—γ ’χ οφ'—α'β
E*. 10. Find the equation o f the plane through the point
(2, —1, I) and the line 4x—3y + 5 = 0 = y —2z—5.
Sol. The equations of the line are
4*—3 y + 5 = 0 , y —2z—5=»0.
The equation of any plane through the given line [using
Ρ + λ β = 0 ] is 4 x —3y+5 + X (y ~ 2 z —5 )-0 . ---(I)
If the plane (I) passes through the point (2, —I, I), we have
4 ( 2 ) - 3 ( - D +5+A { - 1 - 2 .1 - 5 ) = 0
or 16—8A= 0 or λ = 2.
P uttingthevaIueofA in (I), the equation of the required
plane is
4x—3 y + 5 + 2 (y —2z—$ ) - 0 or 4x—y —4z—5.
114 Analytical Geometry 3-D

Ex. II. Prove that the equation to the two planes inclined at an
angle a to xy-plane and containing the line y —0, z cos β = χ sin β is
(x*+y*) Iant β + ζ* -2 ζ χ tan fi=y* tan* a. (M.U. 1990P)
Sol. The equation of any plane through the line Ot
z cos j8=jc sin β is given by
(x sin β —ζ cos β) + Xy=O9 ...(I)
The other plane is the xy-plane, whose equation is z = 0 i.e.
0.*+<X y+l.z= 0. ...( 2)
The d.r’s of the normal to the plane ( I ) are sin β9 X9 —cos j8,
and the d.r/s of the normal to the plane ( 2> are 0, 0, I. Also the
angle between them is a and hence we have
cos a— s*n 0·Ο+λ 0 —cos βΛ
"V isio* β + > + c o s " β)~ν (θ + 0 + 1 )
or cos a v^sin 1 0+cos 2 β+Χ*)= —cos β
or cos a \/( \ + λ 1)***—cos β
or (λ1 + 1) cos* a=cos* β
or A1 cos* a=cos* /3—cos* a
or A= ± ^(cos* β —cos* a)/cos a.
It gives two values of A and hence there are two planes con­
taining the given line and inclined at an angle a to the xy-plane.
Substituting these values o fA in (l) and multiplying both the
equations thus obtained the combined equation of the two required
planes is given by
I * sin f - , cos 0 + v W J > Ti « ! i i L , I
I cos a J
t o a COS* β -COS* a ) I n
J r r cos a I
or / sin
(* - B—z
O cos β)*--------
OsI cos* β5----------
—COS* a v,*= 0Λ

or x* sin* β+ζ* cos* β —ΐχ ζ sin B cos /3+j»*^=_y* cos* β sec* a.


Dividing throughout by cos* β, the above equation becomes
jc* tan* β+ ζ*—2χζ tan β +y* sec* B=y* sec* «
or x* tan* β+ζ*—2ζχ tan β + y* ( 1 4 -tan* 0)=y* ( 1 +tan* a)
or (x*+y*) tan* β+ζ* —2ζχ tan /3=>* tan* «.
Ex. 12. The plane Ix +m y=Q is rotated about its line o f inter­
section with the plane z—0, through an angle λ. Prove that the
equation of the plane in its new position is
/x+ffiy±zV (i*+m 2) tan « = 0 .
The Stndght line
SaL The equations of the given planes are
Ιχ+m y= 0 .'..(I) and r = 0.
The eqoation of any plane throngh the line of intersection f
the planes (I) and (2) is
lx+ m y+X z= 0. . -· ··*)
Suppose the plane (I) when rotated through an angle a aN Jt
its line of intersection with the planer= ( has the equation·'?).
Thns the angle between the planes (I) and (3) is cc.
l.l+ m .m + 0 ,X
cos <
’ W 1+ «i*) VrC/*+« * + A*)
V tP + ni*) /*+»n*
or cos a= or cos* X=
V ((*+«*+A *l *" ‘ ^ !•+ «•+ A »
or (/*+»!*) (I —cos* e)=A* cos* a
or . A*=(/*+m*) tan* a, X = + y /( P + tfP) *·“ «·
Potting this value of A in (3), the required equation «!’ the
plane in its new position is given by
lx+ nty± (l*+ n?).z tan a = 0 .
§7. Fsnt and length nf perpendicular f in · apmnt to a Hm
(A) L ineiasyam etrkalfe
To find the equations and length of the perpendicular distance of
a point P (xu r,) from a given line

I m n
The equations of the given Iioe in symme: rical IVI Ul WV
x —a y —B z —y . ,
— = i_ r = * _ Z = r (say).
m (i)
The co-ordinates of any point N on the lire (I) are
(e+ /r. β +m r. y+nr). (2)
Let this point ATbe the foot of the perpendicular h i: , the
point P(xi, yi, r,) to the line (I), so that the line PN is perpendi­
cular to the line (I).
The direction ratios of the line PN are given by
n + lr—x„ p + m r -y „ y + n r—Zi. ...(3)
Since PN is perpendicular to the line (I), using the condition
O^i1+ - J - C 1Cj =O, we have
I (* + lr—x ,)+ in (fi+m r—y t)+ n (y+ n r—z,)= 0
or r (/*+m *+n*)=/ (jr,—e)+m (yt - f iH n (z,—y)
or r ~ [ / (x ,—a)+m ( γ , —β ϊ+ η (Zi—y )] ’(P + n P + iP ).
(4)
I Analytical Geometry 3-L·

The equations of the perpendicular from the point


Pfxt, yu Z1) to the line (I) are given by
* -* i _ y~y\ z—z\
u+ lr—x i β + m r -y , y+ n r—Zt .(5)
where r is ,iven by (4).
Also s bstituting the value of r from (4) in (2), the co-ordi­
nates of the foot N of the perpendicular are determined, and then
the perpendicular distance PN is easily calculated.' See Ex. I
.which follov s after part (B) of this article.
(B) Li ie in general form :
To find the equations o f the perpendicular line from the point
P(Xi, y ,t Zj) to a line whose equations are given by
ay+ by+ cz+ d= 0= a’x+ b'y+ c'z+ d'.
The equt boos of the given line in general form are
- ax+ by+ cz+ d—0, a’x+b'y+c'z+d'=sO. ...(I)
The perpendicular from a point P to the given line (I) Is the
intersection o f the two planes namely (i) the plane through the given
point P and also through the given line and (if) the plane through
the point P perpendicular to the given line. (Remember]
Let l.m .n be the d.c.’s of the given line (I), then we have
al+bm+cn—0, and o7+b'in+c'«= 0.
Solving these, we have
I _ m _ «
bc'—b'c ca'—c'a ab' - tfb ...(2)
Now the equation of any plane through the line (I) is
given by
(ttx+ by+ cz+ d)+ \ (α'χ-f b'y ±c'z+ d')= 0. ...(3)
If the plane (3) also passes through the point P(X\, yi, Zi)
then (axi+ byi+ezi+d) + λ (cfx1+b'y,+ c'zl + d') = 0
or λ·= —(OX1+ byt + CZi + d)/(o'x,+ b'y,+ c‘zt+<?).
Substituting this value of Λ in (3), the equation of the plane
' h ough the point P and the line ( I) is given by
ax+ b y+ cz+ d = a'x+b‘y + c'z+d' _
axi+ byi+ czt+ d ~ a'xx+b'j't+c'Z i+d' ...(<)
Novt ve to find the equation of the second plane whicl
pusses through P aud is perpendicular to the line (I).
Since the plane is perpeu ’ -ular to the line (I), therefore tlK
d.r of its normal are proportion1 to /, m, n given by (2)
The Straight Une 117

Therefore the equation of the plane perpendicular to the line (I)


and passing through P{X\t y\. z,) is
/( x - x ,) + m (y-yiM -n (Z-Z1)=O. ...(5>
Therefore the equations of the perpendicular line from the
point P(xi, yt, z,) to the line (I) are given by - the equations (4)
and (5).
SOLVED EXAMPLES (E)
Ex. I. Find the equations o f the perpendicular from the point
X v —2 z _3
(3, —1» 11) to the Iin e -^ = -J - = ^— . Findalsothe co-ordinates
of the foot o f the perpendicular. Hence fin d the length o f the per­
pendicular.
Sol. Thegivenpoint is P (S 1 - I 1 IJ) and the equations
of the given line are
x —O y —2
^ - ? = r ( say).
2 - 3 (J)
The co-ordinates of any point N on the line (I) are
(2r, 3r+2, 4r+3). ...(2)
Let this point N be the foot of the perpendicular from the
point P(3, —I, II) to the line (I).
Then the d r.’s of the perpendicular PN are
2r—3. (3r+ 2)—( —I), ( 4 r+ 3 )- 11,
or 2r - 3, 3r+ 3, 4r 8. —(3)
The d.r.’s of the given line (I) are 2, 3, 4. Now PN is per­
pendicular to the line (I). The condition of perpendicularity gives
(2r—3).2+(3r+3).3-f (4r—8). 4= 0,
or 29r—29=0, or /= 1 .
Putting the value of r in (2), the foot N of the perpendicular
is the point (2,'5, 7).
Putting the value of r in (3), the d.r.’s of PN1are —I, 6, —4.
Hence the equations of the perpendicular PN from the point
P(3, —I, 11) to the line (I) are
x - 3 _y+ l z -ll

The length of the perpendicular PATsthe distance between


the points P(3, —1, (I) and AT(2, 5, 7)
= V l(3 -2 )* + ( - I - 5)* + ( 11-7)*J
= ■^/(1 + 36+ 16)=*V(53).
118 Analytical Geometry 3-D

fix. 2. Find the equations of the perpendicular from origin to


the line
a x + b y + c z+ d = 0 -a 'x + b 'y + c 'z+ d '^ O .
S o l.' First read § 7 (B) again and then solve this problem.
The equations of the given line are
ax+by+cz+d=>09 a'x+ b'y+c'z+d0= 0. - (1)
We know that the perpendicular from a given point P to a
given line is the intersection of the two planes, namely (i) the
plane through the give?i point P(OvO9 O) and also through the
given line and (ii) the plane through the point P perpendicular
to the given line. Thus we proceed as follows :
The equation of any plane through the line (I) is
a x + b y+ cz fr f+ λ {a 'x + b 'y+ c'z+ d ')= 0. -(¾
If the plane (2) passes through the point P(09O9 0), then
d + M '= 0 , or A=
Putting this valot of A in (2), the equation of the plane
through the origin and the given line is
(a x + b y+ c z+ d )—(d/d) (n x + ft> + c'z+ d ')= 0
or (ad0—(id) ;:+ (lx r-b 'd ) y + ( a T - c 0d) z = 0. -.(3 )
Now let I9 JVt9 n b:; the d.r.’s of the given line (I)9 then
al+ bm + cn= O9 and a*l+b0m + c0n=*O.

Solving, bc- - b ^ ‘=‘i a c7a a ab - a 'b ’ ..-(4)


Thus the equation of the plane through Py0, 0, 0) and per­
pendicular to the line (I) is
I (x —0 ) + im O'—0H-n ( z —0 ).» 0
or (4 c'-4 'f) x + ( a '- c 'a ) χ + (< Λ '-Λ ) r= 0 . -.(5 )
Hence (3) and (5) together are the equations of the perpendi­
cular from the origin to the given line (I).
E s 3. Show that the distance d of the point P{*, β, y) from
the line (x—x , ) / / = 0 —^,)//π= (z—z,)//» measured parallel to the
plane u x + h y + cz+ d = 0 ύ given by
^ ( a *^6*+c*) £{m{Zt—y)—Itjyl-/*)}*- |.T(x, —a)(bn—cm)]*
a (al-j-hm+cn)*
Sol. The equations of the given line are
(x—xt)/l=(y—J t)/in = (z -z l)/a = r, (say). ...(I)
Any point Q on (I) is
(X1-Mr. yt+ m r, zt +nr).
The Straight Line 119

Now P is the point (α, β, γ) and hence d.r.’s of PQ are


Xi+lr—«, yi+ m r—β, z ,+ n r -y .
It is required to find the distance PQ measured parallel to
the plane ax+by+cz+d·= 0. Now PQ is parallel to this plane
and hence PQ will be perpendicular to the normal to the plane
ax+by+cz+d<=>0.
Therefore we have
( / r + X i - a ) a + iyi+ m r—β) b + ^ + n r - y ) C = 0
or r = - [ a f a - ^ + b & ι-β )+ ο (zt -y)]l(al+bm +cn). ...(2)
Now d * = P 0 * = ( X | + / r — a)*+ (^,+m r-/3)*+ (r, + « r — y)*
or ^ “ { /r+ to - o O P + tm r + ljj- W + in r + tZ j- y ) } *
o r' d*=r* (/* + « * + » ')+ 2r { /(x ,-a)+ m (yt - β ) + η (z,-y)}
+ {(*i - <*)*+ (Pi - /3)*+(2i - y)*}. ··. (3)
. Putting the value of r from (2) in (3) and using Lagrange’s
identity, we get the required result.
Ex. 4. f ind the distance of the point P{3, 8, 2) from, the line
~^3~~ measured parallel to the plane
3x+ 2y—2z+17=0.
Sol. The equations of the given line are
( X — l) /2 = < y -3 )/4 = (2 -2)/3=r, (say). ...(I)
Any point Q on the line (I) is
(2/-+1,4/- + 3, 3r+2).
Now P is the point (3, 8, 2) and hence d.r.’s of PQ are
2 /-+ 1 -3 , 4 /-+ 3 -8 , 3 /-+ 2 -2 i.e. 2 r - 2 , 4 r - 5 , 3r.
Iiisre q u ire d to fin d th e distancePg measured parallel to
the plane
3x + 2y—2z+17 = 0. ...(2;
Now PQ is parallel to the plane (2) and hence PQ will be
perpendicular to the normal to the plane (2). Hence we have
(2 r-2 ) (3)+(4/--5) (2)+(2/-) ( - 2 ) = 0 ,
or 8r—16=0, or r= 2 .
Putting the value of r, the point Q is (5, 11,8).
Λ Required distance=The distance between P (3, 8, 2) and
Q (5, Hs 8)
= V [(3-5)* + ( 8 - 11)2+ ( 2 - 8 ) 2]= V(4 f 9+36) = 7.
Ex. 5. The equations to AB referred to rectangular axes are
jc/2«=»^/—3=z/6. Through apolrit P { 1,2, 5), PN is drawn per­
pendicular to AB and PQ is drawn parallel to the plane 3x+4j>-|-5z=0
120 Analytical Geometry 3-D

to meet AB in Q, Find the equations to PN and PQ and the co­


ordinates o f N and Q-
Sol. Tbe equations of the lin e ABare given as
χ β —y l—3 = ? /6 = r (say). -(1 )
Aoy poiot on (I) is (2r, - I r , 6r). Let this point be N- Also
P is (I, 2, 5).
The d.r.’s of P N are 2r —I, —3r—2, 6*·—5.
Now PN will be perpendicular to the line AB given by the
equations (I) if
2 ( 2 r - 1 )+ (-3 ) ( - 3 /- - 2 ) + 6 ( 6 r - 5)=0, or r - f S ·
/52 —78_ 156Λ
Potting the value of r, we have N \49* 49 * 49 /
52 - 7 8 , 156
and d.r.’s of PN are I, D
49 49 ’ 49
or 3, -1 7 6 , - 8 9 .
Thus the equations to P If are
x — I y —2 z —S
3 " -1 7 6 = - 8 y -(2 )
Again the co-ordinates of any point Q on the line (I) are
(2r, —3r, 6r) and the point P is (I, 2, 5).
The d.r.’s of PQ are 2r—I, —3r—2, 6r—5.
Now PQ is drawn parallel to the plane .
3jc+ 4 j ;+ 5 2 = 0 . -(3)
The line PQ is perpendicular to the normal to the plane
(3), whose d.r.’s are 3, 4, 5.
Therefore we have
3 (2/--1) + 4 ( - 3 /- - 2 ) + 5 (6r—5)=0, or /- = f.
Putting this value of r, we have Q (3, —9/2, 9) and d.r.’s of
PQ are 3 - 1 , ( - 9 /2 ) - 2 , 9 - 5 ie. 4 , - 1 3 , 8.
.*. The equations cf PQ are given by
At-1 y —2 2— 5
4 “ -13 “ 8
The projection (or image) of a line on (or io) a given plane.
The method is explained by the following examples.
The line is in symmetrical form.
Ex. 6. What do you understand by the projection o f a line on
a given plane ? Find the equations of the projection o f the line
AC— I y+1 z —3 . ,,
- j —Β» =>- γ - on the plane x -r2 y + z—6.
The Straight Line 121

Solution. DefloiHon I. The projection o f a line on a given plane


is the line of intersection o f the two planes namely (f) the given plane
and (U) the plane through the given line and perpendicular to the
given plane.
Definition i. Let P be the point o f intersection o f the given
line with the given plane and let Q be the foot o f the perpendicular
from any point on the line to the plane, then the line PQ is said to
be the projection o f the given line on the given plane.
Now wo give the solution of the given problem using both
definftioos one by one.
Using definition I.
The equations of the given line are
ac-l _ y + l _ z —3
2 -1 4 ...(I)
and the equation of the given plane is x + 2y+ z= 6. ...(2)
Ih e equation of any plane through the given line (I) is
A ( x - 1 ) + 5 ( ^ + l)+ C < z-3 )= 0 ...(3)
where 2A- 21-|-4C=0. -- (4)
The plane (3) will be perpendicular to the plane (2), if
A + IB + C = 0. ...(5)
A B C
Solving (4j and (5), we gee — · ■ - =γ = - - ·
Patting these proportionate values of At B9 C in (3), we have
9 (* —1)+2 (>' + 1) + 5 (z -3 )= 0
or 9x-^2y—Sz + 4==0. ...(6)
The equations (2) and (6) together are the equations of the
line of projection.
Using definition 2.
The co-otdinates of any point on the Iine(I) are ( 2 r + l9
—r —l, 4r+3). Lettbis be the point of intersection P of the
given line (I) with the given plane (2). Then P ( 2 r + lt —r —1,.
4r+3) wiM lie on the plane (2).
.. 2r+ I + 2 ( ~ r —l) + 4 r+ 3 « 6 , or r = I.
Putting Uiis value of r, the point P is (3, —2, 7).
Now the line (I) clearly passes through ihe point (I, —If 3)
and hence this is a point on the given line (I). Now we are to
find the foot ot the perpendicular Q from (If - I f 3) to the
plane \2).
The d.r.’s of the normal to the plane (2; are If 2f I and
122 Analytical Geometry 3-2)

hence these are d.r.’s of the line through (I, —1, 3) and perpendi­
cular to the plane (2), and therefore the equations of this perpen­
dicular line are
x —l = jM-J z - 3
Ti (say).
Ϊ 2 I
A nypoioton it is (^ + 1 , 2 ^ - 1 ,^ + 3 ) . Let this be the
point Q and so it will Ιϊς on the plane (2).
·'. ^1+1+2 ( 2 ^ - 1 ) + ^ + 3 = 6 , or T1= I .
Putting this value of , the foot of the perpendicular Q is
(5/3,1/3,11/3).
The equations of the projection i.e. the equations of the
line Pgjoining the points P (3, —2, 7) and Q (f, $, V1) at*·
x —3_ + 2 _ z —7 x —3_y~\~2_2—7
3 ^ t“ - 2 - i " 7 ~ V ° r ~ 'W ...(7)
Remark. If the equations (2) and (6) are transformed} to
symmetrical form, we shall get the equations (7) of the projection
PQ- We note that if we want to find the equations of the projec­
tion in general form then we use definition one, and if we want
symmetrical form then we use definition two.
Ex. 7. i f L is the line tfte direction
cosines of the projection of L on the plane 2 x + y —3z=4 and the
equation of the plane through L parallel to the line
2x+5^ + 3z=4, x —y -3z=6.
Sol. The equations of the line L are
jc—I —O_z+ 2
r, (say).
2 ^ --TI = I .(D
Any point on the line (I) is ( 2 r + 1, —r, /'-2 ).
If it lies on the given plane 2jc+ j >- 3z=4, ...(2)
we have 2 (2 r+ 1 ) + ( - r)—3 (r- 2 ) = 4 , or 0 .r+ 4 = 0 .
This relation is impossible and we do not get any value of r
as the coefficient *of r is zero. This shows that line (I) is parallel
to the plane (2). Hence method of definition 2 (see Ex. 8) cannot
be used here. Now we shall use method of definition I.
The equation of any plane through the line (I) is
A ( x - 1)+ B y+ C (z+ 2)=0, ...(3)
where 2A—2 H C = 0 . . (4)
The plane (3) will be perpendicular to the plane (2) if
2Λ+2?—3C=0. ..-(5)
The Straight Uae 123

Solving (4) and (5), we get


A_ B_JC_ ± =,B_=,C_
2 *8 4 ΟΓ I = 4 = 2 '
Pulling these proportionate values of At B and C in (3), the
equation of the plane through the line £ (1 ) and perpendicular to
the plane (2) is given by
I .(* —1 )+ 4 .^ + 2 (z+2)«=0
or x+4y+2z+3=»0. ...(6)
The equations (2) and (6) together are the equations of the
line of projection of the line L on the plane (2).
Let I9 m9 n be the d.c.’s of the line of projection given by (2)
and (6). Then we have
2 /+ m —3/i=0 and /+4m + 2n«=»0.
_ , . _ ^ l m n
Solving these, we get — = —
I _ m _n __ y/(P + m %+ n%) _ I
or T ~ - I ^ T - V{(2)*+( - 1)1+(17} V6'
Λ /= 2/^/6, rw= —I /·\/6, n= 1/\Λ>·
Hence d.c.*s of the line of projection are 2jy/69 —1/V6, 1/\/θ·
Now to find the equation of the plane through the line L and
parallel to the line
2x+ 5 y + 3 z= 4 , x - y - S z = 6 . ...(7)
Let Il9 mlt R1 be the d.c.’s of the line (7).
Then 2/, + 5//1,+3/1,=0, Il - H i i - S n l=Ot
- . . . /1 nii ni
Solving these, we get
Now the equation of any plane through the line L (I) is given
by (3) provided the condition (4) holds. If the plane (3) is parallel
to the line (7), then the normal to the plane (3) will be perpendi­
cular to the line (7), the condition for which is
22A -\3B + 1C = 0. ...(8)
Solving (4) and (8), we get
A _ B _ C A B __ C
—1+ IJ a I l - 1 4 ^ -2 6 + ^ 2 ΟΓ 3 4 “ —2*
Substituting these proportionate values of A9 B and C in (3),
the equation of tbe plane through the line L (I) parallel to the
line (7) is given by
3 (jc- 1)+4>»-2 ( z + 2)=0, or 3*+ 4.y-2z= 7.
124 Amfyiteat Geometry D-3

The line in general form.


Ex. 8. Find the projection o f the Une 3x—y + 2 z = l ,
x + 2 y —z—2 on the plane 3x+ 2y+ z= 0.
Sol. The equations of the given line are
3x—y + 2 z —I, x + 2y ^ - z —2. ...(I)
The equation of the given plane is 32+ 2 y + 2 = 0 . .-(2)
The equation of any plane through the line (I) is
(32 —y + 2 z — 1)+A ( x + 2 y - z - 2 ) ~ 0
or (3+A) 2 + ( - I +2Λ) ^ + ( 2 —A) 2 - 1 —2A=0. ...(3)
The plane (3) will be perpendicular to the plane (2), if
3 (3 + A )+ 2 (-l+ 2 A )+ l (2 -A )= 0 , o r A = - |.
Puttingthisvaiue of A, in (3), the equation of the plane
through the line (I) and perpendicular to the plane (2) is given by
(3—f) 2 + ( - - 1 - 3 ) y + (2 + $ ) 2—1 + 3 = 0
or 3 2 - 8 ^ + 7 2 + 4 —0. ...(4)
The projection of the given line (I) on the given plane
(2) is given by the equations (2) and (4) together.
Note. The symmetrical form ol the projection given above
by equations (2) and (4) is
ί± ·= ^ ~ ·= _ 5 .
-n 9 15'
§ 8. Coplanar Lines. To find the condition that the two lines
whose equations are given may be coplanar i.e. should Intersect and
to obtain the equation of the plane containing them.
(Gorakkpnr 1982)
Two lines are called coplanar if they intersect. Even if the
two lines are parallel, they are also coplanar because they will
intersect at infinity and will satisfy the condition of coplanahty
given below.
The equations of two lines may be given in three ways :
(A) Both hoes in symmetrical form, (B) one line in symme­
trical form and the other in general form, and (C) both lines in
general form.
Now we shall discuss these cases one by one.
(A) Both lines are given in symmetrical form :
(Avadh 1981, 82; Allahabad 80; Kanpnr 76, 81)
Let the equations of the two given lines be
(2 - 2 ,)//1= ^ - ^ , ) / ^ = ( 2 - 21)/*!. ...(I)
and { x - x t ) f l i = ( y —yi)lmi (Z-Zt)Ini. ...(2)
The equation of any plane through the line (I) is
The Straight Line 125

A (x -X \)+ B ( y - y t)+ C (Z-Z1)-O , . (3)


where All +Bmi + Cnl =O. ...(4)
For the lines (I) and (2) to be coplanar, the plane (3) should
be such that the line (2) also lies in this plane. Then the normal
to the plane (3) will be perpendicular to the line (2) so that we
have Al%+ Bmi +Cn2=O, .. (5)
Again the point (x2*Vf, zf ) through which the line (2) passes
will also lie on the plane (3) so that we have
A{x%- Xi) -l· B(yt --.Vi) + C (z ,- 2i)- 0. . (6)
H encetherequiredconditionthat the lines (I) and (2) are
coplanar (i.e. are intersecting) is obtained by eliminating A9 B9 C
from the relations (6), (4) and (5), and is given by
X t-X i Vi-Vi Z2 - Z i

/. W1 Wi ■ 0. ...(A)

It Mt *2
The equation of the plane containing the lines (I) and (2) is
obtained by eliminating A, Bt C between' (3), (4) and (S) and is
given by
I AC-X1 V-Vi Z -Z j
I
/. Ml Wi = o.

I* Mt Wf

The point of intersection of the lines (I) and (2).


The co ordinates of any point P on the line (!) are (/ιΓ,-f
Wtifi+Vi. n\fi + Zi) and those of a point Q on the line (2) are
U tft+ x 2* wi2r2-f >ί * ntfi+Zt).
If the lines (I) and (2) intersect, then for some values of rx
and rf, the points P and Q should coincide, i.e. we have
Itfx+ x Itft +X i9 m ^ + y ^ m t r t + T t , Wifi+*i=w 2r 2+ z 2
or (Xi—Xt)+liri —Itftc=O9 ...(7)
(Vi-JPi) + Mtfx - mtr2= 0 .. (8)
and (Z1- Ζι)+ΛιΓ| —UtftsssO. ...(9)
Solving the relations (7) and (8) the values of Tj and r2 are
obtained and if these values also satisfy the relation (9), then the
lines (I) and (2) are coplanar otherwise not. In case the lines (I)
and (2) are coplanar, on substituting the value of rx (or rf) the
co-ordinates of the point of intersection P (or Q) are obtained.
126 Analytical Geometry 3 -D

Also by eliminating rk and - r 2 between the relations (7), (8)


and (9) the condition that ihe lines (I) and (2) are coplaoar is
given by
X i - X 2 h It
I
y i - y * mi IW 2 = O

Zi-Zt
B
H2 II

Xl-Xt
*

£
h I
IW 1 n x

It IW 2 W2

(B) One lineisgiveniitsyminetricaIforBi ami the other line


is given in general form.
(Agra 1973 ; Meernt 82S ; Allahahni 7·. 81)
Let the equations of the two given lines be
(* -* i)//= (j-.P ,)/m = (r-Z i)/« —1
and Q\X+biy-{‘C\Z'\- d\*a(}=:aiX-)-bty-\-c&-\-di'. *-*(2)
The equation of any plane through the line (2) is
( a i x + h y + c t f + d j )+ A (fl2x + h 2y + c 2z + J 2) = 0 ...( 3 )
or (A1+ Afl2) χ + ^ + λ δ * ) y-\-(c\ + Ac2) ζ+ « /ι + λ</2)=»0. -..(V)
If the line (I) is parallel to the plane (3'), then the normal
of the plane (3') will be perpendicular to the line (I) and so we
have / (fli+Afl2) + m (6, + Ah2) + w (c, + Ac2)= 0
or A(fl2/+ 6 2m + < y i)= - (^ /+ J jlWi+ Cifl)
or A= —(flj/+^iiw+C1W)/*A2/+Zj2IW+ryi). —(^)
Puttingthis value of A in (3), the equation of the plane
through the line (2) and parallel to the line (I) is given by
,, QiX+bty+CxZ+dL fl2s+ ft2y + r2z + */2
ad+bim+Cxn ““ a j+ ’btm+ctn —(5)
The line (I) passes through the point (jclt y u z,) which most
lie on the plane (5) if the lines (I) and (2) are coplanar t.e.9 inter­
secting. Thus for the lines (I) and (2) to be coplanar the point
(*i, yu Zl) should satisfy the equation (5). Hence the condition
that the lines (I) and ( 2) are coplanar is given by
fli^ i+ h rV i-t-C iZ !+ ^ , U i X l + h 2y i + C2Z 1 + d t

ait-\-bxm ± c xn A2/ + h2"i+<vi


...(6)
If the condition (6) is satisfied the lines (I) and (2) are inter­
secting (or coplanar) auu the plane containing both the lines is
given by the equation (5).
The Straight Line 127

Alternative Method : The co-ordinates of any point P on the


line (I) are (lr+ xu m r+ yu n r+ zj. Ifthe lines (I) and (2) are
intersecting then the point P should also lie on the line (2). Hence
we have
<*\ ilr+Xi) + bx (m r+ y ,)+ ^ (nr+ ζΎ)+ άχ=Ot I
and a2 (lr+ xx)+ b2 (mr+yx)+ c2 (nr+zx)+ d 2^ 0. j
From these, we get
r (ail+bxm + c xn ) = — (QlX ^ b i y l+ C 1 Z 1 + d y ) 9 I
r (a2l+ b2m+ c2n) = —(0a*i+ ^ 2.^1 +Ca^i+^a)· f
Eliminating r, the required condition that the lines (I) and
(2) are coplanar is given by
0i*i+ M i +CiZx+dt a2xx+b2yi + c2zi + d2
axl+ b}m + c}n a2l+ b2m +c2n
(C) Both the lines are given in general form :
Let the equations of the two lines be
a i x + b i y + c t f + d t ^ o ^ a i x + b t y + c t z + d t , ...( I )

and azx+ b9y+ czz+ d3=*0=>a4x+ b4y+ c4z+ d4. ...(2)


If the lines (I) ond (2) are coplanar, these will intersect in a
point say the point (α, β, γ).
The co-ordinates of this point must satisfy the equations of
the four planes representing the two lines, and so we have
Q\<t.+b\fi+C\y+dx=0,
02®+ ύ2β + CzY+d2=0.
03®+ b9fi+c9y + d9= 0,
and 0|<x+ ί>4β +C iy+</4= 0.
Eliminating α, β, y between the above four relations, the
required condition is given by
01 ft. Ci =0

02 ft,. C2 dt
03 ft. C3 dz

04 ft« C4 dt
Remark. The expansion of a fourth order determinant is
usually difficult and therefore it is convenient to solve numerical
examples by first reducing the equations of the lines to symmetri­
cal forms and then proceeding as in case (A).
SOLVED EXAMPLES (F)
Ex. I. Show that the lines £ (jc+ 3 )= J (y + 5 )« —J (z—7) and
^ ( χ + 1 ) = |( ^ + 1)«=* —(z + l) are coplanar. Find the equation o f
the plane containing them. (Rohilkhand 1982; Madras 76)
128 Analytical Geometry 3-D

Sol. The equations of the given lines are


(x + 3 )/2 = ( y + 5 ) /3 = ( r - 7 ) /- 3 = r, (say), -(1 )
and ( x + l ) l 4 ~ ( y + l ) l 5 = ( z + l ) l - l - r t (say). -(2 )
The co-ordinates of any point P on the Iine(I) are (2ri—3,
3fi —5, —3/yf 7) and those of any point Q on the line (2) are
(4r,—I, 5rt - 1. —r2 - I).
If the lines (I) and (2) are coplanar then they intersect and
hence for some values of rt and rt the points P and Q coincide.
Thus we have
Irx-Z = A ri - X , 3r, —5—5ra- I, - S r 1+ 7 = —r*—I
or r, —2rs= l , 3r,—5ra= 4 , 3r,—r2—8.
Solving the first two equations, we get γ2=3, r* = l.
These values of rt and r2 also satisfy the third equation, and
hence the lines (I) and (2) are coplanar (i.e. intersect).
Putting the value of r, (or the value of r») the co-ordinates of
the common point of intersection Le., P (or Q) are (3, 4, —2).
Now the equation of the plane containing the lines (I) and
(2) [i.e. the plane containing the line (I) and parallel to the line
(2)) is
x+3 y+5 z- 7
2 3 -3 =0 [See § 8 (A)]
4 5 -I
or (x+ 3) (3 ( - 1 ) - ( - 3 ) 5 ) - ( ^ + 5 ) (2 ( - 1 ) - ( - 3 ) 4}
+ ( z -7) { 2 .5 -3 .4 )= 0
or (x+3).12—O>+5).10+(z- 7 ) .( - 2 ) = 0 or 6x-5.y-z=l> .
Ex. 2. Prove that the lines J ( x —1 )= J (>’ —2) ■=J (z—3) and
J (x —2 ) = J (y —3 ) = | (z—4) are coplanar; find their point of
Intersection. (Berahmpur I981S ; Lucknow 81 ; Meerut 75)
Also find the equation of the plane in which they lie.
Sol. The equations of the given lines are
(x- 1)/2 = ( y - 2)/3= (z- 3)/4 = r, (say), -(1 )
and (x - 2)/3=(_y—3)/4 —(z 4)/5 =- ra (say). ...(2)
The co-ordinates of any point P on the tine ( I) are (2/¾ + 1,
3rj + 2, 4fj + 3) and those of any point Q on the line (2) are
(3r2+ 2, 4/2+3, 5r2+ 4).
If the lines (I) and (2) are coplanar then they intersect and
hence for some values of rx and r2 the points P and Q coincide.
Thus we have
The Straight Line 129

2fi+ l = 3r*+*2. or 2ri ■■3r2 1 (3)

ut
3r,+ 2= 4ra+3, or

II Il
(4 )

I Ί
and 4/i+3== 5f2+4, or • (5)
Solving (3) and (5), we get T1= - I , r*= —I.
These values of T1 and r%also satisfy the equation (4), and
heqce the given lines (I) and (2) are coplanar. Putting the value·
of Tj (or ofr*), the point of intersection P (or Q) is ( —1, —I, —I).
Now the equation of the plane in which the lines (I) and (2)
lie is given by
x -l y —2 z —3
2 3 4 =0 [See § 8 (A)]
3 4 ‘5
or x-2y+ z= *0.
Ex. 3. Show that the lines
(x + 1 ) / ( - 3 )= (y —3 )/2 = (r+ 2 )/l
and x /l= 0 > -7 )/(—3)=(z+7)/2
intersect. Find the co-ordinates of the point o f intersection and the
equation to the plane containing them.
[Agra 1979, Gorakhpur 82, Kanpur 81, Rohilkhaad 77]
Sol. The equations of the given lines are
^ + 1 ) /( - 3 ) = 0 - - 3 ) / 2 = ^ + 2 ) /1 = * (say) ...(I)
and ( x - 0 ) /l= 0 - - 7 ) /( - 3 ) = ( z + 7 ) /2 = r , (say) ...(2)
The co-ordinates of any point P on.the line (I) are ( —3rj—I,
2 * + 3 , T1- 2) and those of any point Q on the line (2) are (rt(
—3r»+7, 2r*—7).
If the lines (I) and (2) intersect (l.e. are coplanar), then for
some values of T1 and r* the points P and Q coincide. Thus, we
have - S r 1- I = T*, or 3 * + * = —I .. (3)
2tj + 3 = - 3 t,+ 7 or 2 r,+ 3 r,= 4 .. (4)
and * —2=2r*—7, or T1- 2r*= —5 ...(5)
Solving (3) and (4), we get T1= - I , r*= 2.
These values of T1 and r%also satisfy the equation (S), and
hence the given lines (I) and (2) intersect. Putting the value of *
(or of T*), the point of intersection P (or Q) is (2, 1, - 3 ) .
Now the equation of the plane in which the lines (I) and (2)
lie is given by
130 Analytical Geometry 3-D

*+l * -3 z+2

-3 2 I =0

I -3 2
or (X +l) ( 4 + 3 ) - 0 - 3 ) ( - 6 - 1 ) + ( z + 2 ) ( 9 - 2 ) = 0
or * + ^ + 2 = 0.
Ex. 4. In each o f the following cases show that the two given
lines are coplanar :
(0 $ (*—5 ) = i O'—7 ) = —i (2+3); $ (x—8 ) = 0 '- 4)=$ (z -5 ).
Alsoflnd their point o f intersection and the equation of the plane
in which they lie.
(ii) x - J 0 - 2 ) = $ (2+3); $ (x—2)=$ 0 - 6 ) = $ (2-3).
Also find their point of intersection and the equation of the
plane in which they lie. [Indore 1979, Madras 75, 78]
(H i) $ (x -l) = $ 0 - 0 = $ O -i) ; i 0 - 5 ) = J 0 -7 ) = 0 -9 ) -
Also find the equation o f the plane containing them.
[Ranchi 1976]
(/V) ( X - I ) = J 0 - 1 ) = $ 0 - 1 ) ; J 0 - 4 ) = J 0 - 6 ) = J 0 - 7 ) .
Also find their point o f intersection. [Gorakhpur 1978]
Sol. Proceed exactly as in Ex. I (2 or 3) above. The
answers are
(i) (1 ,3 .2 ), 17x—47^-242+ 172= 0.
(ii) (2, 6, 3), x - 2 y + z + 7 = 0 .
(iii) x —2^+ 2= 0. (iv) (2, 5, 7).
Ex. 5. Prove that the lines
(x -a )!a '= O - 6)/6'= O - c)/c’
and (x —a')la= (y—b’)/b= (z—c')lc
intersect and find the co-ordinates o f the point o f intersection and the
equation of the plane in which they lie. [Agra 1982]
Sol. Any point on the first line is P (a'r,+ a, 6 7 i+ 6 , c’ri+c)
and any point on the second line is g (ar»+a', 6r*+6', cr*+c').
The given lines will intersect if for some values of rt and r, the
points P and Q coincide. Thus, we have
a 7 i+ a = a r 1+ a ', i
67i+ 6 = 6 r* + 6 ', I These equations are clearly satis·
and c'ri+c=crt + c’ ) field by r i= r s= l .
Hence the given lines intersect. Putting the value of T1 (or
of r»), the point of intersection P (or Q) is (a + a ', 6 + 6 ', c+c').
The Straight Line 131

Now the equation of the plane in which the given lines lie is
given by
χ —α y —b z —c x y z

a' b' c' = 0, or a b c

a b c a' b' c'


Ex. 6. Show that the lines
x —a + d _ y —a_ z —a+ d
a—8 « α+δ
x —b+ c y —b z —b —c
and
β —y β β +y
ere coplanar and find the equation o f the plant in which they lie.
Sol. The given lines will be coplanar if
(b—c)—(a—d) b —a (b+c)—(a+d)

α—δ a α+δ = 0 . [See § 8 (A),


condition (A)]
β —γ β β+ ν
Adding the third column to the first column, the determinant
on the left hand side
2 (b - a ) b—a b+ c—a —d

2a a OC-f*δ

2/5 β β+Υ
b —a b —a b + c—a—d

a a. α+δ

β β β+Υ
= 0 , the first two columns being identical.
Hence the condition of coplanarity of the two lines is satisfied.
Hence the given lines are coplanar.
Now the equation of the plane in which the given lines lie is
given by
x —a+ d v—a z —a—d

«—δ a α+δ 0.

' β—Υ β β+Υ


132 Analytical Geometry 3-D

Adding the third column to the first column, we get


x + z —2a y -a z —a—d

2a a a +£

2/J β β+Υ
Subtracting two times the second column from the first
column, we get
x + z —2y y -a z —a —d

0 a a -f δ

0 β β+Υ
or x + z —2y=»0 as the required equation.
Ex. 7. Prove that the lines
x__y__ z x y £
α β y' act δβ cy' J m n
N d// lie in one plane i f
(//a) (b-c)+ (m ll3) (c-a)+ (n/y) ( a - b ) = 0.
IGarhwaI 1978; Kanpur 81, 83; Meerut 89]
Sol. We clearly see that the three given lines pass through
the origin O and therefore they will be coplanar if they are
perpendicular to a line through the origin 0.
Lcf it η, ζ be the d. c.’s of this line through the origin 0 .
Hence if this line is perpendicular to the given lines, we have
i* +ηβ + Cy=O, -d )
i βα + φ β + icy = 0, -(2 )
and ξ Ι + ^ + ζ η —Ο. -(3 )
SoIving (I) and (2), we have
i _ v ._____ L _
cj3y—bj3γ any —cay ba-β - ααβ
όΓ ___ (__ _____ n__ ___ __ ζ___
(b—c) βγ (c—a) a y (a —b) <χβ
Putting these proportionate values of i, η, ζ in the equation
(3), we have.
//Jy (b—c ) + m « y ( c — α)+ηαβ (a—f r ) = 0 .
Dividing throughout by αβγ, we get
(//«) (6-c)+(m //3) (c -a )+ (n /y ) (a-b)*~0
as the required condition.
The Straight Line 133

Ex. 8. Show that the lines


— L ·. L· x _z_ 1 £_
α&~~ΰβ ^cy* afa ~ β/b* yfc' a~~β ^ γ
are coplanar i f a*=*b or b*=c or c=>a.
SoK We clearly see that the three given lines pass through
the origin O and therefore they will be coplanar if they are per­
pendicular to a line through the origin 0 . Let I9 m9 n be the d.c.’s
of this line through the origin 0. Hence if this line is perpendi­
cular to the given lines, we have
laa + mb13+ ncy = 0. ...(I)
/a ja + /ttj3Ib+ nyjc = 0, •(2)
and /a+-mj3 + rty«=i0. .(3 )
Eliminating /a, ηιβ and ny between the equations (I), (2) and
(3)v we get the condition for the coplanarity of the given, lines as
a b C

i/« IIb IIc


I I I
Multiplying 1st, 2nd and 3rd columns by a, b, c respectively,
we have
a* b* C2
i I I I
I I I =0,
I a b or
C
I
a b C ; ο* b' c*
Subtracting 1st column from 2nd and 3rd columns, we get
I O O I
b —a c —a
a b—a c—a > « 0 , or =Or
P - Q 2 £ l_ a*
a2 b2- a 2 c*— a* '
or ( b —a) ( c—a) I I
-0,
b+a c+a
or 0, as the required condition·
( b —a) ( c —u) ( c —b ) =
Hence the given lines are coplanar if
b = a or Ct=3Q or b=*c.
134 Analytical Geometry 3-D

Ex. 9. Provethatthe lines x= ay+ b= cz+ d and x ^ ay+β


• γ ζ + S are coplanar If ( Λ β —bc t ) ( y —c)—(cS—dy) (α—a)*=0.
(Bordnaa 1980; Kanpar 82)
Sol.The equations of tbe given lines are
x —0 y+ b/a z+d/c
Xe*ay+ b= cz+ d or 1/c ..(I)
I - 1/fl
X-O y +β/Λ z+Sly
and Xcaty+ β = yz+ 8 or ..(2)
I I /« ” l I y
Tbe lines (I) and (2) will be coplanar if
0 —0 β/m—bla S/y —dlc
I 1/fl II e =0,

I 1/« II y
[See § 8 (A), condition (A)]
0 β/Λ—b/a S/y—d/c
or 0 1/e—1/« II c - H y »o,

I I/* II y
subtracting the third row from tbe second row
or
(αβ—ba) ( y —c) (cS—dy) (a - a ) n
or ------------------------------ e r a ---------------------------------- = v ,
aoLCy cyacc
or (αβ—bx) (y—c)—(cS—dy) (*—o)=0.
This is the required condition.
Ex. 10. Prove that the lines
3x—5=»4j> —9==»3z and x — I =*2y—4=3z
meet la a point and the equation of the plane in which they lie is
3jc— 8>»+3z+ 1 3 = 0 . (Kashmir 1975)
SoL The equations of the given lines are
x —5/3 y —9/4 z
3x—5·= 4y—9=3z or
i/3 1/4 “ 1/3
x —5/3 y - 9/4 z
or
4 “ 3 ”4 ...(I)
X—I y —2 z
and x —l = 2 j —4=3z or
I 1/2 1/3
x —I y —2 z
or
ϋ ~~3~ = 2 -(2 )
The Straight Line 135

Now proceed as io Ex. I above.


Ex. 11. Prove that the lines | (x —9)=·—(^ + 4 )= (2 -5 ) and
6x+ 4y—5z=4, x —5y+2z= 12 are copianar. Find also their point
of Intersection and the equation o f the plane in which they lie.
(Agra 1980)
Sol. The equations of the given lines are
jc—9 y + 4 z —5 .
= — «=»■ (say)
2 0)
and 6 x+ 4 y—5z=4, x —5 y+ 2 z= l2 . • ( 2)
The equation of any plane through the line (2) is
(6;e-f 4y —5z—4)+A (x —5y+ 2z—12)=0
or (6 + λ )χ + (4 -5 λ ) ^ + (_ 5 + 2 λ ) ζ -(4 + 1 2 λ )= 0 . ...(3)
Now if the plane (3) is parallel to the line (I), then we have
(6+A) ( 2 ) + ( 4 - 5A) (—1 ) + ( - 5+2A) (1)=0, or A = - * .
Putting this value of A in the equation (3), the equation of
the plane through the line (2) and parallel to the line (I) is
given by
( 6 - J ) * + ( 4 + f ) .y+(—5—§) z - ( 4 -1 2 /3 )= 0
or 17jc+l7y—17z=0, or x+,y—z= 0. ...(4)
Clearly the line (I) passes through the point (9, —4, 5). This
point (9, —4, 5) satisfies the equation (4) of the plane. This shows
that the line (I) lies in the plane (4). Hence both the given lines
(I) and (2) ate copianar and lie in the plane given by (4).
To find the point of intersection.
The co-ordinates of any point on the line (I) are
( 2 r + 9 ,—r—4, r+ 5). .-.-(5)
The lines (I) and (2) intersect if this point also Iieson the
line (2) Ie., if it satisfies both the equations of the line (2).
Hence we have
6 (2 r+ 9 )+ 4 ( —r —4)—5 (r+5)==4, or r=> —3
and (2 r+ 9)—5 ( —r —4)+2 (r+ 5)= 12, or r = —3.
Since both the equations give the same value of r, the two
given lines intersect. Putting the value of r in (5), the poiut of
intersection is (3, —I, 2).
Ex. 12. Show that the lines J (x —1 )= J (y —2 ) = i (z—3) and
4 x—3y+ l = 0 = 5 .* - 3z -(-2 are copianar. Alsofind their point o f
Intersection. (Burdwan 1978; Gorakhpur 79)
Sol. The equations of the given lines are
136 Analytical Geometry 3-D

χ ~ ι y - 2 z — = r (say),
'2 3 •- ( 1 )
and 4 .^ -3 ^ + 1 = 0 , 5 * -3 z + 2 = 0 . ...(2)
The co-ordinates of any point on the line (I) are
( 2 r + l,3 r + 2 ,4 /+ 3 ) . .(3)
The lines (I) and (2) will intersect i.e., will be coplanar if
this point also lies on the line (2). This point satisfies both the
equations of the line (2), if we have
4 (2r-(-1) —3 (3r4-2)+1 --0, or r- — I
and 5 ( 2 r + l ) - 3 (4r+3)+2--=0, or r —- I .
Since both the equations give the same value of r, the two
given lines intersect. Putting this value of r in (3), the point of
intersection is( —I, —1, —1).
Ex. 13. Show that the lines J ( x + 4 ) = | (jy+6)=» —$ (z—I)
and 3x—2j>+z+S=0=2jc+3.y+4z—4 are coplanar. A lsofind
their point of intersection and the equation of the plane in which they
lie.
Sol. The equations of the given lines are
x + 4 y+ 6 z—I .
I — - - = - 1 = r(say ) . ( 1)
and 3x—2>»+z+5=0, 2x+3.y+4z—4 » 0 . ...(2)
The equation of any plane through the line (2) is
(3χ—2.y+z+5)+A ( 2 jc+ 3 ^ + 4 z — 4)=0
or _ (3+2A) x + ( —2 + 3A)j>+(»+4A) ζ + (5 -4 λ )= 0 . ...(3)
Now if the plane (3) is parallel to the line (I), then we have
(3+2A) ( 3 ) + ( - 2+3A) (5 ;+ (l + hA) ( - 2 ) = 0 , or A=-£-.
Putting this value of A in the equation (3), the equation of
the plane through the line (2) and parallel to the line (I) is
given by
>0
( 3+ ΐ 3) * + ( - 2+ ϊ ? ) ' + ( 1+ Τ3) ί + ( 5~ ϋ ) ·
or 45jc—17^+25z+53=0. -..(4)
Clearly the line (I) passes through the point ( —4, —6, I) and
it satisfies the equation (4) as
45 ( - 4 ) - 1 7 ( - 6 ) + 25 (1) + 53= 0.
Hencethelines (I) and (2) intersect and they liein the
plane (4).
Ty find the point of intersection.
The co-ordinates of any point on the line (I) are
(3 r-4 , 5 r - 6, - 2 r + l ) . .(5 )
The Straight Line 137

The lines (I) and (2) intersect, if this point also lies on the
line (2) if it satisfies both the equations of the line (2). '
Hence we have
3 (3r—4 ) - 2 (5r—6 )+ (--2 r + 1)+ 5 = 0 , or r = 2
and 2 (jr —’4)+ 3 ( S r - 6) + 4 ( —2 r + 1)—4 = 0 , or r= 2 ;
Since both the equations give the same value of r, the two
given lines intersect. Putting the value of r in (5), the point of
intersection is (2, 4, —3).
Ex. 14. Show that the lines
T x - A y + Hz+ 16 = 0 = 4 x + 3 j—2z+3
and x - 3 y + 4 z + 6 —0 ^ x —y + z + 1
are coplanar. (Gorakhpur 1975)
Sol. If the given lines are to be coplanar» then we must have
7 - 4 7 16

4 -2 3 = 0. [See § 8 (C)]

I -3 4 6

I - I I I
Applying Rl - T R i . R2-A R iy R9- R iy the determinant on the
left hand side
0 3 0 • 9

0 7 -6 -I

0 -2 3 5
i
I —1 I I I
3 0 . 9

7 -6 -I >expanding the deter­


minant along the first
-2 3 5 column
3 0 0.

7 -6 -2 2 * by C9- I C 1

-2 3 11
= - 3 {—66—( —66)} = —3 (-66+ 66)= -0.
Hence the given lines are coplanar.
Ex. 15. Show that the lines x + y + z —3=0=»2x+3y+4z—5
138 Analytical Geometry 3-D

ewrf 4*—>>+52—7 = 0 = 2 * —5>>--2—3 are Coplanar9 and find the


plane in which they lie . (Agra 1975)
Sol. We shall first transform the given equations to sym­
metrical forms. The equations of the given lines are
x + y + z —3 = 0 = 2* + 3>>+42—5 -0 )
and 4*—>>+52—7 = 0 = 2 * —5>>-2 --3 . ...(2)
Let I9 m9 n be the d.c.’s of the line (I); then we have
Z+/w+w=0, and 2Z+3wi + 4w=0.
Solving, we get
I _ m _ n /__ m __n
4 --3 ^ 2 -4 ^ 3 -2 0Γ Τ= ^ 2 _ Γ
Λ the d.c.’s of the line (I) are proportional to I, — 2, I.
Now let us find the point where the line (I) meets the plane
2=0. The co-ordinates of this point are given by
*+>>—3 = 0, 2* + 3v —5=0.
SoIving9 * = 4 , >>= —I.
Hence the line (I) passes through the point (4, - I 9 0). Thus
the symmetrical form of the line (1} is
* - 4 _ > > + l_ 2 —0
rx (say).
i = —:2 i
Again let Z1, mi9 W1 be the d.c.’s of the line (2); then we have
4ZX—Wi1+ Sw1= O, and Ilx—Sm1-W1= 0.
Z1 mx W1 Zi _Ot1_ Wi
Solving,
I -+-25 10+4 - 2 0 + 2 ° r 13 7 - 9 *
Λ the d.c.’s of the line (2) are proportional to 13, 7, —9.
Now let the line (2) meet the plane 2=0. Tne co-ordinates of
this point are given by
4*—>>—7=0, 2*—5>>—3 = 0.
Solving, *=16/9, >>= 1/9.
Hence the line (2) passes through the point (16/9, 1/9, 0).
Thus the symmetrical form of the line (2) is
16 I
>—9 z—0
Ij 7 —9 =-- rt (say).
The co-ordinates of any point P on the line (3) are
(^1.+4, —2rj—I, T1)
and that of any point Q on the line (4) are
(13r,+ 16/9, Ir2+ 1/9, - I r 2).
If the lines (3) and (4) [i.e. (I) and (2)] are coplanar, then for
some values of r2 and r2 the points P and Q coincide. Thus we
have
The Straight Line 139

Γι + 4—13r2+(16/9), -*-(5)
- 2 r ,- l- 7 r ,+ ( l/9 ) ,. ...(6)
and Ti = - 9r*. ...(7)
Solving (5) and (7), we get r,=> —10/11, r*= 10/99.
The values of r, and r2 also satisfy the equation (6) and
hence the given lines are coplanar. Putting the value of r2 (or of
r2) the point of intersection P (or Q) is ( —9/11, —8/11, —7/11).
Now the equation of the plane in which the lines (3) and (4)
[t.e. (I) and (2)] lie is given by
x —4 y+ l z

I I =0 [See §8 (A)]

13 7 -9
or ( x - 4 ) ( - 2 ) ( - 9 ) -7.1}- (y+1) { 1 .(-9 )-1 (13)}
+ z { ! - 7 - ( - 2 ) (13)}=0
or (JC-4). I l+ 0 > + l) .2 2 + z .(33)=0 or x + 2 ^ + 3 z = 2 .
Ex. 16. A, A'; B, B'; C, C are points on the axes. Show that
the lines of intersection of the planes A'BC, ABC'; B'CA. BCA
C'AB, CA'B' are coplanar.
Sol. Let the co-ordinates of the points A and A' be (a, 0, 0)
and (a', 0, 0); B and B' be (0, b, 0) and (0, b', 0); C and C be
(0, 0. c) and (0, 0, c‘).
The equations (in the intercepts form) of the planes A'BC
and AB'C are respectively given by
* +* + is I and * + £ + * . . 1 .
a' b c a b c
Therefore, the equation of any plane through the line of inter­
section of these two planes is given by

(^+H - - 1)+* ( s + F + i - 1) = 0’ -(I)


for some value of the constant A.
Choosing A= I, the equation (I) of the plane [containing the
line of intersection of the planes A'BC and AB'C'] becomes

( ^ +τ - ' Κ +Κ - - Ή
or
...(2)
The symmetry in the equation (2) shows that the lines of
140 AnalyticaI Geometry 3-D

intersection of the other two pairs of planes also lie in the plane
given by (2).
Hence the lines of intersection of the given three pairs of
planes are coplanar.
Ex. 17. Find the equation of the plane through the line
x/l<=ylm = zIn
and perpendicular to the plane containing the lines
x\m —yln*=tzH and x/n—y ll—zlm.
I Lucknow 1982, Ranchi 79, Punjab 77, Agra 75, M.U. 89(S)]
Sol. The equation of any plane through the line
x ll—ylm =z/n is
A x+ B y+ C z= 0, ···(!)
where Al+ Bm + Cn=O. ·*·(2)
Also the equation of the plane through the lines
x/ffi=ji/n=>z// arid jr/n=^//=-z/m
(note that both of these lines are passing through the origin) is
X y Z

m n I =:0, [See § 8(A)]


I
n I m

or (»mi—/*) x+ (ln —m*) y + (m i- n*) .z —0. ...(3)


According to the question, the planes (I) and (3) should be
perpendicular the condition for which is
A (mn—i*)+B (ln—mt)+ C (ml—n*)=0. ·*·(4)
Solving the relations (2) and (4), we get
________ A_ _____ _________ B ___
m (ml—n*)—n (In—m*) “ n (mn—l*)—t tml—n*)
C
i </n—m4)—m (mn—I*)
B
or BiiZ-IBBt - B 1Ifn m 11 mn*—Hit - I n lt +in*
C
/*«—Im * -nfn+ml*
B
or (m*—nt)l+ m n (m—n) (Hi - I i) m+ni (n- I)
C
(/*—m*) n+/«t (t—m)
_______ A___ ____ ^ _____ B _____
or (m—n) (m l+ n l+ m n f^in —I) ( ψ ι+ b t +HI)
The Straight Line 141

C
(/—m) (In+ mn+ lm)
A _ B _ C
m —n n—l l —m
Patting these proportionate values of At B, C in (I), the equa­
tion of the required plane is given by
(m—n)+ (n—l) y + (l—m) ζ » 0 .
Ex. 18. Find the foot and hence the length o f the perpendicular
from the point (5, 7, 3) to the line
( x - 15)/3 - {y—29)/8=I z - 5 ) /( - 5).
Find the equations of the perpendicular. Also find the equation o f the
plane In which the perpendicular and the given straight line lie.
Sol. Let the given point (5, 7, 3) be P.
The equations of the given line are
( x - 1 5 )/3 = 0 > -2 9 )/8 = (z -5 )/(-5 )= r (say). ...(I)
Let N be the foot of the perpendicular from the point P to
the line (I). The co-ordinates of N may be taken as
(3r+15, 8r+29, —5r-f-5). ·*·(2)
A the direction ratios of the perpendicular PN are
3 r+ t5 -5 , 8 r+ 2 9 -7 , - 5 r + 5 - 3 ,
l.e. are 3r+IO, 8r+22, - 5 r + 2 . ...(3)
Since the line (I) and the line PN ute perpendicular to each
other, therefore 3 (3r+ 10)+ 8 (8r+22)—5 ( —5r+2)«=0
or 9 8 r+ 196=0 or r —- —2.
Putting this Value of r in (2) and (3), the foot of the perpen­
dicular N is (9, 13, IS) and the direction ratios of the perpendicu­
lar PN are 4, 6, 12 or 2, 3, 6.
A the equations of the perpendicular PN are
(* -5 ) /2 = ( y —7)/3= (z —3)/6. ...(4)
Length of the perpendicular PN
=»the distance between P(5, 7,.3) and N{9, 13, 15)
W { (9 -5 )« -K 1 3 -7 )* + (1 5 -3 )* } = 1 4 .
Lastly the equation of the plane containing the given line (I)
and the perpendicular (4) is given by
x —15 .v—29 z—5
3 ‘
OO

_ 5 =0 [See § 8 (A)]
2 3 6
142 Analytical Geometry 3-D

or (x—15) (48 + 15) -(jp -2 9 ) (18 + 10 )+ iz-5 ) (9 -1 6 )= 0


or 9jc—Ay—z—14«=»0.
§ 9. To determine the equations of a straight line intersecting two
given lines.
Case I. I f the equations of the two lines are given in symmetri­
cal forms.
Let the equations ot' the given lines be
(X-OL1)IU=(^-/3i)//w1= ( z - y i) /n i= r 1 (say), ...(I)
and (x —oLi)!l%= ( y - fii)lm2= (z—yi)lna= ri (s&y). ...(2)
The co-ordinates of any point P on (I) are
(/|Γ ι+α,. IW1V1+/3 u η,Γ,+ y j
and those of any point Q on (2) are
(U't+OLi, mtra+j32, w2r2+ y 2).
Now we are to find the equations of a line which intersects
the lines (I) and (2). Suppose the required line intersects the lines
(I) and (2) in the points P and Q respectively.
Then the required line is one which joins the points P and Q.
The values of r( and rs will be determined by some additional
given conditions.
Case II. I f the equations of the two given line are given in
general form.
Let the equations of the given lines be
U1BaO=Vt and U2=O=Vi .
Now the equations of the required line intersecting both the
given lines are Ki-I-MiVl =O and κ*+/*2ν,=0,
where the values of μι and μ2 are determined by some additional
given conditions.
SOLVED EXAMPLES (G)
Ex I. A line with direction cosines proportional to (2, 7, —5)
is drawn to intersect the lines
(x -5 )/3 = 0 > —7)/(-1 )= (2 + 2 )/1
and ( * + 3 ) /( - 3 ) = ( .y - 3)/2= (z - 6)/4.
Find the co-ordinates of the points of intersection and the length
Intercepted on it. [Kanpur 1980, Lucknow 79, Bundelkhand 78]
Sol. The equations of the given lines are
(* - 5 ) /3 = 0>—7 ) / ( - l) = ( z + 2 ) /l= r , (say), ...(I)
and (x + 3 )/(—3 )= 0 > -3 )/2 = (z—6)/4= r2 (say). ...(2)
Any point P on (I) is (3rt + 5 , - r i + 7 ,/ - j - 2 ) ,
and any point Q on (2) is ( —3r*—3, 2r2+ 3 , 4r2+6).
The Straight Line 143

The direction ratios of PQ are


(3ri+ 3r2+ 8, —/ i —2ra+4, fi —4ra —8). ·*·(3)
Suppose the line with d r.’s (2,7, —5) meets the lines (I) and
(2) in the points P and Q respectively.
Then the d.r.’s 2, 7, —5 will be proportional to the d r.’s
given by (3).
, 3ri+ 3r2+ 8 —Tj—2ra+ 4 ^ r j - 4ra—8
·* 2 7 “ =5 ..(4)
From the first two of (4), we get
7 (3ri+ 3ra+ 8 )= 2 ( —T i- 2ra+4)
or 23r,+25ra+48 = 0. ...(5)
And from the 1st and 3rd of (4), we get
2 (T1—4ra—8) = —5 (3^+ 3/^+ 8)
or 17/^ + 7/8+24=0. ...(6)
Solving (5) and (6), we have rx= / a= —I.
Now putting these values of rr and ra the co-ordinates of the
points of intersection are P(2, 8, —3) and Q(0, I 9 2).
The required length intercepted by the lines (I) and (2) on
the line with d.r.’s (2t 7, —5)
“ ^ e = V [ ( 2 - 0 ) » + ( 8 - l ) 2+ ( - 3 ~ 2 ) a]
= ^ (4 + 4 9 + 2 5 ) = ^(78).
Note. The equations of the line PQ in the above Ex. I are
given by (* -2 )/2 = 0 > -8 )/7 = (z + 3 )/(-5 ).
Ex. 2. Find the equations to the planes through the point
(I, 0, —I) and the lines
4 x—y —13=0*= 3^—4z —I and v—2 z + 2 = 0 —jc—5
and show that the equations to the line through the given point which
intersects the two given lines can be written as x = y + \= z + 2 .
Sol. The equations of the given lines are
4 x —y —13^0, 3j>—4z—1=0 ...(I)
and y —2z + 2 = 0 9 x —5=0. ---(2)
The equations of any planes through the given lines (I) and
(2) are respectively given by
4x—y —]3+pi (3y—4z— 1) = 0,
and (γ —2ζ+ 2)+ μι (x—5)=0.
Iftheseplanespass through the point (IvO9 - I ) 9 we get
4 —0—13+/xi(0+4 —1)=0 giving /^ = 3, and 0 + 2 + 2 + /^ ( 1 - 5 ) = 0
giving/A2= I .
Putting these values of μχ and μ2ι the required equations of
the planes are
144 Analytical Geometry 3-D

4 x—y —13-f 3 (3y—4z—1)=0,


and (y -2 z + 2 ) + 1.( jc—5)=-0
or x + 2 y —3z—4 —0, and x + y —2z—3—0. ...(3)
Tbe equations (3) are. the general equations of a line through
the given point (I, 0, —I) and intersecting the given lin es(l)
and (2).
Transforming the equations (3) to the symmetrical form,
we get
* —0 - ^ + 1 - 2 + 2
or x = y + l= z + 2 .
i i T
Ex. 3. Find the equations to the straight line drawn from the
origin to ‘Intersect the lines
2 x + Sy + 3z—4=0 = x —y —S z - 6
and 3 x - y + 2 z —l= 0= x-{-2y—z—2.
Sol. The equations of any line intersecting the given line are
(2 x + 5 y + 3 z - 4 ) + Ml ( x - j > - 5 z - 6 ) = 0 , 1 r ^ - s o n m
and ( 3 x - y + 2 z - 1)+ μ* (x + 2 ^ —r —2)=0. f ls e e 9 y O ')J
If this line passes through (0. 0, 0), then we have
—4 - 6 /^ = 0 or μι=* —2/3; and —1 —2μ»=0 or μ»=* —1/2.
Putting these values of μχ and μ«, the equations of the required
line are
(2 x + 5 y + 3 z -4 )-(2 /3 ) (x—y —5z—6)=0
and (3x—y + 2 z —l) —\( x + 2 y z—2)=0
or - 4x+17j> + J9z=0 and 5x—4y + 5z =0.
Ex. 4. Find the equations to the line drawn parallel to Jx = y = z ,
so as to meet the lines 5x—6=4y-|-3=z and
2x—4 = 3 y + 5 = z .
Sol. The general equations of the line S x - 6 = 4 y + 3 —z
may be written as
5 x - 6 - ( 4 y + 3 ) = 0 and 5x—6 —z= 0
or. 5x—4>>—9 = 0 = 5 x - z —6. ...(I)
Similarly the general equations of the line 2x—4=»3y+5=z
may be written as
2x—3v—9 = 0 = 2 x —z -4 . ...(2)
The equations of any line intersecting the given lines [i.e.,
the lines (I) and (2)] are
{5x—4y—9)+μ^ (5x—z —6)= 0
and (2x—3 ^ - 9 )+ μ , (2x—z—4 )= 0
or 5 (i + μι) x—4y—μ,ζ—(9+6μ,)=0)
and 2 ( 1 + m»)x —3y—μ*Ζ— (9+ 4μ 2) = θ / -.(3)
The Straigkt U m 145

If the line (3) is parallel to the line j = j = j . thcn this


latter line is perpendicular to the normals of each of the two
planes given by (3), so that we have
4-5 ( Ι + λ ) + 1 · ( - 4 ) + 1 ( - λ ) = 0 giving P1= - 1 6/19
and 4-2 (1 + p.l+M - 3 ) + 1 .(-|* > = 0 giving * = - S P -
Patting these values of μχ and /½ in (3) and simplifying, the
required equations of the line are
15jc—76y+16z—75=0 and 4x—2 1 y + 5 z-4 3 = 0 .
Ex. 5. A line with direction cosines proportional to 2, I, 2
meets each o f the lines given by the equations
x = y + a = z 9 x+ a= 2y= 2z.
Find the co-ordinates o f each o f the points o f intersection.
y (Utkal 1*77 S)
Sol. The equations of the given lines are
*/i = (y + « )/i = * / i = ri («y). -·(*)
and (jc+e)/2=y/l = z /l = r t (say). - (2)
Any point P on (I) is (rj3 rt - a, rjj,
and any point g on (2) is (2rs - a, r*, r,).
The d.r.’s of P g are T1- 2re+ n , r , - rt —a, rt - r2.
If the d.r.’s of P g are proportional to 2, 1,2, then
- 2/*+a r ,- r ,
2 _== Ϊ = 2 -(3)
From the first two of (3), we get r, - 3a= 0 or rt= 3 a and
from the last two of (3), we get rt —rt - 2 a =0 or r,=fl.
Putting these values of rx and Ts, the co-ordinates of the
points of intersection are P (3a, 2α, 3a) and g (a, a, a).
Ex. 6. Pouf toe equations to the line intersecting the lines
x - \= y -2 —1, 2 x f 2=2y z-J-1
and parallel to the line { (x — l)«=(j I )=-1 (z 2). (Agra 1*77)
Sol. Here the required line being parallel to the given line
i (jc -1 )= (^ - l ) = | (z -2), will have its d.r.’s 2, I, 3.
Proceeding as in Ex. I above, the equations of the required
line P g are | (x \)= -y= i (z !).
Ex. 7. Find the equations o f the line intersecting the lines
x—a-=y=z—a, x+ a = y= £ (z-fa) and parallel to the line
i l x - o ) = ( r a )= i (z—2a).
Sol. Thi* equations of the given lines are
('v —a)./i —(ζ --α )Ί ^ r 1 (say) . .(1)
146 Analytical Geometry i-D

and (x+a)/1 = j/l= (z + a )/2 = r t (say). -(2 )


Any point on (I) is P (Ή -α, rlt rt + a) and any point on (2) is
Q Irt -O t r* 2rt - e ) .
The d r.’s o f PQ are rx—rt +2a, r , - rfc r4—2τβ+2α.
If the line PQ is parallel to the given Iin ^
then the d.r.’s of PQ are proportional to 2, 1 ,3.
. . f t—rt + 2a Tt -T t r ,-2 r ,+ 2 e
2 “ I - 3 -(3 )
From the first two of (3), we get r ,-r ,= 2 a I
and from the last two o f (3), we get 2r,—rt= 2a J ...(4)
Solving die equations (4), we get r,= 0 , r ,= —2a.
Fnftingthesevaliies of r, and f*, the co-ordinates of the points
of intersection are F (a, 0, a),and g ( —3a, —2a,,—5a).
the equations of PQ are (x—a )/2 = y /l= (z —a)/3.
Ex. β. Find the equations of the straight line through the origin
and cutting each o f die lines
(x - X jfl1= (y - 7 ,)/n»i= (z -Z1Va1
and (x -J e*W t=(7-7*)/»*=(z - **)/*►
SM. The equation of a plane through the first line namely
(x—χ1)//1= ( 7 - Λ ) /» ι = ( ζ - ζ 1)/η1 is
d ( x —x1) + f t( 7 —7J +C (Z-Z1)=O, -(1 )
where Bml -J-O1= 0 . ...(2)
If the plane-(l) passes through (0 ,0 ,0 ), then
AxH-By1+ Cz4=O. ···(!)
Eliminating At Bt C from (I), (3) and (2), the equation of the
plane through the origin aod through the first line is
X -X 1 J-T i Z -Z 1

JCi 7i Zl
/i IW1 "l
Adding the second row to the first row, we get
y Z
i x
Jl Zl >0 ...( 4 )
i *
! /i mx *1
or (ntPi—Oi1Z1) Jf-K /A —« A )y + (« » A —ZiTk) z= 0 . ...(4')
The Straight Line 147

Similarly the plane through the origin and through the second
line is
X y Z

y* Zl
-.(5)

I2 m 2 M2
or (/¾V2-TntZj χ + ( / 2ζ 2- μ2χ , ) y + (OttXt - I t f t) z = ° C5')
The planes (4) [or (4')] and 5 [or (5') together give the required
line.
Ex 9. Find the equations to the straight line drawn through
the origin which will intersect both the lines
I y+3 z - 5 x —4 y + 3 z —14
and
I “ 4 “ 3 2 " 3 4
Sol. The equation of any plane through the first line is
A (x - 1)+21 (y + 3 )+ C (z—5)=0, ...CD
where 1..4+4.a+3.C =0. ...(2)
Tf the plane (I) passes through (0, 0, 0), we have
—A + 3B —5C=0. ...(3)
Solving (2) and (3). we get .4 /2 9 = .8 /(-2 )= 0 /(-7 ).
.Substituting these proportionate values of A, B, C in (I), the
equation of the plane through the origin and the first line is
29x—2y—7z«0. -C 4)
Similarlythe equation of the plane through the· origin (Md
the second line is
9x —2y—3z=0. ·· C5)
The planes (4) and (5) together give the required line.
Ex. 10. Find the equations o f the line through· the point
(3. I, 2) parallel to the plane 4;c+y+5z=0 so as to cut the line
x + 3 = y + 1= 2 (z—2). Find alsq the point of intersection.
Sol. Let the point P be (3, 1, 2).
The equation of the given plane is 4 x + y + 5 z= 0 . .. (I)
The equations of the given line may be written as
(x + 3 )/2 = ( y + l) /2 = ( z - 2 )/l= r (say). ...(2)
Let the line through P (3, I, 2) parallel to the plane (I) cut
the line (2) in the point Q.
The point Q may be taken as (2 r-3 , 2r—I, r+2).
.·. the d.r.’s of PQ are 2 r - 6 , 2r~2, r. •►■(3)
148 Analytical Geometry 3-D

But PQ is parallel to the plane (I) and hence PQ is perpendi­


cular Io the normal of the plane (I) whose d.r.’s are 4, I, S and
βΛ n/A VlflVP
(2r—6) 4+ (2r -2).1 4-r.5=0 or r=26/15.
Putting this value of r, the co-ordinates of the point of
intersection Q are (7/15, 37/15, 56/15).
Also from (3), the d.r.’s of PQ are
-38/15, 22/16, 26/15 or 19, - II, -1 3 .
.'. the equations of the line PQ are
x —3 y —I z - 2
19 = T f i ^ - T y
Ex. 11. Find the equations o f the line through (a, b, c) which
is parallel to the plane lx-\-my+nz=*0 and intersects the line
OiX-VbiyA C1ZA ^1=
Sol. The required line is the line of intersection of the two
planes given below.
(i) The plane passing through the point (a, b, c) and parallel
to the plane IxA-myA-nz^O. The equation of this plane is clearly
given by
I (x-a)A -nt (v—b)A-n (z—c)=0. ...(I)
(ii) The plane through the point (a, b, c) and the given line
aiXA-b1y-VclzA-di=0=a^xA-biyA-ct2A-dt. .(2)
The equation of any plane through the line (2) is
{.aixA-biyA-clzA-d)+ p(aixA-bty+ ctzA-dd=Q· -(3)
If this plane (3) passes through (a, b, c), we get
(O1O + ^ + C 1C-I- dx)Arp ( + 626+ c,c+ <f2)= O
or μ= —(aja+ftjfr+CjC-t-i/^/iaja+^+CjC+i/g).
Putting this value of μ in (3), the equation of the required
second plane is given by
fli*+ bty -Vc1ZA- dt _ atx -V^y-VciZ-Vdt
ala+bib-Vclc-Vd1 ^ a iO+otb [-C2C-M2 -.(4)
Hence the equations of the required line are given by the
planes (I) and (4).
§10. To find the perpendicular distance of a point from a line and
the co-ordinates of the foot of the perpendicular.
(Punjab 1979; Meerut 84S)
Let P (xx, yx, Z1) be a given point and let AB be a given line
Let the equations of AB in symmetrica- iorm be
X-ot y - p _·? y
I m (I)
The Straight Line 149

where /, m, n are the d.c.’s


of (1): The line (I) is passing
through the point A (ot, β, γ )
and has direction cosines
I, m, n. From P draw PN
perpendicular to AB. Now
it is required to find PN. From
the right angled AAPN, we
have
PNi =APt -A N i. ...(2)
Now A P = the distance between (α, β, γ) and P (x1( Vi, Z1)
= V{(^i α )*+ (^-β)*+ (ζ,-7)*}, ..(5 )
and A N = projection of AP on AB i.e. the projection of AP
on a line whose d.c.’s are I, m, n
=(X1-O ) l+ iy i—β) m + (zi - y ) n. ...(4)
Putting the values from (3) and (4) in (2), we get
PNi = ^ x 1- cOHC fi -β )* + (ζ ι- γ 'ή
—{(*i a) I-Uiy1- β) "'+ (Z 1- y) n f
={(*1 - « )2+ 0 ’ι -β ? + (Ζ ι - Ύ Ϋ ) U 2- U m i + η * )
- { ( * ι - « ) /+ ( λ —Λ w + (z i-y ) «}2
[V /2+ m 2+/»2= l ]
={m ( z x -y ) - n ()·ι-β)}*+{η (X1- O ) - I (Z1- γ)}2
+ {(( y i - P ) - m (¾ -* )} 2
[By using Lagrange’s identity]
m n 2
n I t
I m
+ +
Λ — « Z i - Y Z i- y X 1- O X 1-O t y t - β

-(5)
Note. In the equations (I) of the line AB, I, m, n have been
taken as the actual direction cosines of the line. In case direction
ratios a, b, c of AB are given, we should either first find the direc­
tions cosines of AB or we should divide the R.H.S. of (5) by
(a2+ 62+ c*).
To find the co-ordinates of the foot of the perpendicular N.
Since N, the foot of the perpendicular, is a point on the
line AB given by (I), its co-ordinates may be written as
(Ir+<x, mr+ β, nr+γ). -(6 )
The d.r.’s of PN are Ir+ a - X1, mr+ β —yu nr+ γ —Z1.
Also PN is perpendicular to AB.
150 Analytical Geometry 3-2)

Λ (lr+ct—Xi) l+ {m r+ fi—y 1).m + (nr+ y—zl).n=0


or '* = ! ( X 1 - a ) + m O ’1 - 0 ) + « ( Z 1 - γ)
or r—l (xl ~ *)+ m Iyl - β)+η (ζχ—γ) [V /*+/«*+«*= 1]
Putting this value of r in (6) the co-ordinates of N are
obtained.
SOLVED EXAMPLES (H)
Ex. I. From the point P (I, 2, 3) PN is drawn perpendicular
to the straight line $ (jc—2)= i (y —3 )= J (z—4). Find the distance
PNy the equations to PN and co-ordinates of N. (Rohilkhand 1981)
Sol. The equations of the given line AB (say) are
(x ~ 2 )/3 = (.y -3 )/4 = (z -4 )/5 = r (say). -( 1 )
T h elin e(I) is passing through the point A (2, 3, 4). Since
N, the foot of the perpendicular, is a point on the line (I) [i.e. AB\
the co-ordinates of N may be written as
(3r+2, 4 r+ 3 , 5r+4) . (2)
.*. the d.r.’s of PN are 3 r+ 2 —I, 4r-f 3 - 2 , 5 r+ 4 —3 i.e.
are 3 r+ l, 4 r + l, 5 r+ l. —(3)
The d.r.’s of the line AB whose equations are given by (I), are
3, 4, 5.
Since PN is perpendicular to AB, we have
3 .(3 r+ l)+ 4 .(4 r+ l)+ 5 .(5 r-M )= 0 , or r = - 6/25.
Putting the value of r in (2), we get
W=(32/25, 51/25, 14/5).
Λ /W = the distance between the points P and N

-J S -o r^ N M P ? ·
Putting the value of r in (3), the d.r.’s of PN are 7/25, 1/25,
—5/25 i.e. are 7, I, - 5 .
*. the equations to PN i.e. of a line passing through
P (I, 2, 3) and having d.r.’s 7, I, —5 are
x - l y —2 z - 3
. 7 - 1 --5 ·
Ex. 2. How for is the point (4 ,1 ,1 ) from the line of inter­
section o f x + y + z —4 = 0 = * —2y—z —4 ?
Sol. Let the point (4, I, I) be taken as P.
The equations of the given line .42) (say) are
(x—4 )+ y + z = 0 , (x—4)—2y—z=0.
Solving for x —4, y, z, we get
x —4 y Z- x —4 y z
- 1 + 2 - 1+ 1“ —2 —I ·°Γ I — 2 “ —3* -(I)
The Stntigkt Line ISl

Equations (I) are the symmetrical form of the given line A B .


The line (I) has d.r.’s I, 2, —3 and passes through the point
Λ (4, 0,0).
Draw PN perpendicular to the line AB given by (I), so that
AT, the foot of perpendicular, lies on (I). Hence the co-ordinates
of N may be written as (r+4, 2r, —3r). —(2)
The d.r.’s of PN are
r+ 4—4, 2r—I, —3r—I u . r, 2r—I, —3r—I.
Since P N is perpendicular to AB, we have
r.l+ (2r~ l).2+ (-3r-I) (-3)= 0, or r = -1/14.
Putting the value of r in (2), we get
*=(55/14, -1/7,3/14)
the distance of P (4 ,1, I) from the given line (I)
=Uie distance between the points P and N

Ex. 3. Pm d the equations o f the perpendicular from (I, 3, 7)


on the line x = 3 -5 1, y = 2 + 5 1, z=-7+2/. (P^fah 1981)
Sol. The equations of the given line, say ABt may be
written as
(x-3)/(-5)=CK-2)/5=(z+7)/2=f. ...(I)
Now proceed as in Ex. I above and get
*=(-53/54, 323/54, - 212/54)
and the equations to P N are
x -1 y - 3 z - 7
107 —161 —670 *
Ex. 4. Find the loots e f a point which mooes so that its
distance from the line x —y = —z is twice its distance from the plane
x —y + z = I. (A gp att)
Sol. Let the given point be P (¾, y u z,) whose locus is
required to be found.
The equations of the given line AB are
*/1=7/1 = 2 /(-1 )- --(I)
The line (i) is clearly passing through A (0,0,0) and has
d.r.’s I, I, —1. Hence *Lc.’s /, m , n of (I) are l/v<3, l/y3
—1/\/3. Let Px be the perpendicular distance of P from thcline
(I), then Iqr § 10, we have
1/V3 -1/V3 -I/V3 1/V3 1/V3 W 3 P
Λ*=
7 ,-0 Z1-O Z i— 0 X i-O * i- 0 7 i - ° I
152 Amdftieel Geometry 3-D

or A 1= I Uft+ Λ Ρ + ( - X i - ΖιΡ+(Ρι—* ιΗ
= I (Xia+ V + ft* + A ft+ ftX i-X ift) -(¾
Let be the perpendicular distance of P (Xb yu Zj) from the
plane x p + z = I. Then
„ X i-f t + z .- l
p* ν α + Η » ) ' ·<3>
Aeuxding to the given problem, P1-=Tpt.
Squaring, Pi1= 4A*
or I (ft1+P11+Zi, + P ift+ z1x1- x 1p1) = i ( X i- P i+ f t - 1J*
or Xi*+Jie+Z i*+7A +ftX i -X iPi=2 (ft*+Jft*+*i*+l —2ftPi
+ 2 ftft~ 2 * i—2 ftft+ 2 fi—iz,)
or Xia4-Pis4-Zi1-SpiZ i+ ? z A -3 x iP i-4f t + 4P i - 4A + 2=i0·
the locus bf P (x„ p», Z1) is given by
jZ+p^+z1—5pz+3zx - 3xp - 4x+4p—4z+ 2 = 0 .
K r 5. Pied the Ioaa o f a point whose distmcr from x-axis is
twice its distance farm the yz-plaee. (Agra M l)
SaL Let the given point be P (xlt Jb z,) whose locus is
required to be found.
The equations of x-axis are
(x —0)/1 = (p —0)/0= (z—0)/0, —(I)
<*«»«· the x-axis passes through (0 ,0 ,0 ) and has dx.’s 1 ,0 ,0 .
Ifp 1 be the perpendicular distance of P(XfePfeZ1) from (I),
then
I I 0
A 1=
Pi , Z1 Z1 X1
or A1= [-Zi)*+(Pi)* or Pi1=P1*+Z1*. ...(¾
Let P t be the perpendicular distance of P iiX1, Jf1, f t ) from the
pz-plane. Then p,=x-co-ordinale of P=X1. ...(3)
According to the given problem, we have
Pi=Tpt or p f= 4 p t*
ο*- P i^ ft 1= 4X1*, [using (2) and (3)]
Λ the locus of the point Pix11pto Z1) is 4x* -ρ*--ζ*=0.
Ex. L Fied the length o f the perpendicular drawnfrom origin
to the Uee x+ 2p+ 3z+ 4= 0«= 2x+ 3p + 4z+ 5. Also find the equa­
tions o f this perpendicular and the co-ordinates o f thefoot o f the
perpendicular. (Meant D M S, «4 R)
SuL The equations of the given line AB (say) are
x + 2 p + 3 r+ 4 = 0 = 2 x + 3 p + 4 z+ 5 .
The symmetrical form of the above given line is
The Straight Line 153

(* —2)/1 = (p + 3 )/(—2)= (z—0)/1. ...(I)


The line AB given by (I) is passing through A (2, —3, 0) and
has d.r.’s I, —2 ,1. Here the point P is (0, 0, 0).
Draw PN perpendicular to AB, so that AT, the foot of the
perpendicular lies on (I). Hencethe co-ordinates of AT maybe
written as (r+ 2 , —2 r—3, r). ...(2)
The d.r.’s of PATare r + 2 - 0 , - 2 r - 3 - 0 , r - 0
i.e. r+ 2 , - 2 r - 3 , r. ...(3)
Since PN is perpendicular to AB, we have
(r+ 2).l + ( - 2 r —3) (—2 )+ r.l= 0 or r = - 4/3.
Putting the value of r in (2), we get
AT=(2/3, -1 /3 , -4 /3 ).
Again putting the value of r in (3), the d.r.’s of PAT are
2 / 3 ,- 1 / 3 ,- 4 / 3 i.e. 2 , - 1 . - 4 .
.*. the equations to PAT i.e. the equations of a line passing
through P(0, 0, 0) and having d.r.’s 2, —I, —4 are
Jt-O
- . . —v- - __—-------
0 z—0 or x—— y - —--- 2 ·
2 — —I ~ —4 2 - 1 - 4
The length of the perpendicular PAT
= th e distance between the points P and AT

Ex. 7. Find the equations o f the two planes through the origin
which are parallel to the line (x - 1)/2 = Cv+- 3)/( —I) = ( z + 1)/(—2)
and distant 5/3 from it. (Meerut 1976, 89, 89 S)
Sol. Let the equation of any plane through the origin
be
ajt+Ay+cz=0. ...(I)
The equations of the given line arc
J t - 1 p+3 z + 1
2 ” -I * -2 ’ ...(2)
(f the plane (I) is parallel to the line (2), then the normal to ­
il) whose d.r.’s are a, b, c will be perpendicular to the line (2) and
hence we have
2.a—l.b —2.c=0 or 2a—b—2c=0. ...(3)
According to the given problem, the plane (I) is at a distance
5/3 from the line (2).
Hence the distance of (I, - 3 , - I), a point on the line (2),
from the plane ( I) = 5/3
154 Analytical Geometry 3-D

g .l+ 6 .( - 3 ) + c .( - l) 5
V(a*+b*+c*\ ” 3*
On squaring. 9 (a —3 6 -c )2=25 (a2+ 6 2+ c2)
or 9 (a2+962+ c2 -6a6 —2ac+66c)=25 (a2+62+ c 2)
or 16a2—56b2+ 16c2+ 54ab + 18ac —56be= 0
or Sa2—2862+ 8c2+ Ila b + 9ac —286c=0. ...(4)
Putting the value of 6= 2 (a - c) from (3) in (4), we get
8a2—28 x 4 (a -c )2+Sc2+27a.2.(a-c)+9ac-2bc.2 ( a -c )= 0
or —50a2+ 125ac —SOc2=O or 2a2 -5tfc+2c2= 0
or (a —2c) (2a—c)= 0 or a=2c, Jc.
For a—2c, from (3). 6 = 2c
and for a = £c, from (3), 6=a —c.
Putting a=2c, 6=2c in (I), the equation of one plane is
2 cjc+ 2 c v + c z = 0 or 2x+ 2j’+ z = 0 . -.-(5)
Again putting a= Jc, 6 = - c in (I), the equation of the second
plane is \c x —cy+ cz= 0 or x —2y+2z=0. ...(6)
The equations (5) and (6) are the equations· of the required
planes.
§11. Intersection of three planes. (M.U. 1990)
Let the equations of three planes be given by
U1= O 1X + b Ly + C1Z + J 1 = O 9 ...(I)
U 1= O 2X + b 2y + c tz + d %= 0 , ...(2)
and ih = a t x +b ty + c tz + d t = 0 9 ...(3)
where M1, U 1 and M3 denote respectively the left hand members in
the equations (I), (2) and (3) No two of these three planes are
parallel.
We know that two non-parallel planes intersect in a straight
line and hence we get three lines of intersection by taking two
planes at a time out of the three planes given by (I), (2) and (3).
There arise following three cases :
Case I. The three lines of intersection explained above may
coincide i e. the three given planes have a common line of inter­
section.
Case IL The three lines of intersection explained above may
be parallel to each other and no two of them coincide. In this
case the three given planes form a triangular prism.
Case III. The three lines of intersection explained above may
intersect in a common point. In this case the three planes inter­
sect in a point.
The Straight Line 155

Before proceeding to prove the actual theorem, for conveni­


ence, we make use of some notations given as follows :
Consider the matrix (or a rectangular array) '.
Ul bi Cl di
U2 b3 C2 dt ...(i)

Uj b, U2 dt
Let the determinant obtained by omitting the first column in
(i) be denoted by Λ ι i-c. we put
h Cl Cfl
A 1= bt C2 Cft
bt C2 O3
Similarly, the determinants obtained by omitting the second,
third and fourth columns will be denoted respectively by Δ*. Δ«
and Δ*· Thus we put
Ui Cl di I Ol bi di

Ai= U2 C2 d3 > Δ .8= O3 b3 d3


Uj C3 Cit O3 bt dt
Ul bi Ci

Δ 4= U2 bt ct •
Uj b3 C3
The symmetrical form of the line of intersection of the planes
(I) and (2) is [See § 4]
. I bjdt M i \ ,, ( CtiQf-dtfli\
\α φ 2- α φ ι } y \ Uib2 -UjblJ____ z —0
A1Ca—Aa^1 ““a A -A 2A1* ...(4)
where αφ%—αφ&£ιΟ.
Now we shall discuss the three cases given above in detail as
follows :
Case I. The three planes intersect in a common line.
The equation of any plane through the line of intersection of
the planes (I) and (2) is given by
W1+ An2=O
or (OiX+biy+CiZ+di) + λ (atx +b 2y d t)= O
or (ai+λα*) x +(6 i +A62) ^+(C1+Ac2) 2 + (^ ^ 4 )= 0 ....(5 )
if the three given planes intersect in a common line, then for
156 Analytical Geometry 3-D

some value of λ the plane (5) should represent the plane (3). Thus
comparing the coefficients in the equations (5) and (3), we have
0i+Atfa^ 6i+A6a C1 - l A c 2 dx+Xd2_ . *
az C3 d%
Λ tfi+Atf2 /Xtf3= 0,
Ci+A fy-f^*=0,
C i+ A c s — /xca«=»0,
and —^itZj=O.
Now we are to eliminate two arbitrary constants λ and μ and
this can be done from any three out of the four equations given
above. Hence eliminating A and μ from any three equations taken
at a time out of these four equations, we have the conditions as
01 bi Cl

aI b* Cj = 0 Le. A 4=O

03 h Cj
01 h 4
*<■,

A j= O
O

h
I

0a Cli

03 bz <1,i
01 Cl di

0a Ca dt = 0 i.e A j- O

az <t di
bi Cl dx

d* =0 i.e.
Il

V
>
P

C2

b* C3 d,
Hence the three planes will have a common line of intersec­
tion of Δ λ—Ο, Δ ι= 0 , Δ 2= ° 3η(1 Δ i —O.
Alternative method. If the three given planes (I), (2) and (3)
intersect in a common line then the line (4) [the lineof intersection
of the planes (I) and (2)] will lie in the plane (3), so that we have
[by § 5 (iii)]
03 ( V i - V i ) + * a (cI^a“ C gtfi)+(tfA -02*«)=O
The Straight Litre 157

«1 bi Cl
i.e., A* bt Ct =O i.e., A«=0
a. bt c,
and
Le., Ot (Aid2- A,r/,)+ A3 ( d ^ - d jA ^ + d , (O1A1-A 2A1) - 0
fli bt
K .di
i.e., Oi bt . i/2 = 0 i.e., Aa=O.
d2
O3 N dt
[Note, While writing the symmetrical form of the equations
(4) of the line of intersection of the planes (I) and (2), we have
taken the point on the line for which z —0. If we take the point
for which x = 0 the condition Ai=O is obtained instead of Aa=O.
Similarly by taking the point for which _y=0, the condition Aa“ 0
will be obtained.]
Hence the planes (I), (2) and (3) will intersect in a common
line if Aa=O, Δ 3 = 0 , A t - O and Ai=O.
Case II. The three planes form a triangular prism.
The three planes will form a triangular prism if the line of
intersection of any two pianes is parallel to the third plane and
does not lie in it.
The line of intersection of the planes (I) and (2) is given by
(4). The line (4) will be parallel to the plane (3) if
O3 (AiC2- A 2C1) f A., (V1A2 - C 2A1) + ^, (Atftl - A 2Al ) = O
I O1 A1 Cl 1

i.e., a, C2 I-- - 1' i e„ : \ i .-0.

! “3 c3 1
The line ( I ) will not lie in the p la n e (3 ) if

*(¾ ¾ )
i.e., a3 (AxrZ2-A jd1)+A3 (tlxat—dMy) t-d3 (AlAt- A 2Al) +0
Ar bT di I
i.e., CU bt d3 j^O i.e., AtT^O.

Aj b3. d.
158 Analytical Geometry 3-D

Hence the three planes will form a triangular prism if


A4=O and Δβτ^Ο or AaT^O or Ai9^0.
Case ΠΙ. The three planes intersect in a point.
The three planes will intersect in a point if the line (4) of
intersection of the planes (I) and (2), is neither parallel to nor lie
in the plane (3). Rather than the line (4) must meet the plane (3)
in a point.
Thus the condition that the three planes meet in a point is that
A4^O.
Alternative method. Solving the equations (I), (2) and (3) by
the method of determ inants. [This method is called Cramer’s Rule],
we have
* ______ -y
D1 Cl d x O1 C1 d i

b% c2 d t a, c2 d t

h C3 d z flj Ca d a

Z -I
*1 I 1 " ~ d i Cfl b i Cl

a 2 h d t *2 b t C2

*a b* < k. Qz \ ' c3
X —y Z — 1
or
I 00
\<

II
l< i

A a ~ Δ «

L Δ »
or X = - - z = -
Ni
Il

, ,
Δ / . ( 6)
Hence the three planes will intersect in the point whose co­
ordinates are given by (6) if Δ 4=5^0.
Working Rule. Let the three planes be given by the equa­
tions (I), (2) and (3). Now proceed as follows :
(1) First evaluate A 4- If A 4T^O, then the three planes
intersect in a point whose co-ordinates are given by the relations
(6) above.
(2) If A 4=O, then evaluate Aa, A t and Ai-
(i) If A 3^0 (or A it^O or Aa^O), then the three plants
form a triangular prism.
(ii) If Ae = O, Aa=O and A i= O , then the three planes inter­
sect in a common line.
The Straight Line 159

Remark. If A«=0 and Aa=O and at least one of the three


common minors Agtj -Ojhl and αφ ζ—αφι of Δ ί and
Δ* not zero, then it can be proved algebrically that Aa=O and
Δ ι= 0 . Consequently in this case the three planes will have a
common line of intersection.
SOLVED EXAMPLES (I)
Ex. I. Examine the nature of intersection o f the planes
(i) 5x+ 2 .y -4 z+ 2 = 0 , 4x -2 y - 5 z - 2 - 0 ,
2x1-8y -2 z —1=0.
(//) x+ 2v+ 3z -6 = 0 , 3x+4y+5z -2= 0,
5x+4y+ 3z+ 18= 0.
(Hi) 2x+ 4y+ 2z 7=0, 5x+.y z—9= 0, x —y - z -6 = 0 .
Sol. (i) The given planes are
5 x + 2 j 4 z+ 2 = 0 - ( 1 )
L /i

-( 2 )
N>
II
I

O
N
I
X

2x+8_y—2z—1=0 -(3 )
The rectangular array of coefficients is
I 5 2 - 4 2 j

4 -2 -5 —2 .

I 2 8 -2 -I !
We have,
5 2 -4 I 2 -4
Δ «= 4 -2 -5 -I -2 -5
2 8 -2 0 8 -2
on adding the 3rd column to the 1st column
I 2 -4 , on adding the first row to the
second row
= 0 0 -9
0 8 -2 I
=72^0.
Hence the given planes intersect in a point.
Adding (I) and (2), we get
9x—9z=0 or x=z.
Putting x = z in (3), we get
8y—1=0 or y = 1/8.
160 Analytical Geometry 3-D

Putting x —z in (2), we get


—x —2y—2 or x=> —2y—2. ...(5)
Putting the value of y in (5), we get x — —9/4*=z.
Hence the point of intersection is ( -9/4, 1/8, —9/4).
(ii) The rectangular array of coefficients is
1 2 3 - 6
I
1 3 4 5 -2 .
5 4 3 18
I 2 3
We have, A 4=! 3 4 5
5 4 3
I 2 3 , by Rt—2Ru Rf—5Λ(
0 -2 -4
0 -6 12
= 2 4 - 24 = 0.
Since A 4=O- therefore the three planes either intersect in a
line or form a triangular prism.
I 2 -6
Now A s= 4 2
5 4 18
I 2 -6 , by Rf 3 Ru Rf~ 5 Ri
—2 16
0 -6 48
= - 9 6 - ( - 9 6 ) = -9 6 + 9 6 = 0 .
Similarly we find that As=O and Ai=O.
Hence the three planes intersect in a line,
(hi) The rectangular array of coefficients is
2 4 2 --7
5 I -I --9
i I I 6
The Straight Line 161

2 4 2
We have, Δ 4= 5 I -I
I -I
T1
0 6 4 by Rt - SRat
R \—2Ra
I 6 4
I -I -I
= 24 -2 4 = 0 .
Since A«=0. therefore the three planes either intersect in a
line or form a triangular prism.
2 4 -7 2 6 5

Now A s= 5 I -9 = 5 6 21 ,
I -I -6 II 0 0
j by C*4 Cx, Ca+6Ci
= 1.(126-30)=96=^0.
Hence the three planes form a triangular prism.
Ex. 2. Show that the planes
I x - S y - I z = Q t S x - H y ^ lS z = O t S x -S ly -S S z = O
pass through one line and find its equations. (Meerut 1977]
Sol. The rectangular array of coefficients is
2 - 3 -7 0
3 -1 4 -1 3 0 •

8 -3 1 -3 3 0
2 -3 -7
A t= 3 -1 4 -1 3
8 -31 -3 3
2 -I -I , by C ,+C 1,
C,+3Ci
3 -11 -4
8 T 23 -9
162 Analytical Geometry 2-D

0 0 -I . by Ci+2Ct,
Ct —Ct
= -5 -7 -4
-1 0 -1 4 —9
= -1 ( 7 0 - 7 0 ) = 0 .
Since A«=0, therefore, the three planes either intersect in a
line or form a triangular prism.
2 —3 0
Now At* 3 —14 0 = 0.
8 -3 1 0
Similarly A i =O and At=O.
Hence the three planes intersect in a common line.
Clearly the three planes pass through (0, 0, 0) and hence the
common line of intersection will pass through (0, 0, 0).'' The
equations of the common line are given by any of the two given
planes. Therefore the equations of the common line are given by
Ix - S y - T z = O ,
and S x - I4y— 13z=0.
Λ the symmetrical form of the line is given by
x y z x y z
39—9 8 ~ —2 l+ 2 6 = —28+9 ° r - 5 9 “ 5 _ - 1 9 ’
Ex. 3. Prove that the planes
x—2y+ z—3=0, x + y —2r—3 —0, x—z—1=0
form a triangular prism.
Sol. Proceeding as in Ex. I above, we get A t-O an d Ar^O-
Hence the’given planes form a triangular prism.
Ex. 4. Prove that the planes
x=cy-\-bz, y= az+ cx, z= bx+ ay
pass through one line i f a*+b* +ca+2aftc= I, and show that the line
o f intersection then has the equations
x y z
V (I-A f) " V U -C 2) ’
(Bundelkband 1978; Punjab 77; Rajasthan 75, 77]
Sol. The equations of three given planes are
x - c y - b z = 0, c x -y + a z = 0 , bx+ ay—z —0.
The Straight Line 163

The rectangular array of coefficients is


I —c -b 0
C -I a 0

b a -I o
I —C -b

We have, A«= C -I a
b a I
»1 (I —at)+c ( —c -ab) —b (ac+b)
Ot -H t - Ct -
I —C 0

Also A s= C -I 0 •0.

b a 0
Similarly As=O and Ax=O.
Hence the given planes intersect in a line if A *=0 i.e., if
a%+ IP+c%+Zabc= I.
Clearly the given planes pass through (0, 0, 0) and hence the
common line of intersection will pass through (0, 0,0). Let /, m,
n be the d.r.’s of this line. It being perpendicular to the normal
of each plane, we have
I - cm—bn= 0 ...(I)
cl—m +an=0 ...(Z)
bl+am—n = 0. .-..(3)
Solving ( I) and (2), = -JL jr

or - J _____ ” ____ ! _
ac+b bc+a l~c* ...(4)
Solving (2) and (3), J L _ _ !L . (J)

Solving (3) and (I), - j/— = J-^ri =


' ab+c 1—b* bc+a ...(6)
Taking first two terms of each of (5) and (6) and then multi­
plying, we get
____ P . m3 P m*
(ab+ c) ( I - a 9)- (ab+ c) (1—6*) ° r I - a· = .(7)
Similarly from (6) and (4), we have
164 Analytical Geometry 3-D

m* n*
1—6*= 1—e*’ ...(8)
Now from (7) and (8), we get
/* m2 n*
1—a* =1 -Z>i = l - c 2'
. I m n
V O -a 2)_ VO-**] _ V (i-c 2) -(9)
Λ the d.r.’s of the common line of intersection of the given
planes are given by (9). Since the line passes through the origin,
hence, its equations are given by
x y z
VO - a %) ~ V 0 ~ * 2) = V 0 -c2)’
Ex. 5. Prove that the planes
x+ ay+ {b+ c) z+ d = 0,
, x+ by+ (c+a) z+ d = 0,
x+ cy+ {a+ b) z-M =0,
pass through one line. [Allahabad 1978, Kanpur 82]
Sol. The rectangular array (or matrix) is
I Cl b+c d
I b c+a d
I C a+b d
I a b+c
We have, Δ 4— I b c+a
I c a+b
I a+ b+ c b+c adding 3rd
column to 2nd
I a+ b+ c c+a
I a+ b+ c a+b
I I b+c
- ( a+ b+ c) I I c+a =0
I I a+b
I a ar I a I

Also Δ *= I b a I =M I b I
I c d I c I
<■,\s "■
The Straight Line 165

Similarly Δ 2=0 and Δ ι = 0.


Since Δ 4= 0 , Δβ=0> Δ 8= 0 and Δ ι= 0 , therefore the given
planes intersect in a line.
Ex. 6. Show that the planes
ny—m z= \9 Iz - η χ - μ and m x - l y =V
have a common line i f 1λ+ηΐμ+ην = 09and the direction ratios o f the
line are I9 m9 n. [Punjab 1980; Behrampur 76 S]
Show further that the distance o f the linefrom the originis

Sol. The equations of the given planes may be written as


Q.x+ny—m z—A=O .( 1)
—n x + O .y+ I z - μ = 0 . ( 2)
mx —/y-b O.z—V--0. ...(3 )
The rectangular array (or matrix) of coefficients is
0 n —m —A
—n 0 I -μ
I m -/ 0 —V
0 n —m
Wc have, Δ 4= —n 0 I
m -I 0

-0 -n (0 -
.E

(» /-0 )
E
I

Λ the given planes will either* meet in a line or form a


triangular prism.
0 n Λ
Now Δ a= -/1 0 - M
m -/ —V

= 0 —n (wv+m/i)—λ (/i/-0)
= —n (IX+mp+nv).
If the planes are to meet in a line then Δβ must also be zero.
Then we have n (lX + m p+ nv)= 0
or lX+mp + ny = 0. ...( 4 )
[We have assumed /?^£0.]
If the condition (4) is satisfied, then we can see that Δ -"
166 Analytical Geometry 3-D

and also Δ ι* 0 . Hence (4) is the condition for the three given
planes to have a common line of intersection.
Let a, b, c be the d.r.’s of the line of intersection, It being
perpendiculer to the normal of each plane, we have
0 a+ nb—mc= 0 -(5 )
—n e+ 0 .6 + /c= 0 - ( 6)
ma—lb+0.c=0. -(7 )
Solving (5) and (6),
a___ 6 __ £_ JL — £_
nl ~ m n ~ n * I “m“ u’
showing that the d.r.’s of the line of intersection are I, m, n.
To find the equations of the line of intersection. The line of
intersection meets the plane z = 0 in the point given by
n y —X =0, - η χ -μ β Ο , [Putting z= 0 in (I) and (2)]
or X= —μ/η, y=X/n.
Λ the line of intersection passes through the point
. ( —μ/η, X/n, 0) and has d.r.’s I, m, n and therefore its equations are
given by
χ+ μ/η y - X /n ^ z - 0
I ~ m ~ n •••(8)
The distance of the fine (8) from the origin, Using § 10, the
required distance p is given by
» * = _ _ i _ Ji m » r
* (P + n f+ n %
) 110 —A/« 0 —0 I
n I I* I I m
+ 0 -0 0-X/n
I
(/·+«.■+«»)
I
[using (4)]
Λ p={(Aa+ ^ 2+v‘)/(/2+ m 2+ n 2)}1'a.
Ex. 7. Prove that the planes x = y sin φ+ζ sin φ, y = z sin Θ
+ x sin φ, and z —x sin φ-by sinti witl intersect in the line
x ___ y_ z
cos 9~cos φ"
[Allahabad 1982; Garhwal 78 S]
Sol. The equations of the planes may be written as
x —y sin φ—z sin ¢= 0, ...(I)
x sin φ—y+ z sin ¢=0, ...(2)
x sin φ+ y sin Θ—z=0. ...(3)
The Straight Line 167

Let us find the line-of intersection of the planes (I) and (2).
Let I, m, n be the direction cosines of the line of intersection of
the planes (I) and (2). It being perpendicular to the normals of
both the planes, we have
l—m sin φ—η sin I sin φ -m + w sin ¢=0.
Solving, we get
_______ I_________________ m________ n
—sin φ sin 9—sin φ ~ —sin φ sin φ - sin Θ = —I +sin* ψ
_______ I_______________ m____________w
sin ^ sin ¢ + sin φ '“ sin φ sin ^ + S in i- I-Sin* φ * ...(4)
If 9 4-^+ ^= $π, then we have
sin 0=sin ($π—(φ+φ)}=ξθ& (φ+φ)
=cos φ cos φ—sin φ sin φ
or sin 0-f-sin ^ s:n 0=cos ^ cos ¢. ...( 5)
Similarly sin ^-(-sin φ sin 0=cos φ cos Θ. -"(6)
Using the relations (5) and (6), (4) becomes,
I m______n
cos φ cos 9 ~cos φ cos φ “ cos* φ
ΟΓ
I m n
--------- — --------- C a ---------
COS 0 COS^ COS φ ·*·(7)
Clearlytheplanes (I) and (2) passthrough (0 ,0 ,0 ) and so
their line of intersection will pass through (0, 0, 0) so that its
equations are given by
x/cos 0= yjcos φ= ζ/cos φ. ...(8)
N ow it remains to prove that the line (8) must lie on the
plane (3).
The point (0, 0, 0) through which the line (8} passes also lies
on the plane (3). Also the normal to the plane (3) whoss d.r.’s
are sin φ, sin Θ, —1 must be perpendicular to (8), the condition
for which is
cos 9 sin ^-J-cos φ sin 0 + co s^ ( - 1 ) = 0
or sin (0 + ^ )—cos ^ = 0 , or sin (|π - φ)—cos φ = 0
or cos φ —cos φ—0 or 0 = 0 which is true.
Hence the line (8) also lies on the plane (3). Thus the equa­
tions (8) are the equations of the required line.
Ex. 8. Show that the planes ax+ hy+ gz—0, h x + b y + fz -0 ,
gx+ fy+ cz= 0 have a common line of intersection if
168 Analytical Geometry 3-D

a h g

Δ= h b f

g f C
and the direction ratios of the line satisfy the equations
_m * _ n*
« Δ '_3 Δ ~ 0 Δ ·
da db Sc
Sol. The equations of the given planes are
ax+ hy+ gz= 0 ...(I), hx+ by+ fz= 0 ...(2)
g x+ fy+ cz= 0. ...(3)
The rectangular array of coefficients is
a h g 0
h b f 0

g f C 0
We have
a h g =A (as given in the problem)
Δ*= h b f
I- g f C
or A i= &=abc+2fgh—af*—bg*-chK (4)
Also
a h 0 = 0 . Similarly Aa=O and Δ ι= ° ·

Δ ·“ h b 0

g f 0
If the given planes intersect in a line then A i must be zero
(as Δ». Δ 2 and Ai are already zero). Hence the given planes will
have a common line of intersection if A i=O or A=O
or abcA-Ifgh a p —bg2—Chi -O . -- (5)
Let /, m, n be the d.r.’s of the common line of intersection of
the given planes. It being perpendicular to the normals of the
planes, we have
al+ hm +gn—0, hl+ bm +fn=0, gl-{-fm+cn—0.
Solving any two (say the first two) of these relations, we get
I m n
The Straight Line 169

I2 nf
Squaring, {hf_ b g f- ^ h_ a f) t- {ab_ htf - ...(6)
DifiFerentiating (4) partially w.r.t. a, A, c respectively, we have
0A/9a=Ac—/ 2, 8 /\fb = a c - g2, d/\/8c=ab—A2. ...(7)
Now (A /-Ag)2=A2/ 2+ A V lbfgh
=A2/ 2+A2g2- A ( a f2+bg2+ch2-,-abc) using (5)
= A2/ 2+ A V - f lA/2— A V — AcA2+ ab2c
= h2f 2- a b f 2-b c h 2+ab2c
= - / 2 (aA-A2)+Ac OA-A2)
=O A -A 2) ( A c - /2).
Similarly (gA a / ) 2=(aA A2) (ea—g2).
Sub stitu tin g th ese valu es in (6), we get
/2 ___ wr________
(aA-A2) (Ac / 2) (aA—A2) (ca —g 2) OA-A2)2
. /2 m2 M2
or
A c - / 2 - 'c a - g!2 aA A2
I2 m2 n2
or
ο 'Δ ~ 9Δ ^ 0 Δ [using (7)]
aa 0A' 0C
Ex. 9. Abr vvAar values of k do the planes
x - v + z + 1=0, kx+ 3y+ 2z ~ 3=0, % x+ky+z--2 —0
(i) intersect in a point ; (ii) intersect in a line ; (iii)form a
triangular prism ?
Sol. The rectangular army of coefficients is
I -I I I
k 3 2 3
3 k I --2
N o w w e calcu late th e fo llo w in g d e te r m in a n ts:
Δ *= I -I I O -I O
A 3 2 = A+ 3 3 5
3 k I 3+A k A +1
adding 2nd column to 1st and 3rd
=(A +3) O -I O
I 3 5 = 0 + 3 ) 0 + 1 —5)
I A k +■ I
= 0 + 3 ) 0 - 4 ; ..
170 Analytical Geometry 3-D

Δ»=* I -I I 0 -I 0 , adding I
to 1st i
k 3 - 3 = Jfc+3 3 0
3 k -2 3+fc k k —2
rr i( k+ 3 ) (Jk--2 ),
L^ t = I I ·. I 0 I 0
k 2 -3 Jfc- 2 2 -5
3 I -2 2 I -3
adding ( —1) times 2nd column to 1st and 3rd
-■ - { ( * - 2) (-3 )+ 1 0 } =3Jfc-16,
and Δ ι " -I I I 0 I I
3 2 -3 = 0 · 2 -3

k I -2 k -2 I -2
adding 3rd column to 1st(i)
—- 5 (k—2).
(i) The given planes will intersect in a ,.pint if ArT^O an^
so we must have k -φ — 3 and A:^4. Thus the given planes will
intersect in a point for all real values of k other than —3 and 4.
(ii) If f c = ~ 3, we have Aa=O, Ae=O but Aa^O. Hence the
given planes will form a triangular prism if k ——3.
(iij) If k= 4, we have Aa=O but Aat^O. Hence the given
planes will form a triangular prism if k*=*4.
We observe that for no value of k the given planes will have
a common line of intersection.
Ex. 10. The plane xla-ry/h+ z/c= ]. meets the axes in A9 B
and C. Prove that the planes through the axes and the internal bise­
ctors o f the angles of the triangle ABC pass through the line
x___________ y___________ z
a \/( b 2-\-c2) “ by j(c2+ a2)~~C\/(a2-\-b2)
Sol· Plane x/a+ylb+z/c=*! meets the axes in A (a, 0, 0),
B (0f b, 0), C (0, 0, c).
The equation of any plane through x-axis is (Le. y = 0, z=0)
is .y-t-λζ—0. ...(I)
The d.r.’s of AB are —a, b ' 0
The Straight Line 171

—a
D.C.’s of AB are -,0
Via*+/»*)’ V(a2+*>2)
—a
D.C.’S Of AC y ^ a»+c*yu 0,
v. y/(Q*c+ c*)
D.C.’s of interior bisector of L-BAC are
1 . 1 \ b c
- t (;tV (a2+ * 2) V (a * + c 2) / ’ 2 V ia2+**) ’ 2 V ia2H-C*)
Now if plane (I) passes through the internal bisector of
L.BAC, then normal of (I) will be perpendicular to the internal
bisector of l_ BAC.

*** 0 + 1 ' 2V(a2+i>2) +A-2V(a*+c2)“ °


or . by/(ai A-ci )
Cyf(Oi A bi)
Put this value of λ in (I), the equation of plane through x-axis
and internal bisector of L BAC is
bV(a*+c*)
* Cyf(O2A b i)
or y z
byf(a2A c2) Cyf(Cp-Ybt ) - ( 2)
Similarly the equations of other two planes are
x y
O y f i b i A-Ci ) b y f (O2 AmCi )

Cyf(Oi -Ybi ) - O y f ib i -YCi ) - ( 4)


Planes (2), (3), (4) pass through the lines
x y z
Oyfibi -Yci )- b yf (O2-Yci)- Cyf(Oi-Ybi)
Exercises
1. Find the coordinates of the point where the line
( x - 1 )/2 = (^ -2 )/-3 = (^ + 3 )/4
meets the plane 2x-f-4j·—z + 1=0.
Abs. (10/3, -3 /2 , 5/3).
2. Show that the distance of the point of intersection of the line
(*—3)/1=0'- -4)/2=(z—5)/2
and the plane x-f^+z«=17 from the point (3, 4, 5) is 3.
3. Find in symmetrical form the equations of the line
x+yA-z-Y I = 0 = 4 x + v - 2 z + 2
172 Analytical Geometry 3-D

and find its direction cosines.


I
X+' y + -j 2_o
Ans. -I
- I 2
Direction cosines are —l/\/6 , 2/\/6, ~ l / \ / 6 .
4. Show that the equation of the plane which contains the two
parallel lines
x —4 y —3 z —2 , x --3 >H-2 z

is I lx —v—3z 35=0.
5. Show that the equation of the plane containing the line

ί > - Η ’*=0
and parallel to the line x/a-~zjc=\, is
-a£ _ Zb _ e. £ + 1_ 0.
6. Find the equations of the perpendicular from the point
(I, 6, 3) to the line
Xl y — I z —2
1 "r T " = ~ T ~ ’
Find also the coordinates of the foot of the perpendicular.
Ans. Coordinates of the foot of the perpendicular are
(1,3,5).
Equations of the perpendicular are
x —I y — 6 z—3
0 “ -3 = T "·
7. Show that the following pairs of lines are coplanar :
(i) λ '~ 4 = —J ( j;+ l)= z
and 4x —y + 5z—7 = 0 = 2 x — 5 y — z — 3.
Also find the equation of the plane containing them.
(Kanpur 1980)
(i'i) = O '-Z K -.fz+ l)
and x + 2 y + 3 z = 0 = 2 x + 4 y + 3 z + 3 .
Also find the point of intersection.
Ans. (i) x + 2 y -f-3z=2 (ii) ( 9 ,- 6 ,1 ) .
Show that the lines
x + 2 y — 5 z + 9 = 0 = 3 x —y + 2 z — 5
and 2 x + 3 y — z — 3 = 0 = 4 x — 5 y + z + 3 are coplanar.
9. Prove that the lines
x - 3 y + 2 z + 4 = 0 —2 x + y - t- 4 z + I
The Straight Line 173

and 3 x + 2 .y + 5 z — l = 0 = 2 y + z
intersect at the p o in t (3, I, — 2).
10. F ind the eq u ation s o f the line w hich can be draw n from th e
p oin t (2 , — I, 3) to in tersect the lines
Η * - 1 ) = έ ϋ > - 2 ) = έ (7-3)
and J (x— i ) = (z+3).
Ans. J 2x+4y—9z+7 = 0 = 11x - W y + 2 z - 38.
11, Find the distance of (—2, i, 5) from the linethrough (2,.3, 5)
whose direction cosines are proportional to 2, —3, 6.
f * (6 1 ).
12. Prove that the equations of the perpendicular from th e p o in t
(I, 6, 3) to the line
v— I z—2 x — I V— 6 z—3
X= 2 = — are —0 = ^-— 3
and the coordinates of the foot of the perpendicular are
(1 ,3 ,5 ). (Meerut 1977)
13. Find the locus of a point which moves so that its distance
from the line x —y —z is twice its distance from the plane
x + y + z = I.
A n s. Λ·ί + > · ί + 2 2 + 5 Λ ^ + 5 ^ 2 + 5 ζ Λ Γ — 4 x — 4 v — 4 7 + 2 = 0 .
5
Shortest Distance

Definitions.
§ I. Skew lines.
Two lines are called skew lines or non-intersecting lines if they
do not lie in the same plane. Skew lines never intersect and are
not parallel.
Shortest distance. The straight line which is perpendicular to
each of the two skew lines is called the line of shortest distance.
The length of Jhe line of shortest distance intercepted between the
skew lines is called the length of the shortest distance. The
shortest distance is briefly written as S.. D.
§ 2. Length the equations of the line of shortest distance.
To find the shortest distance between two given lines and to
obtain the equations of the shortest distance.
(Allahabad 1978 ; Behrampur 81; Gauhati 78; Indore 78;
Kanpur 77. 78, 83; Punjab 76; Rajasthan 73, 77; Rohilkhand 81)
Several methods, depending upon the forms of the equations
of the skew lines, are followed to find the shortest distance. They
are as follows :
Method I. Projection method. The equations of the skew lines
being given in symmetrical forms.
Let the equations of the given lines
be
I

X -X l
SSi
*

IN

N
11

I
I

Ii mi »1 (I)

and
X - X t •y—y * Z- Zt
U m2
The line (I) is passing through the
point A (xl9 y u Z1) and has d.c.’s pro­
portional to Iu mu nv The line (2) is passing through the point
y 2 * z%) and has d.c.’s proportional to I29 m%9 n2.
Shortest Distance 175

Let PQ be the line of shortest distance between the two lines


so that PQ is perpendicular to both the lines (I) and (2).
Let Z, w, n be the d.c.’s of the line of shortest distance PQ-
Then we have
/ / 1+ m m , + M M 1 = O and ; / g + f f l m , + n « j - 0 .
Solving, we get
____I_ m n
Ini O2 - tO i W1 ~ O1I2 - O 2I1- I1Ott - Z 1W 1
V ili -Vmi +o') I_______
V W m j 1M 2 - M I 2M 1 ) * ) “ V W m 1IM 2 - M I 2M 1 ) * } '
Now PQ= the length of shortest distance between the given
lines (I) and (2)
= th e projection of the segment AB on the lin eo f
shortest distance PQ
—I (*»— *1 ) + m (yt—yi)+ n (Z2- Z 1)
(MJ1Mt - O T 2M1) (x2— X1RiM 1Z2-M 2Z 1) (yt - y j )
= ___ -____________ + ( Z 1MJ2- Z 2W1) (Zt- Z 1)
-VZi(OT1M2-O T t M1) * + (M1Z2 - M 2Z1) * + (Z1OT2- Z2MJ1)*)
*1 Λ “ J'l Z2- Z 1 + V { 2 (MJ1M2 - M I 2M1) * )
-(3 )
Z1 W1 M1

Z2 JM2 M2
This is the required length of the S.D. between the given
lines (I) and (2).
The equations of the shortest distance. Clearly the line PQ of
the shortest distance is coplanar with both the given lines (I) and
(2). Hence the line PQ of shortest distance is the line of inter­
section of the two planes, namely, (i) the plane containing the
given line (I) and the IineZjC o f shortest distanc; and (ii) the
plane containing the given Iin i (2) and the line PQ of the shortest
distance.
Now the equation of the plane containing the given line (I)
and the line PQ of the shortest distance whose d.c.’s are Z, mj, m is
i1

-X i
I

y -y i
N

lx mi «! = 0. -(4 )
i . m M
Also the equation of the plane containing the given line (3)
and the line PQ of the shortest distance whose d.c.’s are Z, mi, m is
176 Analytical Geometry 3-Z)

X-X2 y -y 2 Z -Z 2

U m2 n* =0. ...(5)

I m n
The equations of the line PQ of the shortest distance are
given by the planes (4) and (5) taken together.
Note. If the shortest distance PQ=0, then the two given
lines (I) and (2) will intersect Le., they will be coplanar. From
(3), we observe that
X2 - X 1 y 2 -y \ Za — Z 1

h W 1 Wl

It W 2 W 9
which is also the condition for the two lines (I) and (2) to be
coplanar. [See chapter 4 § 8 (A)].
Hence we can give an another statement for the two lines to
be coplanar.
itTwo lines are coplanar if the shortest distance between them
vanishes”
M ethodii. General co-ordinates. The equations of the two
lines being given in symmetrical form :
Let the equations of the two lines be given by (I) and (2)
[See method I].
Thegeneral co-ordinates of the points on the two lines(l)
and (2) are given by
M j1+ y l9 ¥ i+ Z i) , say the point P.
and (/2ra+ x 2, Wzr*+y 2>w2ra+ z2), saY the Point Q-
Let P and Q be the points where the line of shortest distance
meets the given lines (I) and (2) respectively, so that the line PQ
is perpendicular to both the given lines (I) and (2).
Now find the d r.’s of the line PQ and apply the conditions
that the line PQ is perpendicular to both the given lines (I) and
(2). Thus two equations in rx and r2 are obtained. Solve these
equations to get rx and r2. Having found i\ and r2, the co-ordinates
of the points P and Q and also the d.r.’s of the line PQ are
known. Now we can at once find the length PQ of the shortest
distance and also the equations of PQ.
Shortest Distance
177

Method III. Theequationsofoneline being given in general


form and those o f the other line in symmetrical form.
Let the equations of one line be Mi=O=Vi ( 6)
and the equations of second line be
( x ~ X i ) ! l i = ( y - y i )lmi = ( z - Z i ) l n l . (7)
The equation of any plane through the line (6) is
«* + Av1=O. ( 8)
Find Aso that the plane (8) is parallel to the line (,7) and
substitute this value of A in (8). Then the length of shortest
distance between the given lines (6) and (7) is equal to the length
of the perpendicular from any point, say the point (*i, yu *i). 00
the line (7) to the plane (8).
The equations of the shortest distance. The shortest distance
is the line of intersection of the two planes namely (a) the plane
containing th? given line (6) and perpendicular to the plaue (8)
and (b) the plane containing the given line (7) and perpendicular
to the plane (8). Hence the equations of the line of shortest
distance are given by the equations of the planes (a) and (b) taken
together.
Method IV. The equations o f both lines being given in general
form.
Let the equations of the two given lines be
M i= O e iv 1 ...( 9 ) and M i = O = V 8 . .(10)
The equation of any plane through the line (9) is .
M j+ A jV l = O. . ( 11)
The equation of any plane through the line (10) is
M2+ A 2va= 0 . .(12)
Now At and A2 are determined with the conditions that the
planes (11) and (12) are parallel.
:. The shortest distance is equal to the distance between the
parallel planes (11) and (12).
The equations of the shortest distance arc given by tbs
equations of the two planes namely (a) the plane through the line
(9) and perpendicular to the plane (11) [or (12)] and (b) the plane
through the line (10) and perpendicular to the plane (I I) [or (12)].
Remark. For the sake of convenience we can reduce the
equations of one or both the straight lines to symmetrical formahd
then follow the methods I, H or 111 as explained above.
178 Analytical Geometry 3-D

SOLVED EXAMPLES
Ex. I. Find the shortest distance between the lines
( * - 1 )/2 = (,-2 )/3 = (2 -3 )/4 ;
( * - 2 ) /3 = ( ,-4 )/4 = (2 -5 )/5 .
Show also that the equations of the shortest distance are
l l x + 2 , —7 r + 6 = 0 = 7 je + ,—52+7.
(Agra 1974, 78; Berahmpor 76S, 8IS; Madras 76;
Meerut 78, 86, 89; Vikram 78)
Sol. The equations of the given lines are
( x - l ) / 2 = ( , - 2)/3= (2-3)/4 = r, (say) ...(D
( * - 2 ) /3 = ( ,- 4 ) /4 = ( 2 - 5 ) /5 = r , (say). ...(2)
M ethodi. (Projection method). Let /, m,n be the d.c.’s of
the line of S. D. Since it is perpendicular to both the given lines
(I) and (2), therefore we have
2/+ 3m + 4n=0; 3/+4m +5n=0.
Solviog, we get
I m n Λ/(/*+/η*+η*) I
- 1 = 2 “ - l “ { ( - 1)»+ (2)»+(-l)*}= V6
A The d.c.’s of S. D. are - 1/V 6, 2 /,/6 , - 1 / ^ 6 .
Now A (I, 2, 3) is a point on line (I) and B (2, 4, 5) is a
point on the line (2). Hence
the length of S.D .=the projection of join of A and B on the line
whose d.c.’s are —l/y/6 , 2 /^ 6 , —\/i/6
= ( - l / V 6 ) ( 2 -1 )+ (2 /^ 6 ) ( 4 - 2 ) + ( - W 6 ) (5 -3 )
= 1/V6.
The equations of S. D. (See § 2)
Theeqdations o fth ep Ia n eth ro u g h th e Iin e(I) and S. D. is
x —I y —2 2—3
2 3 4 =O or l l x + 2 , —7 2 ^ 6 = 0 ...(3)
-I 2 -1
And the equation of the plane through the line (2) and S.D.
U x —2 y —4 2—5 I
3 4 5 I=O or 7 jc+ , - 5 2 + 7 = u ...(4)
-I 2 -1
The equations (3) and (4) together are the equations of the
s.a
Shortest Distance 179

Method 2. Aoy point on line (I) is


(2 /,+ 1, 3 /,+ 2 , 4/-1 + 3), say P. - (3)
Any point on line (2) is
(3 /,+ 2 , 4 /,+ 4, 5 /,+ 5 ), say Q. - (4)
The d.r.’s of the line PQ are
(3r,+ 2 )-(2 /, + 1), (4/, + 4 ) - ( 3 /, + 2). ( 5 /,+ 5 ) - ( 4 /,+ 3)
or 3 /,- 2 /, + 1 . 4 / , - 3 / , + 2 , 5 / , - 4 / , + 2. ..(5 )
Let the line PQ be the line of shortest distance, so that PQ
is perpendicular to both the given lines (I) and (2) and, therefore,
we have
2 ( 3 /,- 2 /,+ 1 ) + 3 ( 4 /,- 3 /, + 2)+4 ( 5 /,- 4 /,+ 2 ) = 0
and 3 ( 3 /,- 2 /,+ 1 ) + 4 ( 4 / , - 3 /,+ 2 ) + 5 ( 5 /,- 4 /,+ 2 ) = 0
or 3 8 /,- 2 9 /,+ 16=0 and 5 0 /,-3 8 /,+ 2 1 = 0 .
Solving these equations, we get / , = 1/3, / , = —1/6.
Substituting the values of /, and /, in (3), (4) and (5), we have
P (5/3, 3, 13/3), Q (3/2, 10/3, 25/6)
and d.r.’s of PQ (the line of S.D.) are —£, £, - £ i.e: —1, 2, —1.
The length of S.D .=the distance between the points P and Q

- M - D ' A t - U M ) ' }

The equations of S.D. are either given by equations (3) and


(4) of method I above or we can write the equations. of a line
passing through the point P and having d.r.’s —I, 2, —I.
Ex. 2 Find the shortest distance between the lines
x —3 y —8 z—3 x + 3 y + 7 z —6
~ ~ i a ~ ==~A~'
Find also Its. equations and the points in which it meets the given
lines. (Avadh 1982; Garhwal 79; Indore 76, 79;
Kanpnr 7+ 80; Lucknow 80, 81; Madrad 78;
Meerot 72, 80, 83, 89S; Rohilkhand 78)
Sol. The equations of the given lines are
(*—3 )/3 = ()'-8 )/ —I = (z—3)/1= / , (say) ...(I)
»nd (*+3)/( —3 )= 0 ;+ 7 )/2 = (z —6 )/4 = /, (say) ...(2)
Any point on line (I) is (3/,+ 3 , —/,+ 8 , /, + 3), say P. ...(3)
Any point on line (2)is( - 3 / , - 3 , 2 /,- 7 ,4 /,+ 6 ) , say Q. ...(4)
The d r.’s of the line PQ are
( - 3 ^ - 3 ) - ( 3 / , + 3), ( 2 / , - 7 ) - ( - / , + 8 * ( 4 /,+ 6 ) - ( /,+ 3 )
or —3 /,—3 /,—6, 2 /,+ /, —15, 4 /,—/,+ 3 . ...(5)
180 Analytical Geometry 3-D

Let the line P g be the Iiaes of S.D., so that PQ is perpendi­


cular to both the given lines (I) and (2), and so we have
3 ( - 3 r , - 3 r , - 6 ) - 1 ( 2 r , + r , - 15)+ 1.(4r,- 2 , + 3 ) - 0
and —3 ( —3r,—3r,- 6 ) + 2 . (2 r,+ r,—15)+4 ( 4 r ,- r ,+ 3 ) = 0
or —7 r,- I l r 1=O and I lr,+ 7 r,= 0 .
Solving these equations, we get 2, = 2, = 0.
Substituting the values of r, and r, in (3), (4) and (5), we have
P (3, 8, 3), Q ( —3, —7, 6). Ans.
And the d.r.’8 of P g (the line ofS.D.) are - 6 , - 1 5 , 3 or
- 2 , - 5 , 1.
The length of S.D .=the distance between the points P and Q
= V { (-3 -3 )* + (-7 -8 )* + (6 -3 )» } = 3 y 3 0 .
Now the line PQ of shortest distance is the line passing
through P (3, 8, 3) and having d.r.’s - 2 , - 5 , 1 and hence its
equations are given by
x —3 y —8 z—3 x —3 y —8 z—3
- 2 ™—5~ “ I ΟΓ 2 ~ l - ” I '
Ex. 3. Find the length of the shortest distance between the lines
x —3 y —5 z —7 'X + l y + 1 z+1
I ~ 7 ~ “ 35" " - 1 ’
(Garhwal 1981; Kanpur 83; Meerut 83S,j86P; Rohiikhand 82)
Show also that Its equations are given by
(x —l)/2 = (y —2)/3=(z—3)/4. (Meerut 1986P)
Sol. The equations of the given lines are
(x —3 ) /1 = 0 - - 5 ) /( - 21=fz—7)/1= r , (say); ...(I)
and (x + 0 /7 = O + 1)/(—6)= (z+ 1)/1= r, (say). ...(2)
Any point on line (I) is (r,+ 3 , —2r,+ 5, r,+ 7 ), say P. ...(3)
Any point on line (2) is (7r,—I, —6r,—I, r ,- 1 ) , say Q. ...(4)
Thie d.r.’s of the line PQ are
( 7 r ,- l ) - ( r , + 3 ) . ( - 6 ^ - 1 ) - ( - 2 ^ + 5). ( r , - l ) - ( r , + 7 )
of 7r,—r , - 4, —6 r,+ 2 r,—6, r , —r , - 8. ...(5)
Let the line PQ be the line of S.D., so that PQ is perpendi­
cular to both the given lines (I) and (2), and so we have
1(7r, - r, - 4 ) - 2( - 6r,+ 2r, - 6) + 1(r, - r, - 8)=0
and 7(7r, - 2, - 4 ) - 6( - 62, + 22, - 6 )+ 1 .(r,—r,—8)= 0
or 20r,—6 r,= 0 and 86r,—20r,=0.
Solving these equations, we get r ,= r ,= 0 .
Substituting the values of r, and r, in (3), (4) and (5), we have
P (3 ,5 ,7 ). g ( - l , - I , - I )
and the d.r.’s of P g (the line of S.D.) are - 4 , - 6 , —8 or 2,3,4,
Shortest Distance 181

The length of S . D . - P g - ^ { ( - l - 3 ) 2+ ( - l - 5 ) '+ ( - l - 7 ) » }


= V{(4)*+(6)*+(8)*}= 2 ^(29).
Now the line PQ of shortest distance is a line passing through
P(3, 5, 7) and having d.r.’s 2, 3, 4 and hence its equations are
given by
Λ—3 y —5 2—7
T -c sI - = 4
or
jc—I y —2 2—3
2 = 3 —4 Proved.
Ex. 4. Find the shortest distance between the lines
( x - 1 )/2 = (^ -2 )/3 = (2 -3 )/4 ; (x -2 )/3 = (y -3 )/4 ~ (2 -4 )/5 .
Hence show that the lines are coplanar. (Meerut 198$)
Sul. We are solving this problem by projection method. The
equations of the given lines are
( x - l) /2 = ( y —2 )/3 = (2 -3 )/4 ; ...(I)
and (x—2)/3= (y—3)/4= (2-4)/5. ...(2)
Let /, m, n be the d.c.’s of the line of S.D. The line of S.D.
being perpendicular to both the lines (I) and (2), we have
2/+3m +4/1=0 and 3/+4m +5/i=0.
Solving these equations, we have
I m n
1 5 -1 6 = 1 2 -1 0 “ 8 - 9
I m n V(/*+»»*+«*) I
0Γ - I " 2 - —i = V « - l)* + (2 )* + (—l)*},eV 6 ’ ...(3)
Clearly the line (I) passes through the point A (I, 2, 3) and
the line (2) passes through B (2. 3, 4).
The length of S.D.= T he projection of join of A and B on
the line of S.D. whose d.c.’s are

= /( 2 - 1 ) + /« (3 -2 )+ n (4 -3 )

Since the length of S.D.=0, hence the given lines are copla­
nar Le. intersecting.
Ex. 5. Find the points on the lines
—x y+ 9 z—2
—— = —(y —7) = z—4 and
which are nearest to each other. Hence find the shortest distance
between the lines and also its equations. (Bnndelkkand 1978)
182 Analy tical Geometry 3-D

Sol. The equations of the given lines are


.{*—6 )/3 = 0 /-7 )/( - 1) = ( 2 - 4 ) /1= rj (say) ...(I)
and x/( —3)=(>’+ 9)/2=--<z—2)/4 = r2 (say) ...(2)
Tbe points on the lines (I) and (2) which are nearest to each
other are the points where the line of S.D. meets the lines (I)
and (2).
Any point P oo (I) is (3r,+ 6, — H-7, rx+4) --(3)
and any point Q on (2) is ( —3r2. Ir2- 9 , 4r2+ 2). ---(4)
The d.r.’s of PQ are
—3r2—3rj —6, 2r2+ r ,—16, 4 /^ - /^ - 2 . . (5)
Let the required points be P and Q9 so that PQ is the line of
S.D. Hence PQ is perpendicular to both the given lines (I) and
(2) and so we have
3 ( - 3 ^ - 3 ^ - 6 ) - 1 ( 2 ^ + ^ - 1 6 ) + 1 . ( 4 / , r ,- 2 ) = 0 .
and —3( —3rt —3 /^ - 6 ) +2(2r2+ / y - 16) + 4 .( 4 / 8 - ^ - 2 ) = 0
or - 7 /8 - 1 1 / ^ - 4 = 0 and 2 9 /^+ 7 /^-2 2 = 0 .
Solving these equations we get r, = —I, r2= J .
Substituting these values in (3), (4) and (S), we have
P (3, 8, 3 ) , 2 ( - 3 , - 7 , 6)
and d.r.’s of PQ (the line of S.D.) are —6, —15, 3 or 2, 5, -r-1.
The length of S.D. = P g = y/{( - 3 - 3)2+ ( - 7 —8)*+-(6—3)*}
= V{ vb)a+ (15)2+ (3)2}= 3 \/( 30).
The line PQ of shortest distance is a line passing through
P(3, 8, 3) and having d.r.’s 2, 5, —I and hence its equations are
a*—3__y —8c_z - 3
2 ~T~ —Ϊ *
Ex. 6. Show that the shortest distance between the lines
x+fl = 2^ = —I Iz and x —y+2a = 6z - t a is 2a.
(Barrahampur 1981)
Sol. The equations of the given lines are
( x + e ) /! 2 = j7 6 = z /( - l) f ...(I)
and x/6=0>+2<7)/6 = ( z - a ) /l. ...(2)
Let I9 m, n be the d.c.’s of the line of S.D. The line of S.D.
being perpendicular to both the lines (I) and (2), we have
12/ + 6m —/i= 0 and 6/+6/n + /i=0.
Solving these relations, we have
I _ m _ / i

6 + 6 = - 6 ^1 2 ~ 72=36
or I _ m _ / i _ V(JJ+/n* + «2) _ 1
Shortest Distance 18)

Clearly the line (I) passes through the point A(—a, 0, 0) and
the line (2) passes through B(0, —2a, a).
The length of S.D.=T he projection of join o f ^ a n d P .
on the line of S.D. whose d.c.’s are
I, m, /i.
= /(0 + a )+ /n (—2a—0 )+ n (a—0)
=(2/7) a —(3/7) ( —2a)+(6/7) a - 2 * .
Proved.
Ex. 7. Ifth ea xesa re rectangular, find the shortest distance
between the lines y= az+ b, z=otx+/3, and y= a'z+ b', z = a 'x + j8'.
Hence deduce the condition for the lines to be coplanar.
Sol. The equations of the given lines in symmetrical form
are given by

Ijx a = 1’ .( 1)
and x + F I* '_ y - b '_ _ z
Ijx' a' i - ( 2)
Let I, m, n be the d.c.’s of the line of S;D. The line of S.D.
being perpendicular to both the lines (I) aud (2), we have
/.(l/« )+ w .a + n . 1=0 and /(I/a ')+ m a'+ / 1.1 = 0.
Solving these relations, we get
I m
a—o' ( l/ « ') - ( t/a ; (1/a) a '—(I/* ').a
I m n
ot
xx' (a—a') a —a' a’x '—ax
_________________ I________________
ta —a 'j'+ t* —x')*+{a’x '—aa)*} ...(3)
C IearIytheIine(I) passes through the point Α(—β(χ, b, 0)
and the line (2) passes through 21(-/37*', b \ 0).
The length of S.D.=Tbe projection of the join of A and B on
the line of S.D. whose d.c.’s I, m, n are
given by (3)
= /( - /3 7 ^ + /3 ^ ) + //^ - 6 ) + /1 ( 0 - 0 )

__________(χ’β —χβ') xx' (a—a’)_______


αα' Vl*"*'*(0 —a f + (a—a')*+ (a'x'—ax)*
. __________ (b '-b ) ( « - «')
vHaV* (a—a'j‘ 'r(a -α')*+(β'α'.—aa)*}
184 Analytical Geometry 3-D

[Putting the values of I, m from (3)]


_ (αβ'-α'β> ( a - a ') + ( b - b ') (« -* ')
"" - y / (α— —β'α')1} ’
. ’ neglecting the negative sign.
This is the required S.D.
Ifthegiven lines arecoplanar i.e. intersecting, then S D = 0 .
Therefore the required condition is given by
(αβ'-α'β) (a -a ') ± ( b - b ’) { « - « ') - 0.
Ex. 8. Show that the shortest distance between any two oppo­
site edges o f the tetrahedron formed by the planes y + z —0, z f x = 0 ,
x+.y=0, x + y + z —a is 2a/y /6 and that the three lines o f shortest
distance intersect at the point x y - z= —a.
(Gorakhpur 1975; Lncknow 76,' Meerut 87)
Sol. The equations of t h e e d g e of t h e t e t r a h e d r o n determined
by the planes .y+z—0, z -f *=-0 are
x/1—j / i —z/( —I). . .(1)
The equations of the edge opposite to that given by (I) are
determined by the planes x + y —O, x~-y+ Z‘-~a and hence are
given by
x /l= .y /( - l) = ( z - a ) /0 . -(2)
Let I, m, n be-the d.c.’s of the line of S.D. between (I) and
(2). This line of S.D. being perpendicular to both the lines (l)
and (2), we have
/.1 + m . 1 + / 1 . ( - 1)= 0 and /. 1+ w ( - 1 )+ / 1. 0 = 0.
l_ m n _ + wj4+ «*) I
Solving,-j j - j VUUk+ U i*+(Z)iIe V b ' ...(3)
Clearly the iine (I) passes through the point ^0» 0, 0) and
the line (2) passes through B{0, (\ a).
Λ The length of S.D .= Ihe projection of the join of A and
B on the line of S.D. whose d.c.’s
/, /W, h are given by (3).
- I (0—0)+//* (0 - 0)-1- /i (ά —0)
=r n a «= 2m I γ / 6 .
The equations of S.D. (See § 2).
The equation of the plane through ihi* ii u e J y and S.D. is
X y Z j

I I
*.Γ ϋ
I
I I 2 J
Shortest Distance 185

or x ( 2 + 1 ) - ^ ( 2 + l) + z ( 1 - 1 ) = 0 .
or x —,y==0. —(4)
The equation of the plane through the line (2) and S.D. is
X y z —a

I -I 0

I I 2
or Jt ( - 2 - 0 ) ~ > ( 2 - 0 ) + ( z - a ) (1 + 1 )= 0
or x + y - z + a —0. -(5)
The equations (4) and (5) together are the equations of the
S.D. These equations are clearly satisfied by the point
x —y--^>z= —a.
By the symmetry of the co-ordinates of the point
X --y = Z = r -a ,
it follows that the other lines of S.D. between other two pairs of
opposite edges will pass through the point jc= j;=z*=»—o.
Hencethe three’lines of shortest distance intersect at the point
x= a.
Ex. 9. Find the length and position of the S.D. between the
lines
* 5x -- 2> —3z+ 6= 0 = x —3>>+ 2z—3.

Sol. The equations of the given lines are


*/4=0>+1);3=ηζ—2).2 ...(I)
and 5x—2.y-~3z-|-6=0=x—3>’-|-2z—3. ...(2)
Here we shall use method HI of $ 2.
The equation of any plane through the line (?) is
(5x—2>·-3ζ+ 6)+λ (x—3>·+2?—3)= 0
or χ ( 5 + λ ) + ) ’ (-2 -3 Α )-|* 2 (-3 + 2 Α Η -(6 -3 λ )ώ .0 . ...(3)
If the plane (3) is parallel to the line ( I), then the normal to
the plane (3) will be perpendicular to the line (I) and hence we
have
4(5+ A )+ 3 (—2—3 A )+ 2 (-3 + 2A)=0 or A=8.
Putting this value ot A »n (i), the equation of the plane
through the line (2) and parallel to the line (I) is given by
13x—26y + 13z — 18 -=* 0. ...(4)
‘ Clearly the line (I) is passing through the point /4(0, —1, I).
186 Analytical Geometry 3*D

Length of S.D .= The length of perpendicular from the


point A(0, —1, 2) to the plane (4)
1 3 .0 -2 6 .(-1 )+ 1 3 .(2 )-1 8
“ V U 13)*+(—26)*+ ( 13)*}
34 34\/d 17V6
-13-^/6°3 13X6 ” 39
The equations of S.D. The equation of the plane through the
Iioe (I) and perpendicular to the plane (4) is given by
x )/+1 z—2
i 4 ,3 2 = 0,
13 -2 6 13
x y+1 z -2
or 13 4 3 2
I -2 I
or * (3 + 4 )-0 /+ 1 ) (4 —2)+(z —2) ( - 8 - 3 ) = 0
or 7 x —2y - llz + 2 0 = 0 . ...(5)
Again the equation of any plane through the line (2) is
(5x—2y—3z+ 6 )+ /t (x - 3 y + 2 z - 3 ) = 0
or jc (5+/i)+.y ( —2 —3/i)+z ( —3 + 2/i)+(6 —3/i)=0. ...(6)
If (6) is perpendicular to the plane (4), then we have
13 (5 + /i)—26 ( - 2 -3 /1 )+ 1 3 ( - 3 + 2 / 0 = 0 .
Dividing it by 13, we get
5 + /1 -2 ( - 2 - 3 /1 ) + ( - 3 + 2 /0 = 0 , or /1 = -2 /3 .
Puttingthevalue of μ in (6), the equation of the plane
through the line ( 2 ) and perpendicular to the plane (4) is given by
1 3 jc — 1 3 z + 2 4 = 0 . ...( 7 )
.'. The equation (5) and (7) together are the required equa­
tion of the S.D.
Ex. 10. Find the length of the shortest distance between the
z-axis and the line
x + y + 2 z —3= 0 = 2 jc+3>/+4z—4. (Gorakhpur 1981)
Sol. Here we shall use Method 111 of § 2.
The equations of z-axis are
x /0 = y /0 = z/l. - ...(I)
The equations of the other line are
x+ y + 2 z—3=0, 2jc+3y+4z—4= 0. ...(2)
The equation of any plane through the line (2) is
(x + y + 2 z -3 )+ A (2jc+ 3)/+ 4 z —4)=0
or x (l+2A )+y (l+ 3A )+ z (2+ 4 A )-3 -4 A = 0 . . ...(3)
Shortest Distance 181

ΙΓ the plane (3) is parallel to the line (I) [i.e. z-axis] then the
normal to the plane (3) will be perpendicular to the line (I) and
hence we have
0.(1 + 2A) + 0.(I + 3A)+1.(2+4A) = 0, or A = - J .
Putting thisvalue of A in (3), the equation of the plane
through the line (2) and parallel to z-axis [i.e, (1)] is given by
x 0 + ^/.(1—3/2) + z.O—3 + 2 --0 , or _y+2=0. ...(4)
Clearly z-axis i.e., the line (I) passes through the point
(0 .0 ,0 ).
Length of S.D. —the length of perpendicular from the
point (0, 0, 0) to the plane (4)
04-2 2
^ “ i
Ex. 11. Find the shortest distance between the z-axis and the
line
ax+ by+ cz + d —Q= a'x+ b'y+ c'z+ d'. (Meernt 1984)
Show also that it meets the z-axis at a point whose distance
from the origin is
(ab' — d'b) (be'—b'c) + (ca'—c'a) (ad’—a'd)
(6t' —b’c)a+ (ca'—c'af *
(Agra 1979; Allahabad 78; Boodelkhand 79;
Kanpur 75, 76, 79, 82; Lucknow 77, 82; Rajasthan 78)
Sol. The equations of the.z-axis are

5 - o - f - r*(say)* ...(I)
The equations of the other line are
ax+ by + c z + d ^ O ^ a 'x + b 'y + c 'z + d '. ...(2)
The length of S.D. We shall rind it by Method HL
The equation of any plane through the line (2) is
(ax + by+ cz+ d) +A (a'x + b'y + c'z+ d ')=*0
or x (fl+Ai/)+^ (6 + A6f) + z (c+Ac')+(i/+-A6f,)=aO. ...(3)
If the plane (3) is parallel to the line (I) [i.e. z-axis], then
0 (a + Aa ) + 0.(6 + A6 ) + 1. (c+ Ac;)— 0 or A= —c/c*
Putting this value of A in (3), the equation of the plane
through the line (2) and parallel to z-axis is given by
x (a—ca'/c')+ y {b -c b '/c ')+ z ( c - c ) + ( d —cd’lc')=*0
OT x (ac'—ca')+ y (b c '- cb')+(dc’—cd’) = 0. ...(4)
Clearly z-axis i.e., the line (I) passes through the point
(0, 0, 0).
188 Analytical Geometry 3-D

Λ Length of S.D .=the length of perpendicular from the point


(0, 0, 0) to the plane (4)
0 + 0 4 (d c '-c d ')___
e y/\(ac'—c a 'f+ ib c '—b'c)*}
dc'—d'c
\/{(ac’ —a'c)%+ (be'—b'c)*} Proved.
The distance from origin of the point where the line of S.D.
meets the z-axis. It will be convenient to use method II.
The equations of line (2) in symmetrical form arc
bd'—b'd a'd—ad’
X ab'—a'b ^ ab'—a'b z —0 .(5 )
bc'—b'c ~ ca '-c'a = ab'—a'b
[See § 4, Chapter 4]
For convenience, let (5) be written as
λ - x, y - y t z—Q
= r2 (say).
I» mu ~ nt /I* ■" ...(6)
Since 0, 0, I are the actual d.c.’s of the z-axis [i.e. the line (I)],
therefore T1 is the actual distance of any point on the line (I) from
(0, 0, 0). Let P (0, 0, r,) be any point on the line (I).
Also any point on the line (6) is (Z2r2+;c2, m trt+ yt, ntrt),
say the point Q.
The d.r.’s of PQ are
Urt + Xt, mtrt + yi, ^ r i - rt.
Let the line PQ be the line of S.D., so that PQ is perpendi­
cular to both the lines (I) and (2), and therefore, we have
0.iZ2r2+ x 2)+0.(Mi2r2+ ^ 2)+l.(M2rs- r , ) = 0 or T1=M2T2
and Z2 (Z2r2+ x 2)+ m 2 im i 2t2 + ;> 2) + m2 ( / I i r 2- T 1) = 0
or Z2V2+Z2X2+/H2V2+/M2.y2+ 0 = 0 [V T1=M2T2]
or T2 (Z22+/w2* )= -(Z 2x2+/r/2.y2)
or T2= - (/2x2+zh2.y2)/(/2s+ /M2*).
Λ T1 - M 2T2 = M2 (/2x2+ /11^ 2)/(/22+ Ml2*)
Substituting the corresponding values of Z2, mi2, m2, x2 and yt
from (5) and (6), we have
-(a b '-aa'b)
'b ) J /i tt.,, tL,^
, A( bbdd'-'-bb 'd
'i )
'(bc'-b'c)>+(ca -c'a)* ] ( 0 C)' ab'-a'b t'b
, , , , . a'd—ad' I
+ '" - " ''( 5 ¾ ¾ }
(db'—d'b) (bc'—b'c)+(ca'—c‘a) (ad'—a'd)
* (bc'—b'c)*+(ca‘—c'a)* Proved.
Shortest Distance 189

Ex. 12. Show that the shortest distance between the lines
x - x \ _ y - y i _ z - z i x —Xj _ y - y % _ *—*t
cos a, cos βι cos γ\ cos a2 aaCos /S2 cos y2
meets the first line in a point whose distance from (xlf yt, Z 1) is
[27 ( ( X 1 - x t) ( cos « i — cos B cos * » ) } ] / j / n * Θ
where B is the angle between the lines.
The epuations of the given lines are
* *' _ r - ? _ (say)
COS «1 COS βι COS yi x ...(D
x —x* y —y* z - z % ,
----- ?=> -— ~5 β -------- =>Γ| (say).
COSa2 COSjSa COSy2 ...(2)
Aoy point P on (I) is
(rt cos * i+ x ,. r, cos /3,+y,. rx cos y , + Z i )
and any point Q on (2) is
(r, cos cc2+ x 2, r2 cos /S2+ y 2, r%cos y 2 + z 2) .
Let the shortest distance meet the line (I) at P and the line
(2) at Q.
Now, siuce cos otlf cos /Slf cos y, are the actual d.c.’s of the
line (I). therefore r2 is the actual distance of P from the point
(x„ yi, z,) on the line (I). It is required to find rx.
D .r/s of PQ are rt cos a ,+Xj - r2cos a*—x2, T1 cos β ι+ yi
—r2 cos β i—y»,rι cos yx+Z i—r2 cos y2—z2.
Since PQ is the line.of shortest distance, therefore, it is per­
pendicular to both the lines (I) and (2), and hence, we have
cos «i (rt cos Ot1-Hx, —rt cos a2—x2)
+cos βιXri cos β ι+ y i- r t cos βt - y t)
+cos yi (r, cos y i + Z 1- T 2 cos y2—z2)=»0 ...(3)
and cos ot2 (T1 cos a, + x, — r2 cos a2—x2)
+cos /J2 (r\ cos βι +y, —r2 cos βt —y»)
+cos y2 (Ti cos y,+ Zi —r2 cos y2—ζ2)=·0. ...(4)
Now
cos* a!+cos® /8|+cos* yi==l, cos® a2+cos* /S2+cos* y2= l I
and cos at cos a2+cos /S1 cos /S2+ cos γχ Cosy2= cos 6, f
-(5 )
B being given to.be the angle between the lines (I) and (2).
From equation (3/, we have
T1 (cos* Ot1 +cos® /S1+cos* Yi)—r%(cos Ot1 cos a2+cos /S1 cos /S2
+cos yi cos y2)+ (x i—Xt). cos y2) cos /S1
+ U i-z » ) Cosy1=O
190 Analytical Geometry 3-D

ot T i - r2 eos Θ+Σ {(xt—x’a’i cos αχ}=0 {using (5)}. . (6)


Similarly equation (4) may be written as
— Γ* + Γχ cos Θ+ Σ {(xt — X i ) cos a2}*^0. -. (7)

Multiplying (7) by cos ¢ and subtracting from (6), we have


Γ| (1—cos*, ¢)+2/ {(xi —x2) cos αχ}—Σ ((X1- Xt) cos a*} cos ¢=0
dr T1 sin* ¢+27 ((X1- x2) (cos ax—cos a2 cos ¢))==0
or Τ\=*\Σ {(xi—x2) (cos «!—cos a* cos ^}]/sin* ¢,
neglecting the negative sign because rt is the distance.
Ex. 13. Show that the equation o f the plane containing the line
7 /ft+ z/c= l, x = 0 and parallel to the line
χ/α—z/c = l, 7 = 0 is x/a—vIb·--zjc + 1 = 0
and If 2d id the shortest distance then show that d_a=a~, + / r , +c~, .
(Agra 1977, 81; Kanpur 82; Punjab 82; Ranchi 74;
Meerut 87P, 90 P
Sol. The equations of the given lines are
y/b + z/c= ), x = 0 -(1)
x z , Λ x —a y z
and
a c a O c ...(2)
the equations of the second line being put in symmetrical form
Tbe equation of any plane through the line (I) is
(yjb+ zjc— 1)+Ax=0 or Ax+(1 /b) y + ( I j c ) z—1=0. . (3)
If the plane (3) is parallel to the line (2), then the normal to
the plane (3) whose d.r.’s are A, 1/b, 1/c will be perpendicular to
the line (2), and so we have
o.A +0.(l/i>)+c.(l/c)=0 or λ = -(1 /α ).
Putting this-value of A in (3), the equation of the plane con­
taining the line (I) and parallel to the line (2) is
- J L + . £ + J _ _ 1=o or — — + 1=0.
a b c Q b C ...(4)
Clearly (a, 0, 0) is a point on the line (2). Hence the length
2d of the S.D .=the perpendicular distance of (a, 0, 0) from the
plane (4).
a fI/a) —0-—(-I 2
2d=
{(1/β)'-t t — t/b;*+! — t/c)'} \/(a“2+ * - *+ c_*)
or d*= I/(η- *+/>■*+£“*) or d-i =a~i + b-i + c - \
Ex. 14. Show that the shortest distance between the diagonals
o f a rectangular parallelepiped and the edges not meeting it are
bejy/(b*+c*), co/V^fca+ ο 1), ab/y/ta*+!)*)
where o, b, c ate the lengths o f the edges.
(Punjab 1981; Garhwal 78S)
Shortest Distance 191

Sol. Consider a rectangular parallelopiped whose three coter­


minous edges OA, OB, OC are taken along the axes of x, y and

z respectively. Also OA=*a, OB=b and OC=*c. The co-ordinates


of the different vertices are as follows ;
O (0,.0, 0), Λ (a, 0. 0). B (0, b, 0), C (0. 0, c), D (0, b, c),
E (a, 0, c), F (a, b, 0), P (a, b, c).
Consider a diagonal AD and the edge OB not meeting this
diagonal. Now we shall find the length of S. D. between AD
and OB.
The equations of AD are
x-------
—a=» v—0 2 - 0 Ο Γx____
------ e=3 ____
- a C 3y- = —z ·
0 —a b—0 c—0 —a b c ...(I)
The equations of OB are
x _ y -b z_
O r' I- e O* ...(2)
Let /, m, n be the d.c.’s of the line of S.D. between the lines
(I) and (2). Since the line of S D. is perpendicular to both the
lines (I) and (2), we have
I (—a )+ m .b + « .c = 0 and /.0+/η.1φπ.0=*0.
Solving, I _n ^ y l(Jl + w,2+ n*)_3____ I
c O a v'lc'+ O + a") y/(c*+a*) ...(3)
Λ The length of S.D. between AD and OB
■=the projection of the join of (a, 0, 0) {a point on (1)}
and (0, b, 0) (a point on (2)} on the line of S.D.
whose d.c.’s /, m, n are given by (3)
«*/ (a—0)+ m (0-:b)+ n (0—0)®=/a
•=*acl\/(c·-t-a*). ...(4)
192 Analytical Geometry 3-D

Similarly the shortest distance between other pairs of lines


can be found.
Ex. 15. A square ABCD o f diagonal I a is folded along the
diagonal AC so that the planes DAC, BAC are at right angles. Find
the shortest distance between DC and AB. (Agra 1976, 80; M.U. 90)
Sol. ABCD is a square of diagonal 2a, so that AC=BD==Ia.
Let O, the centre of the square, be chosen as origin of co-ordi-
nates and the diagonal CA be taken ajong x-axis. Hencethe

co-ordinates of the vertices A and C are (a, 0, 0) and ( —a, 0, 0)


respectively.
Now as given in the problem, the square is folded over along
the diagonal AC so that the planes DAC and BAC are at right
angles. T bisim pliesthattbe IinesO S andO D become at right
angles. Also OA is perpendicular to the plane DOB. Hence the
lines OA, OBt OD are mutually orthogonal. Let us now take OB
and OD as y and z axes respectively.
.*. The co-ordinates of B and D are (0, a, 0) and (0, 0, a)
respectively.
The equations to AB are — - — —®
a —a 0 ...(!)
The equation to DC arc ίΠ ϋ
a 0 a ...(2)
The equation of any plane through DC and parallel to AB
[I.*. through the line (2) and parallel to the line (1)] is
x—0 y —0 z —a
a 0
a ■a 0
Shortest Distance 193

or x (o*)—y (—tF)+(z—e) ( —a*)=0


or x + y —z+ a= 0. -O )
The S.D. between DC and AB
= th e length of perpendicular from a point (a, 0, 0) on
AB [l.e. (I)] to the plane (3)
_ a 4 0 —0 + a 2a_
vT(l )*+0 ')*+ (- 1)*Γ λ/3
Ex. 16. Find the length and equations o f the shortest distance
between 3 x - 9 y + 5 z = 0 = x + y —z
and 6x-f 8 y + 3 z -1 3 = 0 = x + 2 y + z —3.
(Meerut 1984 P, 86)
Sol. Here we shall use method IV of § 2. Theequationsof
the planes through the given lines are
(3x—9y+5z)+At (x + y -z)« = 0
and (6 x + 8 y + 3 r—13)+At (x + 2 y + z —3)=0
or * (3 + A ,)+ .p (—9+ A ,)+ 2(5—A,)=0 ...(I)
and x(6+A,)+^(8+2A1)+2(3+A ,)-(13+3A ,)=0. ...(2)
If the planes (I) and (2) are parallel, then tbeir coefficients
are proportional and so we have
3+Aj - 9 4 - A1 S-At
6+A ,“ 8+2A, ~ k (say).
3+A* (3)
Taking the ratios 1st, 2nd and 3rd with k respectively in (3),
we get
(3+A1)=k(6+A 1) or 3+A,—6A:—Ar At =O ...(4)
( —9+A1)=fc(8+2Ai) or —9+At —8Jfc—2Ac Aa-=O ...(5)
(5-A,)=A:(3+A,) or 5-A t -3Ar-Ar A,=0. . (6)
Substituting (6) from (4),
—2+2At —3fc=0. ...(J)
Subtracting 2 times (6) from (5),
—19+3A,—2*=0. ...?X)
Solving (7) and (8), A=53/5. Ar=32/5
Putting the values of At and Ar in (4>, we get A»= —31/8.
Substituting the values of A1 and At in (I) and (2), the equa­
tions of t!ie parallel planes through the given lines are
1 7 x + 2 y -7 z = 0 . (9‘
and 17x4 2y~7z—11···= 0. .. (IO)
The required S.D. is the distance between the p rallcl piare.
(9) and (10).
194 Amalyiieal Geometry 3-D
Any point on the plane (9) is (0, Of 0).
Λ The length of S.D .=the length of perpendicular from
(0f Of 0) to the plane (10)
0 + 0 -0 -1 1 Il
V{U7)*+(2)*+(-7)*} V(342)
[Numerically]
The eqntii tfSD.
The equation of any plane through the first given line is
^ (3+AO+y ( - 9 + * ,) + * (5 -A t) - 0 [See (1)] ...(Il)
If the plane (I I) is perpendicular to (9) or (IO)f we have
17 (3+AO+2 ( 9 + A1) T 7 (S-A 1)=O or A1- 1/13.
Putting the value OfA1 in (11) the equation of the plane
through the 1st given line and perpendicular to the plane (9) or
(10) is given by
IQ x -29y+16z=0. ...(12)
Again the equation of any plane through the 2nd given line
is [See equation (2)]
x (6+A ,)+y (8+2AB)+ z (3+Ai)-(13+3A B) = 0 , ...(13)
If the plane (3) is perpendicular to (9) or (IO)t we have
17 (6+AB)+ 2 (8+2A ,)-7 (3+A ,)=0 or A,= -5 8 /7 .
Putting the value of Ab in (13), the equation of the plane
through the 2nd given line and perpendicular to the plane (9) or
(10) is given by
13x+82y+55z—109=0. ...(14)
The equations (12) and (14) are the required equations of the
shortest distance.
Nate. We can solve the above problem by reducing both the
lines to symmetrical form and then using method I or 11. The
problem can also be solved by reducing only one line to symmetri­
cal form and then using method III.
Ex. 17x. Find the shortest distance between the lines
x = 0 f J y + iz = l
andy =O9 J x - J z = I . (Agra 1982)
Sol. The equations of the 1st line in symmetrical form are
x _ y —2 _ z
0= T “ a - 3 * ...(I)
The equations of the 2nd line are
/= 0 , J x -J z = I ...(2)
The equation of any plane through the line (2) is
Shortest Distance 195

(fx—\ z — 1)+Ay=0
or Z x + U X y -A z-M = Q . .(3 )
If the plane (3) is parallel to the line (I), then the normal to
(3) is perpendicular to the line (I), and hence we have
0 .3 + 2 .12A—3 .( - 4 ) = 0 or A=» - J
Putting the value of Λin (3), the equation of the plane through
the line (2) and parallel to the line (I) is given by
3 * _ 6 y —4z—12=0. (4)
The line (I) clearly passes through the point (0, 2, 0).
The length of S.D .=the perpendicular distance of (0 ,2 ,0 )
from the plane (4)
0 —12—0—12 _ 24
(Numerically).
v/{(3)»+(-6)*+(-4)*}= V(^I)
Ex. 18. Prove that the S.D. between the\llnes
ax+by+cz+d=0=>Q'x+b'y+c,z+ d '
and ΛX+ βy+γz-^-δ= 0=Λ'x+ β'y+γ,z+ δ' is
d d' δ δ'

a a’ a d'
+ y/[Z (B C -P C )* ]
b b’ β β'

c c' γ y'
where A ^ b c '—b'c and Α '= β γ'—β’γ etc.
Sol. The equations of the given lines are
ax+by-\-cz-\-d=Q=‘Q,x + b’y + c ,z+ d ' -.--(I)
and ax+ /3 y + y z+ e= 0 = a'x + /3 '.y + y 'z+ 8 '. ...(2)
We shall use method IV. The equations of any planes through
the given lines (I) and (2) are
(ax+by+cz+d)+Xx (a'x+ b'y+ c'z+ d')—0 ...(3)
or x(a+ X la,)+ y (b + \ xb ')+ z(e + \le,)+(d+Xld ')» 0 ...(3')
and (oιx+βy + γz-\-δ)+λ9 (<ι'x+β'y+γ'z+δ')=0 ...(4)
or x (a+fl2a')+_y (/J+A^'J + z (y+Afy') + {?)+A.j8')= 0 ...(4')
If the planes (3') and (4') [l.e. (3) and (4)] are parallel, then
their corresponding coefficients are proportional and so we have
α+λ,ο'^ δ + λ ,δ ' ^ + Α , ι ' ,
a+Asa' β+ \φ’ y+A,y' K
From these relations, we get
0 + A1**' — ArA2Ocr= 0 ... (5)
196 Analytical Geometry 3 -D

b + X ib ’ - Ι ι β - ί Γ λ ύ S'=0 ^ (6)
c+Atc'—Ary-ArAiy“ 0. ---(I)
Eliminating Aj, -Ar, -ArA1 betv^een (3), (5), (6) and (7), the
equation of the plane through the line <1) and parallel to (2) is
given by vv x
ax+ by+ cz+ d a'x+ b'y+ c’z+ d ’ v 0

a a' a a' 0.

b b' β β'
C c' Y Ύ
Adding (—x) times second, (—>*) times third and ( —z) times
fourth row to the first row, we get
d d' —(«x+ fiy+yz)
a a' a a' . —0 ...(8)
b b’ β β'
C c' Y y'
Now we shall evaluate the coefficients of x ,y and z in the
expansion of the determinant in (8).
The coefficient of x
« - a a a' a' ( - « ') a a' a

h V β9 b b' β
C c9 y9 C c' Y.
e= a a! —a a ' a a' aa'

b' -OC0' + b b' /Sot'


Ib
C c' -ay' 1 C c' ya'
=» a a' a a ' — aa' o a' 0

b b' β χ '- χ β ' i■ b b’ -C ,

C c' ya'—ay' c c' B' i


« C ' (ac'-a'c)+ B ' (ab’- a ’b) [V It is given that
Β '^ γ χ '- y ' a , C'—χβ' —χ'β]
Shortest Distance 197

= C (~ B )+ B ' (C)=CB1- B C '. I


Similarly the coefficient of y= A C '—CA' I ...{9)
and the coefficient of z= B A '—AB'.J
Now suppose (Xi, y it Zi) is a point on the line (2), so that we
have
«Xi+/3yi+yZ|+S<=0 and a 'x i+ ^ ’yi+ y^Z j+S ^O '
or «x,+j3y,+yz,~ —8 and Λ’χ ι + β'γι + γ'ζ1= —6’ ...(10)
Now the required S.I^between the lines (I) and (2)
=Ths length of the fterpendicular from the point (xit y lt Zi)
on the line (2) from the plane (8)
d d' - ( a x i + / 3 y i + y r , ) - ( « 'X i + / 3 > , + y 'z i )

a a' a a'

b b’ β β'

C c' Y /
- r -^[(coefficient of x)*+(coeff. of y)*+(coeff. of z)2].
Putting values from (9) and (10), we get the required S.D.
d d' δ δ'

a a' a a'

b b' β β'
C c' Y y' Proved.
Ex. 19. Two straight
X- a i - y - β i ,
/. W1 fli It mt nt
are cut by a third line whose direction cosines are λ, μ, v. Show
that ‘d’ the length intercepted on the third tine is given by
d Λ Wi Πι - 0C|—OC2 β ΐ-β * yi—ya
It W2 W2 /i mx «1
λ V Ii W2W1
Deduce the length o f the shortest distance between the first two
lines.
Sol. The equations of the given lines are
(x -c tiV /i-iy -ft)/» » !—(z-y i)/n i= r! (say) • -(1)
198 Analytical Geometry 3-Z)

and (x - « ,) / /,= (>- / 3.)/ 111.=.(* ^ y»)//i,—r, (say). ...(2)


Any point on the line (I) is P (/ι/ι+ α ι, miri+βι, /^r1-Hy1).
Any point on the line (2) is
Q (/*r»+« 2» wiirs+0*. ” .r* + y*). "-(3)
Let the third line with d.c.’s λ, μ , v meet the line ( I) at the
point P and the line (2) at Q so that PQ=d.
Now 4he third line with d.c.’s λ, μ, v is passing through the
point P, hence its equations are
x — ( h ^ + * , ) _ y - ( m l r1+ fil ) _ z - ( n ir 1+ y ,) (say) (4)

Λ The co-ordinates of the point Q at a distance ‘d’ from


the point P on the line (4) are
ί/λ+ΖιΓι+αι, όμ,-\-ηΐιΤι-\-βϊ%Λ -f-Wiri+yi ·*·(5)
Now the co-ordinates given by (3) and (5) are of the same
point Q. Hence comparing (3) and (S)9 we get
έ/Λ4-/ιΓι+α1= /8Γ2+ α2 )
άμ+ m jy +JB1= mara+ 0a >
^+ W 1Zi1+ y i= n ara+ y a J
or rfA+ (ai.—(X2) 4- ii^i— )
όμ + (βι“ 02Η*^ i rI-W ar2=O ? , ...(6)
rfv+(y i-y a )+ « iri—W2^==O )
Eliminating Ti and ra from the relations (6)9 we get
^A-H(OC1-OCj) /. /a

0μ + {\Si-/3.) W1J m%

dv f (y i—y.' «1 n2
Spilling this determinant into two determinants» we have
d A /i /a + «1 — a2 /1 /. « 0

μ mi ma 01 - / 3 . w, m.

V Wi W2 yi W2
I
e>

d A. /i /2 = — « 1 — «a /1 h

μ mi ma 01 — 02 Wli m2

V Wi W2 n - y* Wl Wa
d /l '« I Wi = — «1 — «2 0 1 -0 . y i - y 2

h m2 W2 /1 Wli Wl

A V ' /2 Wla Wa
Shortest D istt 199

Since d is the distance, hence neglecting the negative sign in


(7), the required result is obtained.
Now if d stands for the S.D. between the given Iines(I) and*
(2). then the third line with <Lc.*s λ, * . v is perpendicular to both
the given lines (I) and (2) and hence we have -
/iA+»»i#»+UiV=«0, |V=0.
Solving, we get
A ^ μ ^ ^ -y/(A*+**+v*)
(Hi1Hf - H t 1H1) VL^
_____ __ I________
“ v [ ^ (WI1II1-IfT1Jf1)*! *...(8)
The value of the distance d given by (7) «rill become the
length of S.D. between the given lines (I) and (2) if the values of
λ, μ, v are substituted in it from (8).
Now the coefficient of d in (7)
— Z1 mi »1
Zi ms «*
A V
**A (IH1IT1- nit»»)+* (Uif1- «A )+ v (hm—hmt)
_ Σ (IH1W 1-IW1Bi)1
^ τ/[Σ (WfjW1-IH1W1)*]
(potting the values of A, * , v from (8)]
(Wf1W1-Wf1Hi)*].
Using this value for the coefficient of d in (7). the SJ>. *<T is
given by
d = * i—«i βι—β* r i —r t V l £ (HI1U1 - I H 1W1)* ].

/. HT1 H1
Z1 HT1 W
1
Exercises
I. Find the equations of the straight line perpendicular to both
of the lines
x - I j - 1 z+ 2 x + 2 y - S z+ 3
T ^ — T~ “ d T - = = T — 2 -‘
x —2 y - 3 z - 1
Aas.
7 “ 4 ~ —5 '
2. Find the length and equations of the common perpendicular
to the two lines
200 Analytical Geometry 3-D

* + 3 y —6 z . *+2 y z -7
- 4 —■— 3 -" 2 ----
*“d - 4 ~ i !Ί
(Meerot 1984; Andhca 68; Gorakbpnr 74; Madras 76)
Ans. The length of common perpendicular (i.e. S.D.)=9.
Tbe equations of S.D. are
32*+34,y+13z—1 0 8 = 0 = 4 * + 1lj»+5z—27.
3. Show that the length of shortest distance between the lines
* —2 .y + l z .
2 ~ 3 =4 ;
2*+3j’—5z—6 = 0 = 3 * —2y—z+3 is 97/(13^6)·
(Rajasthan 1975)
4. Find the length and the equations of the shortest distance
between the lines
5*— z = 0 = * —2_y+z+3
and 7*—4y—2 z = 0 = * —y + z —3. (Meerut 1986 S)
Ans. The length of S. D. is 13ISy/2.
The equations of S. D. are
17*+20j>—19z—39=0, 8 * + 5 .y -3 lz+ 6 7 = 0 .
5. Find the equation of the shortest distance and its length
between the lines
* —3 y —5 z - 2
I “ -2 " I ’
* —I y+ 1 z+1
'7 " - 6 “ Ί (M. U. 1990)
6
Volume of Tetrahedron

§ I. (A) To find the volume o f a tetrahedron, whose three


coterminous edges in the right-handed orientation are a, b, c, where
a, b, c are vectors.
Let OABC be a tetrahedron. Let
O be the origin and let position vec­
tors of the vertices A, B, C be a, b, c
respectively, so that
OA=*a, OB=b, OC= c.
Thenthe volume V of the tetra­
hedron is given by
V = I (area of the triangie OBC) X (perpendicular length from
A on the plane OBC). ...(I)
Now the area of A O B C = I | b x c | . ...(2)
I f n b e th e unit vector perpendicular to the plane of the
A
triangle OBC such that b, c and n are in right handed orientation,
then
V bxc '
!bxci *
since b, c, b x c are in right handed orientation.
Λ the length of the perpendicular from A on the plane
OBC=the length of the projection of OA on the perpendicular to
the plane OBC in the direction of n
*OA· a=a · b x c a*(bxc) [a, b, c]
bxc I bxc I ™ I bxc I ...(3)
Putting the values from (2) and (3) in (I), we get

k x e lT S R T
or V= i la, b, cj. ...(4)
This is the required formula for the volume of the tetra­
hedron.
202 Analytical Geometry 3-D

(B) To find the volume o f the tetrahedron OABC whose one


vertex O is at the origin and the co-ordinates o f the remaining three
vertices A, B and C are (Xl t JUt Z1)t (x*, JU· z»> and (x3#ju. z3)
respectively.
Let a, b, c be the position vectors of the vertices A9 B9C
respectively w.r.t. O as origin. Sincethe co-ordinates of A are
(xlt y u Zi), therefore, the position vector a of the point A is given
by
a «=OA c=X1I+ ju j + Zik.
T^· —►
Similarly b=OB —Xai+j'Ud-Zik and c=0C*=x8i+ jr8|+ z 8k.
Now the volume V of the tetrahedron OABC is giveh by
K =J [a, b, c] [See § I ( A), for complete proof deduce this
result here]
or V= J X1 Jt Zl
X2 ju Z2 I
X3 JU Z3 I
The formula (S) is the required formula.
(C) To find the volume o f the tetrahedron whose vertices have
(Xu yi, Zi). (**, Jr*, Z*), (x8. )'s, Z8) and (x4, y4. zt ) as co-ordinates.
Let A, Bt C, D be the vertices of the tetrahedron DABC. Let
( X 1 , yu Z i ) , (X*. Jr8, Z * ) , (X85 J j , Z9), (x4, y t , z4) be the co-ordinates
of the points A, B, C, D respectively. Then the position vectors
of At Bt Ct D are X1I +JilH-Zik, X8Id-Jr8Id-Zik, Xiid-J’jj+Zjk,
Xiid-J1Ijd-Zik respectively.
We have ^ « (x ^ d -y ijd -Z ik )—(x8id-y«jd-z«k)
=(X1- X 4) Id-(JrI - J r4) Id-(Zi-Z4) k,
DB=(Xi - X 4) Sd-(J8- J r8) Id-(Z8- Z 8) k.
and DC=(X9- X 4) !+O r8- J r4) j+ (z8—z4) k.
A The volume V of the tetrahedron DABC is given by
v = i [DA. DB, DC] [See § I (A)]
X1- X 4 J rI - J U Z i- Z 4

= i i X8- X 4 Ji2-JU Zi - Z 4

! X8- X 8 JU-JU Z3 - Z 4
Volume o f Tetrahedron 203

* 1 - *4 y i-> i Z 1- Z 4 0'

*2 -*4 y *-y 4 Z2 - Z i 0

£
I
*4 Z s-Z 4

CO
*4 y« Z4 I

Adding 4th row to 1st, 2nd and 3rd rows, we get


*1 y\ Z\ I

X2 Z2 I
. ( 6)
Xs ys Z9 I

X4 >’4 Z4 I

The formula (6) is the required formula.


Corollary. Condition for four points to be coplanar.
The four points A9 B9 C9 D will be coplanar if the volume of
the tetrahedron formed by them is zero, i.e.
*1 yi Zl I

λ*2 ys Z2 I

*3 Xs Zs I

*4 ) ’* Z4 I

§ 2. To find the volume V of a tetrahedron, in terms of the


lengths of three concurrent edges and their mutual inclinations.
Let OABC be the tetrahedron. Take the vertex O as the
origin. Let the lengths of the three edges OA9 OB9 OC be a9b9 c
and the angles BOC9 COA9AOB bt X9 μ, v respectively. Let any
three perpendicular lines through the origin O be taken a$ co­
ordinate axes.
Again let the direction cosines of the lines OA9 OB and OC
be Iu mu nl9 I29 n2 and I39 m3%n3 respectively, so that the co­
ordinates of the vertices A9 B and C are (lxa%mxa9 nxa); (ltb9 m%b9
ntb) and (I3C9 m3c9n3c) respectively. We have
OA=Q=lxai +Mia\+ntak9 O B= b=l2b\+ m2b\ +

and OC= c = / 3c i + m * c j + W8c k .
204 Analytical Geometry 3-Z)

Again I=fl6 cos v }


a»b~ ab (Z1Z8H-W1W2+ nxn%)=ab
b*c«^c (Z8Z8+/W8Wa+W2W8)=)^b c cos Λ /
c»a=cfl (Z8Z1-^w8Wi+ ng^Jc
I= Cfl cos μ J
or ZiZt+WiWa + n^aecos v j
Z2Z8+ W 2Wi8 +w2n3=cos λ >. . (U
Z8Zi + w 8W|+/Iafi1= cos /4 ;
Now the volume F of the tetrahedron OABC is given by
F - * [ · , b, c ] = i Ii a mxa nxa
Ub m2b n%
b

UC mzc ZV8C

= £ abc Zi Wi Wi
Z8 W8 /I2

/3 m8 w3

Fi = - I f l W u WI1 W1 /1 W1 W1
Jo
U W8 W2 X Z2 W2 W8

Z3 W3 W8 /3 W3 W8

Z^+Wj·+//!* Z1Z2+ W 1 W 2+ / I i f i 2 Z1Z3+ W 1W 8+ / I 1Zf3


~36e W
Z2Zi+ W 8W i + W2Zii Z8*+ w 2* + / I 2* Z2Z3+ W 2W 8+ Zi2W
8
/ a Z i + W a W i + / I 3ZI1 Z8Z2+ W 8W 2+ / ; 3/i2 /3a + z w 8a + /I32

I cos v cos μ
=■·-T- fl‘CSC*
36
COS V I cos A [using the relations (I)].
cos μ cos Λ I
Λ F = ± J ubc I COS V COS μ *
cos v I cos Λ
COS μ COS A I
The negative sign will be neglected in calculating tbe magni­
tude of the ycIume V.
§ 3. To find the volume V o f the tetrahedron when equations o f
its four faces are given.
Volume o f Tetrahedron 205

Let tbe equations of the planes representing the four faces of


the tetrahedron be
a tx + b ty + c iz + d t^ n ...(I)
a tx + b ty + c tz + d t= *0 ...(2)
a »v 4- b »y + c9z + d3— 0
ot x+ bty+ ct z+ dt =Q. ...(4)
Now a set of any three planes out of the four planes given
above, will intersect in a point, a vertex of the tetrahedron. Hence
the four planes, taking three at a time, will intersect in 1C8 i.e. 4
points, the four vertices of the tetrahedron.
Now solving (2), (3) and (4) by the method of determinants,
we get
X -y
b, Ct dt Q2 C2 d%

b» C» d, Q2 C2 d2

bt ct dt I at ct dt
i

Z -I
Q2 b2 ar2 Q2 b2 C2

Q2 b2 a as bi Cz

Qi bA d ot b4 Ci

Suppose Δ = ai bx Cl dx

Q2 bt C2 dt

Qz bt C2 dt

Qi bt Ci dt

Let the capital letters represent the co-factors of the corret


ponding small letters in the determinant Δ» Ai9-ft* Cii Di9
A29 B29...etc. represent the co-factors of au bl9 cu dlt a2%b29... etc.
respectively in Δ· The result (5) may be written as
x _ —y _ z_ _
Ai **'~Β\ Ci " - :b V
The p >i’i; of inter.ecium of th·; planes (2), (3), and (4) is
(AiIDi9 BiIDl9 C1ID1). Mmilarly solvm ; the other three sets of
three planes, the points of intersection the other three vertices
of the tetrahedron urs
206 Analytical Geometry 3-D

I A t B% ^ \ , /^ i Β» C*V ( A t B t C t Y
DtV \Z V D i ' D t ) '
I i V D tt D t ) ' \ Dat ZV
Λ The required volume V of the tetrahedron is given by
At Bi Cl
I
Di Di Di

At Bt Ct
I
Dt D2 Dt
[See § I (C), equation (6)]
At Bt Cz
I
Dt Dt Dt

At B, Ct
I
Dt Dt Dt
I Ai Bi Cl Bi
6Z)| D] OgDi
At Bt Ct Dt

At Bt Ct Di

At Bt Ct Dt
_ A*-1 [since if A ' be the determinant of nth order
6Z)(DtDtDt formed by the co-factors of the elements of
the determinant A. then Δ '= Δ " -1]·
•'· V*< A8
IiDlDiDtDt

SOLVED EXAMPLES
Ex. I. Find the volume o f the tetrahedron, the co-ordinates of
whose vertices are (2, —1, —3), (4, .1, 3), (3, 2, —1) and (I, 4, 2).
(Punjab Ϊ981, 82)
Sol. The volume V of the tetrahedron, the co-ordinates of
whose vertices are (2, —1, —3), (4, I, 3), (3* 2, —l) and (1,.4, 2)
is given by
2 -I -3 I [using (6) of § I (C)]

4 l: 3 I
3 ’ 2 -I I

I 4 2 I
Vobune of Tetrahedron 207

2 -I -3 I , adding (—I) times 1st


row to 2nd, 3rd and
2 2 6 0 4th rows

I 3 2 0

-I 5 5 0
— i 2 2 6 expanding (he determinant
along the 4th column
I 3 2

-I 5 5
- _ 1· 0 12 16
1st row and adding 3rd row
0 8 7 to She 2nd row

-I 55
12 16 I, expanding with respect to 1st column

8 7 I
>i ( 8 4 - 1 2 8 ) = - (Numerically).
0 3

Ex. 2. I f the volume o f the tetrahedron whose vertices are


(a, I, 2), (3,0, I), (4, 3, 6) and ( 2, 3, 2 ) is 6, find the value o f ‘a'.
(Agra 1974)
Sol. The voleme V of the given tetrahedron is given by
K= I a l TL I [using (6) of §' I (C)]

3 0 1 }

4 3 t. I

2 3 2 I
-3 I I 0 , adding (—1) times 2nd
row to each of the other
3 0 I I rows

I 3 5 0

—1 3 I 0
208 Analytical Geometry 3-D

0 -2 I I , ixparding vtith respect to


the 4th column
I 3 5

-1 3 I
0 -2 --LΛ
· I ,ad d in g 3rd column to
the 1st column and
6 -1 2 5 ( - 3 ) times 3rd column
to the 2nd
0 0 I
0 -2 -2
6 -1 2 It expanding w.r.t., 3rd row

- £ { - 1 2 (a - 2)+6 2} —1 2 α + .:6 )-(-2 β + 6 )

But we arc given that the volume V—6.


Λ —2o+6«=6 or a—0.
Ex. 3. Prove that the volume o f the t e t r a h e d r o n formed by the
planes m y+nz=0, nz+lx --0, ix+ m y·*0 and b:+my+nz=*p is
I p*l(lmn).
Hence deduce the area of the triangle formed on the plane
lx+my+ nz -. p . (Agra 1973, 79; Garhwal 78; Rajasthan 7/)
Sol. The equations of the four planes are
m y+ nz= 0 ...(I*, nz+lx-= 0 . (2)
/a + m y Ci ...(?), lx+ m y+ nz -p. . (4)
The planes ( I ), (2) and (3) pass through the origin, and so
the point of intersection of the planes tU, (2) and (3) is O (0, 0, 0).
Now to get the coordinates of one other vertex of the tetrahe­
dron we solve the equations (I), (2) and (4).
Adding (I) and (2), we have
/x+m>>+2wz=0. ...(5l
Subtracting (4) from (i),
tz = —p or r<= —pin.
Putting this value of z in (I) and (2), we get
X=PlI, y=pjm .
Hence the planes (I), (2) and (4) intersect at
A (pllf pl·** —pi*)·
Similarly the other Jivo vertices or the.tetrahedron are
B ( p / l t —pirn, phi), C (—p l l , p m, p n .
Λ The volume V c·. the ictr;*v. \ ^ O BC *s gsvcu by
Volume o f Tetrahedron 209

P1I plm pin

pll —plm pin

-pll plm p/n


IUsiag (5) of § I (B)]
I I -I
6lmn
I -I I

-I I I
0 0 -I
6Imn
2 0 I ,

0 2 I
adding 3rd
-P z 2' 0
6lmn 0 ■ 2 , expanding al<
= J= V = 2p*_
(numerically). .(6 )
6lmn ilmn
Proved.
To dedoce the area of the triangle. Let the area of the triangle
formed on the plane (4) be Δ· Also let the length of the per­
pendicular from the opposite vertex O to the plane (4) be denoted
by rf. so that

’ ...(7)
Also the volume V of the tetrahedron is given by
Κ = Μ .Δ ·
_3V 3.2p* -\/(/2+/w*-fn2)
Λ “ d 3lmn' p
[Putting the values from (6) and (7)]
2p2y/ (I2+ m2+ n*}/( lmn).
Ex. 4. A9B 9C are three fixed points and a variable point P
moves so that the volume of the tetrahedron PABC is constant. Find
the locus of P and show that i: is a plane parallel to the plane ABC.
Sol. Let the co-ordinates of the fixed points Ay B9 and C be
taken as (a90, 0), (0, b, CMand (0, 0, c) respectively. Also let the
210 Analytical Geometry 3-0

variable point P be (Jitl, y it Z1). Now we are given that the volume
of the tetrahedron / yXBCaconstant, say k.
. I a O O I /=Jfc
*· 6
O b O I
O O c l
Xi yt zt I

0 I . J -X 1 0 0 I
I *
0 C I b 0 I

yt Zl . I I 0 C I

or a {b (C-Z1)-J-I .(0—CV1) ) - X1 {1 .(be—0)}=6A:


or abc—abzt—acyt—bcxt **6k
or bcxiArcayf&abZi^iabc—6k).
Dividing by abc, we get
x tla+ yi/b+ ztlctsa(abc—6k^l(abc).
:. Locus of P (xt, y t, zt) is
xla+ylb+z/c*=*(abc—6k)l(abc)
or jc/e+y/b+z/c=constant. ...(I)
The equation (I) is the required locus of the point P and is a
plane parallel to the plane ABC whose equation is
Λ/α+^/6+ z /c= I.

Ex. 5. A, B, C are (0 ,1 ,2 ), (3 ,0 ,1 ), (2,3, 2). Find the


locus of P if the volume o f the tetrahedron PABC is 6.
Sol. Let the point P be taken as (X1, yu zt). The volume
V of the tetrahedron PABC is given by
0 I 2 I
3 0 I 'l
2 3 2 I

Xt yt Zi I
[Using (6) of § I (C)J
Volume o f Tetrahedron 211

0 I 2 I

3 -I -I 0

2 2 0 0

X\ T i - I zi—2 0
[V F = 6, as given. Also adding (—1) times 1st row to each
of the other rows]

or 36= -J 2 2 0 ,

: Xi T i-1 Zj —2
expanding w.r.t. 4th column
or 36 3 -4 -I

2 0 0 ,

—Χ ι+ Λ -1 «1—2
adding (—1) times 1st column to the 2nd
or 36=2 -4 -I

—*i+Ti —I «1—2
expanding w.r.t. 2nd row
or 1 8 = - 4 (zx—2) + 1 .( - X i + ^ , - 1 )
or 18= —4zi + 8 —Xi+y{—I
or Xi —^ i+ 4 z i+ 11=0.
The locus of P is the plane x —_>’+ 4 z + ll = 0 .
Ex. 6. A variable plane makes with the co-ordinate planes a
tetrahedron of constant volume 64k*. Find
(I) the locus of the centroid of the tetrahedron.
(Meerut 1977; Indore 76)
and (//) the locus o f the foot o f the perpendicular from the origin to
the plane.
Sol. Let the equation of the variable plane be
x/a + ylb + z/c= l. ...(I)
The equations of the coordinate planes are x = 0 , ^ = 0 , z= 0 .
Solving the above four equations taking three at a time, the
vertices of the tetrahedron are given by
<9(0, 0, 0); A(a, 0, 0); 5(0, b. 0); C(0, 0, c).
212 Analytical Geometry 3 D

The volume V of the tetrahedron OABC is given by


r= * a 0 0 (See (3) of § I (B)].

0 b 0

0 0 C
64&3=& abc (V F = 64/c8 as given]
or e*c=384frs. ...(2)
(i) Now let the co-ordinates of the centroid of the tetrahe­
dron OABC be (α, β , y), so that we have
O+a+O+O 0 04-0 + 6 + 0 · 0 + 0 + 0 + c
« - — 4-------. β -------- 4------ , y*=— 4— -
or a —4a, b —4β9 c= 4y.
Substituting these values of a9 b, c iu (2), v/e get
(4α) (4β) (4y)=384 fc3 or αβγ*=>6k*.
A the locus cf the centroid (a, β, y) is xyz*=6k*.
(ii) Let (.X1, y u 2,) be the co-ordinates of the foot of the per­
pendicular from the origin O to the plane (I).
Thedirectionratiosofthis perpendicular are X1- 0, y \ ~ Ot
2i—0 t.e. x u y u Z1. Also the d.r.’s of the normal to the plane (I)
are I /a, 1/6, I/c. The two being parallel, we have

Ma I Ib I Ic .(3)
The foot of the perpendicular (xu yt9 Zi) lies on the plane
(I) and so we have xilo+ yilb+ 2ilc**l. ...(4)
From (3), we have
ill-* a - ' *va+ v + * i a ^ ^ι1+ ^ !1+ ^ 8
Xilo y jb ZlIc (*i/a)+(;Vi/6)+(Zi/c) 1
[using (4)1
or ^»Xi, + y ,a+ 2 ii .
Λ α= (Xi*+.Pif +Z18Vx1, b ^ ix x '+ y t'+ z M y u
c—(xit + yit + zi2)/zl.
Substituting these values of a, b and c in (2), we have
(Xi2+ J i2+ Zit )* = 384 X1 y x Z1 k 3.
Λ The locus of the foot of the perpendicular Ue. the locus
of (Xu yu Zi) is given by
(xa+ y 2+ 22)3= 384 xyz k3.
Ex. 7. The lengths of two opposite edges of a tetrahedron are
a, b, their shortest dista ice is equal to d and the angle between them
ts Θ. - Show that the volume of the tetrahedron is J abd sin Θ.
Volume of Tetrahedron 213

Sol. Let ABCD be a tetrahedron. Let the vertex A be taken


as origin and let A B = a. Let the direction cosines of the line AB
be /, m, n, so that equations to AB are
x —0 y —0 z—0
Ύ a ΪΓ ~ ΊΓ ' ...(I)
Since the point B is on the line AB and is at a distance V
from A(0, 0, 0), hence the co-ordinates of B are (to, ma, na).
The edge opposite to AB is CD. Let the co-ordinates of C be
(α, β , γ) and CD=*b. Let the d.c.’s of the line CD be Λ, μ , v, so
that the equations to CD are

λ μ ' * ■ .(2 )
Sioce the point D is on the line CD [i.e. (2)] and is at a dis­
tance 'b' from C (α, β, γ), hence the co-ordinates of D are
(A6+ «. μb+β, vfr+y).'
Now Θ is the angle between the lines AB and CD i.e. between
the lines (I) and (2), so that
sin θ=*\/ ημ)1]. ...(3)
Also d —the S.D. betweeo the lines (I) and (2)
«—0 /3—0 y -0 («IV—«μ)1]
I m η
A V

« β Y 4-sin 0 [using (3)]


I m η
A β V

α β Y
I m η • ...(4)
λ V

fbe volume V of the t etrahedron ABCD lone vertex A being


originj is given by
al am an [see § I (B)]
α β Y
b\^0L ύμ+ β bv+ γ
214 Analytical Geometry 3-D

al am on I al am an
X β y + a β y
b\ *>μ fcv i a β y
a β y + 0
•81*
I m η
D
I

X μ V
—J abd sin Θ [using (4) and neglecting —ve sign]
Ex. 8. A point P moves so that three mutually perpendicular
lines PA, PB, PC may be drawn cutting the axes OX, OY, OZ at
A, B, C and the volume of the tetrahedron OABC is constant and
equal to k*/6 Prove that P lies on the surface
(x, +y*-\-z*)3—ik 9 xyz.
Sot. Let the co-ordinates of the moving point P be
(xi, y\> z,). Since A is a point on the x-axis (i.e. on OX), there­
fore, let the co-ordinates of ^fbe (a, 0, 0). Similarlylet the co­
ordinates of B and C be (0, b, 0) and (0, 0, c) respectively.
N ow thedirection ratio so fth e lines PA, PB and PC are
Xi- a , y t, z,; x„ yx- b , z,; and X1, ylt Z1-C respectively.
It is given that the lines PA, PB and PC are mutually perpen­
dicular, so using the formula +CiC*=0’, we have
( χ ,- .β ) χ 1+ Λ .(^ ,-6 )+ Ζ ι.ζ ,= 0
[PA and PB being perpendicular]
or Xi1+ ^ + Z i 1=UXi+ ¢^ 1 . -(1)
Similarly PB and PC; PC and PA are perpendicular, so we have
x * + y \%+z\*=byi+czi -(2)
Χι*+Λ*+Ζι*=»ΓΓι + a x i· -(3 )
The volume of the tetrahedron OABC is given to be k3/6.
a 0 0 k»
[using § I (B)]
i 0 ir o b
0 0 c
OT abc=>k*. -(4 )
Now in order to find the locus of the point P(xu y u z,) we
are to eliminate the unknown quantities a, b, c from the relations
(I), (2), (3) and (4).
VolumeofTetrahedron 215

Adding (I), (2) and (3), we get


, 3 (Xii - I - V + V ) = 2 (axx+byxA-czx). ...(5)
Subtracting 2 times (2) from (5), we have
*ι*+ν+ζι*=*2βχι. ...(¢)
Similarly (3) and (5) give
* ι· + Λ , + *ι , -2 6 Λ ...(7)
and (I) and (5) give X i* + V + V = 2 c r,. ...(8)
Multiplying (6). (7) and (8), we have
(xi*+V +Zi*)3= 8 abc x xy xzx
or (Xi*+V+Zi*)s ^8** Xi^1Zi, [using (4)].
Hence the locus of the point P(xx, z) is given by
(x*+y*+zt)a=%ki xyz. Proved.
Ex. 9. Show that the volume of a tetrahedron of which a pair
of opposite edges is formed by lengths r and r' on the straight lines
whose equations are
(x —a)ll**(y—b )/m ~ (z - c)/n;
(x —a')IT=(y —b')lm '=(z—c')/n' is
a—a' b -b ' C—i
I m n
I' m' n'
[Rajasthan 1975]
Sol. The equations of a pair of opposite edges of a. tetra­
hedron are given by the equations
(x -a ) /l= ( y -b ) lm = ( z - c ) ln ~ r .(1 )
and (x—a')/l'= (y—b')lm'=-(z—c')ln'=>rY ...(2)
Let A(a, b, c) be one vertex of the tetrahedron on the line (I),
then the other vertex B on the line (I) at a distance r from
A(a, b, c) has co-ordinates (lr+α, mr+b, nr+c).
Similarly other two vertices C and D on the line (2) are
0a b \ c') and (Pr'-+a', m 'r'+b', n’r’+c') respectively..
Hence the required volume V of the tetrahedron ABC is given
by
a b C I
Ir Ya mr+b nr-\-c I
af b' c' I
Vr1-Q t m'r'+b' n 'r'+ c ' I
216 Analytical Geometry 3-D

a b C I
Ir mr nr 0
a' b' c' I
Vr' m'r’ nY 0
adding (-*1) times 1st row to 2nd and (—1) times
3rd row to 4th
^rr' a b c I
I m n 0
a' b' c' I
I' m' n' 0
~ J rr' a—a' b -b ' C — C* 0
I m n 0
a' b' Ct I
I' m' π' 0
adding ( —1) times 3rd row to 1st
= J rr' a —a b -b ' c c' .
I m
I' m' n' I
expanding along the 4th column. Proved.
Exercises
1. Find the volume of the tetrahedron, the co-ordinates of
whose vertices are (I, 0, ·;), (0,0, I ), (0,0, 2) and (I, 2, 3).
Ans. J.
2. Find the volume of the tetrahedron formed by the planes
y + z = 0 , z + x = 0 , χ+ .κ ~ 0 , and x - r y + z = l
Hence deduce the area of the triangle formed on th ; plane
x + y + z - 1.
Ans. Volume—2/3, area-=2/3.
7
Skew Lines

§ I . The equations of two skew lines.


To show that by a proper choice of axes the equations of two
ska w lines can be given by the equations
y^=x tan a, z= c ; y - - J t tan a, z= -c .
(Calcutta 1974)
In the adjoining figure lei AB
aud A1Bt be the two given skew
(non-intersecting) lines, and let CD
of length Ic be the shortest distance
between them.
Take the axis ot z along DC
and Ot the middle point of DCt as
origin. Draw OK ard OL parallel
to AB and AtBt respectively and
take the plane KOL as the plane
z= 0. Take the internal and exter­
nal bisectors of the angle KOL as
the axes of x and y respectively. In the figure» OX and OY
represent the axes of x and y icspectively. Let the angle between
the lines OK and OL (/.e. between the given lines AB and A9B[)
be 2a.
As explained above the line OK (which is parallel to AB) is
inclined at angles a, \n - a , with x t y t z axes respectively and
therefore, the d. c.’s of OK (i.e. of AB) are cos a, cos (Jw - a ) ,
cos ^xr / e, cos a, sin a, 0.
Again the line OL (which is parallel to AfBf) is inclined at
angles —α, + a. \π with the co-ordinate axes respectively and,
therefore, the d. c.’s of OL \i.e. of A9Bt) are cos ( —a), cos ($π+α),
cos $7T or cos a, —sin a, 0.
Now it is required to find the equations of vhe given skew
lines AB and A'B\
2i8 Analytical Geometry 3-D

Since CD = 2cand O is the middle point of CD9 therefore


OC=OD =c. Hence the co-ordinates of the points C and D
(on the z-axis) are (0, 0, c) and (0, 0, —c) respectively.
Thus we sec that the given line AB has d.c.’s cos a, sin a, 0
and passes through the point C (0, 0, c) and hence its equations
are
x —0 —0 _ z —c
or y = x tan a, z=c.
cos a sin a 0 (I)
Also, the line AfBr has d.c.’s cos a, —sin a, 0 and passes
through the point D (0, 0, - c) and hence its equations are
x -0 y —0 zV c
■ - — : /r OriV = - X t a n a 1Z- • —c.
cos a —sin a 0
The equations (I) and (2) are the required equations of the
given skew lines AB and A'B'.
If we put tarta=m , the equations (I) and (2) become
y= m x, z= c and y= —m x9 z = —c. .(3)
The equations (3) may be written as
x y z—c .x x z+ c
and — —
Im 0 I -m 0 .(4)
SOLVED EXAMPLES
Ex. I. Prove that the locus of a vat table line which interjects
the three given lines y = Oix9 z=c; y ^ —mx, z = —c; y = z t m x = —c
is the surface y 8—maxa= Z 2- C 2.
(Agra 1978; Jodhpor 76; Meerut 74, 89)
Sol The equations of the given lines are
y= m x, z=c\ ...(I)
y-=*—mx9 2«=—c; ...(2)
and y = z , m x = —c. * ...(3)
We know that any line intersecting the lines (I) and (2) is
given by two planes, one through each line.
The equation of any plane through the line (I) is
(y -m x )-M (Z-C)=O. ...(4)
Also9 the equation of any plane through the line (2) is
O’+ m x l+ μ (z+ c)= 0 . ...(5)
The planes (4) and (5) intersect in a line and if this line
meets the line (3)* then putting mx=»—c and z= y in (4) and (S),
we have
(y+ c)+ λ (j;-c )= 0 and (y + c )= 0
»— and μ y - c
or
y - c y+ c
Skew Lines 219

Multiplying these relations, we get Αμ=1. ...(6)


The required locus is obtained by eliminating Aand μ between
(4), (5) and (6), and so eliminating A and μ between these equa­
tions, the required locus is given by
_ ( y - mx)1 χ Γ j + mxl

or
[ Z+ C J [
j 8—m2x*= 2*—c*.
Z -C J
Hence proved.
Ex. 2. Find the surface generated by the lines which intersect
the lines j= w x , z =c; y= —mxt z = —c and x-axis.
(Meerut 1973; Garhwal 81)
Sol. As in Ex. I above a line intersecting the first two lines
namely y ~ m x 9 z - c\ y ~ —mx, z=*—c is given by the planes
( j - wix)+A (z -c )= 0 , ...(I)
and (j-fm x)+ /* (z-i-c)=0. ...(2)
The planes (I) and (2) intersect in a line and if this line meets
the x-axis i e. j = 0 , z - 0, then putting j = 0 = z in (I) and (2), we
have
(0—mx) + A (O-C)=O and (0 + mx)-f μ (0+c) = 0
or A =—-mx/c and μ=» —mx/c.
From these two relations, we get A=μ. -. (3)
Therequiredlocusis given by eliminating A and μ between
(I), (2) and (3) and is given by
y —m x y jT t n x
z —c z+ c
or (j+ m x ) ( z - c ) = ( j —mx) (z—c) or cy=mzx.
Ex. 3. A variable line intersects the x-axis and the curve x = j ,
y2«=cz and is parallel to the plane x = 0 . Prove that it generates the
paraboloid xy cz
(Meerut 1972, 76, 89S; Kanpur 77; Kurukshetra 74)
Sol. Theequation of any pfane through x-axis i.e. j = 0 = z is
J e=sAz. ..( I )
The equation of any plane parallel to the plane x = 0 is
χ® μ. ...(2)
The planes(I) and (2) intersect in a line whichintersects the
x-axis and is parallel to the plane x = 0 . Ifthis line meets the
curve x = j , j 2=cz, we have by puttiug x = j in (2),
y= l·· -(3 )
From (I), A = - = —— [V c z~ y2]
z cz y*
220 ■Analytical Geometry i-D

or A=c/j or A=c/^, using (3)


or λμ—c. -(4)
The required locus is given by eliminating λ and μ between
(I), (2) and (4>, and is
(y/z) x = c or x y —cz.
Ex. 4. Show that the straight lines which intersect the three
lines y - z-----1, x -0 ; z - x — 1, j = 0 and x —y = I, z= 0 lie on the
surface whose equation is
X2-Fj2-I-Z2- 2jz - 2 z x - 2 x j —1 = 0 . (Kanpur 1981)
Sol. The equations of the given lines are
y —z - l - --0, jc “ 0 -(1)
"z—x —I .-0, j = 0 ...(2)
and x - y — l = o, 2=0. -.(3)
The equations of any planes through the lines (I) and (2) are
(y—z - 1)-|-Αχ=^0 ...(4)
ann (z—x —l)-f μ ν= 0 . (.5)
The planes (4) and (5) intersect in a line an-i if this line is to
meet the line (3) then we shall eliminate x, y, z between the
equations (3j, (4) and (5). From (3), we have z= 0 , x = y + l.
Putting these values in the equations (4) and (5), we have
(y.—0—1)+A (J-Fl)=O and i0 —y — I —I) + = 0
or >*—I —AvTA=-O and —y —2 -j- μ^·=0
or y (I -FA)= I —A and y ( —1 + μ )= 2
l-λ . 2
or y —-J--T and J = — -
I +A μ —I
Equating the two values of j , we get
Γ ^ λ ^ ^ ΙΓ ι ΟΓ Μ -(-μ Α + Α = 2 + 2 Α
or μλ-ί-λ - μ + 3 - 0 . (6)
The required locus is obtained by eliminating A and μ between
(4), (5) and 16»; and hence is given by
j - z - 1 j - Z - 1 , Z - X - 1 , ,.
- - - - - -X ---------------------y3==0
y x y
or (z—x —I) ( j —z—I) - j ; j - z - - l ) + x (z—a - l ) + 3 x j = 0 ^
or j z —z2—z—x j - xz-f a - j -Fz-F I—j *+ jz
-i-y+ xz- x- - x i 3a j = 0
or x*+ j*+ z*—2jz - 2zx - 2xj - i = <). Proved.
Ex. 5. Prove that the lorus of a line which meets two lines
y**±m x, z = ± c and the circle x ‘ -(-Js=Usj 2 = 0 u
c*m* (cy—m z x f + c t (yz—c m x f —v m 1 ( Z 1 - C i ) .
Skenf Unes 221

Sol. The equations of the given lines are


y —mx=*09 z —c=*0 ...(I)
and y + m x = 0 , z+c=*0. ..-(2 )
Also the equations of the given circle are
x*+y*x=*a*9 Z ^ 0 . ’ . (3)
The equations of any planes through the lines (I) and ( 2 ) are
(y —mx)-\-X (z—c)*= 0 ...(4)
,and (y + nix)+ μ (z^c)=*0. ...(5)
The planes (4) and.(5) intersect in a line and if this line meets
the circle *3) then we are to eliminate x, y, z between the equa­
tions (3), (4) and (5).
From (3), we have z ~ 0 . Putting z —0 in f4) aod (5).s we get
(y—mxj —cA«0, ( γ + η ι χ ) + μ θ 0.
C fλ i-μ) C (Λ—μ)
Solving for x and »\ wc have x
2m z
Putting these values o f.t and y in X2+ y 2= Q1n the X, 2
eliminant of (3), (4) and (5) is given by
c 2 (λ-f μ,1 c* (A- # * ) * at
c*
4m*
or Ct (λ + μ)* + c*m* (λ—μ)2- 4a*ma. . ( 6)
The required locus is obtained by eliminating Λ and μ between
(4), (5), and (6 ), and hence is given by
C* + c*m* l τ L ·I!!^ + y Λ Ξ ? y ^ 4 a 'm *
I z —cc z \f e JI Ii ' z —c z± c J
or c%{(y—mx) (z+ c )+ (z -c ) (y+ m x ) } 2
+ c2m* {(y+ m x)(z—c) (y —m x)(z ^c) ) 2 ~4a*m2 (z* CiP
or c%(yz—cmx)*+c*m* (cy mzx)*--n*w* (z* -C 2 J3.
Ex. 6 Prove that the surface generated by a straight line which
intersects the lines x + y ~ 0 z; x y —zf x + y- 2a and the para­
bola x2= 2 az, y ~ 0 is x*-. v2 -2az (Kanpur 1983)
Sol. The equations of the given lines are
x f y ~ 0 f z- 0 ..(I)
and x —y —z=*0 , x + y - l.i = 0 . ...·2)
Also the equations of the given parabola are
Xt =^laz. y=--0. ...(3)
The equations oi any planes through the lines (I) and (2) are
x-UH-Az-O (4)
and ( x —y - ζ)+ μ (jc+ v — 2e)-=0. ...*5)
These planes intersect in a line end if this line meets the
parabola (3) then we are to eliminate x, z between (3),(4)
and (5).
222 Analytical Geometry 3-D

We have y= O from (3). Putting >>=0 in (4) and (5), we get


* 4 - Az= 0 and x —ζ + μ (x —2 a ) = 0 .
Solving for x and z, we have
_ 2aμ λ _ — 2αμ
I - f /\ -f · μ λ I -I- A - I- μ λ

Putting these values of x and z in the equation x**=2az


belonging to (3), the λ, y, z eliminant of (3), (4) and (5) is given
. 4α 2μ 2 λ 2 —4Q i μ
^ (l-i-A + /iA )2 ( 1 - | - λ + /χλ;

or μλ*= —( \ + λ + μ λ ^
ΟΓ μλ*-\-μλ + 1 +A = Ot o r μ λ (A-I- I)-I- I .(A4 - 1 ) = 0

or (μλ+ l ) (A4-l)“ 0 or ZiA-I- I=O (as A # - 1 ] . (6 )


The required locus is obtained by eliminating Aand μ between
(4), (5) and ( 6 ) and hence is given by

or (x—y —z ) ( x + y ) + z ( x + y —2o)=‘0
or x»—y*c=2az.
Ex. 7. Find .the locus o f the variable line which cuts the three
linesy= b, z=*—c ; z*^c, x ——a ; x=*a,.y— —b.
[Garhwal 1978 (S); Kanpur 73]
Sol. The equations of the given lines are
y—6= 0, z+ c= 0 . (1), Z - C = O t x+a~-0 . ( 2 )
and x —a=Q, y + b = 0 . .. .(3)
The equations of any planes through the lines (I) and (2) are
( y - b ) - X (z+ c)= 0 ...(4)
and ( ζ - ο ) - μ (x+ a)= 0. ...(5)
The planes (4) and (3) intersect in a line and if this line meets
the line (3) then we are to eliminate x, y, z between the equations
(3), (4) and (5).
From (3), x= a, y = —b. Putting these in (4) and (5), we get
—2b—λ (r+ c ) = 0 and ζ —ο—2αμ=0
Ot z«=(—2ί>/λ)—c and 2 = c+2o/i.
Equating the two values of z, the x, y, z eliminant of (3), (4)
and (5) is given by
(—2 ί>/λ)—c=*c+2au or au+ b/\+ c= 0. ...( 6 )
The required locus is obtained by putting the values of λ and
μ from (4) and (5) in (6 ) and is
Skew Lines 223

a j z - c ) b jz + p 0
x+ a ~ y —b
or a ( y - b ) (z -c ) + b (x+a) (z+c)+c (x+a) ( y - f t ) = 0
or ayz+bzx+cxy+abc·=* 0 .
Ex. 8 . How many lines can be drawn from a point to intersect
two non-coplanar lines neither of which passes through the point ?
Find the equations o f the lines or line which can be drawn from
the point (2, —1 , 3) to intersect the lines
<x—l)/2«=(>.—2 )/3 = (^-3 )/4 ;
(*—4)/4=y/5=(z+3)/3.
Sol. Let (α, βt y) be the co-ordinates of a given point. Let
the equations of the two non-coplanar lines be
y —m x= 0, z —c = 0 ...(I)
and y+mx-^-0, z + c = 0 . ...( 2 )
The equations of any planes through the lines (I) and (2) are
( y - m x ) —X ( z - c ) = 0 ...(3)
and (y+ m x)—p (z+ c)= 0 ...(4)
Since the planes (3) and (4) pass through the point (α, β, y),
so we have
(β - m a)~ Λ(y—c) = 0 and (β + .via) - μ (y + c ) = 0
or A= ()9—m a)/(y -c) and μ = (β+Μ<χ)/(γ+€).
Putting the values of λ and /· in (3) and (4), the equations of
the line through the given point (α, β, γ) and intersecting the
given lines (I) and ( 2 ) are given by
(y~ m x) ( y - c ) - ( z - c ) (j8 - / n a ) = 0 , I
and (y+ m x) (y + c ) - ( z + c) (β + Μ<χ.)τ=0. J
Thus only one line can be drawn from a point to intersect
two non-coplanar lines neither or which passes through the point.
Second part. The equations of the given lines are
x - \ y —2 z - 3 x - 4 y z+ 3
2~ = 3 ; ”4 “ 5.~ I
These lines can be reduced to general form as
i x —2y+ U-O= 2 jc—z + 1 .(1)
and 5x—4y—2 0 = 0 = 3 jc—42—24 ...(2)
The equations of any planes through the lines (I) and (2 ) are
f i x —2y+ \ )+X (2 Xr - z + 1 ) = 0 ...(3)
and (5.x —4y—20)+ μ, (3x—4z—24) = 0. ...(4)
The planes (3) and (4) pass through the point (2, —1 , 3), so
we have
274 . A n a lytica l G eo m etry 3 -D

( 6 .+ 2 4 η + λ ( 4 - 3 + 1)--¾ O or A = — 9/2 ;
(10+4- 20) + μ ( 6 - 1 2 - 2 1)=() or /*= -1 /5 .
Putting the values of A and μ in (3) and (4), the equations of
the line through the point ( 2 , —I, .'*) and intersecting the given
lines (I) rnd.( 2 ) arc given by
12x + 4v Oz+ 7 - 0, 11jc - IOy+ 2z— 28 = 0.
Ex. 9. A line of constant length has its extremities on two
fixed straight Urns. Prove that the locus of its middle point is an
ellipse whose axe* are equally inclined to the lints.
Sol. Let the equations of the two fixed straight lioesbe
given by
Jt/! = y /m = (z—c)/0= rT ·· Π)
and , ' xf\ =y/( ~ m )= (z+ c ) / 0 =^r2. -
The co-ordinates of any two points on (I) and (2 ) are
P (ru mrJ$ c) and Q (r2. —mr^ —c) respectively.
Let (α. β, γ) be the middle point of PQ9 so that we have
a-= J (γ! + γ2) , 0 -~■:* m Oil - r B), y = i (c~c) = 0. ...(3)
Also the length of PQ is given to be const ant, say 2d.
Then 2d=· PQ ·-- distance between P ami Q
or 4d*--“ (r 1 —r2 )2 + m2 (r, f r ,) a-f (c + c)*. -(4 )
Let locus of (α, β, γ) is obtained by eliminating the parameters
Tj and rt between (3) and (4). So putting the values o** Γι —r2and
ri + r 2 from (3) in (4), we get
4d2 (2film)2+ m2 (2a)*+(2c>2, y - 0
or m2(x2+fiilm2+c2=^d2t y - 0 .
Hence the locus of the middle point (a, β, γ) is given by
JWfJt*+y*/m*=rf* —cf, z -0. ...(5)
The equations (5) are the equations of an ellipse in the
jry-plane, whose axes lie along the axr;s of Jt and y. But we know
(see § I) that the axes of x and y bisect the angles between the
lines (I) and (2) Hence the axes of the ellipse (in the present case
axes of Jt and y ) are equally inclined to the gi ven lines.
Ex. 10. A point moves <o that the line joining the feet o f the
perpendiculars from it to twy given straight lines subtends a right
angle at the middle point of their shortest distance (S,D .). Prove
that its locus is a hyperbolic cylinder.
Sol. Let AB and AtB* be the two given linies. Let the
middle point O of the S.D between these lines be taken as origin.
ScewLm a 225

The equations of the lines AB and λ 'ΒΤ are respectively given by


r**sn m
and x/l=jr'( - ι»)=(ζ+γ)/ο—f*. —(2)*
Let P (a, P , y) be the moving point and M , N be the feet of
perpendiculars from P to the lines (I) find (2) so that we have
M ir,. mr,, c) and N (γρ <Wn c)-
The d r.’s of PM are a r„ 0-m r„ γ - c and the d.r.’sof
P N are a -r* β J Hirr. γ + e.
Now PM is perpendicular to (I) |i>. AB\ and PN is perpendi-
cohi to (2) [ije. A tB0Imtherefore, we have
I.(* - rt)+m i β IMr1) + 0.(/-0= °
or *!«=»(*+m/f)?( I+ mi*) ---(3)
and !.(*—ra) m (£+011^)+0.(7 +*)=°
or rf=(* μι/?)/(1+μι*). - (4)
According to the problem M N subtends a right angle at the
middle point O Le. O M is perpendicular to ONmso that we have
τχ*±+ μτχ ( Mir2)+c ( — r)=0 or rfrt (I —mi*)—c*=0 ...(5)
Eliminating r, and ra between (3), (4) and (5), we have

*—
or («*- m*/P)—e* (I +n^Jr/fl m*).
Λ The locus of P («, β , γ ) is
x*+ upy*^e* ( I J m *}*Tl —«*)-
This is clearly the equation of a hyperbolic cylinder.
Ex. 11. fa the above Ex. 10. i f M N is equal to the Jistaaee o f
Pfram the Gtiiin O. then prate that P lies on the surface
(I in1)*(**+.»*>=n+n^J*(4e*~ z*).
SnL We have evaluated in Ex. 10 above that
M is the point (rlt mr,. c) ; N is the point (rt, —nw*, - c)
T1= ( « 4 mp)/(l +iff*). r ,= ( i mfi)l { I +m*)
and the moving point P is fa, β, y ).
Now OP* - ** f I M N f (given). (I)
But ( M N i*=Ir1- r,)*+!·5 ^i+r.'H if+f)*
i+m/J * mfi V . - I m f i a—mfi
( T T rf-τ + rf) + r f I i T r f + i+ rf-
4-4c*.
T
Λ (l + #fi*|*(JlfJV)i =4iii^i*4i»i*.4a*+4c=(l lm*)1
or (I +Ifii/ («*+/!*+ 7 *)=4m* <a*ii-A*)+4cI (14-m*)*, using (I)
or (I +OttIt (*r-i-pt)=4mt (**+p*)=4e* (1+m*)*-?* (1+m*)*
226 Analytical Geometry 3-D
or {(I+m*)*-4iit*} (a*+/»*)=(4c*-y*) (I+m*J*
or
The locus of P (a, β, γ) is
(I —m*)* (**+J* )= (i+m *)* (4c*—z*). Prorei.
Ex. 12. Find the surface generated by a straight line which
meets two line y —mx, z—c\ y = —mx, z= —c at the same angle.
(Kanpor 1982)
Sol. The equations of the given lines may be written as
xfl=y!m ( z - c)/0=r, (say) ...(I)
and x/1 =y/( —m)=(z+c)IQ=rt (say).. ...(2)
Let the moving line meet the given lines (I) and (2) in the
points C (rb Iitr1, c) and D (rt, -Oirs, —c) respectively. The
d.r.’s of CD are r , - r*, m (γ,+ γ,), c+c. The equations to CD
are
X-T1 _ y —mr, z —c
r%—rM~ m (A + r,)” 2c - .(3)
Let the line (3) make angles a and β with the lines (I) and
(2) respectively, so that we have
COS H =
L (r,-r,)+i»i.m (r,+ r,)+ 0.2c_______
V(1 +m , + 0)V ((r1—r,)*+»n* Ir1+ Γ ,Ρ + φ )*} -.(4)
Lfti—rt)—m.m ftt+rt)+0.2c
and cos /1=-
V(»+"«, + 0 ) V t t ^ - fs), + « * fti+'t)*+(2c)*} -(5 )
But according to the given problem, cos a = + cos β. Hence
from (4) and (S), we get
Γ ,-Γ ,+m* ( ^ + ^ = ± [ ( ^ - ^ —111* (r,+ r,)],
the denominators beine same have been cancelled.
Taking + v e sign, we get
- 2m* (C1-Hri)=O or rt ——rx
and taking —ye sign, we get
2 (»i—r*)=0 or T1=C,.
Now when r , = —r1( the line (3) becomes
x - r , y —mrx z - c
2 T1 - 0 2c
X - T 1
Le. y ~ C i
Trx 2c

_ Z y_
Le. --1 = and r ,= —
'l . C m
X Z
Le. — = — and #
'i c m
Skew Lines in

Eliminating ru we get
xm __z_
— = — or mcx=yz. ( 6)
c
Again, when T2 =T1, the line (3) becomes
X -T 1 y -H ir1 z —c
0 ” I m r 1 ~~ 2c
i.e. x T1=O and y / m r ^ z / c ; i . e .·, T1=X and c y — m z r i .
Eliminating T1, we get cy=mzx. (7)
Multiplying (6 ) and (7) the required surface is given by
(mcx—yz) (cy - mzx)= 0.
Ex. 13. Show that the locus o f lines which meet the lines
x + a ___ y_____ z__ . x - a y __ z
0 ~~sin oc“~ —cos a’ 0 ~~sin a cos a
at the same angle is
N (xy cos %—az sin a) (zx sin cl—ay cos a)« 0 .
Sol. The equations of the given lines are
(x+ a)/ 0 = ^ s in a= z/( —cos a ) = ^ ...(D
and (x —a)f0=3yls\n a=z/cos a = r2.
Let the moving line meet the given lines (I) and (2 ) in the
points C ( - a , T1 sin a, - T 1 cos a) and D (a, r 2 sin a, rt cos a)
respectively. The d.r.’s of the line CJDare a+ a9 (r* - T1) sin a,
(r*+ ^) cos a. The equations to CD are
x+ a y ~ r x sin « . Z-Ir1Cosq
2 a “ (r 2 -T1) sin a ""(T2 -Hr1) cos a
Let the line (3) r.'ake angles 0 and φ with the lines (I) and
(2 ) respectively, so that we have
cos sin «.sin ot+ ira+ r^ cos « .(-co s a)
\/{( 2 a)2+ (T2 -T 1) 2 sin* a + (^ + T 1) 2 cos2 a}
V^{0 +sin 2 a+ cos 2 a}
...(4)
. __2 a. 0 + ( r t —T1) sin «.sin oc+(r2 +Ti) cos a. cos a
^""V U 2 f l ) 2 + ^ 2 - T 1) 2 sin2 α-Κ τ,+ Ti) 2 cos* a}
V{0+sin2 a+cos2 a} ...(5)
But according to the given problem, cos B= + cos φ. Hence
from (4) and (5), we get
(T 2 -Tj) sin2 OC--(T2 - T 1) cos2 a
= K i r 2 - T 1) sin2 « + (τ* + ^ ) cos2 a}.
Taking +ve sign, we get
2 (r»+^) cos2 a = 0 or τ, +T j —O or T1- - T 1
228 Analytical Geometry 3-D

and taking —ve sign, we get


2 (rt—T1) sin2 a = 0 or T2=T1.
Now when T2= - T 1, the line (3) becomes
x+ a y - r x sin a Z -I-T 1 cos a
2a —2rx sin a ““ 0
— +1 = ----- K----- hi and z + rx cos a= 0
a T1 sin a 1
*·*·» rxx sin a = —ay and T1= - Z / cos «.
Eliminating T1, we get
zx sin a —ay cos a = 0. ...(6)
Again when T2=T1, the line (3) becomes
x+ a V - T 1 sin a__z+Tt cos a
2a = 0 I r 1 cos a
or + 1 -y= rx sin a
τ+·· T1 cos a
or T1Jc cos a= az, Tt = v/sin a.
Eliminating T1, we get jcy cos a - az sin a= 0. (7)
Multiplying (6) and (7), the required locus is given by
(jcy cos α—az sin a) (zx sin a —ay cos a)=0. Proved.
Ex. 14. (a) Find the locus o f a point which moves so that the
ratio o f its distances from two given lines is constant.
Sol. Let P (jct, y l9 zx) be the moving point and the equa­
tions of the given lines be
x/\= y /m = (z c)/0 .. (I)
and , jc/ I ==^/(-m) = (z -|-c)/0. ...(2)
Let px and p2 be the lengths of the perpendiculars from P to
the lines (I) and (2) respectively, so that we have
I
Pit = I + m2 I m
Xi y i I y i zi c Z i-c X1 :'
Il m 0 0 I
2 __ I
or Pi {(mxi - y x)2+ m 2 (zx- c ) 2+ (zx- c)2}
I-HOl2 ,(3 )
Replacing m by - m and c by —c and px by p2, [or proceeding
as above Por^1], we have
A *8= I {(- I i I X 1 - y y f + i n ' - ( S i - h c ) 2 + ( z x + c ) 2}

= i + m* { ^ i +^ i )2+ ^ !+ ^ )2 O"2+ ·)}· (4)


According to the given problem Ρι/ρ2=λ (say)» where Λ is a
constant.
Skew Lines 229

Λ Pi=A/?, or Pi2=A2P,*
or [(w *i-> -i)2+ (m 2+ 1) (Z1-C )2J=A2 {(/η χι+ ρ ,)2
H-C"**+1) (Zi+c)*}
[using (3) and (4)].
The required locus of P (Xi., Z1) is given by
(mx—y)2+(/n2+ l ) ( z - c ) * = A * { ( m x 4 - y ) 2+ ( / H 2+ 1) (z+ c2)}
-(5 )
Ex. 14 (b). Find the locus o f a point which is equidistant from
two given lines.
Sol. Proceeding exactly as in Ex. 14 (a) above and putting
A=I [since here Pi=p,], the equation (3) of the required locus is
given by
mxy+c ( l + / n 2) z = 0 .
Ex. 15. P, P' are two variable points on two given non-inter-
secting lines and PP' is of constant length 2k. Find the surface
generated by PP’.
Sol. Let the equations of the given lines be
Jl

tr
O

(I)
N

Il
II
I

and x H = yl(-m )= (z+ c)IO = rt. ...(2)


The co>ordinates of the variable points P and P' on (I) and
(2) are (T1, mru c) and (r,, —mrt, —c) respectively.
It is given that PP’= constant, say 2A.
Λ (PP')2=4A2
or ( r i- r ,) 2+m 2 (ri-t-r„)2+4c2=4A2. ...(3)
Also d.r.’s of PP' are T1-T2, m (ti+ t,), 2c.
Λ The equations to PP' are
x —Ti y —/MT1 z —c
Ti-T, ~ m (Ti-t-T,)- 2c -(4 )
The locus of PP' is obtained by eliminating T 1 and r, between
(3) and (4). .
From (4), we have
Jt- T1 . z —c J ^ -Wr1 2 c
T1—t, “ 2c m (TiH-T8) 2c
or 2 c x - 2 ctx = T ! ( Z - C ) -T2 (z -c ) ;
and 2cy—2mcr1—mri (z —c)+mrt (z—c)
or Ti (z+ c)—T2 (z—c)=2cx •••(5)
and /MT1 (z+ c) + /M T , (z—c)=2cy. -(6 )
Solving ( 5 ) and (6) for T1 and t „ we get
Tj=c (y+mx)f{m (z+c)}, t2 = c (y-m x)l{m (z -c )} . -(7 )
230 Analytical Geometry 3-D

Jy ± mx x y —mx I 2c (y z-c m x )
I z+ c 1 Z - - C J m (z2—c2)
C fy+ m x y - mx \ 2c imzx cy)
and rx r2——
m ( z+ c z - c [ ~~ m ( Z 2 - C 2)
Substituting the values of rx+ r2 and ^1- T 2 in (3), the required
locus is given by
4c2 (m z x - c y )2 . 4c2 (yz mcx)2
+4c2=4A2
m2 (z2- c 2)2 ^ m2 {z2 - C 2) 2
or . c2 (mzx—cy)2+ c2m2 (yz- mcx)2= m 2 (A2--Ca) (z2- c 2)2.
Ex. 16. AP and AtPf are two given lines, A and Af being fixed
and P and P' variable points on them such that AP.A’P' is constant.
Find the surface generated by PPf.
Sol. Let the equations of the lines AP and AfPf be respecti­
vely given by
x !\= y /m = (z-c)IO = r1 ...(I)
and x/\ =y/( —m )= (z+ c)/0 = r2. :..(2)
The co-ordinates of the fixed points A and Af on the lines (I)
and (2) are (0, 0, c) and (0, 0, - c) respectively.
Let the variable points on the lines (I) and (2) be P(rl9 mrv c)
and Pf (r2, mr2> —c) respectively.
According to the problem AP. AfPf=CowstdLnti say k.
:. V i i '! --0)2+ (/Wr1-O )a+ (c - c ) 2)V { (r2—0 )2 + ( - m r t - C ) 2
+(■-c+ c)2}= k
or V i r l2+ m zr12) .^ ( r 22+ m 2r22) = k
or rvr2 (I + m2)= k or ^ = ^ / ( 1 + m 2). ...(3)
The equations to PPf are
jc- r , y -mrx z - c
ri /*2 ///(/*i+ r2) = ~2c .. (4)
The locus of PPf is obtained by eliminating i\ and r%between
(3) and (4). Hence proceeding as in Ex, 15 above and putting the
values of rx and r2 from (7) of Ex. 15 in (3), the required locus is
given by
c (y+mx) c (y -m x ) k
m (z+c) ’ m ( z - c ) ~ i-\-m2
or c2 (l+ m 2) (y2—m2x2)= km 2 (z2- c 2).
Ex. 17. Find the locus o f a straight line that intersects two
given lines and makes a right angle with one of them,
Sol. Let the equations of the two given lines be
y= m x, z= e (I) and y = mx, z = —c. ...(2)
The equations of any line intersecting the lines (I) and (2) are
given by the planes
Skew Umes 231

(y—mx)—Xt ( z —c)=*0 or —m x+ y—AtZ+cA^O —(3)


and (y+znx) A ,(z+ c)= 0 or m x+ y—A1Z-CAa=O. -(4 )
Let Av m v be the direction ratios of the variable line given by
the intersection of the planes (3) and (4). Then
—mA+1 .μ—A1-V=O, /?iA+1.μ—AjV=O.
Solving, we have . \ - = -----
A1- At —m (A1^A1) —2m .. (5)
Let the line with d.r.’s A, μ, v be perpendicular to the line (I)
. *to x—= —*=,——,
i.e. y z—c so A l_ _ wc .have
that,
I m O
!-(A1- At)+m { m(A1+Ae)}+0.( 2m)=0
or (A1-A t)-Zn2 (A1-HAt)=O. -(6 )
The required locus is obtained by eliminating A1 and At bet­
ween (3), (4) and (6). Hence it is given by
Cy —mx y+ m x I ^ iy - m x y+ m x I Q
\ z-c z+ c J I z —c + z+ c I
or -2mzx+2cy—zw2 (2yz—2zncx)=0
or mzx -cy=m* (cm x-yz). ...(/)
If the line would have beeirpeipendicular to the line (2), then
the required locus on replacing m by m and c by —c in (7) is
given by
(m z x -c y )= —ml· (cmx yz).
Ex. 18. Find the surface generated by a straight line which
intersects two given lines and is parallel to a given plane.
Sol. Let the equations of the given lines be
y= m x, z= c - (1) and y = —zzur, z= —c. .'..(2)
The equations of any line intersecting the lines (I) and (2) are
given by the planes
CV—mx)—A1 (z—c)= 0 or —zwx+y—A1ZH-CA1=O . ...(3)
or (y+m x)—As (z+ c)= 0 or m x+ y—AfZ -CAt =O. ...(4)
Let A, μ, v be the d.r.*s of the variable iine given by the inter­
section of the planes (3) and (4). Then
—mA+1 P -A 1V=O, mA+ Lp- AtV=O.
Solving, we have = ---- — r-\= - I ,
A1-A t ZZi(A1-HA1) —2m -.(:>)
Let the line with d.r.’s A, p, v be parallel to the given plane,
say, A x+ B y+ C z+ D = 0, so that we have
^(A1 A2) - ^m (A1+At)+ C t.(-2 m )= 0 . ..(6 )
The required locus is obtained by eliminating A1 and A2 bet­
ween (3), (4) and (6). Hence it is given by
232 Amaljticml Geom etry 3-D

A _ Z ± ^ l ^ i J ^ + Z ± « E l —2mC » 0
I z—e z+ c J I z c T z+c J
or A (2cy -Imzx) - B m (2jz —2mex)-2ntC (z* c*)=0
or >1 (<cjp—wizz) - JtM (fz —mcr)—mC (z*—c*)=0.
Ex. 19. Find the sufaee generated by a Une winchintersects
the Umesy=z=a; x+3z=a, y+z=a emdUyaraIIel to Aeplaee
jr + j= 0 .
SoL The equations of tike give· lioes are
j —e = 0 . z - o = 0 —(I)
I + 3z—o = 0 , y+z- o = 0 . -(2 )
The equations of any planes through the lines (I) and (2) are
( j - e ) —Jt1 ( z - e ) = 0 or j —J^z—o + o ^ = 0 —(3)
and (x + 3 z—o)—Jt (y+z—o )= 0
or z-JizF+ (3—ii) z —o+oJii=0. ---(4)
Any line intersecting the lines (I) and (2) is givenby the
intersection of the planes (3) and (4). Let Jt, p, » be its <Lr.*s. then
OJt+l.ji—V * = 0 , l-J -J ^ + (3 —Jt1) v= 0.
____A_______μ ____
·· 3 J .-J .J . -A 1 ~ - I '
Now the line with cLr.’s λ, μ, * is parallel to the plane
x + j = 0 Le., this line is perpendicular to the nom al to the plane
X-J-J=O whose d.r.’s are I, I. 0. So we have
l.(3 -J ^ -4 A H -M -A J + 0 L (-l)= 0
or 3 -A 1-Jt1-A 1A1=O. ...(5)
The required locus o f the line ir obtained by cfiaiaating
and J^ between (3), (4) and (S) hence is given by
3 I - · x + 3 z-a y—a x + 3 z - « Q
z —o y+z-a z—a y+z—a
or 3 (y + z - a ) (z—· ) —( j —o) (y+z-a)- (z—a) (x+3z-a)
(j-a) (x + 3 z -o )= 0
aaa

—j z - j*+2oz -x z + 2 e x —Xj=O
yz+j*+xz+xy=2az+2ax
(y+z) (x+y)=2m (x+z).
E r. 2 ·. Fimdthe Ioeas ofthe Umeawhkh smoteparallel to the
xy=r*, z = 0 ; j*—4cz, z = 0 .
SaL Let the equations o f any line be
(x -« )//= C r—Λ /ο * = (ζ-7 )/β . ...(IJ
According to the problem, the line (I) is parallel to the zx-
plane Le. the plane y = 0 , so that we have
Skew Lmt' 233

/.0φιιι.1+/ι.0=0 or IH==O.
Putting m =0 in (I), the line becomes
(x -α )//= 0 > -β )1 0 = (ζ -γ)/η . (2)
Now the line (2) meets Xy=C19 z=>09 so that we have
( x - a )//= ( ^ - f ) /0 = ( 0 - y ) /n
or ( x -(X)H=-Jfn9γ = β or x —a—(Ijfn)9 y= fi.
Putting the values of x and y in Xjr=C29 we have

(-?)— K > M ·
Agvn the line (2) meets y*=4cz, x = 0 , so that we have
(0—<*)//= O' -0)/O=(z—y)/n
or -* Ι Ι= (ζ —γ)Ιη, y = fi or z—y-(η*//), γ = β .
Pnttiag the values of y and z in y2=4cz, we have
> - f c ( r - " ) o r ( r - £ ) l- f . ...(4)
Muhiplying (3) and (4)9 to eliminate I and n9 we get

K k M -S U = '
or (a/? —c1) (4cy—/?*)™4ca/?y
or 4cafiy —β* (αβ-c*) -4c*y=4«/?y
or /P (αβ—c*)+4c*y=0.
Now the locus of the line (I) is the locus of the point (a, β, y)
and is given by
y* (xy—γ*)+4<^ζ= 0.
8
Change of Axes

§ I. Transformation of co-ordinates.
In th e study of analytical geometry, the co-ordinates of a
point, the equations of a curve or the equation of a surface are
always considered with regard to a fixed origin and a set of co­
ordinate axes. The co-ordinates of a point, the equations of a
curve or the equation of a surface change when cither the origin
is changed or the directions of axes are changed or both are
changed. The process of changing the co-ordinates o f a point, the
equations of a curve or the equation o f a surface is said to be the
transformation of co-ordinates. This process of transformation of
co-ordinates will be of great advantage to tackle most of the
problems.
§ 2. Change of origin (Translation of axes).
To change the origin o f
L <
co-ordinates to another point
(a, /?, γ ) 9 whereas the directions
of axes remain unaltered. /V /y
Let O be the origin of co­
ordinates and OX, OYi OZ be
the original co-ordinate axes L O1
Let O' be the new origin and 4 \
(a, β 9 yjits co-ordinates referred O
X
to theoriginal axes. / Y
Draw three lines OrX ', O 'Y \
and OtZ r through Of parallel to and in the same directions as OX9
OY and OZ respectively. Let P be a .point in the space whose
co-ordinates are (x9y 9 z) referred to the original axes OX9OY9OZ.
Again suppose that the co-ordinates of the same point P referred
to the new axes OfX f9 OfYt9 OfZ f are (*', y' , z').
Change o f Axes 235

From O' draw OtL perpendicular to OZ. From P draw PM


perpendicular to )>z-plane to meet y'z'-plane in N. Then
OiL= J - N M y PM=Xy PN=x'.
We have PM= PN+NM
i.e. x = x ' + a.
Similarly y= .y '+ / 3 , z= z'+ y.
Also, Xf = X - a, y '= y —β, z' = z—y.
Thus we have the following rule :
Rule. To change the origin to another point (a, /?, γ) replace
x by .T+a, y by β and z by ζ + γ. The transformed equation is
then obtained with respect to (a, β , y) as the new origin and in the
transformed equation also the current co-ordinates are (x9y y z).
Remark. Shifting the origin back. Sometimes it is required
to shift the new origin back. Then replace x by jc- a, y by y —β
and z by z y in any equation referred to the new origin to get
the corresponding equation referred to the old origin.
§ 3. Change of directions of axes. (Rotation of axes)
To find the change, in the co-ordinates of a point when the direc­
tions of axes are changed, the origin being fixed.
Let OXy OY. OZ be the ori­
ginal system of co-ordinate axes.
Let OX', O Y'y OZ' be the new
axes through the same origin 0.
Let Il9 nil*ni I U* M2, n2 and
Z3, m2. nz be the direction cosines
of OXt9OY' and OZf respecti­
vely referred to OX9 OY and OZ.
Therefore, the direction cosines
of OX, OY and OZ referred to
OXr9 OY' and OZ' are Ily I29 Z3; ntl9 m2y mz and n1} n2y n%
respectively.
Let P be a point in the space whose co-ordinates are
(x9y\ z) and (*', V 9 V)
referred to old and new axes respectively. Draw PM perpendi­
cular to OX.
Λ O M = X or X = O M = the projection of OP on OX.
Now’ referred to new axes the co-ordinates of P are (x 'y y ', z')
and the d.c.’s of OX are I19 I29 Z3. Hence we have
x = O M = (x '-0 ) I1+ (y' —0) /a+ (z' -- 0) /8
236 Analyttcal Geometry 3-D

or X = I i X' + l»y'+ Uz'.


Similarly y =W1X' -f m^y'+ Wjz' J. ...(I)
and Z=U1X'+nty'+ nt z'
Now multiplying the relations (I) by Z1, W 1 , /J1 respectively
and adding, .we have
liX -\-m 1y + n i Z = ( l l 2+ m l* + n li ) x ' + ( l i l t + m imt + ηι η*) Ϋ
+ ( / 1/,+ // 1^ , +«i/»,) z'
or lix + m iy + n lz = l.x '+ 0 .y ’+ 0.z' [See § 4, Next]
or x ' = lix + m 1y-\- U1Z. I
Similarly y ' = ItX -^m tf+ n2z >
and z '= lt x + m ty + n 3z. J
Hence an equation referred to OX, OY and OZ as axes is
transformed to an equation referred to OX', OY' and 0 Z ’ as axes
by replacing x by h x '+ l2y '+ h z ' y by w ^ '+ w jy '+ m jz 'a n d z by
HlXr+ η2γ ’+ η2ζ'. [See relations (I)]. In practice we leave the dashes
and hence from (I) the rule to obtain the transformed equation
referred to the new axes OX', OY', OZ' is to replace
x by I1X-Hjy-HaZ y by nj1x + m Jy+m 3z
and z by η ^ + η ^ + η ,ζ .
In the transformed equation also the current co-ordinates are
(x, y, z).
Again if we want to shift the co-ordinate axes back to their
original positions, then replace x (i.e. x') by Ux+m^y+niZ,
y (i.e. y ’) by liX+nify+n2z and z (/.?. z ’) by l^x^m ^y+ n ^ in any
equation referred to the new axes OX', OY' and OZ' to get the
corresponding equation referred to the old axes OX, OY and OZ,
[See relations (2)].
Remark. The results given by (I) and (2) can be conveniently
remembered by the following scheme of transformation.

X 1 y ’ Zt

X U U U

y /M1 m , Wj

Z ” 1 n2 «·
Change o f Axes 237

In the above scheme to get the value of x, first multiply each


element of the row of x with the corresponding elements of the
first row and then add, Le. we have Jc= Z1*'+Z 3V +/*^. Similarly
the values of y and z are obtained.
To get the value of x f, first multiply each element of the
column of x f with the corresponding elements of the first column
and then add, i.e. we have j e 'e / ^ + w ^ + HiZ- Similarly the
values of y' and z' are obtained.
§ 4 (A) Relations between the direction cosines of three mutually
perpendicular lines.
Let OX, OY. OZ be a given set of mutually orthogonal axes,
and OXf, OY f, OZ f be three given mutually perpendicular lines.
Let I19 mu H1 ; I2, m2, n2 and I9, m9, n9 be the direction cosines of
O X \ OY f and OZf respectively referred to OXt OY and OZ.
Therefore, we have the following six relation :
l\2+ w/^+Wj2= Π //,+ W 1W2 ^w1W2=Ol
V + wt22 + w28 = 1 V .(I) = ·· ( 2 )
/ +ZZ7 4-Z7a2= IJ
38 32 /3/1 + ZW
3 W1 +"W3Wi= Oj
Again the direction cosines of OX, OYy and OZ referred to
OX99 O Yf and OZf are I1, I2, /3; W1, m2, wt and Zi1, w2, zi3 respecti­
vely. Therefore, as above, we have the following six more
relations :
V +V + V= 0 / ^ ! + / 27773+ / 3/1 7 , = 0 1
Zzr12 + w 32 +W32= I V -(I) W1Zi^ W2Zr2 4-WaZT3-O V (4)
Wr + W22+ Z732 —IJ ZT1Z1 + w2/ 2 + W3 Z3 = Oj
(B) The direction cosines of one line in terms of the direction
cosines of other t\vo lines.
From relations ( 2 ) above, we have
V 2 +/Zyrr2 + ZT1Zi3 = 0, I1I9 f Zzr1Zzr3 + ZT1ZT1 = 0.
Solving these relations for Iu /W1, nu we get
_I _______ Zzr1 Zrl
W
T2ZT3 /W3 ZT2 - W2 Z3 W3A, /27Z73 73/ztb
^ VW-I 'WiM-Wi+ __V(I) _ = !
Vl- ow//3 --/ZT3 Zi2Ti V (s,n 90°)
Λ I1= +U zt2W3 /w.;w2), mx=*±(n%l9 -W3 Z2), W1= :Γ.(/2ζζι3 - l9m%),
(5)
Similarly other relations can be dete; mined.
238 A n a lytica l G eom etry 3 -D

Let us suppose
D= k Wl1 W1 I

h m* »* j.
I
W3 W3 !
We have
The co-factor of Z1=(Wi2W3-W 1Wi), etc.
Hence the relations (5) show that e a c h constituent in D
= ± (its co-factor).
(C) To show that D= ± I .
We have Dt= h Wi Wl I I1 W1 »1

k W2 W2 X it W1 «I

U W3 W3 h W2 »«
or />*= Six' Shk £ / 1/3
Z hk ΣΙ* Skk
Zkk w1 Skt
OT Df= I 0 0

0 I 0 [Using relations (I) and (2)]


0 0 I
OT Dt = I, or /)= ±1.
SOLVED EXAMPLES
Ex. I. OAy OB9OC are three mutually perpendicular lines
through the origin with direction cosines Zi, W l i W1 ; Z2i w 2, wt and
hi m2, W3 and if OA=OB=OC=O9 then show that the equation o f the
plane ABC is (Zj+Z2+Z3) jc + ( w 1+ w 2+ w 3 ) r-f- (wx- f wa+ w 8) z=a.
Sol. If the lines OA9OB9 OC be taken as co-ordinate axes,
then the equation of the plane which makes equal intercepts (each
equal to a) on these axes is Xja+ Y/a+Z/a = I
Ue. X -\-Y + Z = a. ...(I)
But it is’required to find the equation of the plane referred
to the original axes. Hence using the relations (2), § 3, we put
X = l1x+ m 1y + n iz9 Y = Z2x + w 2y+w2z and Z = hx+ m zy + W 3Z in the
equation (I). Then the equation of the plane ABC referred to the
original axes is given b>
Change of Axes 239

(IiX+OT1^+ B 1Z)+(ItX+tn2y 4 « 3z) \-(ltx + m^y+ ^ 2)=0


Of (/1 +/*+¾) x + i^ + iw j+ m ,) ^ + (/i,4 «*4»»*) z = a .
Ex. 2. The equations referred to rectangular axes o f three
mutually perpendicular planes are p,—IrX+ m ,y—nrz= 0, r= I, 2, 3.
Prove that if («, β , γ ) is at a distance d from each of them, then
d== * —OiPi 4 ItPt+ /«Pa) _ fi—(m,p1+ m ipt +mt p3)
U+U+U "1,4 w,4 W3
= 7 - («iP i+ » « P » + W »P »)
« i4 n 34«a
Sol. The equations of the given planes are
/1jc4-m 1^ + n 1z - p J= 0 , : - (1)
/2x 4 m s y 4 n 3z - p 2= 0 , - ( 2)
and l3x+ m 3y+ n az ~p3=0, ...(3)
where I1, mlt H1 etc. are direction cosines of the normals to the
given planes. According to the given condition, we have
</=the distance of (α, β, y) from the plane (I)
h n + m f+ n tf - P t , _ , „ p ,
~ V '( / i * + O T i * 4 » i * ) _ / l + , ^ + l7 P l ' . (4)
Similarly d= l^t+ m tfi+ nty—pt -(5 )
and d= Z3 Ct4 - OT3^ 4 η3γ - ρ 3· - ( 6)
Mottiplying (4) by I1, (5) by It and (6) by I3 and adding, we get
(/1 + /.+ /.) (Σ Ut)+ β Σ (Z1O t 1) 4 7 Σ (Un1)
—(PiU+PiK+
= « .l+ /?.0 4 )'.0 —(pfi+PtU+PtU)
[using (3) and (4) of § 4]
or j_ g ( P i! t+ P iU + P * la )
u+ u+ u -(7 )
Similarlyniultiplying (4), (5) and (6) by Oti , mt and m3respec­
tively and adding, we get
d={fi -(pxma+Pama+pjnJftKmi+ma+ma). . .(8)
And multiplying (4), (S) and (6) by nt, H3 and nt respectively
and adding, we get
rf={7-(Pi"i+/V»2+/V»3)}/('»i+«i+«·)· -(9)
The relations (7), (8) and (9) give the required result.
Ex. 3. Show that for any transformation of rectangular axes
without change o f origin, a+b+C, and /*4g*4-A* —bc—ca—ab and
abc+2fgh—of*—bg*—eft* are invariants for the expression
ajp+by*+cz* 4 2fyz+2gzx+2hxy.
240 Analytical Geometry 3-D

Sol. Suppose the expression


ax*-f by*+cz*+2fyz+2gzx+2hxy 0)
after any transformation of axes without change of origin becomes
Ax*+By*+Czt + IFyz ±2Gzx+2Hxy. (¾
We know that x*+yt +z* is the square cf the distance of any
point (x, y9 z) from the origin. Hence A(x*+y*+2*) does not
change by change of axes without change of origin H encethe
equation
ay*+by*+cz*+2fyz+2gzx+2hxy Λ(x, +y*+z*)=0 —(3)
transforms to
Ax*+By*+Cz*+2Fyz+2Gzx+2Hxy—X —W
The equations (3) and (4) may be written as
(er-A) Jt* + (4-A) Vs- (c- λ ) z*+2fyz+2gzx+2hxy=0 - ( 3 ')
(A- X)x *+( B X)y*+(C X)z*+2Fyz+2Gzx+2Hxy=0 - ( 4 0
Now if (3') represents a pair of planes non (40 will also
represent a pair of planes for the same value of A, i.e. X is the
same quantity in both the equations.
The condition for (3') to represent a pair of planes is
a λ h g
h h- A f
g / c -X
or (a- λ ) {(fr—A) (c X)f*}—h {h (c X) fg) f g {hf g { b - A)}=0
or X*—(a+b+c) \ l + [bc+ca+ab f* g- IF) X
- (abc f 2fgli af - bg* dfc*)=0. ...(5)
Similarly the condition for (4') to represent a pair of planes is
A3 (A+B+C) X*+(BC+CA + A B -F t Gt H*) X
-(ABC+2FGH—AF* BG2 CHt)= 0 .(6 )
Sincethe roots of (5) and (6) are same i>. the values of A
given by (5) and (6) are same therefore comparing the coefficients
of like powers of A in ό ) and (6), we get
a+ b + c= A + B + C "I
f*+g* + h* bc=cj-ab=F*+G*+H* BC CA AB I
and abc+Ifgh a f-—bg*-ch2 I
ABC+2FGH AFt -B G t CHtJ ...(7)
Relations (7) show the required results.
Ex. 4. Show that if
ax2+by2+cz*+2fyz+2gzx+ Uixy ±2ux+2vy+2wz+d
be transformed by change o f co-ordinates from one set o f rectangular
Change o f Axes 241

axes to another with the same origin, the expressions a + b+c aiul
a*+v*+tv* remain unaltered.
SaL' The given expression is
ax* T by*+ cz*+2fvz+ 2%zx+ 2hxy + 2ux+ 2vy+ 2wz+ d. ...II)
Let the given set of axes OX, OX, OZ be transformed to the
new set of axes OX', O Y', OZ'. Let d.c.’s of OX', O Y', OZ' be
I u m l t Ui; lt,m t,n t ; /,, m,, /1* referred to Ο Χ ,Ο Υ ,Ο Ζ . Now
replacing x by IiZ etc. [See § 3 relations (I)), the ex­
pression (I) transforms into
o (/ix+/*y+/*z)*+& (mtx + /Wjj>-f.injz)2+ c (/I1X-I-Wi V+/¾-)*
+ 2 / (n^x+ m ^+ m jz) (ntx +thy+ /V )
■LigOhX+iv+/^z)( Ιχχ+ Iiy+ I,z) -{-2AiZ1X+ /,y+ UzKmlX+ m*y+ »W*z)
+ 2 a ( /jx - f /^ + /^ ) + 2 f (m1x+/»it y+e%z)
+ 2 ^(/1^+ /1^+ /1,2)+ //. -.(2)
Suppose the expression (2) is written as
Ax*+By*+Cz*+ 2Fyz+ 2Gzx+2Hxy+2Ux+2 V yJtlW z+ D
where A, B7 C7 F7 G, H7 U7 V7W and D are clearly given by
A -O l1* + Am,*+CTt1* + 2//11,/1,+2gn,/,+ 2U1Ot1,
B=Ol1t A-Amt*+ent* + Vm 1Iii + 2 g n ,/,+ Ihljn1,
C= e/|*+ Am3*+ c'/ij*+ + 2gW,/* + IM jn1.
¢/= a /,+ Vm1+MW1, K= a/.+vm ,+»'/»,,
H'=a/,+vm 3+M7»t> D—d.
:. A + B + C = a I l*+ b Σ ΐΒ ,+ ί Γ /r,*+ 2/2 /H1H1
+ ¾ J H1/, +2// Σ Uml
or ^ + e + C = n + A - |c
[using relations (3) and (4), § 4]
and U*-\- Vs+ IK*—(u/,+vm1+M’/r1)*+(a/,+vm,+M’H2)*
+ (u /9+vm, + Hw,»*
=«* Z /,*+v* Σ m ,*+iv* Σ n,2+2av Σ Ijn t
+ 2 ku 2' h1/,+ 2 vm· Σ m jtt
= m* + v* + h’* fusing relations (3) and (4) of § 4],
Hence the expressions α + Α + c and η*+ν* + Μ^ remain
unaltered.
The Sphere

§ I. Definition. A sphere is the locus o f a point which moves in


space such that its distance from a fixed point is always constant.
The fin d point is called the centre of the sphere and the.
constant distance is called the ndlns of the sphere.
§ 2. Efnatioa of a sphere.
(A) Central Form. To find the equation of a sphere when cen­
tre and radius are given.
Let O be the origin of co-ordinates. Let
the eo-oidinates of the centre C of the sphere
be (a, b, c) and r be the radius of the sphere.
Let P (x, y, z) be any point on the surface of
the sphere. Then
OC=ai+bj+ck,
and O i= M iy j-H zk .
Λ C P = O P -O C
= ( x - a ) i+ ( y - b ) j+ ( z —c) k.

We hare £7»=| CP |= r .
I ( x - a ) i+ ( y - b ) j+ (z —c) k | = r
or y/{lx - a f + { y b)*+(z -c)x}= r
or (x -a f+ (y -b )* + (z-c )* = r * . ...(I)
The equation (I) is the required equation of a sphere whose
centre is (a, b, c) and radius r.
Particalar Case. If the centre of a sphere is origin and
radius is r, then we have : OP I = r,
where O is the centre and OP ( = r) is the radius.
Hence we have
! x i+ y i+ zk ; = r Le. y/( x i + yt + zt)-=r
or x*+y* T-Zi --=rs ...(2 )
which is the rcquired’equation.
The Sphere 243

Remark, it can be easily proved that every diameter of a


sphere subtends a right angle at any point of its surface.
(B) General form. To find the general equation of a sphere.
Tofind also the centre and radius of the sphere in the generaI form.
[Garhwal 1979]
The equation of the sphere with centre (a, bf c) and radius r
is given by [See equation (I) above]
( x - a ) 2+ ( y - b ) 2+ ( z - c ) 2=r2
or x2+ y2+ z2—2ax -2by —2'cz+(a2+b2+ c2—r2)=0. (3)
The equation (3) is of the form
x2+ y2+ z2+ 2ux+2vy+2wz+d—0. ...(4)
The equation (4) is called the general form of the equation of
a sphere.
Comparing (3) and (4), we get
2(7=—2m, 2 b = —2v, 2c= - 2 w, a2+b2+c2—r2=*d.
or a = —u, b= —v, c— —w, r2=u2+v2+w2—d.
:. the centre of the sphere (4 ) given in general form is the
point ( —u, —v, - w) and the radius is y/(u2+v2+w2—d).
(C) Conditions for a sphere. The most general equation of
second degree in jc, y and z is
to?+by2+ Cz2+ 2fyz -(- 2gzx + Ih x y + lu x+ 2vy+ 2w z+ d= 0. ... (5)
Comparing (5) with the general equation (4) of the sphere,
we have :
(i) The equation of a sphere is of second degree in X9y and z.
(ii) The coefficients of X29y 2 and z 2 are equal in magnitude
and sign both, and
(iii) The coefficients of the product Jerms involving yz, zx
and xy are zero i.e. in other words, in the ^equation of a sphere
the terms involving yz9 zx and xy must be absent.
Remark I. The radius of the sphere (4) is \/(w2+ v2+ w2- d)
which is real when u2+ v2 + w2—d > 0 .
If u2+v2+ w2—d—0, the radius is zen the sphere reduces
0 point and is called a point sphere. If u2+v2+w2- d < 0,
radius is imaginary whereas the centre ( - t/, - v, —w) is real
in this case the sphere is called a virtual sphere' or pseudo-
sphere.
Remark 2. The general equation (4) of the sphere contains
°uf arbitrary independent constants namely w, v, w and d and
244 Analytical Geometry 3-D

hence the equation to a sphere can be determined if four condi­


tions are given.
(D) Four-point form. To find the equation o f a sphere which
passes through four given points A9 B9 C and D whose co-ordinates
°re (xl9 y l9 Z1), (x2, y t9 z*), (x3, y3, z9) and U 4, yi9 z4) respectively.
Let the general equation of the sphere be
x2 + y 2 + z * + 2 wx+ 2 v>+ 2 H'z+rf==0 . . (6 )
Ifit passes through the four given points A9B9C and/),
then their co-ordinates will satisfy the equation (6 ), and therefore
we have
X12+ Vi2 +z,*+ Iux1+ 2V y 1 + Iwz1+ d= 0'
x%+y* + Zte+ Iux 2 + 2 vy2 + 2 wzr + rf= 0
Xa2 + V32 + *32+ 2uxs+ Ivyz+ 2Wz9+ d= 0 ... (7)
X42 + V42+ Z42 + 2ux4 + 2vy4+ 2Wzl 4 d= 0
Eliminating the constants u9 v, w and d between the equations
( 6 ) and (7), the required equation of the sphere is given by
x *+ y + z 2 X y Z I

* i a+ .V i * + Z i * Xi yi Zi I
'" ·~·'Α v Q 3
***+ Λ *+ 2 ** *1 y% Zf i = 0. -.(8)

*» *+ Λ *+ *» * y* Zs I

* 4 yi Zi I
Remark. The equation ( 8 ) has *a determinant of 5th order
and hence in numerical problems, the evaluation of 5th order
determinant will require much labour and therefore, it is advi­
sable, to solve four equations given by (7) for U9 v, w and d9 then
the required equation of t,he sphere is obtained by substituting
the values of t/, v, w and d in equation (6 ).
(E) Diameter form. To find the equation of a sphere on the
join o f two given points A U 1, y l9 Z1) and B (x2>y t%z2) as diameter.
(Berahampur 1981 (S); Indore 79; Lucknow 78; M. Dayanand 79)
Let P with co-ordinates (x, v, z)
be a current point on the surface of a
sphere drawn on the line joining the
given points
A Ui> Λ. Z1) and B (x2, y 2, z2)
as diameter. Then clearly the lincs^P
and BP are perpendicular to each other.
The Sphere 245

The direction ratios of AP are


*-* i. y-yi> Z -Z 1 ;
and the direction ratios of BP are x —JC1, y —y t9 z—z2.
Since the lines AP and BP are perpendicular to each other,
therefore using the formula
iOlOt + bib^+c^t= 0 ’, we have
( x - X 1) (X -X 1R C y (y -} ’%)+ ( Z - Z 1) (Z- Zi)=O. ...(9)
The equation (9) is the required equation of the sphere.
SOLVED EXAMPLES (A)
Ex. I . Find the equation of the sphere whose
(i) centre is (--3, 4, 5) and radius 7,
(ii) centre is (2, —3, 4) and radius 5. [Meerut 1972]
Sol. Let P (x, y , z) be a generalpoint on the sphere, then
using § 2 (A), equation (I), we have
(i) Theequation of the sphere with centre ( —3, 4, 5) and
radius 7 is given by
\x - ( - 3)}»+(y - 4)2+ (z - 5)2=(7)2
or x 2 + 6 x + 9 +jv2 - + 16+ Z2- 1 Oz+ 25= 49
or x 2 + };2 + z 2 + 6 x -- 8 v —IOz+ i =0. Ans.
(ii) The equation of the sphere with centre (2, —3, 4) and
radius 5 is given by
(x —2) 2 + {v —(—3) } 2 + (z—4)2 = (5 ) 2
or X2 -4x+4+>» 2 + 6 v + 9 + z 2- 8 z + 16=25
or x 2+ y2+ z2 4x-f6.y—8z+ 4=0. Ans.
Ex. 2. Find the equation o f the sphere whose centre is
(?., - 3, 4) and which passes through the point (1,2, -- I).
Sol. Clearly the radius of the sphere.
—the distance between the points (2, —3, 4) and
(1,2,-1)
■=V{(2-·1)*+(-3-2)* + (4+1)*} = ν(1 + 2 5+ 25)-ν(51).
Hence the equation of the sphere whose centre is (2, —3, 4)
and radius \/(M j is given by [see § 2 (A)]
(x — 2)2+ {y - ( —3)}2 + (z—4) 2 = {vX51) ] 2
cr x2- 4x + 4 + iy2+ 6 j'+ 9 + z 2—8 z + 16=51
or x2+ y2+ z2—4x+6v--8z—22=0. Ans.
Ex. 3. Find the centre and radius o f the following given
spheres :
(i) x 2-\-y* + z2 — 2x+ 4y—6z— 11,
and (U) 2x2+2y2+2z2—2x+ 4y—6z= 15. [Burdwan 1976]
246 Analytical Geometry 3-/)

Sol. (i) The equation of the given sphere is


x 2+ yt + z 2- 2x + 4y ~62 - 11 =0. ...(I)
The most general equation of the sphere is
x*+;y*+za+2Mx+ 2vy+2w z+if=0. ...(2)
Comparing (I) and (2), we get
Iu = - 2 , 2v=4, 2 w = - 6 , d = ~ 11
or M= —I, v=2, w = —3, rf= —11.
Λ The centre is ( —u, —v, —w) i.e. (I, —2, 3)
and rad iu s= v/(Mls+v*+wa- </)«=■/{(-1)*+ (2)*+ (-3) * " ( ~ 1*)>
= V (l+ 4 + 9 + ll)= 5 .
(H) The equation of the given sphere is
2x*+2y2+ 2za- 2 x + A y - 6z - 15= 0.
Dividing throughout by ‘2’, we have
x*+>,2+ 2a—x + 2 y —3z—(15/2)=0. -.(3)
Comparing (3) with the most general equation (2) of the
sphere, we have
u= - b v = I, w= -3 /2 , <f= -15/2.
Λ The centre is ( —a, —v, —w) i.e. (i, —1, | )
and r a d i u s = ( -V )}
= V (i+ i+ !+ ¥ )= \/(ii).
Ex. 4. Find the equation o f the sphere on the join o f (2, —3, I)
and (3, —1, 2) as diameter.
Sol. The equation of the required sphere is given by [See
§ 4 (E), equation (9)]
( x - 2 ) (* -3 )+ 0 > -(~ 3 )} {y—( —1)}+(z—I) (z—2)= 0
or Xa- 5 x + 6 + y a+ 4 y + 3 + z a—3 z+ 2 = 0
or xa+ ya+ z2—5 x + 4 j> -3 z + ll= 0 . Ans.
Ex. 5. Find the equation to the sphere through the points
(0, 0, 0), (0, I, —1), ( - I, 2, 0) and (I, 2, 3).
(Meerut 1975, 85 ; Madras 74)
Sol. Let the equation of the sphere be given by
xa++ a+ za+2MX+2vy+2wz+if=0. ...(I)
If(I) passes through (0, 0, 0), then if=0.
Putting if= 0 in (I), the equation of the sphere becomes
xa+.y2+ z a+2ttx+2vy+2wz=0. ...(2)
If (2) passes through (0, I , —I), we get
0 + l + l+ 0 + 2 v —2w=0 or v—w + l= 0 . ...(3)
If (2) passes through (—1, 2, 0), we get
1 + 4 + 0 —2 u + 4 v + 0 = o or —2u+4v+ 5=0. ...(4)
If (2) passes through (I, 2, 3), we get
1 + 4 + 9 + 2 m + 4 v + 6 w = 0 or 2 u + 4 v + 6 w + 1 4 = 0 . ...(5)
The Sphere 247

Adding (4) and (5), wc have


8v+6w+19—0. ...(6)
Multiplying (3) by 6 and adding to (6), we get
14v+25=0 or V= --25/14.
Putting the value of v in (3), we get
—25/14—w + 1=0 or W= —11/14.
Putting the value of v in (4), we get
-2 i# -5 0 /7 + 5 = 0 or u = -1 5 /1 4 .
Putting the values of if, v, w in (2), the equation of the requi­
red sphere is given by
χ*+τΗ -ζ*-(15/7) jc—(25/7) y - (I 1/7)7=0
or 7 (χ*+ 7 *+ζ2) —15χ -257—Uz=O.
Ex. 6 (a). Find the equation o f the sphere passing through
(0,0, 0), (a, 0, 0), (0, *, 0) and (0, 0, c). (Agra 1975 : Meenit 85S)
Sol. Let the equation of the sphere be given by
xF + y^+ ^^2ux+ 2ry+ 2w z+ d= 0.' ...(I)
If (I) passes through (0, 0, 0), then rf=0.
Putting d= 0 in (I), the equation of the sphere becomes
x * + 7 1+ 7 , + 2 ujc + 2 v7 + 2 iv z = 0 . ...( 2 )

If (2) passes through (a9 0, 0), we get


Hjr+ 0 + 0 + 2ua+ 0 + 0 = 0 or 2 u = —a or u= —\a .
Similarly if (2) passes through (O9 b, 0) and (O9 O9 c), we get
v= \h and w= —\c.
Putting the values of u9 v, w and d in (I)9 the equation of the
required sphere is given by
x ? + y * + z* -a x -b y —cz=Q. ...(3)
Note. Theco ordinatesof the centre of the sphere (3) are
( - I i9 —v, -w ) i.e. (\a 9 \b 9 £c)
and the radius= v / ( h * + v 2 + w 2 —
= V {(-**)2+ ( **)2+ ( - ^ ) #- 0 } = i V (* + * H * * ).
Ex. 6 (b) Find the equation o f a sphere which passer through
the origin and intercepts lengths a9 b and c on the x 9 y and z axes
respectively. (Raacfci 1976)
Sol. The sphere intercepts a length a on the x-axis and hence
it passes through the point (a9 O9 0). · Similarly the given sphere
passes through the points (0, b. 0) and (0, 0, c). Hence we are
to find the equation of a sphere passing through the four points
(O90, 0), (a9 O9 0)9 (O9 b9 0) and (0, O9 c).
Now sec Ex. 6 (a) above.
248 Analytical Geometry 3-D

_ Ex. 6 (c) The plane ABC whose equation is


x !a-Fyfb F-fc —I
meets the axes of X 9 y and z in A1 B and C respectively. I f O is the
Ohgin9find the eauation o f the sphere OABC.
[A en 1974 ; R ajasthu 78]
SoL The equation of the given plane is
x fa+yfb+zlc= I. --0 )
The plane (I) meets the Λ-axis in the point Awhere y = 0, z= 0,
and so (I) given r /e + 0 + 0 = I or jt= a. Therefore, the co-ordinates
of the point A are (a9 0, 0).
Similarly the co-ordinates of the points 2? and C are (0, b9 0)
and (0, 0, r) respectively.
Hence the required sphere OABC is passing through the four
points O (0, 0,.0), Aia1 <, 0), B (0, b, 0) and C (0, 0, c). Now pro­
ceed as in Ex. 6(a) above.
Ex. 6 (d) Find the equation o f the sphere circumscribing the
tetrahedron whose faces are
jc= 0, v=0, z= 0 , xla+ylb+zfc— l.
[Paajab 1976 ; Meerat 80J
Sol. The equations of the planes [i.e. faces of tetrahedron]
arc
jc= 0 ...(I) y ~ 0 -(2)
z= 0' ...0 ) XfO+ylb+zIc^-X.. *-*(4)
Anythree faces (Lc. planes) meet at a vertex of the tetra­
hedron.
Solving (I), (2; and (.>), wc h;,ve the vertex O (0, 0,0).
Solving (2), (3) and (4), \\c have the vertex A (a, 0,0)
Similarly solving (I), (3), (4) and (1),(2),(4) we have the
vertices B (0, b9 0) and C (0, ? c, respectively.
Therefore the sphere circumscribing the tetrahedron OABC is
the sphere passing through the four points O (0, 0, 0), A (a90, 0),
B (0, b9 0) and C (0, 0. c). Now. proceed as in Ex. 6(a) above.
Ex. 7. Find the equation o f the sphere circumscribing the
tetraludron whose faces are
ylb rz-c—i\ zfc+xfa=C, xfa+yfb=0, x a+yfi+zjc I.
Sol. The equations of the planes (i.c. faces of the tetrahedron]
are
Jfb-Fz c=O1 ...(I) z f c + x j a - U1 ...(2)
x fa + y b = O1 ...(3) xla+y/b+ zfc= I. . (4)
Any three faces (i.e. planes) meet at a vertex of the tetrahed­
The Sphere 249

ron. Hence, solving (I), (2)f (3). we get the vertex O (0, 0, 0) and
solving (I), (2), (4), we get the vertex (o, b, —c).
Similarly solving other two triplets of the equations of the
planes the remaining two vertices are (o, - b9e) and (- a, b9 c).
Hencetherequired sphere passes through the four points
(0, 0, 0), (a , 6,— c), (a, —A, c) and ( - o, b9c).
Let the equation of the sphere be
&-\-yz+ zt +2ux+2vy+2wz+d=0. ·* (5)
If (5) passes through (0, 0, 0), we have d=.0. Hence (5) be­
comes
x~+y*+z*+2ux+2vy+2wz=0. .(6)
If (6) passes through (a, b, —c), (a, —b9 c) and (--a9 b9 c),
we have
a2 F6*+c*+2ua+2v/>—2ivc=0, ...{V
a*+b*+c2+2ua -2vb+2\vc—09 - .(8)
and a*+b2 ±c2 -2ua+2vb+2wc=0. ...(9)
Adding (7) and (8),
2 (0^ + ^ + ^ ) + 41/0 =0 or 2u= —(o*+6*4-c2).o.
Similarly, we have
2v= —(o*+6*+c2;/6. 2w= —(o2+ ^ + ^ )//* .
Substituting the values of //. v, w and J in (5), the equation of
the required sphere is given by
JcM-J*2+ z2—(l a) (a*-J b2+c2) jc— (1/6) (οΜ-^M-c2) v
— (1/c) (uP+A^+c*) z—0
a-+ + 2+22 a* z
ΟΓ a2+b2+c- a b c
Ex. 8. F/W /Ac equation o f the sphere which passes through the
points (I, - 3, 4), (I, —5, 2), (I, —3, 0) and whose centre lies on
the plane x + y + z= 0 . [Jodhpur 1978]
Sol. Let the equation of the sphere be given by
jc2+ y* + z 2+ 2ux+2 vy + 2hz + d ^ 0. ... ( I)
If (I) passes through (I, —3, 4), then we have
I + 9 + l6+2w—6 v + 8 h + J —0. ...(2)
If (I) passes through (I, —5, 2), then we have
I + 25+ 4+ 21/- IOv+4 * + J=O. .(3)
If (I) passes through ( I, - 3, 0), then we have
1 + 9 + 0 + 2 ¾ /-6 r + 0 + Je=O. .(4)
The co-ordinates of the centre of the sphere (I) are ( —i/. —v,
—w) and if it lies on the plane jc+ j *+ j - 0, then we have
—u-—v —iv==0. (5)
250 Analytical Geometry 3-D

Subtracting (4) from (2),


16+8 m»=0 or w= —2 .
Subtracting (3) from (2),
—4+4v+ 4w =0 or —l+ v + w = 0 . ...(6)
Putting the value of w in (6),
--1 + μ —2 = 0 or v= 0.
Adding (5) and (6), —I —M=O or m= - 1 .
Putting the values of u and v in (4), J = 10.
Substitutingthevalues o f u9 v%w and </in(l), the equation
of the required sphere is given by
x2+ y 2+ z2 -2 x + 6 y —4z+ 10=0.
Ex. 9. Find the equation of the sphere haying its centre o*t the
line 2x~:3y=0=5y+2z and passing through the points (0, 2, -4)
and <2, —I, —I). [Agra 1977 ; Lucknow 81]
Sol. Let the equation of the sphere be given by
x2+y*+z*+2ux+2vy+2wz+d=0. -.(1)
If (I) passes through (0, -2, —4), we get
0 + 4 + l6+ 0-4v-8w + rf= 0 ..(2)
If (I) passes through ( 2 , — 1, - I), we get
4 + 1 + l + 4 n —2v—2w+</=0. (3)
The co-ordinates of the centre of the sphere (I) are ( - U 9 v,
—w) and if it lies on the line 2x —3y= 0= 5ey+2z, then we have
- 2m + 3v = 0 - .- ( 4 )

and —5v-2w =0. .(5)


Subtracting (2) from (3),
4 m+ 2 v+ 6 w—1 4=0
or 2m+ v+ 3 w=7. -(6)
Putting the values of u and w from (4) and (5) in (6), we get
3v+v—(15/2) v=7 or v= —2.
Putting v= —2 in (4) and (5), we get M= —3, w=5.
Putting the values of m , v , w in (2), we get rf=12.
Substitutingthe values of m , v, w and d in (I), the required
equation of the sphere is
x2+ y2+ z 2—6x—4y+ 10z+12=0.
Ex. 10. A plane passes through a fixed point (p9 q9r) and cuts
the axes in A9 B9 C. Show that the locus o f the centre o f the sphere
O/iBC P l x + q l y + r j z = 2.
KUahabad 1981; Bundelkhand 78; Kanpnr SO, 82;
know 80; Meerut 83, 83P, 84S, 86, 86S; 89S
Raochi 78; Rohilkband 77, 80; Punjab 77J
The Sphere 251

Sol. Let the equation of the plane be


x/a+ y/b+ z/c= I. ...(I)
The plane (I) meets the eo-ordinate axes in A, B, C. Hence
the co-ordinates of A. B and C are (a, 0, 0), (0, b, 0) and (0, 0, c)
respectively. The equation of the sphere OABC is
[See Ex. 6 (a) above]
x2+.V2+ z*—ax—by—cz—0.
If its centre be (*, β , γ), then x—\a, fi= \b , y= $c.
Λ α=2α, 6 = 2 /?,c=2y. --(2)
Itis g iv e n th a tth e p la n e (I) passes through the fixed point
(p, q, r). Hence we have
pla+ qlb+ rjc= l
or p/(2«)+g/(2)9)+r/(2y)=1 [Putting the values from (2)]
or p /a + 0//? + r/y = 2 .
.·. The locus of the centre (a, β, y) is p /x + q / y + r / z = 2.
Ex. 11. A sphere o f radius k passes through the origin and
meets the axes in A, B, C. Show that the centroid o f the triangle
ABC lies on the sphere 9 (x2+ y 2+ z 2)= 4it2.
[Allahabad 1980; Avadh 82; Meerut 84P, 86; Indore 78;
Kanpur 79, 83; Nagpur 77]
Sol. Let (a, 0, 0), (0, b, 0) and (0, 0, c) be the co-ordinates
of the points A, B and C respectively. The equation of the sphere
OABC is [See Ex. 6 (a) above]
x?+y2+ z2—ax - b y —cz=0. ···(!)
The radius of the sphere (l)= V { (ia)2+($h)*+(i<02}. so that
\ ’\Z(ai -\-b2+ cv) —k (given).
Squaring, we get
a2+ 6 * + Ct - A k 2. ...(2)
Let (α, β, γ) be the centroid of the &ABC, so that we have
ot= i (a + 0 + 0 ), 8 = i (0+ 6+ 0), y= $ (0 + 0 + c).
Λ a= 3« , b = 3β, c—3y.
Substituting the values of a, 6, c in (2), we have
9α2+ 9β 2+ 9 ?2=4Α:2.
Λ The locus of the centroid («, β, γ) is 9 (x2+ y ’+ z 2)=4Jfc2.
Ex. 12. A sphere of constant radius 2k passes through origin
and meets the axes in A, B and C. Prove that the locus o f the cen­
troid o f the tetrahedron OABC is
. x 2+ y 2+ z2= k 2. [Meerut 1984, 86P]
252 Analytical Geometry 3-D

Sol. The equation of the sphere OABC is [See Ex. 11 above]


x1+y*+z2—<*x—by—cz= 0. ...(U
The rad;us of (1) = ^/(( JtfV24 (Ib)2A (ic)2}=2k (given).
Squaring, we gel a2+.b2-\-c2=*\6k2. ..(2 )
Let (*, β %γ) be the centroid of the tetrahedron OABC.
Then OC=S=J (θ+α + 0+Ο) or a= 4a.
Similarly b—Αβ and c=4y.
Substituting the values of a, b, c in (2), we have
(4a)2-f(4j5)2+ (4 y )*= 16Ar2 or <x2+ fi2+ y2- k - .
\ The required locus of the centroid (a, β, γ) is
x2f y 24- Z2^ k 2.
Ex. 13. Find the equation of the sphere with centre at (2, 3,
—4) and touching the plane
2x+ 6y—3z4-15=0.
Sol. The point (2, 3, -4) is given as the centre of the
sphere. Since the sphere touches the plane
2 x + 6 y -3 z + l5 = 0 , ...(I)
therefore the radius of the sphere=the length of the perpendicular
from the centre (2, 3, —4) to the plane ( I )
2.24-6.3 3 .(-4 )4 15 49

Λ The equation of the required sphere is


( x - 2)2 r(y - 3 ^ ( - 4 4)2= (7)2
or x2 1- v24-z2- 6;>4 Sz 20=0.
Ex. 14. Obtain the equation of the sphere which passes through
(4, I, 0), (2, --3, 4), (1,0, 0) and touches the plane
2x4-2;» - z = II. | Avadh 1982; Garhwal 78]
Sol. Let the equation ot the sphere be given by
x* + y2 4 z2 f2w x+2)+ 4-2*vz+</='+ . .(I)
Tiie sphere (I) passes through the points (4, 1 ,0 ),1 2 ,.-3 ,4 )
and (1, 0, 0), therefore, we ha\e
16 + 1+ 0 + 8 « t-2i--H)-M=0 ...(2)
'4 + 9 + 16 + 4 m—6v+ 8 m·+ «/■=(> .(3)
1 1 0 i - 0 + 2«+t)+0+</=.(). . .(4)
From (4), —H + I). (5)
From (2), 2 v = - - 8 m - </— 17—4</+4— i7, using (5)
or i-= i (3</ 13). - (6)
Froni (3), 8 ti’= —4mτ·6γ—</—29=2</-f 2+9(/—39—i/—29.
using (5) and (6 )
T h e Sphere 253

or w = l (54-33). ...(7)
Now it is given that the sphere (I) touches the plane
2x+2y - z = l l . ...(8)
·'· The length of perpendicular from the centre ( —u, —v,
—w) to the plane (8)—the radius of the sphere (I)
or 2 ( - « ) + ? . ( —νϊ --( -w) —11
= y/(ui + vi+wi -d).
V{(2}* + (2)* + ( - I)2}
Squaring, ( - 2u--2v + w*~l l)a= 9 (u*+v2+ w *- d). ...(9)
Putting the values of u, v, w from (5), (6), (7), we get
{d+ I- 3 4 + 13 + 1 (54—33)—J I}2= 9 {J (d+ 1.2+ * ( i d - 13)·
+ (1 /1 6 ) (5 4 — 3 3 )*— 4 }
or (1/16) {- 3</—21)2=(9/!6) {4 (4 + :)2+ 4 (34--13)2
+ ( 5 4 - 3 3 ) * - 164}
or (4+7)*=4 (42+ 2 4 + lj+ 4 (942 -784,-169)
+(ZSdi- H O d + 1 0 8 9 )-\6d
or 644*-6614+1720=0 or 842- 834+215=0
or 842—404 434+215=-0 or (4 -5 ) (8 4 -4 3 )= 0
or 4=5,43/8.
When 4= 5, the equations (5). (6) and (7) yield « = 3, v= I.
»v= - 2 .
Substituting the values of u, v, tv and 4 in (I), the equation of
the required sphere is
Xi + y 2+ Zi —6x+2y ■ 4 z+ 5 = 0 .
Similarly when 4=43/8, find the corresponding equation of
the sphere as above.
Ex. 15. A sphere of constant radius r passes through the origin
O and cuts the axes in A, B, C. Find the locus o f the foot of the
perpendicular from O to the plane ABC.
[Allahabad 1975, 79; Kanpur 81; Meerut 73, 83 S, 87, 89, 90
Rohilkhand 78. 79, 81: Nagpur 87]
Sol Let (a. 0. Ot, (0, b, 0) and (0, 0, c) be the co-ordinates
of the points A, B and C respectively. The equation of the sphere
OABC is [See Ex. 6 (a) above]
x2+ y2+ z2 ax by - c;= 0. - (1)
C learlytheinterceptsonthe co-ordinate axes are a, b, c and
so the equation of the plane ABC is
x/a+ ylb+ zjc= I. ...(2)
The radius r of the sphere (I) is given by
V'{(ia/ + (i*)2+ !$ c)2i = r or ai +bi +ci =4r-. ...(3)
254 A n a ly tic a l G e o m etry 3-D

Now the equations of the line through the origin and perpen­
dicular to the plane (2) are
x —0 y - 0 z - 0
\la ~ \jb ~ ijc A’ (say)‘ .(4)
The co ordinates of any point on the line (4) are (X/a, λ/ft,
X/c). If this is the foot, say (α, β, γ), of the perpendicular from
the origin to the plane (2), then we have
Xfa=OL9 X /ft=0, X / c = y ο τ α = λ / α, ft=A//ϊ, C=Xjy. ...(5)
Since the foot of the perpendicular lies on the plane (2), we
have <x/a+$/b+ylc=\. .(6)
Now putting the values of a, ft, c from (5) in (3) and (6),
we get
λ2 ( l/α2+ 1//?2+ 1 Iy2i=Ar2 .. (7)
and (l/A)(a2+ £ 2+ y 2) = l . -...(8)
Eliminating λ between (7) and (8), we have
( a 2+ / ? 2+ ? 2) 2 ( o r 2+ / M - | - y - 2) = 4 r 2.
.·. The locus of the foot of the perpendicular ία, β, y) is
(x2+ y2+ z2)2 (x~2+y~2+z~2)= 4r2. Ans.
Ex. 16. Find the equation o f the sphere which passes through
the points (1 ,0 ,0 ), (0, 1,0) and (0,0, I) and has its radius as
small as possible.
Sol. Let the equation of the sphere be given by
X2 + y2+ z 2+ 2 u x + 2 v y + 2 w z + A=0. ...(I)
If(I) passes through the three given points (I, 0, 0), (0, I, 0)
and (0, 0, I), we have
I +2u+X =-0, I 4-2ν+λ=0, I-f-2w—λ= 0.
Solving, we have
U = V = W= -h ίλ-hl). ...(2)
Let r be the radius of the sphere (I), so that we have
r2=u2+ V2+ w2- Χ= μ (say)
or p = r2=± (λ-f I)2- λ.
[Putting the values from (2)]
If the radius r is minimum, then r2 i.e. μ is minimum. The
necessary condition for μ to be minimum is
dpfdX = 0 or (3/4).2 (λ+ l ) - 1 - 0 or A = - i
Also ^ = ' T = +ve at A= ~έ·
Hence μ is minimum at A = - J .
Putting A=*—I in (2), we have u=v=w =- —
The Sphere 255

Λ Sabstitating the values in ( I), the equation of the required


sphere is given by
X*+yt + zi - I ( x + j’+ z ) - i = 0
or 3 (jc*+y*+z*) - 2 ( x + y + z) - 1= 0.
Ex. 17. A point moves such that the sum o f the squares o f its
distances from a given number o f points is constantx Prove that the
locus o f this point is a sphere and show that its centre is the centroid
o f the given points.
SoL Let (xr, jVt zr), r = l , 2, n be the co-ordinates of n
given point. Let (x, y y z) be the moving point. It is given that the
sum of the squares of the distances of (x, y, z) from n given points
(Xr, yr, zr) is constant, say k r so that we have
Σ { ( x - jir y + ( y - y ,) * + (z zf)2} = k
or n (x^+j^-f-z2) - 2 x Σ xr —2y Z y 9 - 2 z Σ zT
+ Σ χ92+ Σ yr2+ E z 2- k = 0.
Dividing by n , the locus of (x, y, z) is given by
^ + K - 2 * ( % ) ■ -» (% -)-* ( % )
+ ( Γ χ ,2- \ - Σ ) > τ 2-\γ Σ ζ τ 2— fc) == °
which is clearly the equation of a sphere the co-ordinates of whose
Σχ Σν Σζ \
( - r-, ~ I and evidently the centre is the cent­
roid of the given points (xr, y„ zr), r = l , 2, . ,/i. Hence proved.
Ex. 18. A point moves so that the sum o f the squares o f its
distances from the six faces o f a cube is constant. Provethat its
locus is a sphere. (Meerut 1979)
Sol. Consider a cube with the vertex O as origin aud edges
OA, OB and OC as co-ordinates axes. Let the length of each
edge be a .
The six faces of the cube OBDCy OAEC9 OBFA9 AFHE9
BFHDy DHEC and their equations are
x= 0 . y « 0 , z=0, x = a y y = a 9 z= a respectively.
Let P (α, p, γ) be the moving point. It is given that the sum
of the squares of the distances of P (α, β 9γ) from the six faces of
the cube is constant, say k y so that we have
256 Analytical Geometry 3-D

or 2 (0 ^+ /^ + ^ )-2 * ! (α+β>+7)+(3α*—fc)=0.
Λ The locus of P (α, β %γ) is
2 (xH-J8+**)—2o (x+ y+ z)+ (3 a * -k)= 0
which is clearly the equation of a sphere.
Ex. 19. OA9 OB9 OC are three mutually perpendicular lines
through the origin and their direction cosines are I19 ml9 nx ; /* ml9 nt \
l%9 mlf /I1. IfO A=O 9 O B= b9 OC=C9 prove that the equation o f
the sphere OABC is
x*+^*+r2—x (a^+bU+cli) -y (am ^bm ^+ cm ^
z ( Ai i 1 + Α · ? * + ~ 0 .

SoL The co-ordinates of the points A, B and C are


(0, 0, 0), (all9 aml9 OO1), (A/f, A///„ bn%) and (c/a, Cm99 cn9) respec­
tively.
Since the sphere passes through- the origin O9 its equation
can be put as
x*+y*+z* f 2ux+2vy+2wz^i). -- ( »)
Substituting the co-ordinates of A (I), we get
*i* ( / i * 4 /W12+ H 12) + 2 a (i//j + V m 1+ W n 1) = 0
o r *!+ 2 i i /1 + 2 v >w 1+ 2 h 7 i1= 0 . - .- ( 2 )

[ Y I f+ m S + n f = ^
Similarly substituting the co-ordinates of B and C in (I), we
get
b + 2 m/ , + 2 vwr, + 2 m7i , -= 0 , -(3 )
and c i- 2i /4 i- 2vm%+ 2m//3=0. - (4)
The Sphere 257

Now I1, Ii, Ia ; mx, mt, mt ; nt, nt, na are the d.c.’s of OX, OY.
OZ respectively with reference to the three mutually perpendicular
given lines.
Λ Λ*+ /»*+ /**= W1*+ m**+ m»*= «i*+ «**+ = I.
and I1My^mlatrtt“\~ΙζΜ$~
Multiplying (2), (3) and (4) by I1, I9 and I9 respectively,
adding and using the above relations, we get
οΙχ-\· blf\-cl9-\· Iu=O or 2u = —(ali~\~ bla~\~da) ■
Similarly multiplying (2), (3) and (4) by mu m% and mlt and
then by nu nt and n3> and proceeding as above, we get
2v— — (ami+bnta+cma), Iw = — (e » i+ b « i+ c n *).

Putting the values of 2«, 2v and 2w in (I), we get the equation


of the sphere as given.
Ex. 20. A plane passes through a fixed point (a, b, c), show
that the locus of the foot of the perpendicular to it from the origin is
the sphere OABC
xt + yi Arzi —ax—b y~ cz= 0.
Sol. The equation of aoy plane through (a, b, c) is
I (x —a)+m (y —b)+n ( z - c)=0. .. (I)
The equations of any line through the origin and perpendi­
cular to the plane (I) ie . the equations of any line through the
origin and parallel to the normal of the plane (I) whose d.r.’s are
/, m, n are given by
X/1=y Im=Zfn. ...(2)
The line (2) intersects the plane (I) in the foot of the perpen­
dicular and hence the locus of tbe foot of the perpendicular is
obtained by eliminating /, m, n between (I) and (2). Therefore
the required locus is given by
x (x—a)-\~y (y ~ b )+ z (z—c) = 0
or x*+y* + z* - ax by—cz= 0
which is the equation of the sphere OABC. [See Ex. 6 (a) above].
§ 3. Plane section of a sphere. To prove th t the section
of a sphere by a plane is a circle and to find its raditis and >
''ie
centre.
238 Analytical Geometry 3-D

Let us consider tbe section of


a sphere by a plane and let P be any
point on this section. Let O be the
centre of the sphere. From O draw
OC perpendicular to the plane so
that C is the foot of the perpendi­
cular and is a fixed point. Join C
to P. Clearly CP is a line in the plane and hence CP is perpendi­
cular to OC. (Note that OC is perpendicular ^to the plane and
therefore, OC is perpendicular to every line, lying in the plane).
Thus we have
OP'^OCP+CP' or C P= V(O P2-O C t). ..(I)
But OP= the radius of the sphere and is constant
and OC=the length of the perpendicular from the centre
of the given sphere to the given plane and hence
is constant.
Thus OP and OC are constant and hence from (I) it follows
that CP is constant for all positions of the point P and hence the
locus o f the point P is a circle with centre C and radius CP9 which Is
given by (I).
Therefore9 if a sphere is cut by a plane then the plane section of
the sphere is a circle.
Theeqoations of a circle. Let the equation of the sphere be
Xt + y%+ z2+ 2ux+ 2vy + 2wz+d=0 .. .(2)
and the equation of a plane be lx+m y+ nz=p. ...(3)
We have shown above that the section of a sphere by a plane
is a circle. Hence the equations (2) and (3) together represent the
equations of a circle.
Great Circle. A great circle is the section o f a sphere by a
plane passing through the centre o f the sphere. Clearly the centre
and the radius of a great circle are the same as those of the
sphere.
§ 4. Iolersection of two spheres.
To prove that the curve o f intersection of two sphere Is a circle.
Let the equations of the two spheres be
Si&*xt + yt + zt +2ulx+2vly+2wiZ+di=0
and S1=Xa+ y%+Zt + 2u2x +2 v% y +2 w% z+ 0.
The Sphere 259

The equation of the plane which contains all the common


points of the spheres SY=O and 5»--=0 is given by
S1- S t= 2 (Ui-M8) * + 2 (v, —v8) y+ 2 (W1-W t) z+ d t—dt —0.
Thus the points of intersection of the two spheres are the
same as those of the intersection of any one of the two spheres
and the plane S1- S8=O and hence these points lie on a circle.
Hence the curve o f Intersection of the two spheres is a circle,
and the two equations o f the spheres together represent the equations
of a circle.
§ 5. The system of spheres through a .given circle.
Let the equations of a circle be given by
Saixt + yi +z*+2ux+2vy+2wz+d=0 ...(I)
and P = lx+ m y+ n z—p<=0. ...(2)
Consider the equation S+AP=O ...(3)
or x*+y*+z*+2!ux+2vy+2wz+d+X(lx+my+nz—p )= 0
where A is a constant. The equation S-I-AP=O for all values of A
is satisfied by all those points which satisfy both S=O and P=O
l.e. which lie on the circle S = O = P .
Also the equation S + AP=O is of second degree in which the
coefficients of x*, yx, z* are equal and the terms contpinipg yz, zx.
xy are absent.
Rere the equation S+AP=O represents a system of spheres
passing through the given circle whose equations are given by (I)
and (2).
Similarlythe equation S i+ AS8=O represents a system of
spheres through the circle of intersection of the two spheres
S8=O and S8=O.
Remark. The value of A in § 5 above is determined by an
additional given condition.
SOLVED EXAMPLES (B)
Ex. I. Find the radius o f the circle given by the equations
3x*+3y*+3z*+ * —5y —2 = 0 , x+y<= 2. (Calcutta 1973, 81)
Sol. The equations of the given circle are
* * + y * + z * + J * - |y - i= 0 ...(D
and *+y=2. .(2 )
The centre of the sphere (I) is the point O ( —«, I, 0)
and radius O P = y/{(—$)*+($)*+(0)1—( —£>} [See figure of § 3J
or O P = 5/(3^2).
260 Analytical Geometry 3-D

Now 0 C = th e length of the perpendicular from the centre


O (—-J, f, 0) to the plane (2)
- 1 /6 + 5 /6 -2 4 ......................'
~ V(O)W ID·) - 3 V T
A The radius of the circle—C/*·» V (O P t - O C t)

" « / ( I* _ V2
Ex. 2. FiW the radius and centre of the circle o f intersection of
the sphere x* +y*+ z*—2y - 4z -= 11 and the plane x + 2y + 2z = 15.
/Agra 1976; BundeIkhand 78; Madras 74; Meerut 84 S, 85 (S)
Sol. The equations of the giver, ciicle are
x * + j> * + z * -2 y ~ 4 z -l!= 0 . ...(I)
aod x + 2 y + 2 z —15=0. ...(2)
See figure of § 3 above. The centre of the sphere (I) is the
point O (0, I, 2) and its radius
O/*= t/{(0)*+( I )*+ (2)*- ( —I >)}=4-
Now OC—t he length of the perpendicular from the centre
O (0. I, 2) to the plane (2)
0 4 - ? . J+ 2 7—I s
3 (numerically).
A T heradiusofthecircle
--=CP= V iO P i - OCi) = V ( I *·-9) -- V7 ·
To find the co ordinates of the centre C of the circle. The line
OC being perpendicular to the plane (2), is parallel to the normal
of the plane (2) and hence d.r.’' vi the line OC are I, 2 2.
Λ The equations of the lire OC the equations of the
line passing through O (0, I. 2) and having d.r.’s I, 2, I arc
x —0 y - z-2
r (say).
1 "~'~2 2 -(3 )
Any point of (3) is (r. 2r j-l, 2r+ i) If this is the foot of
the perpendicular (i.e. the centre C of the circle) then it must lie
on the plane (2), so that we have
r + 2 (2 r+ l) + 2 (2 r+ 2 )—15—0 c r r = J .
Putting the value of r, the co-ordinates of the centre C of the
circle are (I, 3,-4)..
Ex. 3. I f r is the radius of ike circle
x* A-Pt -^ zi -Trlux y 2 v y - \- 2 * : - \- d -0, I x + m y + n z '= 0,
prove that
(r*+J) (/* -j- niJ + rt*) =.(wu> —,,r)1+ (iiu— ^ -f (/r—m uf.
(Rohiikhttnd 1982; Delhi 75/
The Sphete 261

Sol. Tbs centre of the given sphere is the point


O (—u, —v, —iv) and its radius {?/>= v'( m* + v* + m'2—</).
[See figure of § 3]
Now OC=Uic length of the perpendicular from the centre
O ( —u, —v, —tv) to the plane /jc+m,y+«z=0
_ l (—u)+m ( —v)+n lu+mv+nw
^ (Is-Tm3-Frit) V(f*+iw*-t-n*)
(numerically).
The radius r of the circle is given by
(lu 4 m v + m v ) *
r> = 0 /,*-O C*=(«*+v, + iv*-d)-
or (r*+d) (Ι*+/η*+π2)= (ΐ/2+ ν 2+»ν2) (/*+m2+fi*)
— ( / u + IH V + IJIV )2

or (r*+d) (/*+m2+n*)=(mw—ην)*+(πΜ—/ ^ ‘ + (/v —/nu)*,


using Lagrange’s identity.
Ex. 4. Find the equation of the sphere through the circle
Jt*+?*+z***9, x+,y—2 z+ 4 = 0
and the origin. (Lucknow 1978)
Sol. The equation of any sphere through the given circle is
(**+.y, +2*—9)+ λ (x + y —2 r+ 4 )= 0 . ...(I)
If the sphere (I) also passes through the origin (0, 0, 0), then
we have (0 + 0 + 0 -9 > + A ( 0 + 0 - 0 + 4 ) = 0 or Λ=-9/4.
Putting the value of Λ in (I), the equation o f the required
sphere is given by
**+.T1+z*—9+ (9/4) (x + y —2 z+ 4 )= 0
or 4 (x*+y*+z*)+9x+9y-liz=>0.
Ex. 5 (a). Find the equation of the sphere which passes through
the point (α, β, y) and the circle x*+y*^a*t z= 0.
(Agra 1980; Gorakhpur 82; Meernt 71, 83.85P;
Kanpur 77, 81; Punjab 78)
Sol. The equations of the given circle are
jc*+y*=o*, 2=0.
The above equations ot the circle may be rewritten by intro­
ducing the term z* as follows
Jt2+y*+z2= e 2, 2=0. ...(I)
The equation of any sphere through the circle (I) is
Xi -Fyi -Fzi - β*+λζ=0. ...(2)
If the sphere (2) passes through the point (ά, β, y), we have
*2+/3, + y '—e*+Ay«=0 or A =—(ot*+/3*+y2—e*)/y.
262 Analytical Geometry 3-D

Putting this value of Λ in (2), tlie equation of tbe required


sphere is given by
(**+y*+z*—a1) y —(a*+j8*+y*—a*) z= 0.
Ex. 5 (b). Find the equation of the sphere through the circle
x*+y*=a*, z= 0 and through the centre of the sphere
(,χ -Λ )* + \ γ -β ΐ+ { ζ-γ)* = τ* . (Nagpur 1977)
Sol. The co-ordinates of the centre of the given sphere are
(α, β , y). Hence it is tbe same problem as Ex. 5 (a) above.
Ex. 6. A circle, centre (2, 3, 0) and radius I. is drawn in the
plane 2=0. Findthe equationofthesphere which passes through
this circle and the point (I, 1,1). (Lucknow 1979)
Sol. T heequationsofthe circle of radius I, in the z—O
plane and with centre (2, 3, 0) are given by
(* -2 )* + G — 3 )* = l» ,z= 0 .
These on introducing the term of z* can be rewritten as
(X -2 )» + (y -3 )* + z » = l,z = 0 . (I)
The equation of any sphere through the circle (I) is
(x —2)*+(y—3)*+z*—1+Az=0. ...(2)
If (2) passes through (I, I, I), we have A =—5.
Putting the value of A in (2), the equation of the required
sphere is
(x -2 )» + (y -3 )* + z * -1 —Sz=O
;or x, +y, + z2—4x—6y—5 z + 12=0.
Ex. 7. Find the equations o f the spheres through the circle
x*+y*+z*= I , 2x+4y+5z - 6 and touching the plane Z = 0.
(Avadh 1979; Meerut 77, 81)
Sol. The equations of the given circle are
x, + y*+ z, - l = 0 , 2 x + 4 y + 5 z—6=0. ...(I)
T heequationofany sphere through the given circle (I) is
given by
(x, +y, +z^ -l)+ A (2 x + 4 y + 5 z—6)=0
or x, +y*+ z, +2Ax+4Ay+5Az-(6A + l)= 0 . ---(2)
The centre of (2) is ( —A, —2A, —}A) and radius
= ·(^ λ * + 4 Α * + ^ + ( 6 λ + 1) } = |ν '( 4 5 λ * + 2 4 λ + 4 ) .
Now if the sphere (2) touches the plane z= 0, then we have
the length of the perpendicular trom the centre ( —A, —2A, —*A)
to the plane (z= 0)= th e radius of tbe sphere (2) Le.,
(- ^ 7 ^ = έ ν ( 4 5λ,+24λ+4).
The Sphere 263

Squaring, we have 25A*=45A*+24A+4


or 20A»+24A+4=0, or 5A*+6A+1=0, or A = - I , - J .
Sincew ehavetw ovalues of λ and so we shall have two
spheres satisfying the given conditions. Now substituting these*
values of Λ one by one in (2), the equations of the required spheres
are given by
x, + yt + z, - 2 x —4 .y -5 z+ 5 = 0
and 5 ( χ * + ) > * + 2 * ) — 2 χ — 4)>— 5 ζ + 1= 0.
Ex. 8. Find the equations of the spheres which pass through
the circle x*+.y, + z , = 5 , x+ 2 y+ 3 z= 3 and touch the plane
4x+ 3 y= lS .
(Agra 1981; Kanpur 79; Lucknow 82; Nagpur 78;
Bundelkhand 78; Kurukshetra 76)
Sol. The equation of any sphere through the given circle is
x*+>*+2*—5+A (x + 2 y + 3 z—3>—O
or xt + y, + zi + hx+ 2Xy+ 3\z—(3A+5)=0. ...(I)
The centre of the sphere (I) is ( —JA, —A, —}A) and its radius
‘=V {(-*A )*+(-A )i +(-fA)*+(3Af5)}-*V(14A*+12A+20).
Now if the sphere (Γ) touches the plane
4 ^ + 3 ^ -1 5 = 0 , ...(2)
then we have the length of the perpendicular from the centre of
the sphere (I) to the plane (2)=the radius of the sphere (I)
or 4 ( JA)+3 ( —A) 15 , / / u \ i i , ί ι i
------V W + ( 3 7 j ------- iV ( 4A*+ 12A+ 20)
or 2 (-5 A -1 5 )= 5 v '(l4 A i +12A+20).
Squaring, 100 (A*+6A+9)=25 (l4A*+12A+20)
or 5A*—6A—8 = 0 o r A=2, —4/5.
Substituting these values of A one by one in (I), the equations
of the required spheres are given by
x* +y*+ z*+ 2x+ 4y+ 6z— 11 —0
and 5 (x*+^*+z*)—4x—8 ^ - 12z—13=0.
Ex. 9. Prove that the circles
x*+>*+ z*—2x+ 3y+ 4z—5=0, 5.y+6z+1=0;
xt + y i +z*—3x—4^+5z—6= 0, x + 2 y —7z=0
lie on the same sphere and find Its equation. Also find the value 'a'
for which x + y + z = a \ / 3 touches the sphere.
. (Meernt 1982S; Rohilkhand 81; Garhwal 82; Kanpnr 78;
Madras.77; PnnJah 78)
264 AnalyticalOeometry 3-D
Sol. The equations of any spheres through the given circles
are respectively given by
X*+,?*+**-2 x+ 3 y+ 4 z—3+At (5y+ 6z+ 1)=0, ...(I)
and x*+>*+**—3*—4y+ 5z—6+Af (x+ 2y —7z)=0. .(2 )
Now if the given circles lie on the same sphere, then the
equations (I) and (2), for some values of A1 and At, must represent
the same sphere. Hence comparing the coefficients of x, jy z and
constant terms in the equations (I) and (2), we get
-2 = -3 + A f ...(3); 3+5A»— 4 + 2A,; ...(4)
4+6A1= 5 -7 A , ...(5); - 5 + A ,= —6. -.(6).
Solving (3) and (6), we get A1= - I , Af= I .
Clearly the equations (4) and (3) are satisfied by these values
of A1 and Af and hence the given circles lie on the same sphere*
PuttingA1= - I in (I) [or Af = I in (2)], the equation of the
required sphere is given by
x * + /1+**—2x—2.y —2z—6=0. -(7 )
The centre of the sphere (7) is (I, I, I) and its radius=3.
The equation of the given plane is
X+y+z<=Oy/5. -(8 )
The plane (S) will touch the sphere <7), if the length of the
perpendicular from the centre (1 ,1 ,1 ) to the plane (8 )= ± th e
radius of the sphere (7).
. I + l + l-^flV 3____ . . /iit
'* -^ 1 (1 ^ + (1 ^ + (1 )* } ^ 3 orfl,= v^ ± 3 · An·.
Ex. 10. Prove that the sphere
S |= x 2+ > , + z , +2w1jc+2v1ji+2w,1z+ i/1= 0 cuts the sphere
S1=Xt +,y*+ za+ 2i/4x+ 2V1Ji+ 2W1Z+ J 1= 0 in a great circle if
2 ( « t , + " » , + f » , | - < * t = 2 ( U 1M1 + V 1V1 + w , H - t )—dt
or i f 2 ( « 1« t + v 1V t + H ' 1H ' t ) = 2 r t * + < / 1 + d t
w h e r e rf is the radius o f the second sphere S1= 0.

Sol. The equation of the plane through the circle of inter·


section of the given spheres S1=O and S1s O is S1- S 1ssO1
Le. 2 ( U 1 - K 1) * + 2 (V1-V 1) y+ 2 ( W 1 - W 1) z+Ji-<*»“ 0. . . . ( I )

If the sphere S1=O cuts the sphere S1=O in a great circle,


then the plane (I) will pass through'the centre ( —ut, -V 1, —w%)
of the sphere S1=O, so that we have
2 (U1-U 1) (—uf)+ 2 (V1-V 1) ( —vt)+ 2 (W1-W 1) (—wt)
+ J 1—J 1=O.
or 2 (u1*+v1, +w 1*)—d1= 2 (U1U1-I-V1V ^w 1W1) - J 1. ...(2)
The Sphere 265

Again r, is the radius of the sphere Sa*=0, hence


r%%=W2H v aM-VVa1- d%
or rSfe5*έ {2 (W1Wa-J-VlV1^-W1Wt) - d i+ d2} using (2)
or 2 (wlwa + vlv2+ w1w’a)^ 2 r t2+rfi+rft , .. (3)
The relatious (2) and (3; are the required conditions.
Ex. II. Find the equation of a sphere for which the circle
x* + y%4- 2%+ Ty—2z+ 2 0 , 2x + 3y+ 4z --8
is a great circle. [Garhwal 1981]
Sol. The equation of a sphere through the given circle is
(*l +>'*+z*+7j>- 2z+2)+A (2x+3j>+4z-8j«»0
or Λ1+ ^ , + ζ β+ 2λχ-|-(7+3λι Λ—2 (1—2Λ) z + 2 —8λ*=0. ...(I)
The centre of the sphere (I) is
( - λ , ^-i(7+3A) I - 2λ).
If the given circle is a great circie of the sphere (i), then the
centre of the sphere (I) will lie on the plane 2x+3)H -4z=8 and
hence we have
2 ( —A)-f 3 {—J (7 4-3A»+4 ( —2A)=8 or A= - I .
Putting this value of Λin (I;, the equation of the required
sphere is given by
a*+.?1+** ~ 2X+4.V—6z+10«=»0.
Ex. 12. Prove that the plane x + 2 y —z*=4 cuts the sphere
x t + yt +z*—x + z ~ 2 --0 In a circle of radius unity and find the
equation o f the sphere which has this circle for one o f its great
circles. [Gorakhpur 1975; Madras 76]
Sol. The equation of the given sphere is
**+.fM-22- - jc+ z --2 c=0 -(1 )
and the equation of the given plane is
x -f-2y—r·—4. ...(2)
Refer figure of § 3. The centre of the sphere (I) is the point
O (J. Of —i) and its radius 07*= V {(i)*+ 0+ (-$)*+ 2} = V(f)·
Now OC*= the length of the perpendicular from the centre O
to the plane (2)

ϊ )· num" l“ "y'
Λ The radius of the circle
= C P = V (O P t - O C t)
1=3Vt I —I) =*V d ) ■*I·
This proves the 1st part of the problem.
For the second part proceed exactly as in Ex. 11 above. The
equation of the required sphere is ^iven b>
166 Analytical Geometry 3-D

& + y9+z*—2 (x + y —z —1)=0.


Ex. 13. Find the equation o f the sphere passing through the
circles + 2*=9, x = 4 and y*+z9=36, x = l .
[ PonJab 1975]
Sol. The equations of the given circles are
y, + z * = 9 ,x - 4 ...(I) and y*-Hz*=36, x = l -. (2)
Introducing the terms of x1, these equations can be written as
* * + 3 ', + z * = 2 5 , x = 4 ...( 3 )
and * -l. ...(4)
We are to find the equation of a sphere passing through the
circles (3 ) and (4·), or in other words to find the equation of a
sphere so that the circles (3) and (4) lie on this sphere.
The equations of any spheres through the circles (3) and (4)
are respectively given by
*, + y , +2*-25+ A , (x - 4 ) = 0 ...(5)
and x*+ j*+ z*-37+ A J (x —1)=0. ...(6)
Now if the given circles lie on the same sphere, the equations
(5) and (6), for some values of A1 and As, must represent the same
sphere. Hence comparing the coefficients of x and constant terms
in equations (5) and (6), we get
Ai=As and -4A| —2 5 = —As- 37.
Solving, we get A1=A1= 4.
Putting the value of A1 in (5) [or that O f As in (6)], the requi­
red equation is
x*+3'*+2*+4x—41=0.
Ex. 14. Find the equations to the circle whose centre is (ots β,
y) and which lies on the sphere X1-Hy1-I-Z1=O1.
So). The equation of the given sphere is
X1-Kv1-Hz1=O1. .(1)
The centre of the sphere (I) is the point O (0, 0, 0) and the
centre of the circle is given to be the point C (α, β , γ).
.·. The d.r.’s of the line OC are a —0, β —0, y —0 or «, β, y.
The equation of the plane through C (α, β , y) and perpendi­
cular to OC is
« (x —a)+ 0 ( y - 0 ) + y (z,—y)=0
or az+ fly + y z—o^-H^-Hy1. ...(2)
The equations (I) and (2) are the required equations of the
circle.
The Sphere 267

Ex. 15. A variable plane is parallel to the given plane xja


+ ylb+ z/c^O and meets the axes in A, B, C respectively. Prove
that circle ABC lies on the cone
yz (blc+c/b)+zx (c/a+alc) +xy (a/b+b/a)=0.
(Agra 1975; Allahabad 76, 82; Bundelkband 79; Kanpnr 83;
Lucknow 76; Madras 77; Meerut 74, 89, 90, 90P; Rajasthan 78)
Sol. The equation of any plane parallel to tbe given plane
xja+ yjb+ zlc= 0 is xla+ylb+zlc=X. . (1)
The plane (I) meets the co-ordinate axes in tbe points A, B
and C whose co-ordinates are (aA, 0, 0), (0, bX, 0) and (0, 0, cX)
respectively. The equation of the sphere OABC is
•x* +y*+z* —aXx—bXy—c\z= 0
[See Ex. 6 (a) of Solved Examples (A)]
or x a+y* + z*—X (ax+by f cz)~ 0. ...(2)
The equations (I) and (2) together represent tbe circle ABC.
Eliminating the variable Λ between (I) and (2), tbe required
locus is given by.
x* A-y*+z*—(x/β+ y/b + zjc) (a x-fh y + cz)= 0
or yz (b/c+c/b)+zx (cja+alc)+xy (ajb+ bfa)^0.
Ex. 16. P is a variable point on a given line and A, Bt C ate
Its projections on the axes. Show that the sphere OABC passes
through a fixed circle. (Allahabad 1982; Kanpur 79; Lucknow 77)
Sol. Let the equations of a given line be
( x - <*)ll=(y—fi)'m = iz—y)jn—r (say). .(1 )
The co-ordinates of any point P on (I) are (Ir+x, mr+β,
nr+y). The projections of the point P on the axes are A, B and C
whose co-ordinates are (Ir + «, 0,0), (0, mr+β, 0) and (0, 0, nr+y)
respectively. The equation of the sphere OABC is
x*+y*+z, —(lr+x) x -(m r +β) y ~ (n r+ y ) z = 0 .
[See Ex. 6 (a) of Solved Examples (A)]
or x*+ya+ zt - x x —fiy—y z~ r (lx+ m y+ nz)= 0. . (2)
The sphere (2) is of the form S+ XP=O. This sphere for all
values of Λ passes through the fixed circle 5 = 0 , P=O
i.e. χ*+ρ*+ζί —χ χ ~ β γ —γζ—0, ix+my+nz*=*0.
Ex. 17. Find the equation o f the sphere through origin and
whose centre lies in positive octant, and which cuts the planes x = 0 ,
y = 0 , z = 0 In circles of radii ay/2, by/2, cy/2 respectively.
Sol. The equation of any sphere passing through tbe origin
is xa+ ya+ za+2ux+2vy+2wz=0. ...(I)
268 A n a lytica l G eo m etry 3-/>

The intersection of the sphere (i) with the plane x = 0 is


y* + z*+2vy+2wz—Q9 x « 0 . ...(2)
Equations (2) are the equations ot a circle lying in the yz-
plane. The radius of the circle (2) is clearly \Λ ν*+Η>*).
But we are given that the radius of the circle (2) is ay/2.
Hence we have
y/[y%+ w% )^Q y/2 or va+ Iva=^n1. ...(3)
Again since the sphere ( I ) meets the planes y«=0 and z~ 0 in
circles of radii by/2 and cy/2 respectively, we have proceeding as
above W2-Vu2-^lb g ...(4)
and n*+v*=2ca. ...(5)
Adding (3), (4) and (5), we have
2 (fia+ v a+W'2i= 2 (a*+b*+c*) o r ui + vi+wt ^ u 1-\-bt ^-c2 ...(6)
Subtracting (3), (4) and (5) from (6), we respectively have
u2^ b t + ct —a%t va= c 2+fl2—b29 u2^ a 2+ 6 2—c2
or u=±>y/(b*+c2—a*), v ^ ± \ / ( c 2+ a2—b2)f w = ± \/( a z+b2—c2).
Putting the values of n, vv w in (I), the equation of the
sphere is
Xt -Vy2+ z2± 2 y /Xb1^rC2- Qi) * ± 2 \/(c2+ a2—b2) y
± 2 y/(a 2+b2- c 2)z~ Q . ...(7)
It is given that the centre of the sphere lies in the positive
octant, hence the co-ordinates of the centre of the sphere are. all
positive. The co-ordinates of the centre of the sphere (I) are
(—ii, —vr -tv). Thereforctaking everywhere t h e —ve sign in
equation (7), the equation of the required sphere is given by
x *+ y*+ z* -2 y/(b 2+ c2- a 2) x - 2 y /( c 2+ *2- b 2} y
—I y/(a2+/;2—c2) z « 0.
Ex. 18. POP' ts a variable diameter of the ellipse 2=0,
X2Ia2-Ty2Ib2-- I and a circle is described in the plane PP zzt on PP9
os diameter, prove that as PP9varies, the circle generates the surface
(x*+y2+ z2) (X2/a2+ y2/b2) = a2+ y 2.
Sol 'Recall the analytical geometry of two dimensions.
The parametric co-ordinates of the extremities of any diameter of
the ellipse Jt2/a2+ j 2/62=»l are (a cos 0, b sin Θ) and ( —a cos 0,
—6 sin 0).
Here in the present problem (of three dimensions POP' is a
variable diameter of the ellipse X2Ia2-Vy2Ib2^ l 9 2=0. Hence the
co-ordinates of the extremities P and P 9 may be taken as (a cos 0,
b sin 0, 0) and (—a cos 0, —6 sin 0, 0) respectively.
The Sphere 269

The equation of the sphere on PP' as diameter is given by


(x—a cos Θ) (x + a cos 0)+O>—b sin 0) (y+ b sin Θ)
+ (z—0) (2- 0 )= 0 [See § 2 (E)]
or xt + yt + z2—dt cos1 0+6* sin2 0 - (1)
To find the equation of the plane PP'zz'.
The equationsofzz' t.e. z-axis are x^O , y>=0.
The equation of any plane through zz' is
* + \ y —0. ...(2)
If the plane (2) also passes through P(a cos 0, b sin 0, 0) then
ocos 0 + Ah sin 0=»O or A *= — (o cos 0)/(6 sin 0).
Putting the value of A in (2), we get
x - { ( a cos 0)/(6 sin 0)) ^ O or Q· (3)
The plane (3) clearly passes through
P t ( —a cos Θf —b sin Θ, 0).
Hence (3) is the equation of the plane ΡΡ*τζ\ Thus the
equations of the circle described in the plane PPtZit on PP9 as
diameter are given by the equations (I) and (3).
The required locus is obtained by eliminating the variable Θ
between the equations (I) and (3) From O). we have
<*M.JyLb) \ζν*Ι<*ν+(νΙι>Ά β \ Ί(χ!α)*±
cos ' VHcos2 β i
.. a cos j{{xla)*-\-{yjb)2}, b sin θ=>
Putting the vaHies of a cos Θand b sin Θin (: \ the required
locus is given by
.Jt2
{(*/<*)*+ ( y / b >*} x Ux j a f τ i y / b )*}
or (jca4ψ2+ ζ·, ( j t y + j ^ p x 1+ ·/·
Ex. \9. A is a point :;n OX and B on OY\ so that the angle
OAB is constant and equal to a. On AB os diameter a circle is
drawn whose plane is parallel to OZ. Prove that as AB varies, the
circle generates the cone Ixy -z* sin 2oc«=»G.
Sol. Since A is a pr int on OX, \vc may lake A as (a. 0, 0).
Similarly B being on UY may be taken as ;C\ b, 0) Given
/_OAB a, therefore from right angled trhngled AOBt we have
tan x —OBjOA=>bla. ...(I)
Ihe equation of toe spiiere on AB as diame.cer is
(.V- a* (x Oj-HiV-O) (y - b ) + ( z - 0) (Z--O)--O (See § 2 (E)J
or x l^-/2 + z*^ax-\-by. ...(2 )
270 Analytical Geometry 3-D

Also the equation of the plane passing through the points A


and B and parallel to OZ is
x/a + ylb — I. (4)
Thus the equations (2) and (4) are the equations of the circle
on AB as diameter and having its plane parallel to OZ.
The locus of this circle is obtained by eliminating the varia­
bles a and b between (I), (2) and (4).
From (2) and (4), we get
x t + y i + z, ^ (a x+ b y ) (x/a+y/b)
ot xt + y*+ zi’=‘xt +y*+xy (a/b+b/a)
or x* +y*+z*=x*+y*+xy (cot α+ tan a), using (I)
or z*—xy (cos a/sin «+sin a/cos a)
Of z* = .t)>/(sin « cos at
or sin a cos a z*=xy or 2x y - z * sin 2a=0,
which is the required locus.
Ex. 20. Find the equations o f the circumcircle of the triangle
ABC, whose vertices are A (a, O10), B (0, b, 0) and (0, 0, c). Find
also (I) the co-ordinates of its centre and (Il) its diameter.
(Meerut 1984)
Sol. The three given points are A (a, 0, 0,), B (0, b, 0) and
C (0, 0, c). Let the fourth point be O (0, 0, 0). The equation of
the sphere OABC is [See Ex. 6 (a), set (A)]
x*+y2+ z%—o x—by—cz=0. ...(I)
The equation of the plane ABC is given by
xla+ yjb+ zfc= ). ...(2)
The equations (I) and (2) together are the equations of the
circumcircle of the AdBC.
To 6nd the co-ordinates of the centre of the circle.
A line perpendicular to the plane (2) of the circle and passing
through the centre (Jo, ±b, £c) of the sphere (I) will pass through
thfc centre of the circle. The equations of this line are given by
x —l<*—y —i b _ z —ic
—r (say).
I fa I /b I /c ...(3)
Any point on the line (3) is
(la+ rla. lb+ rlb, lc+r/c). ...(4)
If this is the centre of the circle, then it will lie on the plane
(2) and hence, we have
The Sphere 271

-I
or Γ(ϊ*+ * + ? ) — 5 or r= 2 (a-*+b-*+c-*)
. a , r ^ja I _a* (a- , +b-*+<r*)—I
” 2 a = 2 2α (a-*+b-»+c-*) “ 2α (a-*+tr*+c-*)
_ a* (6-*+c-*) _ a (fe-«+c-«)
2a (a-»+l»-*+c-*)“ 2lo-*+fr-*+c-V ‘
Similarly - + —=> ^ (<τ*+α~*)
y 2 b 2 (a-*+6-*+c-*)
and c (a- *+&-*.)
T + · c 2-(e-*+ft-»+c-*)
Putting these values in (4), the co-ordinates of the centre of
the circle are given by
I g (b-*+c-*) b (c_,-fa_i) c (a - , -fb_2)
\2 (fl-*+h-»+c-»)' !(a -'+ b ^ + c -* ) * I (a-*+h~*+c-> )
To find the diameter of the circle. Let r be the radius of the
circle and R the radius of the sphere. If p be the length of the
perpendicular from the centre of the sphere to the plane (2), then
r*=R*—p*. ...(5)
Now /{«radius of the sphere (l)=$-v/(a ,+ £ * + c*)
and /!«the length of the perpendicular from ($a, $c) Le.,
the centre of the sphere (I) to the plane (2)
________________________________ I________
Vtl !/«>■+ (I /W+d/*)*} 2V {o-*+ b-*+ c-·)
Putting the values of R and p in (5), we get
i _ a 2+ i 2-fc2 I
' ~ 4 4 (a-*+hr··+ c-*)
a*^2C1 ___
or 4r2= a 2-f b*-f c2—
h*c*+c*a*+a*/>*
(b*+c*) (c»+a*) (a'+b*)
b»c*+c«a* +a*b%
The required diam eter«2r

-M f(h2+ C2Mc2-fa2) (a*+h*)l


(6*C*- f C1Vl2-fa*/>2) J
§ 6 (A). The intersection of a straight line and a sphere.
Let the equation of the sphere be <
x*+yt +z*+2ux+2vy+2wz+d=0, ...(I)
and that of the straight line be
(X -X i)II^ iy - y i) lm = ( z - z i) ln ^ r (say). ...(2)
The co-ordinates of any point on the line (2) are
(Ir+Xi, mr+yu n r+ r,). —(3)
272 Anilytlcal Geometry 3-D

Tf this point lies on the, sphere (I), we have


(Ir+ Xif+ im r+ yt)* + {nr+ zrf + lu (Ir + xx)+2v (m t+ yx)
+2w (nr+zx)+d=*0
or r* (/* + m*+n?) + 2r{/ (x ,4 w) + w (^ι + ν) + η (ζ, + w)}
+(X\2+ y\*+Zt* i 2i#x,+ 2 ^ , + 2 ^ , + 1/)^0. . (4)
The equation (4) is a quadratic in r and hence gives us two
values of r which'may be real and distinct, real and equal or
imaginary and, therefor. , accordingly the line (?) will meet the
sphere (I) in two points which may be real and distinct, real and
coincident or imaginary.
The co-ordinates of the points of intersection are obtained by
putting the values of r from (4) in (3).
(B) Power of a point. If a straight Line in any direction and
through a given point A meets the sphere in two points P and Q then
AP.AQ- is constant and is said to he the power of the point A.
Let the given point A be taken as (*,, y x%Z1) and let /. mt n
be the actual direction cosines of the line (2) through Λ so that
l*+m2+nl —l. The line (2) meets the sphere (I) in two point P
and Q and hence rx and r2. tee roots of the equation (4), are the
actual distances of the points P and Q from the point A
i.e. AP -T1 ai d AQ r,.
Λ A P .AQ /Va - * ,2 + V.2 i- Zi2 + 2uxt + 2v>·, + Iwzi + d ...(5)
or ΛΡ. AQ —cvnstant, being +dependent of I, m, n.
(C) To find the condition that the given line (2) may touch the
sphere (I).
The line (2) will touch the sphere (I) if the roots of the equa­
tion (4) are equal and so the required condition is given by
{/ (xx+ u)+m 0>i + v)+w (z\ + w)}f
^ ( l l + ml +nt)(xx2+ yxt + zxi + 2uxx+ 2vyi + lwzl +d). ...(6)
§ 7. The equation of the tangent plane.
(A) To find the equation of the tangent plane to the sphere
x t + y t + z%+7ux + 2vy+2wz+J=O at any point (xu yl9 z,) on it.
(Agra !973; Avadh 80; Madras 73; Kanpur 80, 82; Ranchi 77, 80)
The equation of any sphere is
x*+ y'+ zl + 2ux + 2vy+2wz+d—0. ...(I)
The equations of any line through (xi9 y u Zi) are
(x - X i)//=(y-yi)/m <=(z- zt)/n=r (say). ...(2)
The points of intersection of-the sphere (I) with the line (2)
are given by [See § 6 ( A), equation K). Give complete proof
berej
The Sphere 273

r' (i*+m*+n,)+ 2 r { / (* ,+ a )+ m (Ti + v) + « ( z , + h >)}


+ (*i8+ T i * + Zi8+ 2vx,+ 2vyl + 2m>z, + </)=0. ... (3)
The point (X1, Vl l z,) lies on the sphere (I), and so we have
Jfi8+ Ti*+ Z18+ 2ux, + 2vvj+ 2ivzi + d —0. .. .(4)
Using (4), the equation (3) reduces to
T* (/8+>n8+ n ,)4-2r{/(x1+ a)+OT (Ti + v)+« ^ + ^ ) ) = 0 . ...(5)
From (5), we get r—0
and r = —2 {/ (xt + u )+ /n (Ti + v)+ « (Zi 4 H’)}/(/, +/n*+n8). ...(6)
Now one root of the equation (5) is zero i.e., one value of r
given by (3) is zero. But r is the distance from the point (xlt Ti. Zi)
of a point of intersection of the sphere (t) with the line (2) and
so one point of intersection coincides with (X|, Ti. Zi). Therefore»
if the line (2) is a tangent line to the sphere (I), the other root of
(S) [i.e. the value of r given by (6)) must also be zero and for this
we have
/ (X i+u)+m (T i+v)+« (Zi + w)= 0 . ...(7)
Hence the line (2) is a tangent line to the sphere (I) at the
point (xt. Ti. Zi) on it if the direction cosines /, m, n of the line
(2) satisfy the condition (7). The locus o f all such tangent llnesfor
different values o f I, m, n is called the tangent plane at the point
(Xi, Ti. Zi) to the sphere ( I).
Therefore, the equation of the tangent plane at (Xi, Ti. zt) to
the sphere (I) is obtained by eliminating /, m, n between (2) and
(7), and is given by
(X-X1) (x,+ «)+ (T-T i) (Ti + v) + (z -z i) (Zl + μΟ·=0
or x x i+ x u +TTi+Tv+ZZt +zw
/ - ( - X 18 + T i 8 + Z |8 + UX1 + VT1 + WZi) = 0 '
or xx1+yTi4-zz,+ux+vT+Mz+(uxi+VTi+H'Zi+</)“ 0
fusing (4))
or XXi+ T T i +ZZi+U (χ + χ ,) + ν (t +T i ) + w (z+Zi)+<+=0. .. (8)
(B) To find the equation o f the tangent plane to the sphere
*8+T8+ z 8=a* at the point (x,, Ti, Ti) on it.
(Kanpur 1976: Madras 77; Gnru Nanak 75)
This is a particular case of § 7 (A). Proceeding exactly as in
§ 7 (A), the equation of the tangent plane at (X1. Ti. Z1) to the
sphere χ*+τ8+ ζ8= ο 8 is given by xxi +TTi+ z z i= a 8·
Rule. The equation of the tangent plane at any point
(•*1, Ti, Zi) to a sphere can conveniently be written by replacing
x* by Xx1, T8 by TTi· z* by zz„ 2x by x+Xi, 2t by τ +Ti. 2z by
z+zi and retaining the constant term.
274 Analytical Geometry 3-D

(C) The tangent line at any point of a sphere Is perpendicular


to the radius through that point.
Let the equation of the sphere be
&+y*+&+2ux+2vy+2wz+d=>Q. ...(I)
Let the equations of any line through the point (xt, yu Zi) on
the sphere (I) be
x —x, y —y, z-z,
I W e U ...(2)
. The line (2) will be a tangent line to the sphere (I) at the
point (xi, y u Zi) on it if [See equation (7), § 7 (A)I
I (xi+ «)+ m (y i+ v)+ « (γ, + μ»)—0. ...(3)
The co-ordinates of the centre of the sphere (I) are ( —u, —r,
—w) and so the direction ratios of the radius through the point
f*i. Pu Zi) are X1- (—«), y x - ( - » ) , Z1- ( —w) i.e. are xt +u, y t +v,
Zi+w. Hence the condition (3) shows that the tangent line (2) at
.the point (xt, y t, Z1) is perpendicular to the radius through the
point (x„ y u Z1).
(D) Thetangentplaneat any point is perpendicular to the
radius through that point.
The equation of the tangent plane at any point (X1, y u Z1) of
the sphere
xt + p + z* + 2 u x + 2 vy + 2 w z+ d = 0 is given, by
x x i+ y y t+ zzt+ u (x+ X |)+ r (y + y t)+ w ( z + z ^ + d - 0
or * (* t+ u )+ y (yi+v)-|-z (z,+ w )+ ux,+Vy1-Hwz1-Hd=O. ...(I)
The d.r.’s of the normal to the tangent plane (I) are X1-I-M,
y t+ r, Z1+ w.. But these are also the d.r.’e of the radius through
the point (xl t yu Z1) (See § 7 (C) above].
This shows that the normal to the tangent plane (I) is parallel
to the radiiis through the point (x,, y u z j and h4nce the tangent
plane at any point (Xuy1. Z1) of the sphere is perpendicular to the
radius through the point (xi, y lt Z1).
(E) Condition of taageocy. To find the condition that the
plane lx+ m y+n z -.- p touches the sphere.
x*+yt + z t + 2ux+ 2vy+2w z+ d=0.
(Calcutta 1981; Gorakhpur 82; Madras 77; Kanpur 78)
The equation of the given sphere is
x, + y l + z*+ 2ux+2vy+2w z+d=‘0. ...(I)
The equation of the given plane is lx+my+nz-=p. ...(2)
The Sphere 275

If the plane (2) touches the given sphere (I), then the length
of the perpendicular from the centre ( —u, —v, —h’) of the sphere
(I) to the plane (2 )= the radius ■v/(a*+v*+*·'·—</)of the sphere (I)
l ( —u)+m (.—»)+ n ( - w ) - p
Le., y/(u*+ V2+ a»·—d ).
V (/x+»n*+n*)
Squaring, the required condition is given by
(lu+mv+nw+p)*=^*+v*-\-w*—d)
Corollary. The condition that the plane lx+m y+nz=p
touches the sphere x2+ j*+ z*= e* is given by
p*=e* (Proceed as in § 7 (E) above].
SOLVED EXAMPLES (C)
Ex. I. (a). Find the equation of the sphere described on the line
joining A (2, —1, 4) and B (—2, 2, —2) as diameter and find also
the equation of the tangent plane at A.
Sol. The equation of the sphere described onl the line
joining the points A (2, —1, 4) and B (—2, 2, —2) as diameter
is given by
( * - 2 ) {*—(—2)}+{y—(—1)} O’—2)+ (z—4) ( z - ( - 2 ) } = 0
or *, +.)’, +z*—y —2z—14=0. Ans.
The equationofthe tangent plane at A (2, —1,4) of the
above sphere is given by
* 2 + ^ .( - 1 ) + 2 .4 - * O'- Ι ) —I .(2 + 4 )-1 4 = 0 (See § 7 (A)]
or 4 x —3^ + 62—3 5 = 0 a u b .

Remark. The tangent plane at A can also be found by writing


the equation of the plane passing through A and perpendicular
to AB.
Ex. I. (b). Find the equation of the sphere described on the line
joining the points (3, 4, I) and ( —1, 0 ,5 ) as diameter andfind also
the equation of the tdngent plane at ( —1, 0, 5). (Madras 1976)
Sol. Proceeding as in Ex. I (a) above we have the required
equations as
x t +y*+z*—2x—4y—6z+2=C, x + y —z + 6 = 0 .
Ex. 2. (*). Find the equations of the tdngent planes of the sphere
'xt + y t +z^+2x—4y+6z—T—0 which intersect in the line
6 x - 3 y -2 3 = 0 = 3 2 + 2 . (Rohilkhand 1977)
Sol. The equation of the given sphere is
^ + 3 ^ + ^ + 2 * '- 4,y+6z—7=0. ...(I)
The equation of any plane through the given line
6x—3 y -2 3 = 0 = 3 z + 2
276 Analytical Geometry 3-D

is 6 x -3 .y -2 3 + A (32+2)=0
or 6x—3.y+3Az+(2A—23)=0. ...(2)
If the plane (2) is the tangent plane to the sphere (I), then
the length of the perpendicular from the centre (—I, 2, —3) of
the sphere (I) to the plane (2 )= the radius of the sphere (i)
i e 6 ( - 1 ) —3.2+3A (— 3)+2A —23 m \w , / ι _τ»\
i e · --------V{(6)*+(3)*+ ( 3A)*T-------- ν { ( Ι ) · + ( - 2 ) + ( 3 ) - ( - 7 ) }
or - 7 A - 3 5 = v /(21)v/(45+9A»).
Squaring and simplifying, we get 2A*—7A—4 = 0
or 2A*—8A+A—4 = 0 or A=4, —$.
Putting the values of Ain (2), the required equations of the
tangent planes are given by
2x —y +42—5=0 and 4x—2y—z—16=0.
Ex. 2. (b). Find the equations o f the tangent planes to the sphere
**+y*+z*+6x—2 z + l= 0 which pass through the line
3 (16—x)=3z«=»2y+30. (Punjab 1977)
Sol. The equations of the given line may be re-written as
3 (16—x )—3z=0=2_y—3z+30.
Now proceed as in Ex. 2 (a) above, the equations of the tan­
gent planes are given by
x + 2 y —2z+ 14=0 and 2x+ 2y—z= 2.
Ex. 3. Find the’equatlons of the tangent planes to the sphere
x*-\-yi +z* ==24 which are parallel to the plane 2 x + y —z=*0.
Sol. The equation of the given sphere is
*, +3’*+z, =24. ...(I)
Let the equation of the tangent plane parallel to the plane
2x-f y —z - 0 be 2x+.y—z+A=0. . (2)
If the plane (3) touches the sphere (I), then the length of the
perpendicular from the centre (0, 0, 0) of the sphere (I) to the
plane (2)= ± th e radius of the sphere (2) 1
. __ 2 .0 + 0 —0+A __ .
■’ V{(2)* + ( !)*+( —I )*} - ± V U '
or A= ± V (2 4 ) . v '6 = ± ( 2 .
Putting the values of A in (2), the equations of the required
tangent planes are given by
2 x + ^ -z ± 1 2 = 0 .
Ex. 4. (a). Show that the plane 2x—2_y+z+12=0 touches the
sphere x t + yt + zt —2 x —4y+ 2z—3 = 0 and find the point of contact.
(Avadb 1981; Agra 82; Berahmpor 81; Gaohati 77;
The Sphere 277

Sol. The equation of the given sphere is


x*+y* + z*—2x—4y4-2z—3= 0. ...(I)
Its centre C is (I, 2, —I) and its radius
= V { ( - l) * + ( - 2 ) » + ( l)* - ( - 3 ) } ~ 3 .
The equation of the given plane is
2x—2y+z+\2=>0. ...(2)
The length of the perpendicular from the centre C (I, 2, —I)
2 . 1 - 2 . 2 - 14-12 9
to the plane (2)=
V {(2)*4-(-^+(T)*}
= the radius of the sphere (I).
Since the length of the perpeudicular from the centre
C (1, 2, —I) of the sphere (I) to the plane (2) is equal to the
radius of the sphere, therefore, (he plane (2) touches the given
sphere (I).
To find the point of contact. The d.r.’s of the normal to the
tangent plane (2) are 2, —2, I, and. therefore, the d.r.’s of the
radius CP are 2 , - 2 , I. [See § 7 (D)].
Thus the equations of the line (radius) CP are
(■*—1)/2--=(^-2)/(- 2 )= (z+ \)ji —r (say). ..(3 )
The co-ordinates of any point on (3) are (2 r+ lt —2r-f-2, r— I).
If this is the point of contact P, then it will lie on the tangent
plane (2) and so we have
2 (2r-|-1 )-2 ( - 2 r + 2 ) + ( r - 1)4-12=0 or r —- I .
Putting the value of r, the co-ordinates of the point of contact
P are ( 2 ( -1)4-1, - 2 ( - 1 ) 4 - 2 , - 1 - 1 ) or ( - 1. 4 , - 2 ) .
Ex. 4. (b). Show that the plane 2 x + y —z= 12 touches the sphere
x 2+ y * + z'^ 24 and find its point of contact.
Sol. Proceeding as in Ex. 4 (a) above the point of contact
is (4, 2, -2).
Ex. 5. (a). Find the equation of the sphere which touches the
sphere x*+y*+z2-\-2x—6 y + 1«=0 at ( I, 2, —2) and passes through
the point ( I, —1, 0). (Indore 1979; Meerut 82)
Sol. The equation of the given sphere is
xi + y i + zi + 2 x—6y4-1=0. ...(I)
T heequationofthetangentplane to the sphere (I) at the
point (I, 2, —2) is
x .l+ y .2 4 - z ( - 2 ) 4 - ( J c 4 - l) - 3 ( y + 2 ) + l- 0
or 2x—y -2 z -4 = * 0 . (2)
Therefore, the equation of the sphere which touches the
sphere (I) at (I, 2, —2) is given by
278 Analytical Geometry 3-D

^ + ^ + ^ + 2 ^ -^ 4 1 + ^ (2 ^ -^ -2 2 -4 )= 0 . ...(3)
If (3) passes through the point (I, —I, 0), then we have
1+ 1 + 0 + 2 + 6 + 1 +A ( 2 + 1 - 0 - 4 ) = 0 or A = Il.
Putting the value of λ in (3), 'the equation of the required
sphere is given by _\ ' ^ '
* * + ^ + z * + 2 4 x - l7 V - 2 2 r - 4 3 = 0 .
Ex. S (b). Prove that tfte equation of the sphere which touches
4 (x*+y*+2*)+1O x - 25y - 2z= 0 at (I, 2, - 2 ) and passes through
the point (—1, 0 ,0 ) is \'Λ \
x*+y*+z?+'^x—6 y + l= 0 . (Meerut 1978)
Sol. Proceed as in Ex. 5(a) above.
Ex. 5 (c). Find the equation o f the sphere which touches the
sphere x*+y, + r* + 2 x —6>>+l=0 at (I, 2, —2) and passes through
the oHgin.
Sol. Proceed upto equation (3) of Ex. 5 (a) above.
If (3) passes through (0, 0, 0), then we have
0 + 0 + 0 + 0 - 0 + 1+A ( 0 - 0 - 0 - 4 ) = 0 oj"VA= 1/4.
Putting the value of A in (3), the equation of the required
sphere is given by n^
(λ', + Ρ , + ζ* + 2 χ —6 y + l) + i.( 2 x —y —2z—4)=0
or 4 (χ’ + ^ + ς^ + ΙΟ χ —25y—22=0.
Ex. 6. Find the equations o f the spheres through the circle
**+ ,^ + Z i =aI. 2x+ 4y+ 5z= 6
and touching the plane 2 = 0. (Gorakhpur 1974; Meerut 85S)
Sol. The equation of any sphere through the given circle -is
ju*+j,*+zs _ l + a (2 x + 4 ^+ 5 2 —6)=0
or xi +y*+2*+2Ax+4A^+5Ar—(1+6A)=0. ...(I)
If the sphere (I) touches the plane 2 = 0, then the length of
the perpendicular from the centre ( —A, —2A, —J.5A) of the
sphere (I) to the plane 2= 0 = + th e radius Of the sphere (I)

ie - V O iT “ ± * /{ λ*+ 4 λ ί+ ^ ’ +6λ)}
25A1 45A*
Squaring = - ^ —+6A+1 or 5A*+6A + 1= 0
or 5A*+5A+A+1=0 or (5A+1) (A +1)=0 or A =—1/5, - I
Putting the values of A in (I), the equations of the required
spheres are given by
5 (x*-+y2+ 2*)—2x—4y—5 z + l= 0
and x*+3'*+z*—2x—4>’—5z+ 5=0.
The Sphere 279

Ex. 7 (a). Find the equations o f the spheres which pass through
the circle **+>>* +z*—2 x + 2 y + 4 z -3 = 0 , 2x + .y + z= 4 and touch
the plane 3x+ 4y—14=0.
Sol. The equation'of any sphere through the given circle is
x * + y * + z* -2 x + 2 y + 4 z-3 + X { 2 x + y + t-4 )= 0
or xt + ^ , + 2 * -2 x (l-A )+ y (2 + A )+ * (4 + A )-(3 + 4 A )« 0 ....(l)
If the sphere (I) touches the plane
3x + 4 y —14=0; ·- -(2 )
then the length of the perpendicular from the centre
(I-A , -(2+A )/2. —(4+A)/2)
to the plane (2) = + th e radius of the sphere (I)
. 3 .f l- A ) - 4 .j( 2 + A > —14

- ± V { ( l- A ) * + i (2+A)*+i (4+A)*+3+4A)
or -<A+3)=±*v'(6A*+20A+36).
Squaring and simplifying. A1—2A=0 or A=0, 2.
Putting the values of A in (I), the equations of the required
spheres are **+>*+**—2x+2y+ 4z—3= 0
and χ*+^*+ζ*+2χ+4>»+6ζ-ΙΙ=·0.
Ex 7 (b). Find the equations o f the spheres which pass through
the circle x1+y*+ z, = 5 , 2*+3,y+z=3 and touch the plane
.N 3 ^ + 4 ^ -1 5 = 0 . (Punjab 1982)
Sol. Proceed as id . βχ. 7 (a) above.
Ex. 8. Find the equations o f the tangent line to the circle
3x*+ 3y*+ 3z*—2x—3^—4z—22= 0, 3x+ 4y+ 5z—26=0
at the point (1,2, 3).
Sol. The tangent line to a circle at a point is the line of
intersection of the tangent plane to the sphere at the point and
the plane of the circle.
The equations Qfthe given circle are given by the sphere
**+>>»+z*-ix-.V-fz-*e*«0 -.(I)
and theplane 3 χ $ % ^ 5 ζ —26=0. ---(2)
The equation of the tangent plane to the sphere (I) at (I, 2, 3)
is x .l- h y .2 + z .3 - J ( x + l ) - J (.y+2)—S ( 2 + 3 ) - ^ = 0
or 4x+9.y+14z—64=0. ---(3)
Thusthe non-symmetrical equations of the required tangent
line are given by (2) and (3) together. To find its symmetrical
form, let /, m, n be its d.r.’s, so that we have
3/+4m -|O/i=0 and 4 /+ 9m+ 1 4 = 0 .
280 AnaJyticaI Geometry 3-D

Solving, wc get ,%
I __m _ n Z_ m _ n
—11_ 22*" —lT ° r T - 2 Γ
Λ The equations of the tangent line at (1, 2, 3) and having
d.r.’s 1 . - 2 , I are
(*—1 )/1 = (3 '-2)/(—2 )= (z—1)/1.
Ex. 9 (a). Ifthreemutually perpendicular chords of lengths
du dt, d3 be drawn through the point (xit y t, Zi) to the sphere
xi +y*+z*c*ai, prove that
dt* + da*+d3* = l2o*—8 (x1*+J’i*+2l1).
Sol. The equation of the given sphere is
x*+y*+z*=a*. ...(I)
Let /,, mlt Hj; /*, mit η» and /8, m3, M3 be the actual d.c.’s of
the three mutually perpendicular chords drawn through the same
point A(xit y u z,). The equations of the first chord are given by
(x -.x 1)//l = ( ^ - 7 ,)/Mii=(z-Zi)/n1= r (say). . (2)
Any point on (2) is (ZiM-Xlt m ,r+ yu Bir+Zi).
Substituting the co-ordinates of this point in (I), the points
of intersection of the chord (2) with the sphere (I) are given by
(Λγ+Χ ι ^ + Ο πιΓ-Η ',^-Η λιΓ+Ζι )*=^* ...(2)
or rs(Zi2+ IWi1-I-B1*)+ 2 r (/iXi+miyi+BiZi)
+ (*I*+>1*+ V - ύ*)= o
or r*+2r (ZiXi+fWiyi+BjZi)+ ^ - + ^ + - 2 ^ - 0 ^ = 0 . ...(3)
Let the chord (2) meet the sphere in the points P and Q so
that AP=rit AQ=rt where Ti and r8 are the roots of (3).
A dt= P Q = A P —AQ=Tt - T i
or </i*=(T1- T i)*=(rt + γ8)*=4 γιΓ8
or </i*={—2 (I1X1^ m ty 1 +Mi Zi )}*-4 (χι2++ι2+ ζχ2- α 2)
o r. </x*=4 (/i Xi +»Wi^ i +Mi Z1)* -4 ( x ^ + y ^ + z ^ -a * ). ...(4)
Similarly </**=4 (/3Xi+m8+i+w8zi)*-4 (Xi*+yi2+Zi*-a*) . (5)
and <Z8*=4 (/8Χι+Βΐ3^ ι+ η 3Ζι)*—4 (Xi*+,yi*+Zi*—a*). . (6)
Adding (4), (5) and (6) and using Zi*+/8*+/3*<=l etc.,
/itMi+ Z 2IW 8 +Z3Wi3=O etc., we get
<Zi *+</3*+ </3*=4 (Xi2++i*+Z12J - 1 2 (Xi2+ y i2+Z i*-fl2)
= 12e*—8 (Χι*+Λ*+Ζι2).
Ex. 9 (b). Show that the sum o f the squares of the intercepts
made by a given sphere on any three mutually perpendicular straight
lines through the fixed point is constant. \
Sol. Let the equation of the given sphere be x*+>*+ z*= ar
and the fixed point be (xx, y lt Z1).
The Sphere 281

Now this problem is nothing but a different'statement of the


Ex. 9 (a) above.
Ex. 10. I f m y tangent plane to the sphere x*+j>*+z*«=r* makes
Intercepts a, b and c on the co-ordinate axes, prove that
a~2+b~2+c~2=r~2.
(Agra 1975, 79; Kanpnr 73. 81; Pnnjab 82)
Sol. The equation of the sphere is given to be
*a+.y*+**—r*. --(I)
Let (xx, yu 2|) be a point on the sphere (I), so that we have
Xi*+y,*+zi2= r2. -(2)
The equation o fth etan g en tp lan eto the sphere (I) at the
point (xlt y lt z,) is XX1+yyx+ ZZ1=T2
x , y , z ___ .
Γ TltJxl TtIyl TtIzl ~ ‘ -(3)
Λ The intercepts made by the tangent plane on the axes are
TtIxu Tt Iyl and Tt Iz1; but these intercepts on the axes are given to
be a, b and c respectively.
.·. a =T2Ixu b= T2Iy1, C-=T2Iz1
or X1-=r*la, y x=T2Ib, Z1=T2Ic.
Substituting the values of Xi, y u Z1 in (2), we get
(r2la)2+ (r2lb)2+(r2lc)2= r2 or a~2+b~t +c~2-=r~2.
Ex. 11. Find the equation of the sphere touching the three co­
ordinate axes. How m m y such spheres cm be drawn ?
Sol. Let the equation of the sphere be
x 2+ y2+ z2+2ux+2vy+2wz+ d=0. -.(1)
If the sphere (I) meets the x-axis i.e., y*=0, z= 0, then from
(I), we have
x*+2«x-|-rf=0. —(2)
S incethespbere(I) is touching y = 0 = z , the roots of (2)
must be equal and so we have
(2«)*—4 .!.< /--O [using B2-AAC=O]
or U2= d .

Similarly if the sphere touches j> and z axes, we have


v2= d and· w2=d.
U2=V2=W2Bad=X2 (say).
.·. i/C=V=W= ±A.
' Putting the values in (I), the equation of the sphere (I)
becomes
x 2+ y* + z2±2X (x H-j H-z)+A2= 0. ...(3)
282 Analytical Geometry 3-D

Since Λ may have any real value, an infinite number of spheres


can be drawn satisfying the given conditions.
In case the radias of the sphere be giveo, say r, we shall have
only eight spheres as shown* below.
Now r a th e radius of the sphere = ^(**1+*'* +
Λ r * ~ d + d + d -d \ [ Y U8=V2= w*=d)
or r a=2tf.
ua=va --- w2=*d~\r2 or u = v = w = ± r/V (2 ).
Hence the equation (I) of the sphere becomes
x2+ y 2+ z%±. rx y/2 ± try ± rzV 2+ Ir 2= 0. ... (¾)
The equation (4) represents only eight spheres satisfying the
given conditions.
Ex. 12 (a). Find the equation o f a sphere touching the three
co-ordinate planes. How many such spheres can be drawn ?
(Rohilkhand 1979)
Sol. Let the equation ot the sphere be ^
x*+y*+z*+2ux+2vy+2wz+d=Q. ...(I)
In the sphere (I) touches the jrz*plane ie .t jc» 0, then the
length of the perpendicular from the centre ( —i/, —v, —w) of the
sphere (I) to the plane (x = 0 )» th e radius of the sphere (I)
i.e. —w/1 = y/{u2+ v2+w2—d).
Squaring, u2=*u2+v* + w2—d or va+w a=rf. · -(2)
Similarly if the sphere (I) touches the Planesjr=O and z= 0,
then we have
w2+Ma= d .(3), M*+ va=rf. —(4)
Adding (2), (3) and (4). we get
2 (u2+ v2+w2) ^3rf or Ma+ v2+w 2«=»frf. --»(5)
Subtracting (2), (3) and (4) from (5) in turn, we get
m ? = v2=w>2= |< /~ r2 (say), or m= v= w= ± ^ ·

Also radius of the sphere= \/(u 2+ v2-f w2—d)


= V (r2+ ra+ ra—2r1)= r.
Substituting the values in.iij*Jh^ equation of the required
sphere is given by
X1+ y2+ z2± 2rx ± 2ry ± 2rz+ 2r2= 0. ... (6)
Since r may have any real value, an infinite number of such
spheres are possible. In case the radius, r of the sphere be given
then only eight such spheres are possible.
Ex. 12(b). Find the equation o f a sphere which ties in the octant
OXYZ and touches the co-ordinate planes.
The Sphere 283

Sol. Proceed upto equation (6) of Ex. 12 (a) above.


Now if the centre of the sphere lies in the octant OXYZ9then
the centre of the sphere is (r, r, r) and hence the equation of the
required sphere is of the form
x*+yt + zi —2rx—2ry —2rz+2ra= 0. ...(7)
Ex. 12 (c). Show ihatingeneral two spheres can be drawn
through a given point to touch the co-ordinate planes and find for
what positions of the point the spheres are (i) real and(ii) coincident.
Sol. Let the given point lie in the positive octant. Obviously
the centre of a sphere passing through this given point and touch­
ing the co-ordinate plants will also lie in the positive octant.
From Ex. 12 (b) above» th^.equation of any such sphere touching
the co-ordinate planes is given by
x2+y*+z*—2r (x+ y+ z)+ 2 rt *=0. ...(I)
If this sphere passes through the given point (Jt1, y u zx) (say)»
then we have \
* ι , + Λ , + * ι * - 2 γ (Jt, + λ + * ι ) + · 2 γ · — 0
or 2r*—2r (At,+ ^ , + 2,)+(¾ ,*+^, 2+z,*)=0. »··(2)
This equation (2) is a quadratic in r and hence gives two
values of r and accordingly two spheres are given by (I) passing
through a given point (At, , ) ,, z,).
The two spheres will be real or coincident according as the
roots of ( 2) are real or equal and the condition for the same is
i - 2 (Ar,+^,+z,)}*-4.2.(At,*+>i*+Zi*) > 0
[using Bt -A A C ^ 0]
or (A t, 4-^1 - 2 (At,*+J',*+Zi8) > P
or 2 Ο',Ζ,+Ζ,Χ,+ΑΤ,^,) Jr At,*+^,*4-Zl*.
Note. If the co-ordinates of a point are given, the point will
lie in any one of the eight octants. Any sphere touching the co­
ordinate planes and passing through (Ms given point will have its
centre in the same qctant. Vv γ ^
Ex. 13. A Sphere touches three cp-prj^tieplanes and passes
through the point ( 2, I, '). Find its equation. (Burdwan 1978)
Sol. The given point (2, I, 5) lies in the positive octant
OXYZ. Heucc the equation of the required sphere is of ,the form
[See Ex. 12 (b) above]
*2+ i'2+z*-2r(A c+.y+z}+2r»=0. ...(I)
(Deduce this result here]
If the sphere (I) passes through the point (2, I, 5), we have
4 + 1+25-2»· (2 + l+ 5 )+ 2 r* = 0
284 Analytical Geometry 3 -D

or 2r·—16r+30=»0, or r2—8r+ 1 5 = 0 ,
or (r—3) (r—5 )« 0 or r= 3 , 5.
Substituting the values of r in turn in (I), the equations of
the two required spheres are given by
s* + ^ * + * * -6 x -6 y —6 z+ 1 8 « 0 f
and x 2+ y2+ z2—IOx —IO y -10z+50*=»0.
Ex. 14 (a). Find the equation o f the sphere inscribed in the
tetrahedron whose faces are
y=0, 0 and 2*+6y—3 z+ 9= 0.
Sol. The equation of the fourth plane is t2x+6y—3z+9«=»0.
This plane clearly meets the x 9y and zaxes in the —ve, —ve and
+ve directions. Hence the sphere touching the three co-ordinate
planes and the plane 2x+ 6y—3z+9*=0 lies in OXtYtZ octant and
so the centre of such a sphere is of the form (—r, —r, r) where r
is the radius of the sphere.
Since the sphere touches the plane
2 x + 6 y -3 z + 9= 0, ...(I)
therefore the length of the perpendicular from the centre ( —r, —r,
r) to the plane (1) = the radius r of the sphere
—2 r - 6r—3r+9
i.e. V (4+36+9) - « r
or —I lr + 9 = 7 r or r«=J.
Λ Thecentre ofthe required sphere is ( — $, — $, and
radius is £. Therefore its equation is given by
(* + *)f+(y + i)a+ (* --* )f- U ) e
or x 2+ y*+ z*+ x+ y—z + $ = 0 . Ans.
Ex. 14 (b). Find the equation of the sphere inscribed in the
tetrahedron who.se faces are
x = 0 , .v^-0, z - Oand 2 * + o y + 3 z—14—0.
Sol. Here the sphere lies in OXYZ octant. Proceeding
as in Ex. 14 (a) above, the equation of the required sphere is
given by
81 (*a+ y 2+ z 2) - 126 (x + y + z)+ 98 = 0.
Ex. 15. Prove (hat the centres of the spheres which touch the
lines y —mx, z=c; y = —mxt z=>—c lie upon the conicoid
mxy+cz (l+ //i2)= 0 .
(BundeKkhand 1979; Lncknow 74, 81; Kanpur 79; Meerut 89S)
Sol. Lu the equation of the sphere be
JC2+ > a + z 2+ 2 w x + 2v#K + 2 n ’z + J = i 0 .
The Sphere 285

Thepoinrs of intersection of the sphere Π) wirh the line


y= m x9 Z ^ c are given by
Xt + TntXt + c9+ l u x + 2vm x+ 2w c+ d = O
or Xt (l+ m a)+ 2 (ιι+vm) JC-Hc*+2wc+i/)=p. (2)
If the sphere (I) touches the line y= m x9 z= c then the roots
of the equation (2) will be equal and so we have
{2 (w+vm)}2—4 (l+m*) (ca+2w>c+d)«0
or (ii+vm)a^ ( l + / n e) (ct +2wc+d). -(3)
Again the sphere (I) also touches the line
y=*—mx, z = —c.
So putting - m for m and —c for c in (3), the required con­
dition is given by
(m—vm)2= (\+ m 2) (c2—2wc+d). ---(5)
Subtracting (4) from (3), we get
Auvm=(X^mt) (Awe),
or muv—cw (1+/η*)=0.
Λ The locus of the centre ( —i/, —v, —w) is given by
m ( —X) ( - y ) - c ( z) (I +W1JeO
or m xy+cz (l+ m 5)= 0.
§ 8 (A). Plane of contact.
The plane of contact is the locus of the points o f contact o f the
tangent planes which pass through a given point (not on the sphere).
(B) Tofind the equation of the plane o f contact.
Let the equation of the sphere be
x t + yt + zt +2ux+2vy-\-2wz+d=0. ---0)
Let (α, β, y) be a given point external to the sphere (I). The
equation of the tangent plane at any point (*i,yi, Zi) of the
sphere is
xxi+ yyi+ zzi+ u (χ+-*ι)+ν O '+ j'i)+ » ’ (z + z ^ + d = 0.
If it passes through (α, β, y)9 we have
a * i+ A f t + y *i + w ( a + * i ) + v ( 0 + y i) + w ( y + Z i) + d = 0 .
Λ The locus of the point of contact y\% *i) is
(ix+ fiy+ yz+u (x + a )+ v (y+β) + w (z+ y)+ d = 0
which is the equation of the plane ol contact of the point (a, β9 y)
with respect to the sphere (I).
§ 9. Pole and polar plane.
Definition. Consider a line through a fixed point A to inter­
sect a given sphere in the points F and Q. Take a point R on
286 Analytical Geometry 3-D

I I 2_ (i.e. the distance AR is the


this line such that
AP+AQ' 'AR
harmonic mean of the distances AP and AQ), then the locus of the
point R is a plane called the polar plane of the point A with
respect to the given sphere and the .fixed point s is called the
pole of the polar plane.
The equation of the polar plane.
Let the equation of the given sphere be
xi + y i +z*+2ux+2vy+2wz-td=0. · (0
Let the co-ordinates of the fixed point A be (xi,yi* zi) end let
I, m, n be the actual direction cosines of the variable line through
A (Xi, y t, Zi) and so its equations are given by
x - x , y~ y, Z - Z i .
-r (say).
I ~ m “ n . . ( 2>
The co-ordinates of any point on the line (2) are
(Ir+Xu m r+yu nr+z,).
The distances AP and AQ of the points of intersection P and
Q of the variable line (2) and the sphere (I) are the roots of the
equation
(/r+ x ,), + (m r+^,)*+(nr+ Z |)s-|-2a (/r+ x,)+ 2v (m r+ y j
+ 2w (nr+Zi) + d = 0
or r*+ 2r {/ (x ,+ « )+ m (y i+ v )+ n (z,-f h>)}
+ ( ^ + 7 ^ + ^ + 2 1 ^ + 2 ^ ,+ 2 ^ ,+ ^ ) = 0 ...(3)
[V /*+»!*+«*=-1]
·’. fi+r»= AP + AQ = - 2 { /(x ,+ ii)+ m (y»+v)+n (Zi + h>)}
and r i. r»= A P . AQ=Xi*+Jp1*+ Zi*+ 2vx, + 2Vyi+ 2wzx+ d.
. _l_ _1_ AP+AQ —2 {/ Cxi -i-«)+m Iv1+?)+*! (ζ, + ηΊ)
AP- A Q ^ A P .AQ ssXi*+yit + zi*+2uxi +2vyl +2wzt +(l
2
C=-^ [See definition above]
or * i , + ^ 1»+ z,*+2 mx1+ 2 vj'i + 2 ivZ!+</
= - { I A R (Χ!+ ιι)+>μ.ΛΛ (yi+ v)+ n.+ ft (z,+w)} ...(4)

>\ ; Λ ’(
let the ^o-ordinates of the point R be (x , y 9z). The
distance of JR frdnp A (xlt y u zx) is AR i.e. putting r —AR in (2),
we get \ '
\I
x \ - Xi=LAR9y —y ^ m .A R , z - z x=h.AR. ...(5)
The locus of the point R is obtained by eliminating the
variables I9 m9 « between (4) and (5). Hencethe locus of R Le.
The Sphere 287

the polar,plane of A (xlt j^, Z1) with respect to the sphere (I) is
given by
*i*+y\'·+ zi*+ 2«x,+ 2vyt + 2h»z, + d
= —{ (* -* i)(* i+ « 0 + 0 '-J'i) (y t+ v )+ (z -zt) (2|+W)}
or xx\+ yyi+ zzt+ u (x+*i)+v(y+yi)+»*’ (2 + 21) + ^ = ° - (6)
To flod the pole of a given plane. Let the pole of the given
plane be (X1, y u Z1). Write the equation of the polar plane of
(X itji, zr) with respect to the given sphere. Now compare the
given plane with this polar plane and then obtain X1, y r, Z1.
§ 10. The properties of the pole and the polar plane.
Property I. Salmon’s Theorem. The distances of two points
from the centre of a sphere are proportional to the distance o f each
from the polar plane o f the other.
Let the equation of the sphere with centre O (0,0, 0) be
xt + y t +z*=a*. ···(!)
Let P (xlf y u zt) and Q (x%, y t, z*) be the two points whose
polar planes with respect to the sphere (I) are
xxi+ y yi+ zzi-a * —<2)
add xxt+yya+zztesa*. —(3)
J^ow the distance of P from (3)
the distance of Q from (2)
(xtx*+ yiyt+ zrzt —a*)l ViXt9^ y t t +Ztt )
“ (** *i+ y ty i+ ZtZl - a*)l^(X1"jCyit + zx*)
OP
^V iX t'+ ya'+ z»*) ~OQ Proved.
Property II. ThepolarpIaneof apointP with respect to a
sphere Is perpendicular to the line joining the point P to the centre o f
the sphere.
Let the equation of the-sphere with centre O (0, 0, 0) be
x*+y*+z*z=a*. ...(I)
Let the co-ordinates of the point P be (X1, y r, 2i) and so the
equation of the point plane of P (X1, y u zr) w.r.t. the sphere (I) is
x x i+ y y i+ ZZ1^ a t. .(2 )
The direction ratios of the normal to the polar plane (2) are
Xi, y u zt. Also the d.r.’s of the line OP are X1-O , J11-O , Zj -O
i.e. X1, y r. zt. Hence the normal to the polar plane (2) is parallel
to the line OP i.e., OP is perpendicular to the polar plane (2).
Property III. I f the line joining the centre O o f the sphere and
a point P meets the polar Of P in the point Q, then
OP. OQ =»(radius)*.
288 Analytical Geometry 3-D

Let the equation of the sphere be


2*=a*. ...(I)
The centre of sphere is O (0, 0, 0) and radius=a
Let the co-ordinates of the point P be (Xi, ylt Z1). The
equation of the polar plane of P (X1, y u 2,) with respect to the
s p h e re (l)is x x , + ^ , + 22, = 0*. ...(2)
Now OP is perpendicular to the plane (2) and meets it at Q.
.·. O g = the perpendicular distance of the centre O (0, 0, 0)
from the polar plane (2)
O2 ' a2
eV i* .* + * * + * ,* ; " o p
Property IV. Ifth e polar plane o f a point P (x,, yu *1) posses
through another point Q (xa, y t, za), then the polar plane of Q will
pass through P. The two points P and Q satisfying this property ore
called conjugate points.
Let the equation of the sphere be x*+y*+ 2*=o*. ...(I)
The polar plane of P (x,, y lt 2,) is x x,+ y*+ **.= fl*
If it passes through Q (x8, yt, Zi), then
***1+ * * + * » * ,*=a*. —(2)
The polar plane of Q (xa, yt, z*) is xxa+ yyt+ zzt —a*.
If it passes through P (x,, y u z,), then
* i* s+ * * + * ,* s= e* . ...(3)
The relation (3) is true since the relation (2) istrue. Hence
the result.
Property V. L etu x=O and Ua=O be the equations of two
planes. Ifth ep o leo fth ep lo n eu i =Olies ontheplane U a = O 1 then
the pole of the plane Ui = O will lie on the plane U1= 0. The two
planes u ,= 0 and ua= 0 satisfying this property are called conjugate
planes.
The proof is similar to property IV above.
§ It. The polar line.
■The polar line of a given line AB is another line CD such that
the polar planes o f all points on the line AB pass through the line
CD.
Let the equation of the sphere be χ*+^*+ζ*=α*. ...(I)
Let the equations of the given line AB be
(x—oe)//=(y—/3)/m=(z—y )/n = r (say). ...(2)
The co-ordinates of any point on the line (2) are(/r+ « , mr+ β,
ttr+y). ^s polar plane with respect to the sphere (I) is
x (/r+ * )+ y (/nr+/3)+ z (nr+y)=o*
or («x+/Jv+ yz—n2) + r (/r+ m y+ nz)= 0.
The Sphere 289

This plane for all values of r passes through the line


αχ+β)>+γζ—a*=0=/x+m;y+nz. ---(3)
Thus the equations (3) are the equations of the polar line CD
of the given line AB [given by (2)].
§ 12. The angle of intersection of two spheres.
If the two spheres intersect then the angle of intersection of
these two spheres is defined as the angle between the tangent
planes to them at their common point of intersection. Since the
radii of the spheres through the common point are perpendicular
to the tangent planes at that point, therefore the angle of inter­
section is equal to the angle between the radii of the two spheres
through their common point.
Let P be the point of intersection of the two spheres whose
centres are Ci and C2 and radii rt and r2. Let Θ be the angle of
intersection so that /.C iPC 2=A. Also let CiC2= d.
Applying cosine formula for the ACiPCi, we get
r,*+r,«-d* / r,*+/·,*-</*)
cos A* --· or A=COS-1
2r,rt \ 2rirt }
Twospheresare saidtointersectorthogonally if their angle
of intersection is a right angle. In this case the spheres are called
orthogonal spheres.
To find the condition for orthogonality of two spheres.
(Avadh 1981; Kanpnr 78)
Let the equations of the two spheres be
x*+J’*+z*+2u,x+2v1y+2w ,r+</,=0, ...(I)
and **+y*+z* t-2u2x-f2v2y+2w2z+i/2= 0 . -. (2)
Let Ci and C2 be the centres and r, and rt the radii of the
spheres (I) and (2) respectively. Then
Ci is ( - « „ -V,. -tv ,), C2 is (-M 2, - v t, - w 2)
and r , = -v/(Mi8+ v ,* + h>i *-</,), rt ■=>vX"**+ v»2+ w2*—</2).
Let P be a common point
of intersection of the spheres
(I) and (2). If the sphere's (I)
and (2) intersect orthogonally
then the angle between the
radii through the common
point P is a right angle and
hence we have
CiP1-I-C2P i = C 1C2* or r^ + iV -C iC ,1
290 Analytical Geometry 3-D

+(W^1-W a)*
or 2i/1w*+2 v1vi + 2 h’iWf=^Z1+</,, ...(3)
which is the required condition.
§ 13. Tooching spheres.
If the distance between the centres of the two spheres is equal
to the difference of their radii, then the two spheres touch
internally.
If the distance between the centres of the two spheres is equal
to the sum of their radii, then the two spheres touch externally.
SOLVED EXAMPLES (D)
Ex. I. Prove that the equation o f the polar plane of (xlf ^1, Z1)
with respect to the sphere a 1+;'*+Z1=A1 is
xxi+yyi+zzi'=** (Delhi 1974)
Sol. proceed as in § 9 above.
Ex. 2. Find the pole o f the plane lx+ m y+ n z~ p with respect
to the sphere x*+y2+z*=a*. (Gorakhpur 1976; Rajasthan 78)
Sol. The equation of the sphere is
^ + / + Z1= A1. ..-(I)
The equation of the plane is lx+ m y+ nz= p. .(2 )
Let the pole of the plane (2) w.r.t. the sphere (I) be (X1, yl9
Z1). The equation of the polar plane of (xlf yu *i) with respect to
the sphere (I) is **i+XVi+ZZ1=A1. - (3)
The planes (2) and (3) are the same, so comparing the coeffi­
cients of like terms, we get

*1 yi ^ z l A1
A1/ i orn
or , zx= — ·
The pole of the plane (2) is (a*l/pf a*mlp, a*njp).
x —I y —2 z —3
Ex. 3. Show that the polar oj ~ ~ =»-^— with respect
to the sphere x*+y*+z*=9 is given by
x+ 2 y+ 3 z—9<=0, 2x+3y+4z*=0.
Sol. ThisprobIem isbased on § 11. The co-ordinates of
any point on the line
are (2r+ l» ir+ 2 , 4r+3).
Its polar plane w.r.t. the sphere x*+<y>^2 * = 9 is
The Sphere 291

(2 r+ l) * + (3 r+ 2 ) y + (4 r+ 3 ) z= 9
or (x+2j»+3z—9 )+ r (2*+3y+ 4z)=0.
This plane for all values of r passes through the line
x + 2 j'+ 3 z - 9 = 0 . 2 x + 3 y + 4 z= 0 .
Thus these are the equations of the polar line of the given
line.
Ex. 4. Prove that the polar plane o f any point on the line
jx = $ Cp - I ) = * U + 3) With respect to the sphere 0 + 0 + 0 = 1
passes through the line
(2 x + 3 )/l3 = (y —I)/(—3)=z/(—I). (Allahabad 1976)
Sol. This problem is based on § 11.
The co-ordinates of any point on the line
|x = $ O' —1)=·! 0 + 3 ) are (2r, 3 r + l, 4r—3).
Its polar plane w.r.t. the sphere 0 0 + 0 + 0 = 1 is
oc.2r+,y.(3r+l)+z.(4r—3)=1
or (y—3z—l)+ r (2x+ 3y+ 4r)= 0.
This plane for all values of r passes through the line
y —3z—1=0, 2oc+3.v+4z=0. .··(!)
The symmetrical form of the line (I) is
* + 3 /2 _ y -l_ z -0
13 ~ - 6 “ - 2
-2oc+3 y —I z 2x+3 y — I z
or
26 -6 -2 13 -3 - I
Ex. 5. Find the angle of intersection o f the spheres
x * + 3 ', + z* - 2 x - 4 > ' - 6 z + 1 0 = 0
and oc*+0+z*—6x—2 ^ + 2 z + 2 = 0 .
Sol. The equations of the given spheres are
* * + 0 + 0 - 2 o r - 4 j> - 6 z + 1 0 = 0 , ...(I)
and x*+y, +z*—6x—2 ^ + 2 z+ 2 = 0 . ...(2)
The centre of (I) is Ci (I, 2, 3), its radius
r ,= V ( l , + 2 * + 3 * -1 0 )= 2 .
The centre of (2) is C* (3, I, —1), its radius
= ^ + 0 + (-1 ^ -2 )= 3 .
Now CiCt=the distance between the centres C> and C»
={(3—l)*+ (l —2)*+( —I —3)*}= -v/(21).
Let Θ be the angle of intersection of the spheres (I) and (2),
then
292 Analytical Geometry 3-D

2
cos 9 _ r«*+ r,* - (C1C2)*_ (2)*+ (3 I*- 21 - 8
^ 2r,r, " 2.2.3 12 3
= } (taking the acute angle).
9=cos_1 (}).
Ex. 6. Show that the two spheres x* +y*+ z, + 6 y + 2 z + 8 = 0
and x*+y, +z*+6jc+8.y+4z+20=0 are orthogonal.
Sol. The equations of the two spheres are
x, + y*+ z, + 6 y + 2 z + 8 = 0 ···(!)
x, + 7*+ z* + 6 x + 8 ^+ 4 z+ 2 0 = 0 . ...(2)
For the sphere (I), we have t/,=0, v,=3, Wi = It </ι=·8, and
for the sphere (2), we have «,--=>3. v,=4, tv,=2, </,=20.
We know that the condition of orthogonality of two spheres
is [See § 12, equation (3)]
2«iW»+2v,Vj + 2 h',mv-</|+</,. ---(3)
Putting the values in (3), we have
2 .0 .3 + 2 .3 .4 + 2 .1 .2 = 8 + 2 0
nr 0 + 2 4 + 4 —28 which is true.
Hence the given spheres are orthogonal.
Ex. 7. Two points P and Q are conjugate with respect to a
sphere S; prove that the sphere on PQ as diameter cuts S ortho­
gonally.
Sol. Let the equation of the sphere S be given by
S s x 1 + y * + z* + 2ux + 2vy+ 2 m»z + < /= 0 . - - - ( I)
Suppose the co-ordinates of the given points P and Q are
(xi. .V i. Ti) and L r , , y t, z f ) respectively.
T heequationofthe polar plane of P (Xtf yt, Zj ) w.r.t. the
sphere (I) is
* * ,+ v y i+ zz,+ t/ (x + x ,)+ v (y + y ,)+ iv (z+zO+</=0.
The point Q (xt, y t, z,) will lie on this plane, since P and Q
are conjugate points and hence, we have
*ix*+yiy»+z,z,+i< (*!+**>+v O't+y»)+»' t o —z,)+</=o
The equation of the sphere on PQ as diameter is
(X -^i) (x -x » ) + ( y - y i) (y -? * )+ (z -z i) (z-z,)*=*o
or x, +y*+z*—x (x i+ x ,)—y (y i+ y » )-z (zi+z,)
+(xiX,+yiy,+ZiZ,)=0. ---(3)
-. Now the sphere (3) will cut the sphere (I) orthogonally, if
-2 -2 _ 2 < h+ I*l.w
=</+(x,x,+yiy,+ziZi)
or xix»+yiy»+z,z,+u ( χ ι+ χ ,)+ ν (>i + y ,)+ w (zi+z,)+</=o
which is true by virtue of (2). Heace proved.
The Sphere 293

Ex. 8 (a). Obtain the condition that the spheres


a (x*+ / + 2*j+ 2/*+ Im y+ 2/iz+p=O
and b (x*-)-yi + zi)=k*
may cut orthogonally. (Punjab 1975. 81)
Sol. The equations of the given spheres may be re-written
as
x*+yt +z*+2 (Ito) x + 2 (m/a) y+ 2 (n/a) z+ (p/a)^0, ...(I)
end x * f y i -\-z*~(k*lb)=0. ...(2)
The spheres (I) and (2) will cut orthogonally if
2 (IJa).0 + 2 (m/a).0 + 2 (/ι/α).0=(ρ/ο)+{-Α:2/*)} (See § 12]
or -amaTm
b or pb^ak*
which is the required condition.
Ex. 8 (b). Find the condition that the spheres
x t + y, +z*=at
and xi +y*+zi -\-2ux+2vy+2wz+d=0
may cut orthogonally. (Punjab 1979)
Sol. Proceeding as in Ex. 8 (a) above, the required condition
is a*—d.
Ex 9. Twospheresofradiirx an.·/ r* cut orthogonally. Prove
that the radius of the common circle is Z1^/V V 1*+ r**)·
(Kanpur 1977, 78; Rajastbau 77; Meerut 85, 87P)
Sol. Let the equations of the common circle be taken as
xt + yt —a \ z~ 0 . ...(I)
The radius of this circle is clearly a and we are required to
find it.
The equations ol the two spheres through the circle (I) are
given by
(x*+y + Zi -O i^ A 1Z=O, ...(2)
and (x*+.V* i-z*-o*) + 2A2z --0. ...(3)
The radii rt and r2 of the spheres (I) arid (2) are given by
r.= V{(A.)*-(-o*)} or ^ i = A1H o i
and rt*~ A**+e*.
Again the spheres (2) and (3) intersect orthogonally, so apply­
ing the condition
•2«,U2+ 2V1Vi -1-2W1vt, = (I1+ d2\ we have
0 + 0 -i-2 A iA 2= — Ut -a * or A1A2= - O 1.
Squaring, A1iAii =O4
or ( r f -a*) (rt*-a*) - a* or T1V**-Oi (Γι 4+ γ**)=-0
294 Analytical Geometry 3-D

of fl, «=r,,r1*/(r1*+r**) or a**rtrtl\/(r,*+ rt*).


Ez. 10. Prove that every sphere through the circle
x*+y*—2ax+r**=0, z= 0
cuts orthogonally every sphere through the circle x2+ z * = r \ ; = 0 .
(P odjob 1977 (S))
Sol. The equations of any spheres through the given circles
are given by
**+)’*+**—2ex+r*+2AiZ=0. ···(!)
and *■+>*+z*'—r*+2At.y=0. -( 2 )
If the spheres (I) and (2) cut orthogonally, then applying the
condition
‘2tiiUt+ 2v|Vi +2w'1W1= J 1+d%, we have
2 . ( - o).0+2.0.Ai+2.Ai.0=r*+(—r*)
or 0=0
which is true for all values of A1 and A,. Hence the result.
Ez. 11. Find the equation of the sphere that passes through the
circle Jci -Ky*+**-2x+3y—4 z+ 6 = 0 , 3 j c — 4y+5z—15=0 and cuts
the sphere x*+yt + zt + 2 x+ 4 y—6 z + 11=0 orthogonally.
Sol. The equation of any sphere through the given circle
is
Jt, + y * + * * — 2* + 3 ^ —4z+6+A (3x—4.y+5z—15)=0
°r z*+<y*+**+(3A—2) z + ( 3 —4A) >>+(5A-4) z+ (6-15A )= 0.
- ( 1)
The equation of the other given sphere is
**+< y, +**+2JC+4y— 6 z + ll= 0 . ...(2)
If the spheres (I) and (2) cut orthogonally, then applying the
condition ■
•2uiU»+ 2r,Vi + 2»v,Wi = </,+ , we have
2. (i ^ l ). l + 2 . (^ ^ - , .2 + 2 .(- ^ f i ).(-3 )= (6 -1 5 A )+ ll
or 3A—2 + 6 —8A+12—15λ=17—15A
or 16-5A =17 or A= —1/5.
Putting the value of Λin (I)9 the equation of the required
sphere is given by
5 ^ + ^ + ^ )-1 3 ^ + 1 9 ^ -2 5 2 + 4 5 = 0 .
Ex. 12. Find the equation of a sphere which touches the plane
3*+2y—z + 2 « 0 at the point (I9 —29 I) and also cuts orthogonally
the sphere xi+ yt + zt—4x+6y+4'=i).
(Avadh 19789 80; Agra 80; Punjab 76; Jodhpur 76)
The Sphere 295,

Sol. The equation of the given plane is


3x+ 2 y—z+ 2 = 0 . ...(I)
It is given that the plane (I) is a tangent plane to the required
sphere at the point A (l, —2, I) and hence the line joining the
centre C, of the required sphere and the point A is perpendicular
to the plane (I). The d.r.’s of the normal to the plane (I) (i.e. of
the line ACi) are 3, 2, —I and so the equations of the line ACi are
given by
x —I y+ 2 z 1
(say).
3 ~ 2 ...(2)
Since the centre C1 lies on the line (2), therefore the co-ordi­
nates of the centre Ci may be taken as
( 3 r + l ,2 r - 2 , - r + 1 ) .
The radius of this Sphere=C1/*
= V{(3r+1- l)*+ (2r-2+ 2)*+ (-r+ 1- 1)*}= v/(14r*).
The equation of the other given sphere is
4x+6y+4=0. (3)
The co-ordinates of the centre C* of the sphere (3) are (2, —3,
0) and its radius= v'(2*+3*+0*—4)=3.
Now the two spheres will cut orthogonally if
(radius of one sphere)*+(radius of other sphere)*=(CiCa)1
t.e. (y/( Mr*)]*+ (3)*=(3r + 1- 2)*+ ( 2 r - 2+ 3)*+ ( - r+ 1 - 0)*
or 14r*+9=9r*—6 r + l + 4r*+ 4r+ l-|-r*—2 r + l
or 6 = —4r or r = —3/2.
Putting the value of r in the co-ordinates of Cu the centre of
of the required sphere is
( - 1 + 1 . - 3 - 2 , 1 + 1 ) or ( - * , - 5 , f )
and its radius= V (M r1)=VX 14.f)= f V(I4).
.‘. the equation of the required sphere is given by
(x+7/2)1+ (>-+5)1+ ( 2 - 5/2)1= [f V ( 14)]1
or x1+ J’*+*, + 7 x + 1 0 y —5 z + 12=0.
Ex. 13. Prove that the general equation o f all spheres through
the points (a, 0, 0), (0, b, 0) and (0, 0, c) is
x , + y, + zt —a x—by—cz—λ (x/a+ y/b+ z/c—1)=0.
Find also the value o f λ so that this sphere may cut orthogonally
the sphere
x 1+y*+ z1—2ax—2by—2cz= 0 .
Sol. The three given points are A(a, 0, 0), £(0, b, 0) and
C(0, 0, c). Let us take the fourth point as 0(0, 0, 0).
296 Analytical Geometry 3-D

The equation of the sphere OABC is given by


S= Xi + + 2*—ax —by—cz-A). —(1)
[See Ex. 6 (a) of the set (A)]
The equation o( the plane ABC is given by
P sx /e + .y /b + z /c —1= 0. *··(2)
The general equation of the sphere through the intersection
o f(l) and (2) is S -A P -O
or xi + yi + zi —ax—b y - c z - λ (x/a+ ylb+ z/c—1)=0. ...(3)
This proves the first part of the problem.
The equation (3) of the sphere may be rewritten as
xi + yi + zi —(a+X/a) x —(6+A/h)>—(c+A/c) z+A=0. ...(4)
The equation of the other sphere is given as
x9+ yi + zi —2ax—2by—2cz**>0. -(5)
The spheres (4) and (5) will cut orthogonally if
(-« ) ( - a - A /u )+ ( -6 )(-6 -A /6 )+ (- c ) (-c -A /c )- A+0
or (o*+/)*+c*)+3A--A or A——(a*+ ^ + ^ ) /2 .
Ex. 14. Prove that a sphere S=O which cuts the two spheres
Sj=O and S*=0 at right angles will also cut the sphere A1S1-I-A4S4--O
at right angles. (Allahabad 1975; Garhwal 82)
Sol. Let the equations of the spheres be given by
S = x 9+ y9+ z9+2ux+2vy-r2wz+d-- 0,
S1mx*+yt + zi +2ulx +-2V1^ + 2»v1z+</1= 0 ,
and St BXi + yi + zi +2uix+2vty+2wiZ-\ </4= 0 .
The sphere S=O cuts the sphere St =O orthogonally, so we
have
2mti + 2 vvi+ 2 hh·.-- ...(I)
The sphere S=O cuts the sphere S4=O orthogonally, so we
have
2mw4+-2vv4-J-2h’h4'—</+d8. ..(2)
The equation ut the sphere AjS1+ A4S4=O is given by

ϊ Γ ΐ ; ΐ Α'Η ·
The sphere S=O will cut the sphere A1S1-I-A4S2=O [i.e., the
sphere (3)} orthogonally if

(*¾¾*)+*')
_ j i / iM i I^ A 4 i
V A1 , A4' I
The Sphere 297

0Γ 2 ( / ( A 1 IZ j + A 2M 2) + 2 v ( λ ι V 1 + A j V i ) T 2 w

(A1+A2)+(/Z1A1+ J 2A2)
or A1 (2IO/1 + 2W I+ 2 IVW0 +A 2 (2uu2+2vv2+2ww2)
t^A1 (J+ d|)+ A 2 (J + d 2)
or Aj (J+ J|)+ A a ( J + J 2) -- A1 (J+ Ji)+ A 2 (^/+^/2)
fusing (I) and (2)]
which is true for all values of A1 and Aa. Hence the result.
Ex. 15. Prove that the tangent planes to the spheres
X2 + y 2+ Zt + 2ujjc + 2vjy + 2W1Z + J1^ 0
and X2 + y %+ z2H- 2i/2jc+2v2y + 2 w2z + J 2 0
at any common point are at right angles if
2t/iM2+ Zv1V2+ 2 W1w2= J 1+ J2. (Allahabad 1975^78)
Sol. By § 12, we know that at a common point the angle
between the tangent planes to the two spheres is defined as the
angle of intersection of the two spheres. As the angle between the
tangent planes is given to be a right angle, the given spheres will
cut at right angles (i.e. orthogonally). Hence this is the same
problem as § 12.
Ex. 16. Find the equation of the sphere which cuts orthogonally
each o f the four spheres
x 2+ y2+ 2%^ a 2+b2+c2, x2+y* + z2+2ax*=a%9
x2+ y2+ z2+ 2by fca, x2+ y%+ z2+2cz—c2.
Sol. Let the equation ol the required sptiete be
x2+ y2+ zt -{2u\ \ .lvy+ :wz+ct^O. ...(I)
The equations ol the given sphetes arc
x 2+ / - + z * - ( a 2+ b 2+ c * ) = 0 . ...(2)
x1+ > a+ z* + 2ίΐλ - a2= 0, ...(3)
x 2+ y 2+ z 2+ 2 b y ~ i > * ~ 0. ...(4)
and **Hr.)^ + ^ + 2 ^ - c2<=0. ...(5)
It the sphere ( !) cuts the sphere (2) orthogonally then apply­
ing the condition ' 2ulu.1-\-2vxV%+ 2 w l \\2 ·--Ji + Ja\ we get
2u.0+2v.0+2w .U =J - (a2’\-o2+ c i )
or d = a 2+ b 2+ c \ >..(6)-
Similarly it the sphere (I; cuts the sp)ku> ii), (4) and (5)
orthogonally, wc respectively hai.«
lUQ*=*d-Q2 ...v. ; 2\b d -b* ...(«)
u ν’.
298 Analytical Geometry 3-D

Patting the value of d from (6) in (7), (8) and (9), we get
2u=(b*+c2)la, 2v=(c*+a*)/b, 2w—(a*+b%)/c.
Substituting the values of 2u, 2v, 2h>and d in (I), the equa­
tion of the required sphere is given by
x*+y*+zr+{(bi +c1i)la} x+{(c*+a*)lb) y+{(a*+b*)lc) z
+ (a»+6*+c*)=0.
Ex. 17. The variable sphere x t + y*+ zt + 2ux+ 2vy+ 2w z+ l*= 0
always cuts the sphere 3x*+3y* + 3z*—6x+ lOy+z—8 = 0 orthogo­
nally. Show that the point («, v, w) moves on a fix e d plane.
Sol. The equations of the given spheres are
x t + y*+ zt + 2 u x + 2 vy + 2 w z+ l= 0 ...(I)
and x H ^ + z * ^ * + j ? y + | z-* = 0 . (2)
The spheres (I) and (2) will intersect orthogonally if
2«.(—l)+2v.(5/3)+2H \(l/b)= I —8/3
or —όΜ+ΙΟν+Η'= —5 or 6u—IO v -w+5=>0.
The locus of the point (u, v, w) is
6x— 10y—z + 5 = 0
which is a fixed plane.
Ex. 18. Show that the spheres
x* +y*+r* =s25, x *+>* + 2*—I S x - 24y—40z+225=»0
touch and show that their point of contact is (9/5, 12/5, 4).
(Berahampur 1981S)
Sol. The equation of the first sphere is
x*+yi + zi —25. ...(I)
Its centre C1 is (0, 0, Oj and radius ri= 5 .
The equation of the second sphere is
■**+>’2+ 2 i —18x—24y —40z+225—0. ...(2)
Its centre Ct is (9, 12, 20) and radius
r*= V(9i +12»+208- 225)«=20.
The distance between the centres- CiCi
= V U9—0)»+ ( 12—O)2+ (20 - 0)* = V(625)= 25.
Also the sum of the radii = r,+ r* = 5 + 2 0 ^=25.
Thus the distance between the centres of the spheres (I) and
(2) is equal to the sum of their radii. Hence the given spheres
touch externally. (See § 13).
To find the point of contact. Let (xt, y lt Z1) be the point of
contact. The equations of the tangent planes at (xi, yu Z1) so the
spheres (I) and (2) respectively are
'The Sphere 299

χχι + y y \+z*i=25 -.(3)


and x x i+ y y i+ z z,—9 ( x + x ,) - l2 (y+ yt)—20 (z + ri)+ 2 2 5 = 0
or x ( X i - 9 ) + y ( yi -1 2 )+ r(z ,-2 0 )
—(9 x ,+ 12yi+20z1—225)=0. -(4)
Since the spheres (I) and (2) touch each other, therefore the
tangent planes (3) and (4) must be the same.
Comparing tbeir coefficients, we have
9_>’. - l 2 _ r l - 2 0 _ 9x1+12>'l +20r1- 2 2 5 _ fr ( } (5)
Xi yt Zi 25 \ Jt K r
X i - 9=Acx, .yt — 12=Acyi.z,—20=&z,
or x,=9/(l-A c), Λ - 12)/(1-*), z,-=-20/(1—A:). ...(6)
Also from the last two relations of (S), we have
9 x ,+ 12λ +20 ζ, —225=25*
or 225—25Λ [putting the values from (6)]
or 6 2 5 -2 2 5 (I -Ac)=25 (1—Ac) Ac, or Aa+8fc4-16«=-0
or (Ac+4)*=0 or Ac=—4.
Putting the value of k in (6), we get
* i-9 /5 . Λ - 12/5,z,=4.
Λ The co-ordinates of the required point of contact are
(9/5.2/5,4).
$ 14. The length of the tangent.
To find the length o f the tangent from a point A (x,, y u Z1) to a
sphere.
Let the equation of the sphere be
5=x*+ ^*+z*+2ux+2vy+2w z+d=0. ...(I)
Its centre is C(—u, —v, —w)
and radius= V iu*+ v* + h»*—d).
Let AP be the tangent line
from the point A to the sphere
(I) and P be the point of con­
tact. The length o f the tangent
Is defined as the distance between
the points A and P.
Clearly the radius CP is perpendicular to AP. Hence
APi =ACt - C P t
= {(*1+ + i) ',+ V)*+Ui+ W)*}-(u * + v*+W*—d)
—Xi*+yi*+zit +2uxl +2vyi +2wzl +d. ...(2)
The expression on tL« right hand side of (2) is usually deno­
ted by S1 i.e.
300 Analytical Geometry 3-D

APt - S i- Xit + yx'+ zJ + lu X i+ lv y^lw zx+ d .


This represents the square o f the length of the tangent AP9 or
the power o f the point A. [See § 6 (B)].
WorkiogRole Fitst make the coefficients o f Xt 9 yf and z2
each unity in the equation o f the sphere; then replace X 9 y and z by
the co-ordinates of the point from which we are to determine the
length o f the tangent; the value so obtained represents the square o f
the length of the tangent.
§ IS (A). The radical plane.
Definition I. The radical plane o f the two given spheres is the
locus o f a point from where the squares o f the lengths o f the tangents
to the two given spheres are equal.
Definition 2. The radical plane of the two given spheres is the
locus o f a point whose powers with respect to the two given spheres
are equal.
Clearly the two definitions of the radical plane given above
are equivalent.
To find the equation of the radical plane.
Let the equations of the two given spheres be
S i=X2+ ; 2 f z2+ Iulx + 2vxy + 2W1Z+ dx«=»0,
and Si^ a2 f V2+ Zt + 2 u% x + 2vty + 2w%z+ d%-0.
Ler (xl9 V1. Z1) be the co-ordinates of any point A. By defini­
tion the !ocus ol the point A is the radical plane if the powets of
A with respect to the two given spheres be equal. So we have, the
power of A w.r t. the sphc.«e (Si - O)
«■the power of A w.r t. the Sphcre(S2=sO)
i.e. Xi *+Λ β+ *ι1+ 2ι#ιΧ1 f 2v1<y,+2»v1Zi + rf1
Xit + yi* + V + Iu3tXl + 2vty i +2wtzl + dt
or 2 (U1- u >) X1 b > i V1- V5,? yk+2 (W 1 - Wi ) Z1 p dx- rf2-~=’0 .
The locus Ki A (.Vji Y1, Z1) i.e. the radical plane of the
two given >;ihere$ is
2 {u i" x v 2 \ V | v 2) y -i L (W|-· W8} zT rfj-^rfae O ···(*)
or S \—S8=O.
§ IS (B). The properties of the radical plane.
Property I. The radical plane i f two spheres is perpendicular
to the tine joining their centres.
The d .i/s oi the uoriwl io the radical plane (I) are Ol-Iz2,
Vi-V2, H1 H,.
I he co ordinates oi this centres of the spheres 6*i—0 and S8=O
arc uu —V|, -W 1) and V--Ifa, vi9 —w2) respectively.
The Sphere 301

Hence the d.r.’s of the line joining the centres* are Iz1- if2,
V1-V 2, W1- wi which are same as that of the r.orihal to the radical
plane.
Property II. The radical plane of two spheres passes through
their points of intersection.
Let the equations of the two given spheres be given by Si=O
and S2eaO. Theequation ofthe radical plane is given by S1- S 2=O.
Clearly the points which satisfy both S1-O and S2*=0 also satisfy
S1- S 2=O.
Hence the radical plane S1- S 2=O passes through the common
points of intersection of the spheres Si=O and S2-O .
§ 15 (C). The radical line (or radieal axis). (Avadh 1982)
Definition. The radicat planes o f three spheres taken two at
a time pass through a line which is said to be the radical line (or
radical axis) of the three spheres.
Let the equations of the three spheres in their standard form
be given by S1=O, S2=O and S3=O.
Taking two spheres at a time, the radical plane of
S1=O and S2=O is Si —S2=*0.
Similarly the radical plane of S2=O and S3=O is S2- S 3=O
and the radical plane of S3 =0 and Si=O is S3- S 1= 0.
These three radical planes clearly pass through the line
S1= S2= S 2
which is the equation of the radical line (or radical axis) of the
three given spheres.
§ 15 (D>. Radical Centre.
Definition. The radical lines of the four spheres taken three at
a time meet in a point which is said io be the radical centre o f the
four spheres.
Let the equations of the four spheres in their standard form
be given by S1=O, S2=O, S1=O, Si=O.
Iaking three spheres out of these four spheres 4C3 i.e. 4
radical liues will exist. The equations of these four radical lines
are given by
S1= S 2= S 3J S2=S3= S1J S3= S 3= S 1j S3= S 1= S 2.
These lour radical lines clearly intersect in a point, called the
radical centre, given by Si = S2=S 3==S3.
Clearly the radical centre will also lie on all the radical planes
of these four spheres taken two at a time.
§ 16. Coaxial system of spheres.
Definition. A system (or family) of spheres is called a co-axial
302 Analytical Geometry 3-D

system o f spheres If for all spheres any two o f them have the same
radical plane.
Remark. We have proved in property I of § I S (B) that the
line joining the centres of any two spheres is perpendicular to
their radical plane. In the case of co-axial system of spheres any
two spheres have the same radical plane, hence the centres of all
the spheres of a co-axial system always lie on a straight line.
Note. Some authors write co-axat for co-axial.
(A) To find the general equation of the system of co-axial
spheres.
Let 5,==0 and Si - 0 be the equations of two spheres. The
radical plane of these spheres is given by S i - S i = 0.
Now 5 ,+ μ ( S i - S i )-O ...(I)
represents a system of spheres for different values of μ and any
two members have the same radical plane. Let μχ and μ» be the
two values of μ and so two members of the system (I) are given
by S i+ μι (S1- S 1)=O and 5 ,+ μ , (5 ,—5,)= 0.
Subtracting, the radical plane of these two spheres is given
by (μ ι—μ») (S i—5 ,) - 0 or 5 , - 5 , = 0 ( V μιΦμ»].
Hence any two members of the system of spheres given by (I)
have the same radical plane. Hence (I) represents a co-axial
system of spheres.
Again (I) may be re-written as
(1+ μ ) 5 ,—μ 5 ,—0 or S i—{μ/(1 +μ)} 5, = 0.
T h isiso ftb efo rm 5 ,+ λ 5 ,= 0 ...(2)
The equation (2) is the general equation of a co-axial system
of spheres.
Similarly if 5 , - 0 be a sphere and P=O be a plane then
S+XP=0 ...(3)
represents a co-axial system of spheres.
(B) To find the simplified form of the equation of co-axial
spheres. (Punjab 1980; Madras 76)
Let us take the centres of all the spheres of the system to lie
on the x-axis so that the y and z co-ordinates of the centres of all
the spheres of the system are zero whereas the x-co-ordinate is
different for different spheres.
The equation of such a system of spheres is given by
**+y*+ i*+ 2kx+ </=»0. ...(I)
The Sphere 303

Let two members of the system (I) be


x 9+ y1+ z2+ Iu 1X^dieaO9 x2+ y2+ z2+2u2x+ d t =Q.
The radical plane of these two members (spheres) is
2 ( i/j—ι/t) *··(2)
The equation of the radical plane (2) depends upon the values
of d and hence will be in general different for different pairs of
spheres belonging to the system (I). If any two spheres belonging
to the system (I) should have the same radical plane, then d must
be an absolute constant. Then the common radical plane of any
two spheres belonging to the system (I) is the plane x=»0.
Hence the equation (I) is the equation of a co-axial system
of spheres with the common radical plane as the plane x«=**0#
where u is a variable and d is constant.
(C) Limiting points of a co-axial system of spheres.
Definition. The centres of the spheres o f a co-axial system which
have zero radius are called the limiting points of the co-axial system.
To find the limiting points. Let the equation of the co-axial
system of spheres be given by
x*+yt + z%+ 2ux+ d= 09 . (I)
where u is a parameter and d is an absolute constant.
When written in the centre-radius form, the equation (I)
becomes
(x + m)2+ —0)*+ (z—O)1« (u* —d). ...(2)
The radius of the sphere (2) is y/(u*—d). If the radius is
zero, then u2—d=0 or m—± \'d . The co-ordinates of the centre
of a sphere of the system (I) are (—u9 0,0). Putting the value
of u*=±y/dy the centres of the spheres of zero radius (i.e.9 the
limiting points of the system) are (± y/d , 0» 0).
Thus the limiting points of the co-axial system of spheres (I)
are\< fd %0, 0), (--y/d , 0, 0).
Role. To find the limiting points o f a co-axial system, equate
to zero the radius of the system and then the corresponding co-ordi­
nates of the centres give the limiting points.
SOLVED EXAMPLES (E)
Ex. I. Prove that the members of the co-axial system intersect
one another, touch one another, or do not intersect one another
according as d < t = or > 0.
Sol. Let the co-axial system of spheres be given by
x 2+ y2+ z2+ 2ux+ d= 0 ...(I)
where d is an absolute constant and u is a parameter.
304 Analytical Geometry 3-D

The common radical plane of the system (I) is x = 0 .


The radical plane Jt=-O intersects the sphere (I) in a circle
given by
*4“t~.V2+2* + 2ojt + (/= 0# JCeaO
or given by
y* + z2= —d, X=O. ...(2)
The equations (2) represent a circle of radius s/( —d) in the
jz-plane.
Now the following three cases arise :
Case I. When d < 0 i.e. d is —ve.
In this case (—d) is + ve and the radius y /(—d) is real I.e.
the circle of intersection is real, and hence the two spheres o f the
system Intersect.
Case II. Whend=O. The radius y/(—d) of the circle
reduces to zero I.e the circle reduces to a point circle and hence
the two spheres of the system touch.
Case III. When d > 0 i.e. d is +ve. In this case the radius
y /(—d) is imaginary I.e. the circle of intersection is imaginary,
and hence the two spheres of the system do not intersect.
Ex. 2. Prove that every sphere that passes through the limiting
points o f a co-axial system cuts every sphere of that system ortho­
gonally.
Sol. Let the system of co>axial spheres be given by
+>'*+ z*+ 2u.t+ (/= 0. -.(1)
Thelimiting points of the system (I) are (y/d ,0 ,0 ) and
( - S / d , 0,0).
Let the equation of aoy sphere be
**+^*+z*+2u1x + 2 v ,y + 2 tv ,z+ 4 = 0 ( ...(2)
If (2) passes through the limiting points of the system (I),
then we have
( /+ 0 + 0 + 2 0 ,^ ( /+ 0 + 0 + 4 = 0.
and (/+ 0 + 0- 20,-^ (/+ 0 + 0 + 4 = 0
or (/ + 2k, v ^ + 4 - - 0 and d —2ο,·γ/ί/+4=0.
Solving these, vie have u,<=0, 4 = - 4
Hence the equation (2) of the sphere becomes
*, +>’i + zi + 2v,y+2H’,z —(/=0. ...(3)
If the sphere (3) cuts the system of spheres (I) orthogonally,
then using ‘2u,i/,+2vIv2+ 2 tv ,tV i= 4 + 4 ’, we have
2.0.0+2.0.V, + lfy.wx= d - d or 0 = 0
which is true for all values of v, and tv,.
The Sphere 305

Hence (3) intersects the system of co-axial spheres ortho­


gonally.
Ex. 3. Provethat the equation x 2+ y2+ z* + 2 p y + 2 v z J = 0 ,
where μ and v are parameters, represents a system o f spheres passing
through the limiting points of the system X 2+ y2+ Z2+ lu x + J=- 0 and
cutting every member o f that system orthogonally,
Sol. This is another way of stating the Ex. 2 above. Simply
replace V1 by μ and W1 by v here.
Ex. 4. Show that the spheres whirh cut two given spheres along
a great circle all pass through two fixed points. [Kanpur 1978]
Sol. Taking the line of centres as the x-axis and the equa­
tion of the common radical plane as x= 0 (i.e. yz·plane), the
equations of the two given spheres are given by
x2+ y2+ z2+ :u lx+ d^Q , —0 )
and x8+.y2+ z 2+ 2w2x + J = 0 . .-*(2)
Let the equation of any other sphere be given by
x2+ V2H-Z2+ 2wx-f 2vy+2wz+fc=0. ...(3)
The sphere (3) will cut the sphere (I) along a great circle, if
their radical plane namely
2x (ii -W1) T Ivy+ Iw z+ k —J=O
passes through the centre (-W 1, 0, 0) of the sphere (I), and there­
fore, we have
—2wl (W-W1)-Kfc-J=O. ··(■*)
Similarly the sphere (3) will cut the sphere (2) along*a great
circle if
—2 w 2 ( w -w2)+fc—J= 0 . · (5 )
Subtracting (5) from (4), we get W=^w1-Kw2.
Putting the value of w in (4), we get fc=2ii|Ut+J.
Since the spheres (I) and (2) are given, the quantities W1, W2
and d are constants and hence w and fc are also constants.
Now it remains to prove that the sphere (3) passes through
two fixed points.
The abscissae of the points of intersection of the sphere (3)
with the x-axis i.e. with y= 0 = z are given by
x2+2wr+fc=0.
Both the roots of this equation depend upon w and fc, which
have already been proved to be constants.
Hence every sphere given by the equation (3), [different
spheres exist for different values of v and w) passes through two
fixed p >ints on the x-axis.
306 Analytical Geometry 3-D

Ex. 5. Find the limiting points o f the co axial system o f


spheres determined by the spheres
x*+J*.+z*+3x—3 j+ 6 = 0 , 6 v -6 z + 6 = 0 .
Sol. The equations of the given spheres are
S 3 x*+ j *+ z*+3 x - 3 j + 6 « 0 , 0)
and S"=x*+j*+z*—6 j - z + 6 = 0 . - (2)
The radical plane of the spheres (I) and (2) is given by
S - S ' = 0 i.e. f = 3 x + 3 j+ 6 z = 0 or P = x+ y+ 2 z= 0 .
.*. The equation of the co-axial system of spheres d.tcrrr.:r.-d
by the given spheres (I) and (2) is given by S+A P=^
or x*4.J*+ z*+3 x - 3 j + 6+A (x+v+2z)«=0
or * * + J * - f z 2+ x (3+A)—j (3 —A)+2Az+6=0. -.(3)
3 - A A I and ns radius
The centre of (3) is ( - 3 - A
2 * 2 ’
(A-3)* ■(A)2 - 6 Y \ V(6A* - 6) - ( 4 )
- j r 4+ x " 1■ 4
We know that the limitingpoints are the centres of the
spheres when radius=0. Hence equating the radius given by (4)
to zero, we get
6A2 -6 —0 or Af= I or A= ± I .
Putting the values of A in the co-ordinates of the centre of
the sphere (3), the limiting points are ( - 2 , 1, - I) and ( 1,2, I).
Ex, 6. Prove that the locus o f points whose powers with respect
to two given spheres are in a constant ratio is a sphere co-axial with
the two spheres.
Sol. Let the equations of the two given spheres be
X2+ y*+z*-\-2ulx-\-2v1y+2w1z+ d l —0, (1)
and X2+ y *+ z2+ 2u2x 4- 2v.2y 4 2w2z -f d9= 0. ( 2)
Let P tx 1, yl9 Z 1) be the variable point.
By the given condition,
the power of P (xu >y Z 1) w.r.t. the sphere (I) =r Jfc, (constant)
the power of P (X1, y ly Z1) w.r.t. the sphere (2)
* 1 *+ Vr24 ·Tl24- Iu1X1+ 2V1Jy1+ 2 W1Zx+ dx ,
or
*i2+.Vi4+ 4- 2m2Xi + 2vf Vi4- 2W2Z14 d - Γ
Cross-multiplying and simplifying, the locus of P (X 1, Ji, Zi)
is given by
2 (Zcw2 «i) Y , 2 (Zev2 V1 ) 2 (Zewa - W 1)
x 2+y*+z2+- y -t- k _ { -
Zc- I * k -1
. ( k d f—d\)__r\
k —I ...(3)
The Sphere 307.

The equation (3) clearly represents a sphere.


Now subtracting (3) from (I), the radical plane of the spheres
(I) and (3) is given by
2 (U1-U 1) x + 2 (V1-F 1) y + 2 (W1-W ,) z + M - J ,) = 0 . ...(4)
Similarly the radical plane of the spheres (2) and (3) is
2 (U1 - 1½) x + 2 (v, v,) y+ 2 (W1-W 1) ζ+ irf,—rf,)=0 ...(5)
The equations (4) and (5) are same and hence the sphere (3)
is co-axial with the given spheres (I) and ( 2).
Ex. 7- Show that the locus o f a point from which equal tan­
gents may be drawn to the spheres x*+y*+z*—I,
x*+y*+z*+ 2x -27 + 2z - 1 = 0, x*+ 7 *+z*—x + 47 —6z—2 = 0
is the straight line (x - 1)/2=-(7 -2)/5= (z—I
Sol. Let P (x, y, z) be the moving point. Then the lengths
of the tangents from the point P to the given spheres are
V(**+v*+z* - I), V(**+:v*+z*+ 2x—2> + 2z—l)
and \/(x*+ 7 *+z, - - x + 4}'—6z —2).
By the given condition, these lengths are equal. Hence on
squaring and equating, we get
x*+ 7 *+ 2*—I = ^ + y a+ z * + 2 x - 2 y + 2 z - 1
= x*+ y4+z·—x + 4 y —6z—2. ...(I)
From the first two relations, we get
2x - 2y + 2z = 0. ...( 2)
From the first and third, we get
—x + 4 y —62—1 = 0. ---(3)
The symmetrical form of (2) and (3) is
(x -1 ),/2=(7 -2 )/5= (2-1)/3. ...(4)
Remark. By definition the line (4) is the radical line (t.e.
radical axis) of the three given spheres.
Ex. 8 . Find the equation o f the radical axis in the symmetrical
form o f the spheres
Si=5x*+;y*+z*+2x+2.y+2z+2=0,
5 , = x*+ 7 *+ 2* + 4 x + 4 z+ 4 = 0 ,
and Sa= x*+ y * + z* + x +<>7 4z—2= 0. (Garhwal 1978)
Sol. The radical plane o f . the spheres 5 ,= 0 and S1= 0 is
S1 - S 1=O i.e. x -7 + 2 + 1 = 0. ...(I)
Again the radical plane of the spheres Si=O and St -O is
S1-S^=O i.e. x —4 7 + 62+ 4 = 0. ---(2)
The equations (I) and (2) are the non-symmetrical form of the
radical axis. The symmetrical form of the radical axis is
x/ 2 = ( 7 - 1)/:.= 2/ 3.
308 Analytical Geometry 3-D

_Ex· 9. Show that the locus o f the centre of a circle o f a radius


a, which always intersects the co-ordinates axes (rectangular) is
X V i o 2 - J t 2 - Z 2^ y V io t - Z t X 2) + z V i o 2 - X t - y * ) = a 2

Sol. . Let the centre of the circle be (α, β, γ) and let it cut
intercepts p, q, r on the axes so that (ο, β, γ ) lies on the plane
x v z ,
— + - - + — =1
p~ q~ r •••(I)
u B y ,
—+ — + — = 1.
P q r - ( 2)
Since radius of the circle=*».
Distance of («, β, y) from (p, 0, 0 )= a
( —cr.+p)2+fi!+ f = a i
^ P = V id t - β2—? )+ *
Similarly ς=ΒΛ/( α*~γ»-α*)+ β
r= V io * —ot*-fi*)+y
. J _ _________ I U - V jd t -Pt - / 2)
P u + V (tP -β * —Ύ*)ί **—(β*--β2—'·*)
U - V ia t Pt - tY*)
~ (ο*+β2+ γ2) - α 2
c . ·, , I β —ν ( & — Yt - **)
Similarly3 q—=7ixt. +fii
. Λ +, ty ic~
* )-d .t
I__ y—Vfrta—a» - β2)
r ~ (χ*+β2+)*)—α*
Put these values in (2;
« { * - V(O2^ f i i - V i )) +β {β -V (O 2- Y t -O t))
+7 I r - V io t- a* - β*)) =(α*+β2+ γ* - a 2)
Λ Locus of (α, β, γ) is
χ V (o t —y 2—zt)+ y V i a*- ζ* —χ)+ ζ V (ο* - Xt - y 2)= a2
Exercises
Find the equation of the sphere which passes through the
origin and makes equal intercepts of unit length on the
axes.
Ans. V + y 2+ z2- χ - y —z = 0.
2. A point moves so that the ratio of its distances from two
fixed points is constant. Show that its locus is a sphere.
3. Find the centre and radius of the circle
xt + yt + 2t + x + y + z —4=0, x + y + z = 0 .
Ans. Centre is the point (0, 0, 0) and radius is 2.
4. Find the equation of the sphere through the circle
**+,y*+2* =9, x+3y+4z<=5
The Sphere 309

and the point (I, 2, 3).


(Agra 1982; Gaahati 78; Garakhpar 81)
Aas. 3 (**+.y*+z*)—2x - 3 ^ - 4 2 - 2 2 —0.
5 Find the equations of the spheres which pass through the
circle x*+y*+z*=4, 2 x + 4 y + 2 = 9
and touch the plane Z=O.
Ans. Xi 2jc—4>—z f 5 = 0
and 5 (jc*+ j 4+2*)—8oc—16^—4z-l-16=0.
6. Show that the two circles
χ*+]Ρ+ζ*--γ+2ζ—0, x —y + z = 2 ;
x*+y*+zi + x —3j 4 z —5=0, 2x—y + 4 z = l
lie on the same sphere. Find its equation.
(Luckaow 1979, Avadh 79)
Ans. x*+ )*+ z?+ 3x-4y+ 5z—6= 0.
7. Find the equation of the sphere whose centre is the point
( 1 ,2, 3) and which touches the plane 3 ^ + 2 ^ + 2 + 4 = 0 .
Show also that the radius of the circle in which the sphere is
cut by the plane x + y + z = 0 is VOO).
A ds. X--Vy2-Vz - 2 x - 4 y - 6 z = 0 .
8. Show that the spheres
χ*+χ*+ζ2—4 χ -2 ν + 2 ζ 3 = 0
and x2+ y2+z*—8x—8y— 10z+41=0
touch externally.
9. Find the equation of a sphere which cuts four given spheres
orthogonally.
10. Prove that all the spheres, that can be drawn through the
origin and each set of points where the planes parallel to the
plane x/a } y/b+ z'c^Q cut the co-ordinate axes, form a
system of spheres which are cut orthogonally by the sphere
X2 Vy2+ z2+ l u x + 2 vy+2wz«=0,
if •au+bv+LW=0.
1 0

The Cylinder

§1. Cylinder.
(A) Definition. fKnnpnr 1979 ; Lucknow 77)
A cylinder is a surface generated by a moving straight line which
mares parallel to a fixed straight line and intersects a given curve or
touches a given surface.
The curve is called the. getting curve and the fixed straight
line is called the axis of the cylinder.
Any line on the surface of a cylinder is called its generator.
(B) To find the equation o f a cylinder whose generators are
parallel to the line x/l= ylm = zln and intersect the conic
ax*+2Jhxy+by*+2gx+2fy+c=Q9 z=0.
(Agra 1981 ; Garhwal 78S ; Kanpnr 77, 81; Rohilkhand 81)
The generators of the cylinder are given parallel to the line
xll= ylm = z/n. ...(I)
H encetheaxis of the cylinder is given by the equations (I);
'tLet P (xl9 yl9 Z 1) be a point on the cylinder, then the straight line
through the point P (xlt y l9 zx) and parallel to the axis (I) is a
generator and so its equations are
X - X 1 y - V 1 z -Z1
I m n . (2)
The equations of the conic are given to be
ax*+2hxy+ by*+2gx+2fy+c=09 z=0. ...(3)
The generator (2) meets the plane z = 0 in the point given by
( * - ¾ .* - ¾ 0 ).
Since the generator (2) meets the conic (3), hence the point
(Xl - I z lIn, y i—m zjn, 0) will satisfy the equations of the conic
given by (3), and so we have
The Cylinder 311

+ ^ ( * · - 'τ ) + ν ( Λ - ν - ) +£“ °
or a (Wjc1- Zzx)2+2A (Wjc1-Zz1) (Wy1-Wiz1)+ ^ (WV1-Twz1)2
+ 2gn (WJC1-Z z1) + 2/w (W^1-/WZ1)+w*c=0.
Λ The locus of P(JC1, yx, Z1) *·*· the required equation of the
cylinder is given by
a (nx /z)2+ 2A ( wjc - Zz) (wy - wiz) + b (wy - wiz)2
+ 2gw ( wjc - lz)+2fn (wy—wiz)+cw2= 0. ...(4)
Coro1Iary I. If the axis of the cylinder be taken as z-axis
whose d.c.’s are 0, 0, I, then putting Z= 0, wi=0, w*=*l in the equa­
tion (4) above, the equation of the cylinder becomes
0Xa+2Ajcy+Ay2+2gje+2/y+c==O. *··(5)
Remark. The readers should recall that the equation (5) repre­
sents a conic in two dimensional geometry whereas in three dimen­
sional geometry the equation (5) represents a cylinder with its
generators parallel to the z-axis (the co ordinate which is absent in
the equation).
Corollary 2. In view of corollary I above, the equation of
the cylinder whose axis is the z-axis and whose generators inter­
section the circle xa+ y 2=w2, z —0 is given by
*2+ y 2—a2.
(C) The equation o f the form f ( jc, y )= 0 represents a cylinder
with its generators parallel to the z-axis. (Rajasthan 1975)
Consider a curve in the xy-plane w'hose equation m two
dimensional geometry is / ( jc, y)= 0. If P (a , j? ) be any point on
it, then we have / ( a , /?)=0.
Now the co ordinates of P in three dimensions are (α, β , 0).
Draw a line through P ( a, 0, 0) parallel to the z-axis. Take
g(a, 3, z) any point on this line. The co-ordinates of the point
Q clearly satisfy the equation / ( jc, y)«=»0. Therefore the co-ordi­
nates of every point on the line PQ satisfy the equation/(jc,y)= 0
and hence the whole line PQ lies on this locus.
Thus the assemblage of lines (parallel to the z-axis drawn
through the points on the curve in the jcy-plane) is the required
locus which is, therefore, a cylindrical surface having generators
parallel to the z-axis.
Similarly, the equations / ( y , z) =:0 and / ( z , jc) = 0 represent
cylinders with their generators parallel to jc-axis and y-axis res­
pectively. u
312 Analytical Geometry 3-D

Corollary. The equation of the cylinder which intersects the


CurvejrX*, y, z)—0, φ(χ, y, z>—0 and whose generators are para­
llel to z-axis will be obtained by eliminating the co-ordinate z
between the equations f (*, y, z)—0 and φ(χ, *)=<>.
Similarly the equation of the cylinder intersecting the curve
f (jr, y 9 z)=0, φ(x 9y 9 z) = 0 with the generators parallel to .v-axis
(or y-axis) will be obtained by eliminating the co-ordinate jc (or y)
between the above equations.
§ 2. Right circular cylinder.
(A) Definition. If the generators are always at a constant,
distance from the fixed straight line, the cylinder so generated is
called a right circular cylinder. The fixed straight line is called
the axis and the constant distance is called the radius of the right
circular cylinder
(B) Tofind general equation o f a right circular cylinder.
(Jodhpur 1978 ; Gorakhpur 79 ; Kanpur 83 ; Rajasthan 76;
Meerut 86 P)
Let the radius of the cylinder be r and the equations of its
axis be (x —a)// = (y - 0>/w= (z - y)jn. ... ( I)
Clearly the axis (I) of the cylinder passes through the point
(a, /?, y), say the point A.
Consider a current point
P (x9y9 z) on the cylinder.
PQ perpendicular from the
point P to the axis (I). Then
P g = radius=r.
Now AQ = the projection of the join of A(α, β9 γ ) and ( jc, y 9 z)
on the axis (1-) whose d.r.’s are I9 m9 n
( χ - α ) ./ + ( ^ —β).>η \ (ζ - y).n

AP= the distance between the points A and P

From the figure, we clearly have


APi - A Q i=PQi
or (>— β)+« (z- r·}2
u "'-ex r f p + m 2+ n t
—r2
or { (* -« )2+ (y -p )\+ (2-y)*} (/2T W2+ » 2)
- { / (*-«) + m (y-fi)-tn ( z — y)}2=/·2 ( / 2+ /«*-f ,,2)
The Cylinder 313

or {n (y - β ) - m (z -?)}*+{/ (z ~ γ ) —η (x - a)}2
+{m (x —a)—/ (y ~P)}*=r* (l*+m*+n*) ...(2)
The equation (2) is the required locus of P and hence this is
the required general equation of the cylinder.
Corollary. If the axis of the right circular cylinder be z axis
then putting a=0=y® O , l= m = 0, and n= I in the equation (2),
the equation of the right circular cylinder in its simplest form is
given by x*+ jp—-r*.
§ 3. Tangent plane to a cylinder.
To find the equation o f a tangent plane to the cylinder whose
equation is
ajf*+2hxy+by*+2gx+2fy+ c= 0
at th° point Pixt, y u zx). Also to prove that this tangent plane touches
the cylinder albrig a generator. (Gorakhp'ir 1982)
The equation of the given cylinder is given by
ax*+2hxy+by*+2gx+2fy+c=0. -..(I)
Since Pfx1, y x, Z1) is a point lying on the cylinder (I), we have
OXi*+2hxxyx+ by i*+ Igxi + Ifyl +C=O. ...(2)
Let the equations of any line through the point P (X1, y x, Z1)
be ( x - x 1) / / = i v - y 1)/m =(z *z1)/«= r (say). ...(3)
The co-ordinates of any point on the line (3) are
(Ir + .V1, m r+}\, nr+ zt).
Therefore the points of intersection of the line (3) with the
cylinder (I) are given by
a (lr+ xt)*+2h (lr+xi) (mr+yx)+ b (m r+ ^ )8
+ 2 g (lr+xx)+ 2 f (mr+yi)+c=;0
or r* (aP + 2hlm+bm*) + 2r {/ (axi +hyt+g)+m (hxx+ byx+ f )}
+ (axx*+2hxxyx+byx*+2gxx+2fyx+ c)= 0
or r* (al*+2hlm-{ bm*)+2r {/ (axx+hyx+g)+m (hxx+byx+J ))= 0
-(4 )
[using (2)]
The equation (4) being a quadratic in r, gives two values of r,
Clearly one value of r is zero. Hence, if the line (3) is a tangent
line to the cylinder (I), then the other value of r must also be
zero and the condition for the same is that the coefficient of r
must zero
*·«· I (axx+ hyx+ g)+ m (hxx+byx+ f) = 0 . ...(5]
The tangent plane to the cylinder (I) at the point Ρ(χχ, y x, zx)
is the locus o!' the tangent line (3) for all values of /, m, n and is
obtained by eliminating the variables /, m, n between the equa­
314 Analytical Geometry 3-jD

tions (3) and (5). Hence the equation of the tangent plane to the
cylinder (I) at the point P (xl9 y l9 Z1) is given by
(x - x i) frxi+hy1+ g ) + ( y - y j (Ax1+ ^ y 1+ / ) = 0
or x (axi + hyt+ g)+ y (Ax1+Ay1+ / ) H a x 12+ by12+2Ax1y1
+gXi+fyi)=Q
or x (axi+hy1+g) + y (H x ^b y 1+ / ) + (gxi+fy!+c)=0 [using (2)]
or axxx+h (X jh+ *]/)+ byyx+g ( x + x ^ + / (^ + J 1Rc==O. . (6)
Second part. The equation (I) of the cylinder is of the form
f(Xy y)=~-Q and hence, in view of § I (C) above, the generators of
the cylinder (I) are parallel to the z-axis. Again P (x\9 V15 Z1) is a
point on the cylinder (I) and, therefore, the equations of a gene­
rator through the point P (X 1, y l9 Z1) are
{x - x 1)/0=(y yi)/0= (z -Z1)/1= r ' (say), ..(7)
The co-ordinates of any point on this generator are
(Xi>yi>'r'+Zi)·
Therefore, using the formula (6), the equation of the tangent
plane to the cylinder (I) at the point (xl9 yl9 / - '+ Z 1) is given by
Oxx1+ h (xyi+ x1y)-\ byy1+g ( x + x /) + / (v + J i)+ c = 0 . ...(8)
Clearly we see that the equations (6) arid (8) are identical.
The equation (3) of the tangent plane remains the same for all
values of r, it being free from r and, therefore, there is same tan­
gent plane at every point of the generator (7). Hence the tangent
plane to the cylinder (I) at XhepointP(Xl5Jh izI) touches the
cylinder (I) along a generator through the same point
Pfru yu zi)·
§ 4. Envelopiag Cylinder.
(A) Definition. The enveloping cylinder is the locus of the
tangents to a surface (sphere or conicoid) which are parallel to a
given line or in other words the enveloping cylinder is the cylinder
whose generators touch a given surface and are directed in a given
direction.
(B) To find the equation o f the enveloping cylinder of the sphere
x*+j>2-J-Z2=A2 whose generators are parallel to the line
xll=ylm-=*zln.
^Rohilknand 1982 ; Kanpur 76 ; Meerut 84)
Consider a point P (a, :i, γ) on the enveloping cylinder.
Since the generators of the cylinder are all parallel to the line
xjl=yjm —zln9 therefore the equations of the generator of the
cylinder through the point P are
The Cylinder 315

(x -« )//= (.v -P )/m = (z- y)/n=r (say). ...(I)


The co-ordinates of any point on the generator (I) are
(Ir+a, mr+β, nr+γ).
The equation of the given sphere is
x t + yi + zi —a2. ...(2)
The points of intersection of the generator (I) with the sphere
(2) are given by
f/r-f a)*+(mr+β)2+ (nr+ γ )2=α2
or r2 (P+m2+rP)+2r (U +m fi+ny)+ (e2+ ^2+ ya—a2)=0. ...(3)
The equation (3) being a quadratic in r gives two values of r
and hence two points of intersection of the generator ( I) with the
sphere (2). Ifthe generator ( I) touches the sphere (2) then the
roots of the equation (3) must be equal and the condition for the
same is
(/a+ mfi+ny)2= (P+ m 2+ n2) (α.2+ β 2+ γ 2 - a2)
[using Bt =AAC].
The locus of P (a, β, y) Le. the equation of the envelop­
ing cylinder of the sphere (2) is given by
y2+ z* -a 2) (P+m2+n2)=(lx+my-\-nz)2. ...(4)
SOLVED EXAMPLES
Ex. I (a). Find the equation o f a cylinder whose generators are
parallel to the x = y j2 = —z and whose guiding curve is
Ix 2+ 2y*=l, z= 0.
Or
Find the equation o f a cylinder whose generators are parallel to
the line x = y /2 = —z at. d passing through the curve
3x*+2y*=l, z=0.
Sol. The equations of the given guiding curve are
Ix 2-Y ly2= I , z= 0. *' . .(I)
The equations of the given line are
*/· = ^ /2 = z /(—l). ...(2)
Consider a current point P ( X 1, y u Z 1 ) on the cylinder. The
equations of the generator through the point P (Jt1, yi, zt) which
is a line parallel to the given line (2) are
(x -X ,)!I = ( V - ^ 1) I l = (Z -Z 1)A - I ) . . . (3)
The generator (3) meets the plane z= 0 in the point given by
x —X1 y —Vi O -Z 1 .
—P - l e- U i+Z „ yi+ 2 zlt 0).
316 Analytical Geometry 3-D

Since the generator (3) meets the conic (I), hence the point
(*y+Zi, ^i+2zj, 0) will satisfy the equations of the conic given by
(I)9 and so we hav*
jM*t+ zx)%+ 2 O’i+2zj)*=* I
3 (X12+!*!*! +Z1*)-1-2 0'ι*+4-νιζ1+4ζι2)=1
or .W -f 2^!2+ 11Zlt I-ZylZx+ 6zxxx- 1=0.
.·. The locus of P (X 1. y u Z1) i.e. the required equation of
the cylinder is given by
3x2+2j;2+ I lz2+ 8yz-t-6zx -1 = 0 .
Ex. I (b). Find the equation o f the cylinder whose generators
are parallel to the line x =^y/( 2)= ζ β and passing through the
curve x2+ 2y2= I, z - 0. (Agra 1982; Gorakhpar 74;
Meerut 83 S, 84 S; Kanpar 78; Lackiow 79)
Sol. Proceeding exactly as in Ex. I (a) above the required
equation of the cylinder is given by
3 (x2+ 'j ,3+ z 2) 2zx4 8yz 3=-- 0.
Ex. 2 Find the equation to the cylinder whose generators are
parallel to the line x =yj{ 2) = z/3, and the guiding curve is the
ellipse x* f 2_v2= I, z= 3.
(Avadh 1982; Allahabad 80; Agra 78; Garhwal 799 81;
Kurukshetra 76; Meerut 77. 89; Poojah 77; Rajasthan 77)
Sol The equations of the guiding curve are
x2-f2>>2= !, z = 3. ,..(»)
The equations of the given line are
x/\= y/( -2)=z/3. (2)
Consider a point P (xl9 ylt Z1) on the cylinder. The equations
of the generator through the point P (xl9 yl9 Z1) which is a line
parallel to the given line (2) are
x - Χ ι y - y i Z- Z1
I ~ - 2 3 ...(3)
The generator (3) meets the plane z= 3 in the point given by

= u · (*i *Z1+ I -2 . 3).


Since the generator (3) meets the conic (I), hence the point
(X1- izi + 1, Vi+ Jz1- 2, 3) will satisfy the equations of the conic
given by (I), and so we have
( * i- * * i+ i) f+ 2 (^ i+ f z ! - 2 ) 2= l
or (3xt Z!+3)*+2 ( 3 ^ + 2 z! - 6 ) 2= 9 .
Λ The locus of P (xl9 .Vl9 Z|) i* . Ihe required equation of
the cylinder is given by
The Cylinder 317

(3 x - z + 3 ) * + 2 ( 3 .y + 2 z - 6 ) * « 9
or Q-X2-I-z* Π -9— 6 z x + 1 8 x —6z
+ 2 (9y2+4z2+36+I2yz -3 6 j> -2 4 z ) = 9
or 9 * 2+ 1 8.y2+ 9 z 2+ 2 4 ; ; z — 6 z x + 1 8 x — 72>>— 4 8 z + 7 2 ^=O
or 3 x 2+ 6 > '2 -|-3 z 2+ 8 >yz 2 z x + 6 x -2 4 jr -1 6 z -f2 4 = 0 .
Ex. 3. Find the equation of the circular cylinder, whose gene­
rating lines have the direction cosines, I9 m\ n and which pass through
the f ix e d circle x*+Z1=G2, in ZOX plane. (Agra 1980)
SoL The guiding curve (circle) is given to be in the ZOX
plane Le. y —0 plane and hence its equation are
x2+ z2^a*9y= 0. .. (I)
Consider a point P (X1, ^1, Z1) on the cylinder. The equations
of the generator through the point P (xlf y 1%Z1) and with d.c.’s
I9m9 n are
(x —xi)/l= iy —.V1M m ^iz -ZlMn. . (2)
The generator (2) meets the plane ^=O in the point given by
X Xi 0 —.Pi z—zi i.e. (x, Iyl Sm, 0, Z1 - n v ^ m ) .
I m n
Since the generator (2) meets the curve (I), hence the point
(xi—lyilm, 0, Zl -H y Jm ) will satisfy the equations of the curve
given by ( I), and so we have
(*i - /.Vi/m)H (Z1- nyjm )2—a*
or (mxt—/vl)2+(m z1- ηγ^*·=α*ηι2.
Λ The locus of P (.X1, y x, Z1) i.e. the required equation of
the cylinder is given by
(imx—ly)*+(mz—ny)2~m*a*·
Ex. 4. Find the equation of the surface generated by a straight
line which is parallel to the line y= m x, z —nx and intersects the
ellipse X2Sa- + y2jbl = I, z = 0. (Avadh 1981 ; M .U. 90)
Sol. The equations of the guiding curve (ellipse) are
X2Ia2F y 2Sb2= :, z=0. ...(I)
The equations of the given line may be written as
x/1 =y!m=zjn. .. (2)
Consider a point P (xt, y lt Z 1) on the cylinder. The equations
of the generator through the point P (x„ y x, Z1) which is a line
parallel to the given line (2) are
(x -X1V I=C p- y ^ i m - f z - Z1Mn -.(3)
The generator (3) meets the plane z= 0 in the point given by
X-X1 ,P-JP1 C -Z 1
I m "(* ■ - Ϊ - - " M
318 Analytical Geometry ?-£>

Since the generator (3) meets the co :ic (I), hence the point
(xi—zjn , y x—MZ1In90) will satisfy the equation of the conic given
by (I), and so we have
(IIati) (^1-::,//0+(1/*2) ( ji—m zjn)2= I
or ft2 (nxx - z , ) 2+ 3 2 (η γ τ m z ,)2= a 2ft2n 2.
Λ The locus of P (xl9 V 1 , .Z1) i e. the required equation of
the cylinder is given by
b2 ( n x - z ) 2+a2 (r\y—mz)2 =Iib1U1.
Ex. 5 (a). Find the equation o f the cylinder which intersects the
curve ax2+by2+CZ1= I 9 lx+ m y+ nz= p and whose generators are
parallel to the axis o f x. (Meerut 1978, 82 , Lucknow 82)
Sol. Thfe equations of the guiding curve are
ax 2+ ftV2+ C Z 2 = X ... ( I)
and lx+my+nz=p. :-(2)
Now the equation of the cylinder whose generators are para­
llel to x-axis will not contain the terms of x. [See § I (C)]. Hence
the required equation of the cylinder is obtained by eliminating
x between the equations (I) and (2), and so is given by

[ v Fmm (2,. χ
or a (p my - nz)2+ ft/2+ !+ cl2z2=- /2
or a (p2+ m2y *- \ n Z 2 -''lpmy -2pnz-{ 2mnyz)+bl2y1+cl1z2= l 2
or (am2+ bl2) y'+ (an2+cl2) z* + 2amnyz—lampy
—2anpz+(ap —/2)=0. Ans.
Ex. 5 (b). Find the equation o f the quadric cylinder which
intersects the curve ax2+by2+cz' = I, lx+ m y+ nz—p and whose
generators are parallel to z-axis.
(Kanpur 1982, 83 ; Bundelkhand 79 ; Meerut 85 P)
Sol. Th^ equations of the guiding curve are
ax2+ft;r+C22= l ...(I)
and lx+ tn y+ n z= p. .. (2)
Since the generators of the cylinder are parallel to the z-axis,
therefore the required equation of the cylinder is obtained by
eliminating the z co-ordinate between the equations (I) and (2),
and so is given by
a x 2+ b y 2 +c { ( p — l x — m y ) In}2= \
or an% x + b t v y 2 + c ( P 2 j i- V x 2-1 i;ily 2— 2 / t i x + 2 l m x y — 2 p m y )= /r
The C y lin d e r 319

or (cP+ an2) x 2+ ( cm2+ bn2) y2+Iclmxy Icplx - lcpmy


+ (cp? - n2) = 0. Ans.
Ex. 5 (c). Find the equation o f the cylinder with generators
parallel to the x-axis and passing through the curve
ax2+ by2=2cz, lx+ m y+ nz= p.
Sol. The equations of the guiding curve are
ax2+ by2 -2cz = 0 - (1)
and lx+ m y+ nz= p. .(2 )
Since the generators are parallel to the x-axis, therefore
eliminating λ*betv/een the equations (!) and (2), the equation of
the required cylinder is given by
a {(p my -nz)/l}2+by2—2cz~0
or a (p2+ rri'y2+n2z2 -2pmy+2ninyz—2pnz)+bl2y 2 2cl2z= 0
or (am^+bl2) y 2+an2z2+2amnyz- 2apm y-2 (apn+cl2) z = 0 .
Ans.
Ex. 5 (d). Find the equation o f the cylinder with generators
parallel to z-axis and passing through the curve
ax2+ by2—2cz9 lx+m y + nz--=p.
(Lucknow 1977; Kanpur 79)
Sol. Proceeding as in Ex. 5 (b) above, the equation of the
required cylinder is
anx2+bny2+2c (lx+my) —2pc=Q.
Ex. 6. Find the equ ation of the right circular cylinder of radius
2 whose axis passes through (1,2, 3) and has direction cosines propor­
tional to 2, —3, 6. (Agra 1976, 78; Lucknow 83; Garhwal 78, 82;
Meerut 74, 77, 83, 84 P, 85; Kanpur 80)
Sol. The axis of the cylinder passes through (1,2, 3) and has
d r.’s 2, —3, 6 and hence its equations ere
(jc— 1)/2 = {y 2 ) / ( - 3) —(z --3)/6. -(1 )
Consider a point P (x , y, z) on the cylinder. The length of
the perpendicular from the point P (x ,.y , z) to the given axis (I)
is equal to the radius of the cylinder i e. 2. H encethe equation
of the required cylinder is given by [See § 2 (B) equation (2). Put
/= 2 , m-= 3, /i = 6; « = i, ¢ = 2, γ —3; and r = 2 in the equation 2)
of the cylinder;.
{6 O' - 2 ) - ( - 3) (z-3)}M -{2 (z —3) —6 ( * - ! ; } 2
+ ( ( - 3 ) ( j c — 1 ) - 2 ( y - 2 ) y = ( 2 f { < 2 )* + ( - 3 ) ^ + - ( 6 ) 2}
or (6v+ 3z -- 2 1):·-1 (2z- 6x)2-b(--3x-2;M -7)2= 4 (49)
or 45x'-f-40y'~|- I3z2+ 36;yz—24zx+12xy 42* -280;>
- 126z 1I- 294 = 0. Ans.
320 A n a lytica l G eo m etry 3 -D

Ex. 7 (a) Find the equation o f the right circular cylinder of


radius 2 and having as axis the line
ix - 1) / 2 - ( v - 2 ) = (z -3)/2.
(Avadh 1978; Kanpar 79; Punjab 81: Meerut 85S)
Sol. The equations of the axis of the cylinder are
(x \)/2 = (y -2 )/1 = (2 - 3)/2. -0 )
Consider a point P (x, j \ z) on the cylinder. The length of
the perpendicular from the point P (x, y, z} to the given axis (I)
is equal to the radius of the cylinder i e. 2. H en ceth ereq u ired
equation of the cylinder is given by [See § 2 (B), equation (2)]
(2 (Λ'—2) —I.(z - 3)}2+ { 2 (z—3) 2 (x—1 » ?+{1 ( x - l ) - 2 (jp-2)}2
= (2)2.{(2)2+ (l)* + (2)*>
or (2y—z l)2+ ( 2 z - 2 x - - 4 ) 2-i-(x—2j H-3)2= 36
or 5x2+ Sytljr Sz1-A y z 8zx—4xv+ 22x—I6y—1 —10=0. Ans.
Ex. 7 (b). Find the equation o f the right circular cylinder o f
radius 2 who:e axis is the line (x— l)/2=>V3 = (z -3)/1.
(Berahampur 1976(S); Rohilkhand 82; Punjab 78)
Sol. Proceeding exactly as in Ex. 7 (a) above, the equation
of required cylinder is given by
IOx2+ 5y°· + 13z2— 12 x y —6>>z—4zx—8x+ 30y—74z+ 59=0.
. Ex. 8. Find the equation o f the right cylinder whose axis is
x/2e»y/3 = r/6 and radius 5.. (Gorakhpur 1978, 81)
Sol. The equations of the axis of the cylinder are
( r - 0 ) '2 - ( y -0 } /3 = (z -0 )/6 . . (1)
Consider a point P (x, v, z) on the cylinder. The length of
the perpendicular from the point P (x, y y z) to the given axis (I)
is equal to the radius of the cylinder i.e. 5. Hence the required
equation of the cylinder is [See § 2 (B). Note that here
a=/} = y=0; /= 2 , w - 3 , and r= 5 ]
(6y -3z)2+ (2 z-6 x )>f (3x - 2 j02= 52 {(2)2+ (3)2+(6)2}
or 45x2+ 40y2+ 13z-—36yz—24zx *■ 12xy— 1225= 0. Ans.
Ex. 9. Find the equation o f the right circular cylinder whose
axis is x —2= z , y = 0 and which passes through the point (3, 0, 0).
(Agra 1981)
Sol. The equations of ihe axis of the cylinder may be
written as
( x ~ 2 )/1= ( y - 0 ) / 0 « ( z - 0 ) / 1. .. (I)
First We shall find the raJius r of the cylinder.
We know that
r= th e length of the perpendicular from a point (3, 0, 0) on the
cylinder to the axi* (I)
T h e C ylinder 321

~ V S W c W o W V if0-O-OO)'+(I.ο - 1.(3-2)}"
+ {0-(3-2) -1.0}·]
= 1/V2-
Consider a point P (x, y, z) on the cylinder. The length of
the perpendicular from the point P (x , y, z) to the given axis (I)
is equal to the radius of the cylinder. Hence the required equation
of the cylinder is (See § 2 (B)]
(l.y- 0.z)2+ { l.z—l.(x—2)}2+{0.(x—2)—l.y}2= (l/\/2 )* (1 + 0 + 1 )
of y*+(r—x + 2 )2+y2= l *
or x2+2y2+ z 2~ 2 zx —4 x+ 4z+ 3--0. , Ans.
Ex. 10 (a). Find the equation o f a right circular cylinder des­
cribed on the circle through the points A {a, 0, 0), B (0, a, 0) and
C (0, 0, a) as the guiding curve.
S61. We are given three points A (a, 0, 0), B (0, a, 0) and
C (0, 0, a). Let the fourth point be taken as O (0, 0, 0). The
equation of the sphere OABC is [See Ex. 6 (a), set (A) of the
chapter on spherel
x2+y*+z*—flx—ey— b0. -.(1)
The equation of the plane ABC is xja+ yla+ z/a—l
or x+y+ z= e. --(2)
Theguidingcircleis given by the equations (I) and (2)
together. '
Since the cylinder is a right circular cylinder, hence the axis
of the cylinder will be perpendicular to the plane (2) of the circle
and thus the.d.r.’s of the axis of the cylinder are I, I, I.
Let P (X1, yu Z1) be any point on the cylinder. The equations
of the generator through P (X1, ylt Z1) having d.r.’s 1,1, I are
(X-X1)/! «-(y—y i)l\—{z—Z1)/1= r (say). ...(3)
Any point on the generator (3) is (r+ x l5 r+ ylf r + z j.
Since the generator (3) meets the guiding circle, hence the
point (r+ x „ r+ ylt r+Zj) will satisfy the equations (I) and (2)
of the circle, and so we have
('■+•«i), + (i-+yj)!!+ ( r + z 1)*—a (r+ xfjihe ( r + y ^ - f l ( r + z ^ - O
and ('•+*i)+(>’+ y 1) + ( r + z 1) = a
or 3r2+ r {2x1+2y1+2z1—3a)+(x1t + y1, + z1i —ax1—ay1—az1)=Q
...(4)
322 A n a ly tic a l G e o m e try 3 -D

Eliminating r between.(4) and (5), we get


3 -1 (a—X 1 - . P i - Z 1 J H i (a - X 1 - y t - z t) (2xl +2y1+ 2z1—3 a )
+ ( x P + V + z p —Ox1-Q y1- OZ1)=O
or {a*A-x1*+y1i+ z1i —2ax1—2ay1—2az1+ 2x1y l +2xlz1+ 2y1z1)
+ (2flXj+ Iay1+ Iaz1- I a i- X t* - 2x1y1—Ix 1Z1+ 3ax,—2x,y t
—2 V —I y 1Zi + ^a y t - Ix 1Z1- 2^zi - 2zx*+ Baz1)
+ 3 ( x p + V + z p —axi—ayi—βζχ)= 0
or 2 ( V + V + V ) - 2 (JP1Z1+ Z1Xj + X1^ ) - 2a2= 0
or X1* + V H- V Jf1Z1- Z1X1 - X 1JP1= a*.
Λ The locus of R i.e. the required. equation of the cylinder
is given by
tR+y*H-z*—yz—zx—x y = a*.
Ex. 10 (b). Find the equation o f the right circular cylinder
described on the circle through the points A (I, 0,0), B (0, I, 0) and
C (0, 0, I) as the guiding curve.
(Gorakhpur 1975; Burdwan 75; Meerut 87 Pj
Sol. Proceeding exactly as in Ex. 10 (a) above or putting
a = I there, the required equation of the cylinder is
x*+y*+z*—yz - z x —xy= I.
Ex. 11. Find the equation o f the right circular cylinder which
l passes through the circle x2H-^"+z*=9, x —_p+i=3.
V (Avadh 1978; Meerut 79, 83 (S), 89 (S); Kanpur 80, 81;
79, SG; Jodhpur 73; Gorakhpur 77)
Sol. The equations of the guiding circle are
x*+JP*+z*=9 ...(I)
and x —^ + z = 3. ...(2)
Since the cylinder is a right circular cylinder, hence the axis
of the cylinder will be perpendicular to the plane (2) of the circle
and so the d.r.’s of the axis of the cylinder are I, - I, I.
Let P (x„ ^lt Z1) be a point on the cylinder. Theequations
of the generator through P fx 1, JP1, Z1). Tiaving d.r.’s I, —1, I are
(X-X1)/1e O' - V / ( —I) = (z Z1Vl = r (say). ...(J)
Any point on the generator (3) i.s (r+ x lt —r+ yi, PH-Z1).
Since the generator (3) meets the guiding circle, hence the
point (r+ x lt —r+ ylt r+ z ,) will satisfy the equations (I) and (2),
and so we have
( pH-X1P - K - P + V * + ( r H-Zi)*=9
and (p+x1) - ( - p + V + ( p + V = 3
The Cylinaer 323

or 3r*+2r (« !+ V j+ z O + O rZ + V + z /—9)= 0 ...(4)


and r = i (3-Λχ+,ν,—z,>. ...(5)
Eliminating r between (4) and (5), we get
J (3—Χ ι+ Λ —Zi)*+f (3—JCi+^i—z,) (X1+.V1+Z1)
+ (* /+ 7 /+ 2 /-9 )= 0
or * / + 7 / + 2 / + 7 ι 2ι -2 !* ι + χ,7ι =9. .
Λ The locus of P (xlt y u Z1) or the required equation of the
cylinder is xa+y*+z*+yz—zx+ xy= 9. Ans.
Ex. 12. Find the equation o f the enveloping cylinder o f the
sphere x t + f t+ zt —2x+Ay - 1 = 0 having its generators parallel to
the line x = y = z .
(Avadh 1981; Rohilkhand 83; Kanpnr 83; Meerat 83,88, 89S)
Sol. [This question is based on § 4].
The equation of the sphere is
x*+ 7 *+z*—2x+4y—1=0. ...(I)
The generators of the enveloping cylinder are parallel to the
line x=y= z. .(2)
Consider a point P (α, β, γ ) on the enveloping cylinder. The
equations of the generator of the cylinder through the point
P (α, β, γ) and parallel to the given line (2) are
(x - a)/1- O '- /? ) /! —(z—y)/l = r (say). ...(3)
Any point on the generator (3) is (r+ a , r +β, r+ y ). The
points of intersection of the generator (3) with the sphere (I) are
given by
(r+ a)» + (r+ /?)* + (r+ y )* -2 (r+ a )+ 4 (r+ β) -1 = 0
or 3r*+2r (a+ j?+ y + l)+ (a* + /J* + y * -2 a + 4 j5 -l)= 0 . ...(4)
The equation (4) being a quadratic in r gives two values of r.
Since the generator (3) touches the sphere (I), the two values of r
obtained from (4) must be equal and the condition for the same is
{2 (a + /? + y + 1)}* 4 .3 .(α * + ^ * + ^ -2 α + 4 /? - 1)=0
[using Bi-AAC=O]
or (a+yff f y + l)* - 3 (a*+£2+y* -2 a + 4 /? ^ l)= 0
or a2+/?*+}'2 -yffy—y a —α/?—4α+5β—y —2=0.
.·. The locus of P (α, β, y) i.e. the required equation of the
enveloping cylinder of the sphere ( I) is given by
x i + y<s+z*—y z - z x - xy—Ax+5y—z - 2=0. Ans.
Ex. 13 Show that the enveloping cylinder o f the conicoid
Ws+fey*+cz*=l with generators perpendicular to the z-axis meets
•he plane z = 0 in parabolas. (Meerut 1984P)
324 Analytical Geometry 3-D

Sol. The equation of the conicoid is


ax2+by2+cz2= ]. ...(I)
The direction cosines ofthez-axis are 0 ,0 ,1 . Hence the
direction ratios of a line perpendicular to the z-axis can be taken
as I9 m, 0 because O.Z+O./w+1.0=0.
Consider a point P (a, /?, γ) on the cylinder. The equations
of the generator of the cylinder through P (α, β 9 y) a:;d having
d.r.’s I9 m9 0 are
(x -* )/l= (y — 3)/m = ( 2—y )/0 = r (say). ...(2)
Anypoint on the generator (2) is (lr+<x, mr+ β, y). The
points of intersection of the generator (2) \v:th the conicoid (I)
are given by
a (ilr + a f+ b (ΜΓ+β)2+ογ2= \
or r2 (al2+ bm2) + 2r (alot+bmfi) + (aa2+bfi2+cy2-1 ) = 0 . ...(3)
The equation (3) being a quadratic in r gives two values of r.
Since the generator (2) touches the conicoid (I), the two values of
r obtained from (3) must be equal the condition for which is
{2 (ala+bmfi)}2—4.(aF+bm2) (acP+bfi2+ Cy2- X )
[using B2—4AC==0]
"or (afa+bmfi)*—{aP+bm2) (a<x2+bfi2+ cyt —X)=0.
Λ The locus of P (α, β, y) i.e\ the required equation of the
enveloping cylinder of the conicoid (I) is given by
(alx+brnyf—faP+bm2) (ax2+by2+cz2— 1)=0. .. (4) Ans.
The sections of (4) with the plane z= 0 is
(alx+bmy)2—(aP+bm2) (ax2+by2 1)=0, z= 0
or OtI2Xt+ lablmxy+ b2m2y2—O2Px2—abm2x2 abPy2
- b 2m2y2+(al2+brh2)= 0, z=0
or lablmxy—abm2x2—abPy2+ (at2-f bm2)= 0, z = 0
or ab (mPx2—Ilmxy+ l2y 2)=--(al2 bm2), z= 0
or ab ( w z - ly)2=(aP+bm2)9 z= 0. ...(5)
In the first of the equations ( 5), the second degree terms form
a perfect square and hence (5) represents a parabola in the z~fl
plane.
Ex. 14. Find the equation of a right circular cylinder which
envelopes a sphere o f centre (a, b, c) and radius /*, and has its gene­
rators parallel to the direction I, m, n.
Sol. Proceed exactly as in Ex. 12 above. The required
equation of the enveloping cylinder is given b y
{/ (x—a)+m (y—b)+n (z —c)}2
= ( / 2+ / h 2 ',-H2; if.v—a)2+ ( y —6 ) * - L ( z —c)e- r *
1 1

The Cone

§ I. Cone.* Definition. Acone is a surface generated by a


moving straight line which passes through a fixed point and intersects
a given curve or touches a given surface.
The fixed point is called the vertex and the given curve (or
surface) is called the guiding curve (or guiding surface) of the cone.
The moving straight line is called a generator of the cone.
Quadric cone. A cone which is cu* by a straight line other than
the generators only at two points is called a quadric cone or in other
words ‘a cone whose equation is of second degree is called a quadric
cone\
§ 2. The cone with the vertex at the origin.
To prove that the equation o f a quadric cone with its vertex at
the origin is a homogeneous equation of second degree in X 9 y, z.
(Lucknow 1981 ; Kanpur 78)
Let the cone with its vertex at the origin O be represented
by the general equation of second degree in X9y and z i.e. by
ax2+ by*+ Cz2+ Ify z + Ig zx + Ih xy+ lux+ 2vy+ Iw z+ </«0. ... ( I).
Consider a point P ( X ^ y l 9 Z 1 ) on the cone. Theequations
of the generator OP are
O y - O 2—0
X1 -O ^1- O Z 1 0 0Γ JC1 y t Z 1 r ^say)* ...(2)
Any point Q on the generator (2) is (rjclt rylt Tz1). Since the
line OP is a generator of the cone (I), therefore every point on it
like Q must lie on the cone (I) whatever the value of r may be.
Thus, we have
r * (Ctx12+ b y f + c z f + Ify1Z1+ 2gz1x1+ 2 h x 1yf)
+ Ir (w*i+Vy1+WZ1H rf= O . ...(3)
Since (3) holds for all values of r, therefore it is an identity
in r and so the coefficients of r8, r and constant term must be
separately zero i.e., we have
326 Analytical Geometry 3-D

ax\ *+byx2+ Cz12+ 2fylzl+2gzixl + Ih x tf1=O9 .. .(4)


!/^+ViV1H-WZ1=O (5). andrf=0. .(6)
The relation (5) shows that the point P (X1, ^,-Z1) lies on the
plane wx+vy+wz=0 if u, v and w are not all zero but this con­
tradicts our assumption that the surface is a cone. Hence
W=V=W=O.
The relation (6) is clearly true since the cone passes through
the origin.
Λ Substituting the values of U9 v, w and d in (I), the equa­
tion of the cone with its vertex at the origin is given by
ax2+ by2+Cz2+2fy z + Igzx+ Ihxy=O f ... (7)
which is a homogeneous equation of the second degree.
Converse. To prove that every homogeneous equation o f second
degree in x, y and z represents a cone whose vertex is at the origin.
(Lucknow 1981; Kanpur 78)
The most general homogeneous equation of second degree in
Xp y i z is given by
ax2+by2+CZ2+ 2fy z + 2gzx+ 2hxy= 0. ... (I)
If equation (I) is satisfied by any point P (X1, y l9 Z1), then for
all values of r, the point (/iX1, ryl9 Tz1) clearly satisfies the equation
(I). Since (rxl9 Tyl9^rzl) are the general co-ordinates of a point on
the line through the origin O (0, 0, 0) and the point P (X1, y l9 Z1),
therefore every point on the line OP lies on the equation (I) and
ro the line OP lies wholly on the surface (I).
Therefore, the surface (I) is generated by the straight lines
through the origin and hence it is a cone with its vertex at the
origin.
Note. Incase abc+2fgh a f2 -bg2—ch2=Q the equation (I)
will represent a pair of planes. See the chapter on The Plane.
Working method. I f the two equations representing the guiding
curve are such that the one equation is of first degree then the requi­
red cone with the vertex at the origin is obtained by making the other
equation homogeneous with the help of the first equation.
[Note that by § 2 the equation of the cone with the vertex at
the origin is homogeneous.I
Ifboth the equations representing the guiding curve are not
of first degree, then we introduce a new variable say. We make
each of the two equations homogeneous (in x, y 9 z and /) by
multiplying the terms with appropriate powers of t. Now the
The Cone 327

required equation of the cone with the vertex at the origin is


obtained by eliminating the new variable t between these two
homogeneous equations.
SOLVED EXAMPLES (A)
Ex. I. Find the equation of the cone with vertex at (0, 0, 0)
and patsing through the circle given by
x*+yi + z* + x—2.V +3Z-4-0, χ —y+ z= 2 .
(Bundelkhand 1978)
Sol. The equations of the given circle are
x*+ y*+ z*+x- 2y+3z—4=0, ---(I)
and x - y + z = 2 or (x -,v + z )/2 = l. ...(2)
Making (I) homogeneous, with the help of (2), the equation of
the required cone with the vertex at the origin is given by
(x*+y2+ z * )+ i (x —2y+3z) ( x - y + z ) - 4.J (x—y+z)*=!)
or 2(x*+y*+z*)+(x*—3xy+4zx+2y*—5yz+3z*)
- 2 (x*+y*+z*—2xy+ 2zx—2yz)=0
or x*+2ya+3z*+xy—yz=0.
Ex. 2. Find the equation o f the cone whose vertex is (0, 0, 0)
and which passes through the curve of intersection o f the plane
lx+ m y+ nz= p
and the surface ax*+by*+cz*= I. (Meerut 1983)
Sol. The equations of the given curve are
ax1+by*+CZi - 1, ~ ( 1)
and (lx+my+nz)fp= I. - ( 2)
Making(I) homogeneous with help of (2), the equation of
the required cone with the vertex at the origin is
(ax*+ by*+ cz*) =-{(lx+my+ nz)/p}*
or p* (ax*+by*+cz*)=(lx+my+nz)*. Ans.
Ex. 3. Find the equation o f the cone whose vertex is the origin
and base curve is given by
ax*+by*=2z, lx+ m y+ nz= p. (Gorakhpur 1976)
Sol. The equations of the given cutve are
ax*+by*=2z, ...(I)
and (lx+ m y+ nz)jp—\. ...(2)
Making (I) homogeneous with the h;lp of (2), the equation of
the required cone with vertex at the origin is
ax*+by2—2z.(lx+my+nz) \p
or p (ax*+by*)=2z (lx+my+nz). Ans.
328 Analytical Geometry 3-D

Ex. 4. Prove that the equation o f the cone whose vertex is the
origin and base the curve z = k ,f(x ,y )= 0 isf(xklz, ykjz)=0.
(Kanpur 1979, 81; Meerat 87, 89 S)
Sol. Let x/l—ylm=zln -(0
be a generator of the cone.
Since it meets the base curve z = k , f (x, y)= 0, therefore
x y k ... I , m ,
-r = — m=—, SOthat X·=- k, V=-- AC.
I in it η n
Putting these values of x, y i n / (x, y)= 0, we get

' ( H - H - 0· .( 2)
Eliminating /, m, n between (I) and (2), the locus of
xll= ylm = zln is the cone

' ( T t- T i H '
Alternative Solution. We know that the equation of a cone
whose vertex is at the origin is homogeneous in x, y and z. From
z —k, we have kjz·= I. Therefore m a k in g /(x, y )= 0 homogeneous
with the help of z= k , the equation of the required cone is
f(x k fz , yk/z)—0.
Ex. 5. Find the equation of the cone whose vertex is the origin
and base the circle x —a, y*+z*=*b2 and show that the section o f the
cone by a plane parallel to the plane XO Y is a hyperbola.
(Meerut 1973, 84)
Sol. The equations of the circle are
ya+ z* = p ...(I), and jc/e-1. (2)
Making (I) homogeneous with the help of (?.), the equation
of the required cone with the vertex at the origin is
y*-\-za= P (x/e)* or a* ()ΐ*+ζ*)=Ρχ*. ...(3) Ans.
The section of the cone (3) by a plane parallel to the plane
XO Y Le. by the plane z —c is the conic given by
a* Cys-S-Ca) -- z= c
OT P t f - O aP t = O i C*, Z e C
which is clearly the equation of a hyperbola.
Ex. d. The plane x/a+ y/b+ z/c= I meets the co-ordinate axes
in A, B, C. Prove that the equation of the cone generated by the
lines drawn from O to meet the circle ABG is 1
yz (b/c+clb)+zx (c/a+a/c)+xy (a/b'+bia)=0.
(Agra 1979; Bundelkhand 78; Punjab 82; Nagpur 78;
Rohilkhand 77)
The Cone ' 329

Sol. The given plane is xja+ yib+ zlc= l. ...(I)


The points A, B, C are (a, 0, 0), (0, b, 0) and (0,0, c).
The equation of the sphere OABC is
&A-yi + zi - a x - b y —cz= \\ -(2)
[Refer Ex. 6(a), page 246, in the chapter on sphere] ,
The plane ( I) meets the sphere (2) in the circle ABC. Thus
the equations (I) and (2) together give the circle ABC.
Making (2) homogeneous with the help of (I), the equation
of the cone generated by the lines drawn through the origin O to
meet the circle ABC is given by
^■\-y2+ zi —{ax+by+cz) (xja+ylb -|-z/c)=0
or yz (b/c+clb)+zx (eja+a/c) +xy(a/b -1- bla) ~ 0.
Ex. 7. Find the equation to the cone with the vertex ut the
origin and which passes through the curve
xt+y*+zt + x —2 y + z—4 = 0 , xt + y2+ zt + 2x—‘i y + i z —5=Q.
Sol. The equations of the guiding curve are
λ*+}’*+**+ *—l y + ϊζ —4=0, ...(I)
and x*+y2+ zi + 2x—3 y + 4 z -5 = 0 . -.(2)
Subtracting (2) from (I), we get
—x + y —2+1==.0 or χ —y + z = I. ...(3)
Making (I) homogeneous with the help of (3), the required
equation of the cone with the vertex at the origin is
& + y*+ z*+(x—2y+ 3z) (x—y+ z) - 4 (x y+z)*=0
or 2x?-\-yi —5xy—3yz+4zx=0. ...(4) Ans.
Note. We can also make the equation (2) homogeneous with
the help of (3) and then also we shall get the equation (4) as the
equation of the required cone.
Ex. 8. Find the .equation of the cone with the vertex at the
origin and which passes through the curve
x*ld*+y2lb>+ZtIci = I, x*l«t + yiWt =2z. (Meerut 1971)
Sol. The equations of the guiding curve are
XiIat^ y tIbt A-ZiIci= I, and XtId2-YyiIpt= Iz.
Making these equations homogeneous with the help of a new
variable t [See working method on page 326], we get
X t Ia t - I f y i I b t A -Z t Ic t = I t , - ( 1)
and XtIat -YytIpt = 2zt, - ( 2)
Eliminating between (I) and (2), we get
XiIat -YyiIbt A-ZiIct=
330 Analytical Geometry 3-Z)

or 4zS( ^ - + £ - + ^ - ) = ( i i - + ^ ) ·
This is the required equation of the cone with the vertex at
the origin.
Ex. 9. Planes through OX, OY include an angle a. Show that
their line o f intersection lies on the cone
z2 (x2+ y2+ z 2) = x 2y 2 tan2 a.
(Agra 1978; Meerut 76, 87, 89; Lucknow 76, 80; Guruiianakdev 76;
Nagpur 76)
Sol. The equation of any plane through OX i.e., the line
y = 0 , z==0 is y +λίΖ—0
or ψ 0.x+ I ^ + A 1Z = O . -O )
ySimilarly the equation of any plane through O Y i.e., the line
x = 0 , z^=0 is
*+Aaz = 0 or l . x + 0 ^+A 2Z=O. -. (2)
The angle between the planes (I) and (2) is given to be V ,
hence we get
_ 0.1+ 1.0+Aj.Af A1A2
cos a“ v ( i + v j v ( T + v r V i(H -V ) (Γ+αλ Γ
• Squaring, we get A12A22 sec2 α = (1 + λ ι2) ( I +A22)
or A12A22 sec2 a= -1+A1s^ A 2H A 12A22
οι A12A22 (sec2 a 1)= I +A a2+A22,
or Ai2A22 tan2 a = I +A12+A22, .. (3)
Eliminating A1 and A2 between the equations (I), (2) and (3),
the locus of the line of intersection of the planes (I) and (2) is
( - y j z ) 2 ( xlz)2 tan2 a = l + (-+ 7z)2+ ( - x / z ) 2
or X2/ tan2 a= za (x2+ y 2+ z 2).
This is a homogeneous equation of 4th degree and hence
represents a cone with its vertex at the origin.
§3. The line x /l—y\tn=-zjn is a generator oj the cone whose
equation (homogeneous) is given by
/ ( x , y 9 z)= ax2+ by2+ czz+ I fy z + 2gzx+ 2hxy= 0,
i f and only if its directiori ratios /, mf n satisfy the equation of the
cone i.e., i f and only i f / ( / , m, /i) = 0.
The equation of the given cone is
/ ( x , y , z ) = A X 2+ b y 2+ cz2+ I f y z + 2gzx+ 2 h x y = 0. / .(I)

The equations of the given line are


xj l ^ y j m = Z}n = r (say). ...(2)
The co-ordinates of any point P on the line (2) are (Ir, mr, nr).
The Cone 331

The line (2) is a generator of the cone (I) if and only if each
point on the line (2) lieso n (I) Le., if and only if the point
(Ir, mr, nr) satisfies (I) for all values of r Le., if and only if
(al2+bm2+cn2-\-2fmn -\-2gnl+ Zhlm) r2= O for all values of r Le., if
and only if al2 \-bm2+cn2s±2fmn+2gnl+2hIm = 0 i.e., if and only
if/ ( / , m, n) = Q.
§ 4. To find the general equation of a cone of second degree
which passes through the co-ordinate axes, the axes being rectangular.
(Avadh 1980; Agra 76, 78; Garhwal 78, 82;
Kurukshetra 76 ; Meerut 87P)
Since the cone passes through the co-ordinate axes, its vertex
will be at the origin and hence its equation will be homogeneous
of second degree. Let the equation of the required cone be given
by ax2+by2+cz2+2fyz+2gzx-¥ 2Axy=O.' ...(I)
The d.c.’s of the x-axis are I, 0,0. The x-axis will be a
generator of the cone (I), if the d.c.’s I, 0, 0 of the χ -axis satisfy
(I) [See § 3 above] and hence we get a = 0.
The d.c.’s of the y- axis and z-axis are 0, I, 0 and 0, 0, I
respectively. Similarly the y and z axes will be generators of the
cone (I) if ft=0, C=^ O.
Substituting the values of i/, b .c in (I), the required equation
of the cone which passes through the co-ordinate axes Le., the
cone which has co-ordinate axes as generators is given by
fyz+ g zx + hxy=0.
SOLVED EXAMPLES (B)
Ex. I. Show that a cone can be found to contain any two sets
of three mutually perpendicular concurrent lines as generators.
Or
Show that a cone of the second degree can be found to pass
through any two sets o f rectangular axes through the same origin.
(Rohilkhand 1979)
Sol. L e to n e se to f three mutually perpendicular lines be
chosen as the co-ordinate axes and the other set of three mutually
perpendicular lines through the same origin have their direction
cosines Ilf mx, nx ; I2f m2f n2 and I3, m3f n3.
Thegeneral equation of the cone passing through the co­
ordinates axes (/.<?., one set of rectangular axes) is
fyz+ gzx+ hxy= 0 ,..(I)
[See § 4 above]
332 Armfftical Geometry I-D

If the cone (I) passes through the first two lines of the second
set, then we have
fm xnx+gnxlx+ Iiltfn1=O9 ... (2)
and fni2n2+gn2l2+hl2mt =0. ...(3)
Adding (2) and (3), we get
Z im lHi ±m 2n2)+g ( n ^ + n f i ^ + h (Itfn^U m i)=O
or f ( —m% nz)+g (—/I3Z1)+ * ( - / 8m3)= 0
[V W1H1-I-WaWa+W3H8=O, etc., the lines being mutually
perpendicular]
or /w aWa+gw8/3 +/i/am8= 0 . ...(4)
The relation (4) shows that the cone (I) also passes through
the third line of the second set.
Hence a cone can be found to pass through any two sets of
rectangular axes through the same origin.
Ex. 2. Show that the equation o f the cone which contains the
three co-ordinate axes and the lines through the origin having direc­
tion cosines I19 W 1. W1 and I2i w2, n2 is Σ I1I2 (W1Wa-W 2Wi) yz= 0.
Solution. First we shall show that a cone of second degree
can be found to pass through five concurrent lines.
Take the point of concurrency of the lines as origin. We
know that the equation of a cone with the vertex at the origin is
ax2+ by2+ Cz2+ Ifyz+ Igzx+ Hixy = 0. ' ... (I)
Let α^άΟ. Dividing throughout by ‘a \ we get
xt+ib/a) y 2+(c/a) z2+ 2 (f/a ) y z + 2 (gja) zx+2 (h/a) xy= 0
or X 2+ b y 2+ cfz2+ 2f ' y z + 2g'zx+ 2h'xy = 0, ... (2)
where b'==b/at c '^c/a etc.
The equation (2) of the cone contains five arbitrary indepen­
dent constants na ely b \ c \ f \ g' and h' and as such a cone can
be found to satisfy five independent 'conditions. Therefore, a cone
can be made to pass through five concurrent lines.
The general equation of the cone through the three co-ordi­
nate axes is [See § 4]
fy z+ g zx+ Iixy=0. ...(i)
If the cone (I) also passes through the lines whose d.c.’s are
I11 W 1, W1 and I29 w2, n2 then these d.c.’s will satisfy the equation
(I) of the cone [See § 3] and hence we have
fnixnx+gnxlx+ Mtfn1^O 9 ...(2)
and f i n & t + g n J t + M t f n t =0. .... (3)
Solving (2) and (3), we get
The Cone 333

_______ / _____ _ g______ ^ ______ Jt_____


I1I t <H1Ott - Ot1Hl ) Ht1Ott ( Oi I 1 - !»,/„) ' H1Ht O aOt1 -.- I1Ott )
or _______L . _ S h______
I1It Km1Ht -O ttHl) Oi1Otdnilt-H tI1) H1Ht Oimt IaOt1)
P u ttin g th e se p r o p o r tio n a te va lu e s o f / , g, h in ( I ) , th e r e q u i­
re d e q u a tio n o f th e c o n e is g iv e n b y
Σ {I1Ia (Ot1It2- OT1H1) yz}=0. Proved.
Ex. 3 fa). S how that the lines drawn through the point (λ, β, γ)
whose direction cosines satisfy aP+bnfi+cn*- 0 generate the cone
a (x—«)*+b (y —f ) 2+ c (z—y)*=0. (M a d ra s 1977; Berahntpar 76)
Sol. T h e e q u a tio n s o f a n y lin e th ro u g h th e p o in t ( * , β, γ)
a re
(x—(X)Il= (y —fi)lm= (z - y)/n. . ( I)
Its d ir e c tio n c o s in e s /, m, n s a tis fy th e re la tio n
a l 2-1 hm*±cn*--=Q. .· (2 )
E lim in a tin g I9 m9 n b e tw e e n t h : e q u a tiim s ( I ) a n d (2 ), th e
lo cu s o f th e lin e ( I ) Le. th e e q u a tio n c f th e re q u ire d c o n e is g iv e n
by
a (x - a)*+ft (y— ?)2+ o (z y)2=0.
Find the equation of the cone generated by the
E x . 3 (b )
straight lines drawn through the point ( 1, 2, 3) whose direction ratios
satisfy the relation 2/2+ 3/??-—4«2=0.
S o l. T h e e q u a tio n s o f a n y lin e th ro u g h th e p o in t ( I , 2, 3)
are
(x—I )//= (y 2) Im= (z 3)/w. ... ( I)
Its d ire c tio n ra tio s /, m , n s a tis fy th e re la tio n
2 /® + 3 w 2— 4 / j2— 0. .. (2 )
E lim in a tin g /, m , n b e tw e e n th e e q u a tio n s ( I ) a n d (2 ), th e
lo c u s o f th e lin e ( I ) i.e.9 th e e q u a tio n o f th e re q u ire d c o n e is g iv e n
by
2 (x - 1 ) 3 + 3 0 > - 2 ) 2- 4 (z ~ 3 )2= 0
or 2 x * + Iy2— 4 z 2— 4x — 12y + 24z 2 2 = *0 .
Ex. 4. Find the equation of the cone with vertex at the origin
and direction cosines of its generators satisfy the relation
/ * + 2 m 2 - 3w‘2= 0 .
S o l. T h e e q u a tio n s o f a n y lin e th ro u g h th e o r ig in a re
-x/l=.ylm-zln. ...(I)
Its d ir e c tio n c o s in e s s a tis fy th e re la tio n
/2+ 2m%—3n2—0. . . . ( 2)
334 Analytical Geometry 3-D

Eliminating/, m ,/r between (I) and (2), the equation of the


required cone is given by
x2+2y2—3z*=0.
Ex. 5. Find the equation of the cone through the co-ordinate
axes and the lines in which the plane lx+my+nz=Q cuts the cone
ax*+by2+cz2+2fyz+2gzx-\-2hxy=0.
Sol. The equation o f the given cone is
S= ax2+by*+cz*+2fyz+2gzx+2hxy=0 ...(I)
and the equation of the given plane is
P~lx+my+nzc=0. ...(2)
T heeqnationof any surface through the intersection of (I)
and (2) is
*S+AP=0, ...(3)
where λ is an arbitrary function of x, y 9 z.
Since the required cone passes through the co-ordinate axes,
its vertex is at the origin. Now if (3) represents a cone with the
vertex at the origin, then the equation (3) must be homogeneous
and hence we^choose λ to be a linear function of x, y and z, say
X=ux+vy+wz.
Putting this value of λ in the equation (3)., the equation of the
cone is given by
ax*+by2+czi +2fyz+ 2gzx+2hxy
+ (wx+vy+wz) (lx+my+nz) = 0. ...(4)
The d.c.’s of the x, y and z axes are 1, 0, 0; 0, I, 0 and 0, 0, I
respectively. If the cone <4) passes through the co-ordinate axes
then the d.c.’s of the co-ordinates axes will satisfy the equation (4)
and so we have
0 + m/=O, b+ vm = 0, and c+ wti=0.
These give i / = —ajl%v= - b/m, w— cln.
Putting these values of n, r, w in (4), the equation of the
required cone is given by
ax2+by2+ cz2+ 2fyz+2gzx+ 2hxy
—(axll+by/m+czin) (lx+ m y+ nz) = 0
me bn\ na cl \ , Ib am\ Λ
v ~ n i r ) + i n 2!,“ T - i r ) +),n 2'' m
or I (bn2+cm2 - 2 fmn) yz+ m (cl2-yan2 - 2gnl) zx
-fn (am2+- bll —2hlm) xy= 0. Ans.
Ex. 6. OP and OQ are two lines which remain perpendicular
The Cone 335
and move so that the plane OPQ passes through OZ. i f OP descri­
bes the cone f (yjx, zjx)—Q, prove that OQ describes the cone

/{ M -T -a -» ·
Sol. Let the equations of the line OP be
XlIi=Plmt =Zln1 ...(I)
and that of OQ be x/lt =ylm t =z/nt. (2)
It is given that the lines OPand OQ are perpendicular
therefore
hi»+t»itnt+«i«i= 0. ...(3)
Now it is given that the plane OPQ (i.e. the plane containing
the lines OP and OQ) passes through OZ (i.e. x= 0= y). The
equation of any plane through OZ is
χ+ λ+ = 0 . .. (4)
Let the equation of the plane OPQ be given by (4). Since
the plane OPQ given by (4) contains the lines OP and OQ, there­
fore the lines OP and OQ both will be perpendicular to the nor­
mal of the plane (4), and hence we get
/j.l+ m 1A-|-«i.0=0 and 4 1 + ^ ,^ + / ^ . 0 = 0
or X = - I lIml= - I tImt. ...(5)
Again it is given that the line OP given by (I) describes the
c o n e /(ylx, z/x)=0. Therefore the d.c.'s I1. mlt w, of the line OP
will satisfy the equation of this cone, so that we have
f i m j l i , /1,/4)=0. ...(6)
Dividing (3) by Iu we have
4 + («»/ 4 ) »»i + ( « j /4 ) « * — o

or 4 + (m,/4) IW21 (/1,/4) «*=0


[V from (5), m ,/4 -m 2/4]
or /1,/4= - 4/nt-m 2V(4«i) - ( 7)
Puttingthevalue ofm ,/4 from (5) and w,//, from (7) in (6),
we get

Hence the line (2) i.e. the line OQ whose d.c.’s are It, mt, nt
generates the cone

Proved.
336 Analytical Geometry 3-D

§ 5. The equation of the cone with a given vertex and a given conic
as base. To find the equation o f a cone whose vertex is the
point (α, β, y) and base the conic
ax*4-2hxy4-by*+2gxA-2fy-\-c=0, z= 0.
(Agra 1980; Delhi 75; Indore 76; Lncknow 78; Kanpnr 76)
The equations of the base of the cone are given to be
ax* Af^hxy Jr by2 f 2gx+2fy+c=0, z= 0. · ( I)
The equations of any line through the vertex (α, β, y) are
(x -a )//= (^ -p )/m = (z -y )/n . ..(2)
The line (2) meets the plane z = 0 at the point given by
x —a y —β 0—y . ( Iy my n\
—r = -—- = — - i.e. at the point I a —— , B---- -, Ol
I m ■ η \ n r n /
If this point lies on the given conic (I), then we get

a,
The relation (3) is the condition that the line (2) intersects
the conic (I), and hence the locus of the line (2) i.e. the required
equation of the cone is obtained by eliminating /, mf n between (2)
and (3).
From (2), we have 1
/ x- λ , m y -β
— =----- , and — = - — —·
η ζ-γ η ζ —γ
Putting these values in (3), the required equation of the cone
is given by

+ V { P - ( y: :_ y ) v } + C- 0
Multiplying throughout by (z y)2 and simplifying, we get
a 1<χζ~γχ)*+2Ι] ( j ~ γχ) (βz - y y ) + b (βζ—yy)*
+2g'(az—yx) ( z - y)+ 2f ( β ζ -y y ) (z - y ) + c (r - 7)*= 0 .
§ 6. Tofind the condition for the general equation o f the second
degree to represent a cone and to find the co-ordinates o f its vertex.
(Avadh 1982; Allahabad 77, 80; Jodhpur 78; Rajasthan 75)
The Cone 337

The most general equation of the second degree is


ax1+by*+ cz*+ 2fyz+ 2gzx+ 2hxy + 2ux+ 2vy+2wz+ d—0. ...(I)
Let (I) represent a cone with the vertex at (α, β, y).
Shifting the origin to (α, β, y), the equation (I) becomes
o (*+«)»+* (r+ W 1+* (r+y)*+ 2/(y+j9) ( 2 +y)
+ 2 g (z+y) (x+a)+2A (x+*) (y+fi)+2u (x + a)
-ί-2ν 0>+|3)+2 η· (z+ y )+ d —0
or ax*+ by*+ cz8+ 2fyz+ Ig zx + 2hxy
+ 2x (a&+hfi+gy+u)+2y (Aot+A/3+/y+v)
+ 2z (g* + ffi+ cy+ w )+ (e«*+ A/J*+ cy*+2/j8y
+ 2gya + 2Aaj3+ 2υα.+2\·β + 2wy + d ) - 0 . ....(2)
Now the equation (2) represents a cone with the vertex at the
origin and hence it must be homogeneous. Therefore, the coeffi­
cients of X, y, z and the absolute term must be zero separately.
So we must have
o*+A/3+gy+«=0, .(3)
hιι+bβ+fγ+v**0, .(4)
gn+ fk+ ey+ w ^O , ...(5)
and a&*+bfi*+ ey*+2/j8y+2gya+2Aa/3+2na+2v/3+2Hy+d=0
Le., a (e«+Aj3+gy+w)+/3 (A«+A/3+/y+v)
+ y (g*+ffi + ey + w)+(ua+vfi+wy+d)=0.
Using (3), (4) and (S) this last condition gives
uix r νβ + M-y + d - 0 . « ..(6 )
Therequtred condition that the equation (I) represents a
cone is obtained by eliminating α, β, y between the equations (3),
(4), (S) and (6) and hence is given by
I- e A g U
i
i A b f V

g f C W

U V W d
If the condition (7) is satisfied, then the equation (I) represents
a cone. The co-ordinates (α, β, γ) of its vertex are obtained by
solving the equations (3), (4) and (5) and are given by
X - β _ y -I
A g u a g «II a Λ u a h g

b f V A / v I H b v A b f
f e w g C W I g f W I g f C
338 Analytical Geometry 3-Z>

Working rule for numerical problems. Forsolving numerical


examples, an easier method can however be adopted. We intro­
duce a new variable t. Make the given equation (I) homogeneous
by multiplying the different terms in it by proper powers of f.
Let the homogeneous expression so obtained be denoted by
F (x9y %z91) and thus the homogeneous equation is given by
F (x t y , Z 9 t)m άχ%+ by*+ cz%+ Ifyz + 2gzx+ Ihxy
+ Iu xt+ 2vyt+ 2wzt+ dt%= 0.
P iF P iF P iF P iF
Now find S-, -S-, -T- and — and then put /= 1 in each. We
ax oy ox ot
. ... . . BF „ dF ~ BF μ . SF Λ .
observe that the equations ^ -= 0 ,- ^ = 0 ,- ^ = »0 and = 0 at
/= 1 clearly represent the equations (3), (4), (5) and (6) respecti­
vely. Now solve any three equations for x, y, z and if the
remaining fourth equation is satisfied by these values of x, y and
ztben the given equation (I) will represent a cone with the vertex
at the point (x, y, z) found by solving the three equations.
SOLVED EXAMPLES (C)
Ex. I. Prove that a line which passes through (α, β, γ) and
intersects the parabola z*=4ax, y = 0 lies on the cone
(J3z-yy)*-4fl (|3 -y ) (/3x-ay)=0.
(Meerut 1983)
. Sol. Clearly in the present problem, we are to find the equa­
tion of a cone with the vertex at (α, β, γ) and base curve as
z*=4ax, y = 0 . ...(I)
The equations of any line through (α, β, γ) are
(x-<x)//=(>>-j8)//n=(z-y)//i. ·· (2)
The line (2) meets the plane j>=0 at the point given by
x —a 0 —β z —y
— -«*— - t.e. at the point £m, 0 . r - = m
S )/
I m n
This point will lie on the curve (I), if
(y—πβ//η)*=»4α (a—Ιβ/m). ...(3)
Now from (2), we have
n z —y , I x —a
m y —β m y —β
Putting these values of n/m and Ijm in (3) /.¢., eliminating
I9m9 n between the equations (2) and (3), the required equation
of the cone is given by
The Cone 339

M ^ W 1= -K -= J W
or {γγ-βζ)*= 4α {&γ—βχ) (y—β)
or { β ζ - yy)*= 4a (β- y ) (βχ - ay). Proved.
Eu. 2 (a). Find the equation o f a cone whose vertex is the point
(α, β, y) and whose generating lines pass through the conic \ ·
x*la*+y*Jb* = I , r= 0 . (Gorakhpur 1975; 82; Gaahati 77)
Sol. The equations of the given conic are
XtIat -^yiIbi = I t Z=0. Ο)
The equations of any line through the vertex (α,.β, y) are
(x -a )//= ( y - 0 ) //n = ( z - y ) /« . -(2)
The line (2) meets the plane 2«=0 at the point given by
X - a y —β O y . ' . . ■, I W a mY nV
I m n \ π n /
If this point lies on the conic (I), then

Eliminating I, m, n between the equations (2) and (3), the


required equation of the conic is given by

or b%(<xz—yx)*+a* ^ z —yy)i = atbt (z—y)* ...(4),'


Ans'
Ex. 2 (b). The section of a cone whose vertex is P and the base '
curve the ellipse XtIat -^yiIbt = I , z = 0 by the plane x = 0 Is a rect­
angular hyperbola. Show that the locus of P is
XtIat + (y*+ 2*)/h*«£i I .
(Meerat 1978, 84; Punjab 8t (S); Gorakhpur 75)
Sol. Let (α, β, y) be the co-ordinates of the vertex P of the
cone. The equation of the cone whose vertex is the point (α, β, y)
find base curve the ellipse XtIat -^yiIbt = I, z=Q is
b1 (az—yx)*+a* (βζ—yy)t = atbt (z—y)*. -. (I)
[See eqn. (4) of Ex. 2 (a) above. Derive it here].
The section of the cone (I) by the plane x = 0 is obtained by
putting x = 0 in the equation (I) and is given by
0***2*+a* (βζ -yy)*=o*h* (z-y)*, x = 0 . ...(2)
If the equations (2) represent a rectangular hyperbola in the
x = 0 plane, then in the first of the equations (2) we should hove
the coefficient of _y*-J-the coefficient of 2*=0
340 Analytical Geometry 3-D

ie . (αV ) + ( 4 V + α*β* - a W )= 0
or Ρα·+α*(β*+γ*) -a*4*.
Λ The locus of the point P (a, β, γ) is
4*x*-fe* (y*-Vzt)= atbt or x lja2-V(y* -Vzt) jb*= I. Proved.
Ex. 3. Find the equation to the cone whose vertex is the point
P (a, b, c) and whose generating lines intersect the conic
px%-\-qy*= I, z=0. ' '
(Agra 1982; Avadb 81; Calcutta 77; Meerot 85)
Sol. The equations of the base curve are
PXi Vqyt = I t Z=O. ...(I)
The equations of any line through the vertex P (a, b, c) are
( x —a)ll=(y—b)fm=(z—c)ln. \·- (2)
The line (2) meets the plane z= 0 at the point given by
x —a y —b O—c . . . ( Ie . me A
_ l.e. at the potat ( a - - , b - - , o)
This point will lie on the conic (I), if
p (a—Icjn)*-Vq(b —mc/n) = I. ...(3)
Eliminating I, m, n between the equations (2) and (3), the
equation of the required cone is given by

' Η ^ * ϊ ,+« Η £ - ί Μ ν ι
or p (az—exp+ q (bz—Cyf=(Z-C)*
or c* (px* + qy*)-V(p(P-Vbtq - \ ) z*—2c (apzx+bqyz—z)=c*. Aos.
Ex. 4. Find the equation o f the cone whose vertex is (I, 2, 3)
and guiding curve is the circle Xt -Vy2-Vzt = 4, X-Vy-Vz= I.
(Lucknow 1982)
Sol. The equations of the guiding circle are given by
*, +3'*+2, ==4 ...(I) and x -Vy+z= I. .. (2)
The equations of any line (generator) through (I, 2, 3) are
x — I _ y —2_ z —3_^x-Vy + z ~ 6
I m n ϊ+m+n .--(3)
[Note]
The line (3) meets the plane X-Vy-Vz=I at the point given by
x —I_y —2_z —3_ 1—6 ^ —5
I m n I-Vm-Vn l+m-Vn
. . /. SI „ 5m ^ Sn \
U . at the PO,at 2 - + ^ -
t m-Vn—41 2l+2n—3m 3 /+ 3 m —2n\
V + m + » ' I-Vm-Vn ’ l+m-Vn /
The Cone 341

This point lies on the base circle given by the equations (I)
and (2), hence this point will satisfy the equation (I) and so we
have
(m + n -4/)*+(2/+2n-3m )*+(3/+3m -2n)*«=4 (/+m +n)* ...(4)
Eliminating /, m, is between (3) and (4), we· get the IocUs of
the line (3) i.e., the equation of the required cone. We observe
that the equation (4) is homogeneous in /, m, n, so to eliminate /,
m, n, we can put even the proportionate values of I, m, n from (3)
in (4). Then the required equation of the cone is given by
{(y—2 ) + ( 2 - 3 ) - 4 (x—1)}*+{2 ( x - l ) + 2 ( 2 - 3 ) - 3 0--2)}*
+ {3 (x—1)+3 0 - 2 ) - 2 (2—3)}*=4 { ( x - 1 )+ 0 - 2 ) + ( 2 - 3 )} *
or 0 + 2 —4x—l)*+ (2x+22—3.y—2)*+(3x+3y—2 2 - 3>*
= 4 (x+>-+ 2 —6)*
or 5χ*+3^*+2*—6yz—4rx—2 x y + 6 x + 8 y + 1 0 r—26=0.
Ex. 5. Find the equation o f the cone with vertex (5, 4, 3) and
with 3x*+2>*=6, y + 2 = 0 as base. (Indore 1979)
Sol. The equations of the base curve are
3x*+2>*=6, and y + 2 = 0 . ...(I)
The equations of any line (generator) through (5, 4, 3) are
x —5 y —4 2—3 y + z —7
I m n m+n ...(2)
[Note]
Ih e line (2) meets the plane y + 2= 0 at the point given by
x —5 y —4 2—3 0 —7 —7
/ β m ^~n~ ~ m+ n ^ m+n
Tm , Tn \
i.e., at the point ' I s - J U -
V ' m +n m+n
/ Sm+5*»—Tl 4«-3m 3m—4n\
or
\ m +n * m +n ’ m + n J
If this point lies on the base curve given by (I), then this
point will satisfy 3x*+2^---^6 and hence we have
3 (5m +5n—7/)*+2 (4n —3m)*=6 (m+n)*. ...(3)
Putting proportionate values of I, m, n from (2) in (3) and
thus eliminating /, m, n between the equations (2) and (3), the
required equation of the cone is given by
3 {5 0 - 4 ) + 5 ( 2 - 3 ) - 7 (x—5)}*+2 {4 ( 2 - 3 ) - 3 0~4)}*
= 6 (0 -4 )+ (2 -3 )} *
or 3 (5y+52—7x)*+2 ( 4 2 - 3 ^ / = 6 0 + 2 —7)*
or 147x*+87j'* + 10l2*+90j»2—210rx—210x)'+84<y+842—294*^0.
342 Analytical Geometry 3-D

Ex. 6. Prove that the equation


4x*~yt +2zt +2xy —3 yz+ l2 x— I ly + 6 z + 4 = 0
represents a corie. Find the co-ordinates o f its vertex.
(Rohilkhaud 1982)
Sol. Let
F \ x r y, z)m4x*—y*+2z*+2xy —3 yz+ l2 x~ W y+ 6 z+ 4 = 0 . (I)
Now we introduce a new variable t and making (I) homo­
geneous with the help of t [See working rule, § 6], we get
F (x , y, Zi t)^4 x* —y*+2z*+2xy—3 y z+ l2 x l— Ilyf+6zf+4t* ...(2)
Differentiating (2) partially with respect to x, y, z and t
respectively, we get
dFIBx=’Sx+ 2y+ l2t, BFjdy= —2y+ 2x—3z—Wt I
d F ld z= 4 z-3 y + 6 t,d F ld t= \2 x -l\y + 6 z+ % t f ...(A)
Putting /*=>1 in each of the relations .in (A) and then equating
them to zero, we get
8 x + 2 .y + l= 0 .( 3 ) , 2 * -2 ^ -3 z -ll= 0 ...(4)
—3 y+ az+ 6= 0 ...(5), 12x—lly + 6 z + 8 = 0 ...(6)
Now we shall find x, y, z by solving the equations (3), (4)
and (5).
Eliminating x between (3) and (4), we get
5,y+6z+28=0. ...(7)
Solving (5) and (7), we get y = —2, z = —3.
Putting the value of y in (3), we get x = — I.
Substituting these values i.e. jc=>—I , y= —2, z = —3 in (6),
we have 1 2 ( - 1 ) - 1 1 ( - 2 ) + 6 ( - 3 ) + 8 = 0 or 0 = 0
i.e. the equation (6) is satisfied. Hence the given equation (I)
represents a cone with the vertex at ( —1, —2, —3).
Ex.7. Prove that the equation
ax*+by, +cz*+2ux+2vy + 2wz+d=0
represents a cone if u’/a+ v ’/i»+ WiIc= d.
( Avadh 1980; Garhwal 79, 81; Kanpur 82; Meerut 80, 82, 86, 90;
Rajasthan 75, 77; Delhi 76)
Sol. Let
F (x, y, z)=qxi +byi +cz*+2ux+2vy+2wz+d=0. .. (I)
Now introducinga new variable t and making (I) homo­
geneous with the help of t. [See working rule, § 6], we get
F ( x , y , z , t) s a x i +by*+czt + 2uxt+2vyt+2wzt+dit. ...(2)
Differentiating (2) partially w.r.t. x, y, z and t respectively,
we have
The C one 343

dFIdx =»2ax4-2w/, 8F/dy=2by + 2vt I


dF/dz*=2cz+2wt9 dFldt = 2ux+2vyA-2wz+2dt J ..,(A)
Putting t= I in each of the relations in (A) and then equating
them to zero, we Lave
2ax+2u=09 2by+2v = 09 2cz+2w=0, 2ux+2vy+iwz+2d=0.
From the first three equations, we have
X = -IiZ jf y =>—vlb9 z = —w/c.
Putting these values of x 9y 9 z in the fourth equation namely
2ux+2vy+2wz+2d=*09 we have
2u (—ju/'a)+2v ( —v/ft)+2iv ( —w /c)+ 2d^0
or ut/a+ vl/b+w% lc ^ d . ...(3)
Hence (3) is the required condition that the given equation
(I) represents a cone.
We also note that the vertex of the cone (I) is the point
(~u/a9 —vjb, —w/c).
Ex. 8. Two cones with a common vertex pass through the
curves
Z2- A a x 9 y —0 and Zt C=Aby9x=*0.
The pL, j z - 0 meets them in two conics which intersect in four
con-cyclic points Show that the vertex lies on the surface
Zt (x /a + y /t)= 4 (**+^4).
Sol. Let (α, β, γ) be the common vertex. Proceeding as
Ex. I,above, the equations of two cones are respectively
given by
(W*-J8z)* (a y - 0 * ) (y -J 3) ...(I)
(z x - OLZ)2- Ab (/L r-ay) (x —a) ..:(2)
T hepianez= O m eetsthecones (I) and (2) in two conics
whose equations in xy·plane are respectively given by
'S l =y*y*-4a ( a y - /Sx) ( y - β ) , 2= 0 ...(3)
St —y'x*—4b (βχ —ay) (x —a), 2 = 0 - C4)
Any curve through the intersection of (3) and (4) is given by
S i+AS2==O, 2= 0
i.e. γ Y - 4 a (ay- β χ ) (y - β )
+A —4b (β χ —ay) (x—a)]= 0, 2=0.
It the last equation represents a circle, then wc must have :
The coeffi. of Jt2= Ihe coeffi. of y* and the coeffi. of *y=0.
i.e. A(y2—4f»/3)=y*—4a* ...(3)
and 4a/8+4h«A=0 i.e. λ ~ —(αβ)/(bx). ...(4)
344 Analytical Geometry 3-D

Eliminating Λ between (3) and f4), we have


y* (bx+afi)=4ab («*+0*)
/. The locus of (a, 0, y) is z* (x/a+y/6)=4 (*, +y*).
Ex. 9. Find the equation o f the cone with vertex at (2a, b, c)
and passing through the curve x* + y * = 4 a * and z —0. F lndbandc
if the cone also passes through the curve y*=4a (z + a ), x —0. Also
show that the cone Is cut by the plane y —0 in two straight lines
and the angle 6 between them is given by tan 6=2.
Sol. The equations of any generator through (2a, b, c) are
(x -2 a )ll= (y -b )lm = (z -c )/n ...(I)
T h elin e(I) meets the plane z= 0 at the point given by
x —2a y —b — c . . ,. -. . / . Ic , me ~\
I m n K \ n n /
This point will lie on the conic Xi -Yyi =Aat, z= 0 if
(2a—lc/n)*+(b—meIn)*= 4 a* .(2)
Eliminating I, m, n between (2) and (I), the equation of the
cone with the vertex at (2a, b, c) is given by

or (2az—cx)*+(6z—c<y)*=4a* (z—c)* -(3 )


If the cone passes through the curve
y*=4a (z+a), x«=0 ...(4)
the curve of intersection of (3) will be same as (4). Now put
x*>0 in (3), we get
(2αζ)*+(6ζ—cy)*«»4a* (z—c)*
or b*z*+c*y*=»26cyz—•8ca, z+4e*c* ...(5)
Comparing (4) and (S)1 we have
c* -Hcai 4a*c*
b' o b=0, c = —2a.
* I “ 4a ~ 4a*
Now putting b=0, c = —2a in (3), the equation of the cone
becomes (on simplification)
xt + yi+ 2 zx—4az—4ai = 0 , ...(61
The plane y«=0, meets the cone (6) in
**+2zx—4αζ—4a*«=0, y = 0
It clearly represents two straight lines and the angle Θ
between them is given by
2VU1)»-1X0}
Tan 0« •2 .
1+ 0
The Cone 345

$ 7. The tangent line and the tangent plane to a cone.


To find the condition that a given line through the point (α, β, y)
on the cone ax*+byt +czi + 2 fyz+ 2 g zx+ 2 h xy^0
be a tangent line to the cone and to find the equation o f the tangent
plane to the above cone at the point (α, β, y) on it.
. v
The equation of the given cone is .
F (x, y. z)= ax%-\-byt + cz*+ 2fyz+ Ig zx+2Axy= 0. ...(I)
Let P (α, β, γ) be a point on the cone (I), pt which the
tangent plane is to be obtained. Also let I, m, n be the actual
direction cosines of the given line. Therefore, the equations of
the given line through (<χ,:β, y) are
(at-a)//=fy-j3)//w «=(r-y)/n«=r (say). ...(2)
Since the point Pji*, β, y) lies on the cone (I), we have
F («, β, y)maxi +b$*+cyi ^-2fβy + 2gγx+ 2Λα/)=0. ,..(3)
The co-ordinates of any point on the line (2) arc (a+ Ir, β +tnr,
y+nr). If the line (2) intersects the cone (I), then for some values
of r this point will lie on the cone (I) and, therefore, the points
of intersection of the line (2) and the cone (I) are gived by
a (a+lr)*+b (β+mr)*+c (y+nr)*+2f (β+mr) iy+ nr)
+ 2 g (y+nr) (x+ lr)+ 2h (x+ lr) (/J+m r)=0
or r* (al*+bm, +cni +2fmn+2gnl-\· 2hlm)
+ 2r {(ax+ hβ-t^gy) l+ (h a + bfi-\-fy) ffJ + (g«+//3+cy)n}
+ ( + bfi*+ cy* -J-2//Jy 2g yx + 2A«/J)=0.
Putting the value from (3) and using F (I, m, n) for the coeffi­
cient of r*, the above equation becomes
r*F(l. m, n)+2r {(ax+hf)+gy) /+ ( hx-^-bβ+fy) m
+(g*+//S+cy) n}=0. ...(4)
The equation (4) being a quadratic equation in r shows that
the Iioe (2) cuts the cone (I) in two points and, therefore, if the
line (2) be a tangent line to the cone (I), the two values of r should
be coincident. Clearly one value of r is zero and hence the condi­
tion that the line (2) be a tangent line to the cone (I) is that the
other value of r should also be zero. Hence the required condition
is obtained by putting the coefficient of r in (4) equal to zero and
is (fla+AjB+gy) /-HA« + A]8+/y) m + tgx+ ffi + cy) n = 0 . ...(5)
The tangent plaoe, The tangent plane at a point P (α, β, y) to
the cone is the locus of the tangent lines through·'P (a,‘ β, y) drawn
in all directions.
Hencethe tangent plane to the cone (I) at the point P(x, β ,γ)
346 Analytical Geometry 3-D

is the locus of the tangent line (2) under the condition (5) and is
obtained by eliminating /, m, n between the equations (2) and (5).
Thus the equation of the tangent plane at P (α, 0, γ) is given by
(a«+-A0+gy) ( χ - α ) + ( A*+A0+/y> (y- β )
+ - ( g a + /0 + cy) (2 — y ) = 0
or x (ax + hβ+gγ)+ y (Ιιχ+ϊβ+]γ) + ζ (ga+/0 + cy)
—(aa*+ 60* + Cyt +-2/0y-j-2gya + 2A*0)*=»0
or x (act.+-hβ+gγ)+y(hx+bβ+fγ)+z^gx-{-fβicγ)=0 ...(6)
[using (3)]
Note. The equations (S) and (6) can be more conveniently
remembered and written with the help of differential calculus as
follows :
Differentiating (3) partially w.r.t. a, 0, y respectively, we
have
BFjdx = 2 (a* + A0 fgy), BF/Ββ 2 (hx+όβ -\-fy)
BFjdy —2 (gx +/0 + cy).
Thus the condition (5) that the line (2) is the tangent line to
the cone (I) may be written as
. BF , B F , BF n
' - I S m Te+ " Tr - * ...(S')
The equation (6) of the tahgent plane at the point P (a, 0, y)
to the cone (I) may be written as
B F , B F 1 BFi .
x T S y Te+ 2 Tr - 0- (e/)
Corollary. To prove that (he tangent plane at any point
P (ά,β, γ) is the tangent plane at every point o f the generator through
P (α, β, y). In other words, the tangent plane at P touches the cone
along the generator through the point P.
The vertex of the given cone (I) is O (0, 0, Uj, and the tan­
gent plane at P (α, β, γ), i.e. (6) clearly passes through the vertex
O (0, 0, 0).
The equations of the generator OP are clearly given by
X -Q n J - O = 2—0 j 2
——k (say).
a —0 )3-0 y —0 €' α β V
Any point on this generator is (/ca, Ar0, ky). The equation of
the tangent plane at (kx, A0, ky) to the cone (I) is
X (akx+hkfi+gky)+y (hkx+ bkβ+ fkγ)+ z (gkx+fkβ^\-ckγ)■=0
or * (aa+-/j0+gy)+y (A a+60+/y)+z (ga+ -/0 + cy J-0
which is the same as the tangent plane at P (a, 0, y) given by (o).
The Cone 347

Hence the tangent plane at every point of the *generator OP


is the same as the tangent plane at the point P. This proves our
desired statement.
We also conclude (hat the tangent plane at any point P
touches the cone along the generator OP.
This generator OP is called the generator of contact.
The vertex of the cone is called a singular point since all the
tangent planes of the cone pass through the vertex of the cone.
§ 8. The condition of tangency.
To determine the condition that the plane ux+ vy+ w z^O may
touch the cone
ax2+ by+ cz1-f 2fyz + 2gzx + 2hxy<= 0.
(Avadb 1979; Agra 77; Kanpur 77)
The equation of the given cone is -
F (x,y> z)tsax2+f>y2+ cz2+2fyz+2gzx+2hxy'=0. ...(I)
The equation of the given plane is ux f vy+wz*=0. ...(2)
Let the given plane (2) be the tangeilt plane to the given
cone (I) at the point (α, β, γ). The equation of the tangent plane
to the cone (I) at (α. β, γ) is
x (aa+hfi+gy)+y (Λα + ΰβ f/y)+z.(ga+//*+C y)~0. v ...(3)
The equations (2) and (3) represent the same plane. Hence
comparing the coefficients of x 9y and z %we get
= v H’ - 1 (SftV)
βα+Λ/3+gy hx + bfi+Jy g x+Ζβ+ογ μ '
Λ αν.·\·Ιιβ+2γ*~μΐί*=0, . ..( 4 )
—μνβΟ, *··(5)
and gx·\■Jβ■^■cy—μw=0. ...(6)
Also ( x , β , γ ) lies on the plane (2), so we have
u«+ vj8+wy=.0. ...(7)
Eliminating «, β, y and —μ between the equations (4), (5),
(6) and (7), the required condition is given by*
a h g U

h b f V

g f C W

U V W 0
348 Analytkal Geometry 3-D

The equation (8) [on expanding the determinant] may be


written as
Au*+ Bvt + Cw*+ 2F*>w+ 2Gwu+ 2Hav=0, ., .(9)
where A, B, C, F, G, H are respectively the co-factors of the
coiresponding small letters a, b, c ,f, g, H in the determinant A
given by
A= a h gg Le., A =‘abc+2fgh—of*—bgt —ch*.

h b f

g f C
Thus
» ~ b c - f* - JMf , B = c a - g * J J ± .C = a b - h * J £
(IO)
F ^gh- α / ^ ψ . G ~ h f-b g -i
§ 9. The Reciprocal cone.
Definition. The reciprocal cone o f the given (one is the locus
of the lines through the vertex and at right angles to the tangent
planes o f the given cone, Le. the reciprocal cone a f the given cone is
the locus o f the normals through the vertex to the tangent planes o f
the given cone.
(A) To find the equation of the reciprocal cone of a given
cone.
Let the equation of the given cone be
ax*+by*+cz*+2fyz+ 2gzx+ 2hxy =*0 ...(I)
and let ux+vy+wxeaO ...(2)
be the equation of a tangent plane to the cone ( I). The condi­
tion that the plane.(2) touches the cone (I) is given by [see equa­
tion (9), § 8]
Au*+Bv*+Cw*+2Fvw+2Gwu+2Huv=*H. ...(3)
The d.r.’s of the normal to the plane (2) are u, v, w. Hence
the equations of the normal to the plane (2) passing through the
vertex (0,0, 0) of the given cone (I) are
xlu=ylv*az/w. ...(4)
Eliminating u, v, w between (3) and (4), the locus of the
normal (4) is given by
Ax*+ By*+Cz*+2Fyz+2Gzx+2 Hxy =- 0. .. .(5)
The equation (5) being a homogeneous equation of second
The Cone 349

degree represents a cone with the vertex at the origin. Thus the
cone (5) is the reciprocal cone of the given cone (I).
Working rule. In order to find the reciprocal cone of the
given cone ox*-|-hy*+er*+2/yz+2£ZX+2/ury=»0, we have to simp­
ly replace the small letters a, b, c,f, g, h by their corresponding
capital letters A, B, C, F, G, H, where the capital letters are the
co-factors of the corresponding small letters in the determinant
a h g
h b f

Z f C
(B) To find the reciprocal cone of the cone
Axl +Byt +Czt +2Fyz+2Gzx+2Hxy=‘0. ..:(6)
To find the reciprocal cone of the cone (6) we should only
replace A, B, C, Fi G, H by their co-factors in the determinant
A H G

H B F

G F C
Let A', B', C , F', G', H be the co-factors of A. B, C, F, G, H
in the above determinant. We have
A '^ B C - F t ^ ( c a - g t) (ab—h*)—(gh- a/)*
[See equations (IO) of § 8]
—aibc -f- 2afgh—a’/ 1- Obgt -Ocht
■»« (abc+2fgh~ a ft —bgt —cht) - a Λ.
Similarly B'=*bA> C’—c A ,
and F '= G H ^ AF=f A tG ' HF—B G -gA * H'= F G -C H ^ h A-
Now the reciprocal cone of the cone (6) is given by [See
working rule of § 9 (A) above]
A'xt + B 'yt 4-C z t + 2F'yz+2G'zx+2H'xy*= 0
or A (ox*+ by1+ cz*+ 2fyz+ 2gzx+ 2hxy)*=* 0
or axt +byt +czt +2/yz+2gzx+2hxy=0 .. (7)
[as A ?*0 because ( I) represents a cone].
But in § 9 (A) we have proved that the reciprocal cone of (7)
is given by (6). Hence we see that the cones (6) and (7) are such
that each Is the locus of the normals through the vertex (the origin)
350 Analytical Geometry 3-D

to the tangent planes to the other and on account of this property


the cones (6) and (7) are the reciprocal o f each other.
Important Note. We have defined above that the reciprocal
cone is the locus of the normals through the vertex of the tangent
planes to the given cone. Hence In order to show that a given
plane P= 0 (say) is a tangent plane to a given cone F (x , y, z)= 0
(say), we should show that the normal to the plane P =>0 through
the vertex of the cone F (x , y, z)= 0 is a generator of the recipro­
cal cone.
SOLVED EXAMPLES (D)
Ex. I. Find the equation of the cone reciprocal to the cone
fyz+ gzx+ hxy= 0.
Sol. The equation of the given cone is
fyz+ g zx+ h xy= 0 . ...(I)
Let the reciprocal cone of (I) be
Ax*+By*+Cz*+2Fyz+2Gzx+2Hxy=0. .-(2)
Comparing the equation (I) with the equation
ax*+ by*+ cz*+ 2fyz+ 2gzx+2hxy= 0, we have
a = Or A=0, c= 0, f = \ f , £«=}£ and h= \h.
Λ A = " b c - f" '.---- * /* ; B = - W C— -JA*;
F = “g h - ar = \ g .\ h - 0 = \ g h ·, G = W , H = \fg .
Putting these values in (2), the equation of the cone reciprocal
to (I) is given by
—J f* x*- ig V - ih*z* 4- 2 . ig h yz+ 2. \hfzx + 2. \fg x y = 0
or f tx*+gty t + htzt —2ghyz—2hfzx—2fgxy=0. .. (3)
Ex. 2. Prove that the general equation to a cone which touches
the co-ordinate plane is
a*x*+ b*y*+ c*z*—2bcyz —2cazx —2abxy= 0.
(Agra 1975, 80; Gorakhpor 82; Meerut 70, 79(S); Korokshetra 76)
Sol. The co ordinate axes are the normals to the co-ordinate
planes and hence the cone which touches the three co-ordinate
planes is the reciprocal cone to the cone which contains the three
co-ordinate axes.
The equation of the cone which contains the three co ordinate
axes is given by
fyz+gzx+hxy*=*0. [See § 41 ...(I)
Now rewriting the steps of Ex. I. above, the cone reciprocal
to (I) is given by
Xhe Cone 351

/ a*2+ g'y*+ h9Z2 - 2ghyz—Ihfzx - 2fgxy= O', ... (2)


which is of the form
QtX1+ b*y*+ CtZt —Ibcy z—Icazx—2abxy·=* 0 .
Ex. 3. Show that the tangent planes to the cone
fyz+ gzx+ hxy= 0 are perpendicular to the generators o f the cone
f*x*+ giy'+h*z2-2 g h y z-2 h fzx -2 fg x y = 0 . [Meerut 84 (Re)]
Sol. In view of the definition of the reciprocal cone, to
prove the given problem it is sufficient to prove that the two
cones are reciprocal to each other.
Rewriting the steps of Ex. I above, the cone reciprocal to
fyz+gzx+hxy=* 0 is
f 2Xi W y 2+ AaZ2- 2ghyz - Ihfzx - 2fgxy=>0.
Ex. 4. Find the equation of the cone reciprocal to the cone
ax*+by*+cz2-- 0. (Rohilkhand 1981; Kanpur 74; Meerut 85)
Sol. The equation of the given cone is
'AXa-I-Aya+ CZa=O. -.(1)
Let the cone reciprocal to the cone (I) be
Ax2+ By*+ Cz2+ 2Fyz+2Gzx+ 2Hxy=* 0 . -.(2)
Comparing (I) with the equation
ax2+ by*+ cz2+ Ify z+ 2gzx + 2hxy= 0,
we have a= at b —b, c=c, /= 0 , g = 0 , A=0.
Λ ^ = " A c - / 2” =Ac-0=Ac.
Similarly 2?= ca and C= aA.
Also F = "tfA -a / ” c=0.0- a . 0=0.
Similarly G=O and H =0.
Putting these values of A9 B9 C9 F9 G9 H in the equation (2),
the required equation of the reciprocal cone is given by
bcx* + cay*+abz2=>0 or 'xa/A-+ya/A+ za/c=0.
Ex. 5. Prove that the perpendiculars drown from the origin to
the tangent planes to the cone ax* + by*+cz**=* 0 lie on the cone
x2la+y*/b+z2lc=0. (Kanpur 1978)
Sol. By the definition of reciprocal cone (§ 9), we know that
the locus of the normals (i.e. perpendiculars) drawn from the
vertex (i.e. the origin in the present case) to the tangent planes of
a given cone is called the reciprocal cone. Therefore, in the
present case, we are required to find the cone reciprocal to the
cone ax* +Ay2+cz2=.0. Henceit is the same problem as Ex. 4
above.
352 Analytical Geometry 3-L>

Ex. 6. Find the condition that the plane ux+ vy+ w z= O may
touch the ccne axi +byi +czi =0. (Meerut 1984 S)
Sol. Tbe equation of the cone is
<7X*+'6}>*+ C2*= 0. -.-(I)
The equation of the plane is ux-l-vy+wz-O. ...(2)
We know that the condition that the plane ux+vy + wz^O be
a tangent plane to the cone ax i + b y i + c z i + 2 / y + 2 g z x + 2 h x y = 0
is (See § 8)
Au*+ Rv*+Ctvi + 2Fvw+ 2Gwu+ 2Huv ^=0. .(3)
Here a = -a ,b = b ,c = c ,f= 0 ,g = 0 ,h = 0 .
.·. A = i,b c ~ fi"=>bc—0=1be.
Similarly B=ca,C=ab.
Also F= "gh—a f” =0.
Similarly G -O and H = 0.
Putting these values in (3), the required condition of tangency
is given by
bcu*+ cav*+ab w*+ 0 + 0 + 0 -- 0
or u*/fl+ v*/^+M’i/CKr 0
Alternative method. In view of the Mmpottant note’ to § 9,
the plane ux+vy+w z—0 will be a tangent plane to the cone
ax*+6.y*+cz*~0 if the normal to this plane through the vertex
of the given cone i.e., through the origin in this case is a genera·
tor of the cone reciprocal to the given cone namely
UX3 + (>>>*+ C Z 1 :=0.

Now by Ex. 4 above the equation of the cone reciprocal to


the cone ax*+by*+cz%= 0 is
Xi I a + y*lb+z*lc—0. ...(I)
The equations of the normal to the plane ux+ vy+ w z= 0
passing through the vertex (0, 0, 0) are x/u=ylv=zlw. . . (2)
If (2) is to be a generator of the cone (I), then the direction
ratios u, v, w of the line (2) should satisfy the equation (I) and so
the required condition of tangency is given by
u*la+v*/b-{· WiIc= 0.
Ex. 7. Prove that the equation y/(fx )+ \/(g y )+ y /(b z)= 0
represents a cone that touches the co-ordinate planes.
( Bundelkhaod 1979; Lucknow 74, 81; Kurokshetra 76;
Rajastban 74)
The Cone 353

Show also that the equation o f the reciprocal cone is


fyz+ gz Jt-f hxy=0.
(Agra 1973, 77; Robilkhand 80; Kanpnr 75; 80;
Meerut 83S, 84P, 85P, 89S)
Sol. The given equation is
V ( fx ) ± V ( s y ) ± V ( h z ) —o ...(I)
or V (fx )± V (8 )')= T (h z ).
Squaring, we have fx+ gy± .2y/( fgxy)= hz
or ( fx + g y - h z ) = + 2 y /( fgxy) ·
Agai n squaring, P x i + g V + h*z* + 2fgxy—Ifh zx—2ghyz—4fgxy
or f*x*+g*y*+h*z*—2fgxy—2fhzx—2ghyz=0 -.(2)
which is a homogeneous equation of degree two, hence the equa­
tion (2) Le. the given equation (I) represents a quadric cone with
the vertex at the origin.
To'find the reciprocal cone of (2) (i.e. of (I)]. In the equation
of the cone (2), we have
a = /*, b=g*, c=A*, / = —gh, g= -fh ,\h = - fg .
Λ A="bc -f*"=g*h*-g*h*=0. Similarly B = 0, C=O.
Also F ^ ‘g h - a f '’= { - fh ) (- f g ) - P ( —gh)=2f*gh.
Similarly G=Igth f and H~2h*fg.
Hence the equation of the cone[reciprocal to (2) [l.e. to (I)] is
given by
Axt +By*+Czt + 2Fyz+2Gzx+2Hxy= 0
pr 0+0+0+4f*ghvz+4g*hfzx+4h*fgxy= 0 .
Dividing throughout by 4fgh, we have
fx y+ g zx+ h xy= 0 . Proved.
Ex. 8. Prove that the perpendiculars drawn from the origin to
the tangent planes to the cone
3x*+4y*+5z*+2yz+4zx+6xy=0 lie on the cone
. 19**+1ly* + 3 z* + 6 yz-I0 zx-2 6 xy= 0 . (Kanpur 1979)
or Prove that the tangent planes to the cone
3*8+4>2+ 5 z*+ 2y z + 4zx+ 6xy= 0
are petpendicular to the generators o f the cone
19**+1ly*+3z*+6yz—IOzx—26xy=0.,
Sol. In accordance with the definition of the reciprocal cone,
in the present problem it is required to prove that the two given
cones are reciprocal to each other.
The equation of the first cone is given by
3**4 4y* + 5z*+ 2yz+ 4zx+ 6xy= 0. .. (I)
354 Analytical Geometry 3-D

Let the cone reciprocal to (I) be


Ax*+By*+Cz*+2Fyz+2Gzx+2Hxy—0. ...(2)
For the cone (I), we have
a = 3 . 6= 4, c ~ 5,/= 1 , £ = 2 , A=3.
Λ A=bc—f * = 4 .5 -(1 )* = !9, 2?=ca—g*= 1 5 —4 = 11,
C=eA-A*= 1 2 -9 = 3 . F=»gA-fl/ = 6 - 3 = 3,
G = A /-Ag= 3—8 = - 5 , H = fg —ch= 2 - 1 5 = - 1 3 .
Putting these values in the equation (2), the cone reciprocal
to (I) is given by
19x*+l ly*+3z*+6yz—IO z* -26xy=0
which is the equation of the second given cone. Hence proved.
§ 10. The angle between the lines in which a plane cuts a cone.
Tofind the angle between the lines In which the plane
ux+ vy+ w z= 0 cuts the cone
ox*+by*+cz*+2fyz+2gzx+2hxy=0. (Rajasthan 1976)
LetthepIane ux+vy + wz=0 ---(I)
cut the cone
F (x, y, z)m ax*+ by*+ cz*+ 2fy i + 2gzx+ 2Axy= 0 ...(2)
in a line given by x//=y/m =z/n .. (3)
where /, m, n are the actual direction cosines, of the line (3). The
line of intersection (3) passes through the origin since the vertex
of the cone (2) is (0, 0, 0) and the plane (I) also passes through
the origin.
Since the line (3) lies in the plane (I), therefore we have
ul+ vot-|-w» = 0 . -(4)
Again the line (3) is also a generator of the cone (2) and
hence its d.c.’s namely I, m, n will satisfy the equation (2) and so
we have
al*+ bm*+ cn* -|- 2'fmn + 2gnl+2hlm= 0. ···(5)
From (4), we have n = —(u/+vm)/w. Putting this value of n
in the equation (5), we get
ei*+Am*+c {—(ul+vm)lw}*+2fm {—{ul+vm)/w}
+ 2 gl {—(u/+ vm)/w}+2A/m=0
or /* {aw*+ cu*—2guw)+ 21m (cuv —fuw —gvw+hw*)
+m* (Aw*+cv2—2/vw)= 0
or (aw*+cu*—2guw) (//m)*+2 (cuv—fuw —gvw+hw*) {l/m)
-+(Aw2+cv*—2/vw )=0. ...(6)
The equation (6) is a quadratic equation in Ijm and hence it
The Cone 355

•hows that the plane (I) cute the cone (2) in two lines. Let the
direction cosines of these two lines be h, mt, nt and Z1, mt, π».
Thus the roots of the equation (6) are Iilmt and Zs/ma. By theory
of equations, using the formulae for the sum and product of the
roots of the equation (6), we have
Zi . Zt —2 (cuv—fuw—gvw+hw1)
Cut -^awt -Ig u w
Zimt-f ZtTW
l_______ !",In,____ » Zsav)
or —2 (cw —fuw—gvw+hw^^ciA+aw^—lguw 1 3
...(7)
I1 It__ b w * v w
and mt /η*- eu*+aw*—2guw
Z1Zf iw,m, WfW, A
·* hw, +cv*+2/rw"!aeii, +eH^—igsiW- ^ + * ! ^ - 2hwi “ '
...(8)
keeping in view (7) and writing the third fraction by symmetry.
(Z jlW f— Z1WJ1) 1 = (Z j IW f - |- ZfTW j)*— 4 Z jZ f IW1IW f
=4λ* (cuv—fuw—gvw+ Aw*)*
—4A* (Am^ + cv*—2/iwv) (« ι· + αη4—2guw)
~4A»w» { - ( ^ + ^ + 0 ^ + 2 ^ + 2 ( 7 ) ^ + 2 ^ ) }
=4λ·»ν·0*
where the expression within the brackets is denoted by Dt and is
given by the determinant
a h g U
h b f V

g f C W

u v w O
Also the capital letters A, B, C, F, G, H are the cofactors of
the corresponding small letters a, b, c ,f, g, A in the determinant
a h g
h b f .

g f c
Hence Umt - Zf/w1=>2AK'Z). I
Similarly Wi1Wl—/Wfij1■»=2\uD and ntlt —HtIi ^iXyD. J .. (9)
356 Analytical Geometry 3-D

Let O be the angle between the two lines of intersection, then


cos Θ=*IJ i+ mtmt +n,nt
=A (bw*+c**—2frw)+λ (cu*+AH'1—2guw)+A (ατ*-|-6ιΛ—2Awv)
« λ ((6+c) n»+(c+o) κ»+(β+6) w·- 2frw-2guw-2hur]
=A [(α+6+ c) (ut +v*+w*)-(au*+br*+cw*+2fvw
+2gwu+2huv)]
[on adding 8nd subtracting atF+b^+cw*)
=A [(α+6+c) («ι*+ν*+»·)—F(«, v, w)] IIn view of (2)]
sin* O=ImlHi - m*»I1)*+ (nxlt —n j x)*+ (/,mt —ItTnx)*
=4A* D1 (n*+v*+w*) , [Substituting from (9)]
or sin Θ= 2XD\/(u* -f r*+ w*).
Λ tan 9 2P V(tF+v*+w*)______
cos 0*“ (a -1-6 4-c) («*■+»*+»*)—F (u, v, w) ...(10)
P ednction I. To find the condition that the plane (I) cuts the
cone (2) In two perpendicular lines (i.e.generators). (Delhi 1976)
Io the case the plane (I) cuts the cone (2) in two perpendi­
cular generators (i.e. lines), the angle O=Jw l.e. tan 0= tan | π = oo.
Hence the required condition from the formula (10) is given by
(a+b+c) (iF+v*+w*)=F(u, v, w). ...(H )
Deduction 2. To find the condition that the plane
ux+ vy+ wz=0 cuts the cone ax*+by*+cz*=0 In two perpendicular
generators (lines).
In this case the direction cosines of the two perpendicular
lines are given by the relations
a/+vwi+w«=0 and al*+bnP+cnl·=0.
A ls o F (x ,y , z)=ax*+by*+cz*.
Λ F(u, ψ, w)=*arF+bv*+cw*.
From (11) the required condition is given by
(a+b+c) (u*4- ν’+ W2J= au*+ + cw*
of (6+c) a*+(c+fl) v*+(a+b) w*=0. ...(12)
Dednetion 3. To find the condition that the plane (I) cuts the
cone (2) in two coincident generators. In other words to find the
condition that the plane (I) be a tangent plane to the cone (2).
In this case 0= 0 i.e. tan 0= tan 0=0. Hence the required
condition from (10) is given by
2£y(«i*+i>*+w*)=0 i.e. D=O as ιι»+ν*+H-t ^O
l.e. Dt=O
i.e. ArJ+B^+Cw*+2Fvw+2Gwu+2Hur=0 ...(13)
The Cone 357

Note that the condition ( 13) is the same as is the condition


(9) obtained in § 8.
SOLVED EXAMPLES (E)
Ex. I. (a). Find the angle between the lines o f section o f the
olane 3 x+ y+ 5z= 0 and the cone 6 yz- 2zx+5xy=0.
Sol. The equation of the given piane is
4x+ y+ 5z= 0. (I)
The equations of the given cone is 6yz—2zx+5xy*=*0 - ( 2)
Let the equations of a line of section of the cone (2) by the
plane (Ifb e given by x//=> /in= r/« (3)
Then 3/-( m + 5/t= 0 ..(4) and 6mn—2nl+5lm=0 ...(5)
Eliminating m between (4) and (5), we get
6 ( - 3 / - 5n) n -2 n /+ 5 / ( - 3 / -5 « )= 0 or 30w*+45n/+l5/*
or 2ϋ·+3η/+/*= 0 or (2n+/) (n + /)= 0 .
Λ 2λ+/=»0 or n + l= 0.
When 2 n + /= 0 i.e. /=· —2n then from (4), m=n.
Λ //(—2 )= m /l= » /l.
Again when n + l= 0 i.e. I= —n then from (4), m=» —2n.
/ //1= m /2= «/(— I).
Hence the equations of the lines of section are
x y z ■■ x y z
=5“ Γ - Γ “ d Γ - Γ - - Γ
Let 0 be the angle between these lines of section.
Tben cos ¢=- ( - 2 . . . + (1),(2)+ (1).(-1)
V(( - 2)·+ ( I)■+ ( 11’) ν (α )'+ (2 )·+ < - 1)*1 6
Λ the acute angle Θ between the lines of section is given by
0= co s-' (1/6).
Ex. I (b). Find the equations to the lines in which the plane
2 x + y —z=- 0 cuts the cone 4xi —y*+5z*=0.
Find also the angle between the lines o f section.
(Avadh 1981; Agra 82; Rohilkhand 77; Madras 78)
Sol. Let the equations of a line of section of the cone
4jc*—γ*+3ζ*=0 by the plane 2 x + y —z = 0 be given by
x/l= ylm = z/n . ...(I)
Then 2/+WI—n = 0 ...(2)
and 4/*—m*+3n*=0. -.(3)
From (2), n = 2l+ m . ...(4)
Putting the value of n from (4) in (3), we get
4/»_m»+ 3 (2/+iw)»=0 or 8/*+6/m+m*=0
3S8 Analytical Geometry 3-D

or (4/+m) (2/+m )—0. .·. 4/+ m = 0, 2/+ m = 0.


When 4/+ r»=0 t.e. m= —4/ then from (4), « = —2/.
Λ //1=»«/(-4 )= /1 /(-2 ).
Again when 2i+ m = 0 f.«. m = —2/ then from (4), n=0.
Λ //l = ffi/(—2)=n/0.
Hence the equations of the lines of section are
JC )/ 2 x )/ *
r= -4 ---2 “ d r = - 2 - r
Let 0 be the angle between these Uaes of section.
1.1 + '( - 4 ) .( - 2 ) + ( - 2 ) . 0
Then cos 4='
v W + (-2 ? -H 0 ? }

* V (2 l)v /(5 ) " 7 ( 3 5 ) ‘


.·. 4=cos-V {27/35).
Ex. 2. FiW the angle between tl ie Unes whose d.c.*s are given
by the equations ul+vm+w a=0
and aP+bm'+cn*+ Ifmn - t-2gn/+2A/m=0.
Sol. Clearly it is required to fi nd the angle between the lines
of section of the cone ax*+Ay*+ez! 1+ 2 fyz + Ig zx +2Axy= 0 by the
plane ux+vy-)-wz=0. Heneesee $10 above.
Ex. 3. Show that the condltlc m that the plane ux+yy+wz=0
may cut the cone ex, +by*+a, =0 In perpendicular generators Is
(b+c) u*+(e+a) v*+(o+b) w*=0.
Sol. This problem has alre sdy been solved in ’deduction 2 of
§ 10*. But here we are giving it a complete solution.
Let. the equations of a line of section of the cone ax'+hy*
+cz'= O by the plane u x + yy+ wz=0 be given by
Jc/f=y/«·=»*/«. ...(I)
Then a F + b W + c n ·= 0 ...(2)
and u/+vn*+wn=Q. -.-W
Eliminating n between (2) and (3), we get
a/*+dm*+c {—u/+wn)/w}*=0
or (ew»+cu*) /*+ Icuvlm+ (4w*+ cv1) m*=0
or (aw*+cu*) (//m)*+2cuv (//m)+(6w*+ev*)=0 ...(4)
This is a quadratic equation in Ilm and hence it shows that
the plane u x + y y + wz=0 cute the given cone in two lines (or
generators). Let I1, mlt Rt and I1, m», Ra be the d.c.’s of these lines.
Then I1Im1 and Ulm1 are the roots of the equation (4). The pro·
T h e C one 359

duct of the roots of the equation (4) is given by


/i_ U _ bw*+cp*
Wi1 ‘ Wi1 aw*+ cu*
IJa = Wi1OT1__ B1W 1
bM^+cv* eu’ +απ’* αν’+bu*'
writing the third fraction by symmetry.
Now the generators (l.e. the lines) will be perpendicular if
/ ^ + BliOT1+ BtB1=O
l.e. if (bw* 4-cv,)-f-(cu*+flH',)4-(ev*+4u*)=0
l.e. it (b+c'h u*+(c+a) v*-Ha 4-ft) w*=0. Proved.
Ex. 4. Show that tHe plane ax+ by+ cz·*0 cuts the cone
y z + zx + x y **0 In perpendicular lines If 1 /e + l/b + l/c= 0 .
(Garhwal 1982; Indore 78, 79; Kanpur. 80, 82;
Meernt 81, 84S, 85S, 86, 89; Lucknow 77, 80;
Robilkhand 78,82; Rajasthan 76, 78)
Sol. Let the equations of a line of section of the cone
y z+ zx + x y = 0 by the plane ax+ by+ cz= 0 be given by
xll=zym*=zln. ...(I)
Then otb+ b/+ / ot= 0 -(2 )
and al+bm+cn*=0. -(3 )
Eliminating n between the equations (2) and (3), we get
Ot{—(a/+bw»)/c}+{—(al+bm)/c} /+ / ot= 0
or al*+ (a+ b-c) /ot+ 6 ot*=0,
or a (llm)*+(a+b—c) (l/m )+ b= 0. ...(4)
This is a quadratic equation in l\m and hence it shows that
the given plane cuts the given cone intwogenerators(i.e. lines).
Let /„ mi, Bi and Ii,Wt1, B1 be the d c.’s ofthese two lines so that
the product of the roots of the equation (4) is given bv
lj_ l*_= b
Bi1 ' Ot1 aI
L·!* OT1OT1 BiB1 „ 4
,. = Ii Ib /t *= Ii Ic
/. (by symmetry).
II a
These lines of section will be perpendicular if
M*+Ot i Ot1+Bin1= 0 ie . if I /o + l/b + l/c= 0.
Ex. 5 (a). Prove that the angle between the llnesgtven by
x + y + z = 0, ayz+bzx+cxyz-0 is Jw
If a + b + c= 0 but in If l/e + l/b + l/c = 0
(Agra 1981; Meernt 75; Pnnjab 79, 82)
360 Analytical Geometry 3-D

Sol. Let the equations of a line of section of the cone


ayz+ bzx+ cxy=*Oby the plane x+y+z=>0 be given by
x/l^ylm *" z/n. ...(I)
Then amn+bnl+clm = 0 --(2)
and I + / m+ r = 0 . ...(3)

Eliminating n between the equations (2) and (3), we get


am (—l—m)+bl ( — c/ m=0
or bl%+ (a + b —c) Im+am* =-0
or b (l/m)*+(a+b—c) (l/m)+a=0. ...(4)
This is a quadratic equation in Ijm and hence it shows that
the given plane cuts the given cone in two lines (i.e. generators).
Let l„ mi, M1 and /», mt, Rt be the d.c.’s of these two lines, so that
we have from (4), the product of the roots= — . —
fill Wa O
/MlRlt Mt Rt
(by symmetry)
a o c
=A (say). •(5)
Case I. The angle between the lines of section is
In this case, we have
Ii/t + /MiRit +RiRt =O or aA+bA+cA=0, using (5)
or a+ b+c= 0, [ y AwfcOj. Proved.
Case II. The angle between the lines of section is Jv.
The sum of the roots of the equation (4) is given by
lj_ _|_l« a + b -c
Rll Rlt c
Iimi +Umi mxm%
·· c—a—b =A [from (5)).
- ( 6)
Λ (IiR tt - I i R l i ) 1= (IiR it + It Rli )* — 41,It RiiRit
=A* [(c—a—b)*—4.ab], using (6) and (5)
=A* [a*+b*+c*—2bc—2ca—2ab].
By symmetry, we have
(/MiR1-R itMi)*=(RtIt- R tIi)*=A* [a*+b*+c* —2bc—2ca—2ab]
Now tan* 0 = ; Σ (IiRlt - I tRli)*
(ltl*+ RliRI1 + MiRl )*
Since the angle between the lines is ir/3, therefore we have
3A* (a*+b%+c*—2bc—2ca—2ab)
tan* Jir*
A* ( a + b + c j *
or 3 (o+b+c)*= »3 (a*+b*+c*-2bc- 2ca—2ab)
The Cone 361

or (a*+£>*+ c* + 2be+ 2ca+ 2a6)= a*+ b*+ c*—2be —2ca—2ab


or 4 (6 c+ ca+ a6 )= 0 or I fa+ 1/6 + l/c= 0. Proved.
Ex. 5(b). Show that ike angle between the lines given by
x + p + z = 0 and + — H—— = 0 Is Jw.
q -r r-p p -q
Sol. Putting a=>\J(q—r), 6 = 1 / r - p ) , c=-ll(p—q),
the equation of the given cone becomes
ayz+ bzx+ cxy ^ 0 .
Here we see that l/ a + l /6 + l /c = g —r + r —p + p —q=0.
Hence the angle between the lines of section of the cone
JrzH q.-r)+ z x /(r—p)+ xy/(p—¢ )= 0 by the plane x fp+z=>0 is
(See Ex. 5(a) above].
Ex. 6. Find the angle between the lines of section o f the plane
6x—y —2z=*Q and the cone Ι08χ*-7ρ2—20z*=0.
Sol. Let the equations of a line of section (i.e. generator)
of the cone I08x*—7^*—20z*=0 by the plane 6x—y —2z=0 be
given by
x/l=>ylm=zln. ...(I)
Then 108/*-7w2-20n*=0 ...(2)
and 6/—w —2w=0. ...(3)
Eliminating w between (2) and (3), we get
108/*—7 ( 6 / - 2w)2 —20w2= 0 or 144/2-168/w+48n»=0
or 6/2—7/w+2w*=0 or (3/—2«) (2/—»)= 0
or 3/=2n, 2/=n.
When 3/=2w, from (3) we havew=3/.
3 /= w = 2/1 or //2=w/6=w/3. -(4 )
When 2/=/i, from (3), we have m=2l.
2 /= w = /i or //l= w /2 = n /2 . .."(5)
The direction ratios of the two lines of section are given by
the relations (4) and (5). If Θ be the angle between these two
lines of section, then we have
CQS 0 = __________ / , / , + W tW , + W 1W,__________
V (h *+ m,*+ W1*) V (it*+ "h* + «**)
_ 2 .1 + 6 .2 + 3 .2 _2 0 _ 2 0
V U2/*+ lb)*+ (3)2) ν' K I )*+ 12)*+(2)2} e 7 .3 “ 2 l’
Λ O=COS-1 (20/21).
Ex. 7. I f the plane 2 x —y + cz=0 cuts the cone y z+ zx + x y = 0
in perpendicular lines, find the value of c.
362 Analytical Geometry 3-D

Sol. Let tbe equations of a line of section of. the cone


y z + z x + x y = 0 by the plane 2 x —y+ cz= 0 be given by
xll*=ylm=zln. ..( I )
Then mn+nl+lm=^0 ...(2)
and 2l—m+cn=*0. ...(3)
Eliminating m between the equations (2) and (3), we get
(2l + cn) n+ nl+ l (2/+ c«)--=0 o r 2/*+(3+c) nl+cn2= 0
or 2 (//«)»+ (3 + c )(//n )+ c -0 . ...(4)
This is a quadratic equation in Ijn and hence it shows that
the given plane cuts the given cone in two lines. Let Iu mu n,
and U, mt, n, be the d .c ’s of these lines, so that we have from (4)
the product of the roots= — . so that
Tl\ Hg Z,
/»/*/c=«i«i/2. .(5 )
Now eliminating I between the equations (2) and (3), we get
mn f n {(m~cn/2)+{(m—cn)/2} m = 0
or m* i-(3 - c) m n-c n 2—0 or (m/n)*+(3—c) (m/n)—c=0.
the product of the ro o ts= — . — = c, so that
nI H2
Hi1Iti2K—2c)= rtirtj/2. ... (6)
From the relations (5) and (6), we get
IlI2^iriim2 _ nxn2
~F ~^2c ...(7)
According to the given problem, if the lines of section are
perpendicular, then we have
/1/2+ H i1Ttl2 + /IiWgc=O
or c—2c-f-2^0 or c=»2. Ans.
Ex. 8. Show that the plane. ax+ by+ cz= 0 cuts the cone
y z+ zx + x y = 0 in two tines inclined at an angle
-1 [ { ( ^ + ^ + C2) (0s+ fc*+ c2—lb c —2ca—2ab)}1/al
L bc+ca+ab J
and by considering the value of this expression when a+b+c*=0,
show that the cone is o f revolution and that its axis is χ*=*γ=*ζ and
vertical angle tan"1 ( —2%f2).
Sol. Let the equations of a line of section of the cone
y z + z x + x y *=*0 by the plane ax+ by+ cz= 0 be given by
xll=*yjm=zln. - ( 1)
Then mn+nl+lm=>0 - ( 2)
The Ctme 363
and al+bm+cn—O. ...(3)
Eliminating n between the equations (2) and (3), we get
m { - (al + bm)/c) + {- (al+ bm)/c) /+ /« = 0
or a P + (a + b -c ) lm+bm*=*0,
or a (llm)*+(a+b—c) (//m )+6=0.
Let the roots of this equation be /,/« , and /,/m*. then we get
h /, b ,. '4ϋ W1W2 . . v
—-· - = - . so that
w , IVti 0 1 /0 -J jb -= J fi = A(say) (by symmetry).

Also ^ - + /a 0 + 6 —c' oT Z1WI^Z2WIie c— a— b


l»i WI1 a ^WI1WI2 ab
ZllWt2+ ^wi1 Wl1WI2
or =λ, using (4).
(c*—a —b)l(ab) I/b
Now (ZtWi2 Z2Wii ) * = (Z1Wi2 *f Z2Wi1)*— 4 Z1Z2Wt1Itff
.,/(£ -£ --6 )1
{(c— 6)*-4a6}
* I
(a*+6*+C* - 2 6 c - 2 c a - 2 a & H ^ ,
where **= α*-j-6*+c»- 2£>c- 2ca - 2α6. ...(5)
Λ Σ (/,/n1- / i/fi1)*c=A*jfe* (1/(α*6»)+ l/(6*c»)+(l/(c*e*)}
_λ**» (a*+*»+c*)
e*6*c*
and 4 4 + / ^ , +W1B1=A (l/a + t/6 + l/e )= A (bc+ca+ab)/(abc).
Let the angle between the two lines of section (Le. generators)
be Φ, then we have
tan p - V tP ('./*,-/**!,)*} _ Ak (a2+ b'+c*)'i*
iil*+Ifiifnt + ntnt A (bc+ca+ab)
or a {(o*+b*+c*—2bc—2ca—lab) (aV+^+c*)}1/**i
(bc+ca+ab) ...(6)
i [Using (S) for Λ).
Again if e + b + c = 0 , then (a+b+c)*«=0
or e*+b*+c*=—2 (bc+ca+ab).
Putting this value of e s+ ^ + c * in (6). we have
Λ_ [{ —4 (bc+ba+ab)).{—2 (bc+ca+ab)})1'2
t a n f l - --------- --------bc+ca+ab------------------
= V 8 ~ ± 2 v /2 . ...(7)
Since a+fc+ce=«0, therefore the line x /l« .y /l= z /l lies on the
!pane ax+ by+ cz= 0.
364 Analytical Geometry 3 -D

The equation (7) therefore gives the angle between the gene-
rotors of the cone cut by a plane passing through the line
*/1= ^/1= z /l. It is required to prove that this line is the axis of
the given cone. It will be so if we prove that the angle between
this line namely $/1=^/1 = 7/1 and any generator of the cone is
half of the angle Θgiven by (7) Le 9 vertical angle of the cone
contained between the generators in which a plane through the
axis cuts the cone.
Now if x/l= ylm —zln is a generator of the cone
y z + z x + x y = 0, then mn+nt+lm=0. ..(8)
Let a be the angle between this generator and the line
x/l==y/l==z/l then we have
_______L I H-m. I + n. I _____
cos “ - λ/{(/*+«*+«*)
_________ l+m-\-n
V $- y/{(l’+ m+n)*—2(mn+nl+lm)}
I
since mn+nl+lm=*0 from (8).
λ/3·

Λ ta n « = V 2 . Also tan -----2 ^ 2 . -.(9)


From (7) and (9), tan 0=tan 2a = —2y/2. ...(10)
0=2a or a<=>$0.
Hence the line x=y=*z is the axis of the cone and the cone is
of revolution.
Ex. 9. Prove that the equation to the planes through the origin
perpendicular to the lines o f section o f the plane
lx+ m y+ n z= 0 and the cone ax*+by*+cz*=*0 Is
X t (h /ia+ c m * ) + ^ * (cl*+an*)+z* (am*+bl*)—lamnyz
—2bnlzx—Tclmxy= 0.
Sol. Let the equations of a line of section of the cone
ax*+by*+cz*=*0 by the plane lx+ m y+ nz= 0 be given by
χ /λ = ^ /μ = 2/ν. ...(I)
Then AA1-Fhjtia-Fcva=O and l\+mp+nv=*0.
Eliminating v between these relations, we get
AA1-Fhjtia-I-C {—(lλ+mμ)|n)^—0
or (A«a+ c/a) {X/p)2+2lmc {λ/μ)*+(bn*+cm*)=0. ...(2)
This is a quadratic equation in A//t and hence it shows that
the given plane cuts the given cone in two lines. Let Xlt μ,, V1
The Cone 365

and A1, Mt, Vi be the d.r.’s of these two lines so that* from (2) we
have,
the product of roots= —. —=»^—~ r ~ i r *
Mt Mt an*+ c l*

• <b* <«*>·
..(3)
Also the sum of the roots=— + — · lime
Mi Mi an2+cl2
*iMi+*iMt _ MiMa = AT[from (3)]
—2Imc
Miva+Mivi ν>*»+ν**ι (by symmetry) .(4)
. —2mna —2nib
Now the equations of the two lines of section are
*/Ai «=j7mi = z/ vi and χ λ a=JVmi= */vi.
Therefore, the equations of the planes through the origin
perpendicular to these lines of section are
A,*+Mi.?+viZ= O and A2X+ Ma^+V2Z=O.
Hence the combined equation of the planes through the origin
perpendicular to the lines of section is
(λι*+Μι.V+viZ) (A2X+ p% y + v2z)= O
or A1A1X1+ /! ,^ * + V 1ν ^ + ί ^ ν * + ^ ) yz
+ (A1V2+A2Vj) zx + (Aj/z2+Aj/A|) X^==O
or (bnv+cma) x*+(cl*+an2) y 2+(am2+bl2) z 2
—2amnyz—2 b n lzx -Iclmxy=O9 using (3) and (4).
Proved.
Ex. 10. Prove that the conditions that the lines o f sections oj
the plane lx+ m y+ nz= 0 and the cones
ax2+by2+cz2*=*0, fyz+ gzx+ hxy= Q may be coincident are
bn2+cm2 ^ c l 2+ an2^ a m 2+bl2
fmn gnl him (Meerut 1977)
Sol. Let the equations of a line of section of the cone
ax2+by2+cz*«=0 by the plane lx+ m y+ nz^ 0 be given by
χ/Λ=^//χ=ζ/ν. Then, we have
a\2+bp2+ cva=0, /A+m/t+nv=0.
Proceeding as in Ex. 9 above, we get
*i*i _ MiMi _ VjVi
btP-t cm2 cl2+an2 am2+bl2 -...(I)
366 Analytteal Geometry 3-D

Again let the equations of a line of section of the cone


fyz+ gzx+ hxy= 0 by the plane lx+ m y+ nz= 0 be given by
x/p=ylq=z/r.
Then, we have
fqr+grp+hpq=0, lp+ m q+ nr^ 0.
Eliminating r between these relations, we have
fq {~(lp+mq)ln}+g {-(lp+ m q)fn} p + Itpq==O
o t gl(plq)*+ ifl+ gm —hm) (plq)+jm=0. -(¾
This is a quadratic equation in p/q and hence it shows that
the given plane cuts the cone fyz+ gzx+ hxy=0 in two lines. Let
Pu qu Tx and p%, qt, rt be the jd.r.’s of these two lines, so that
from (2), we have
P i P a ^ f m or PiPi (Jll)
qt q t ^ - g i qtq* (g M

··* m ^T T n ^ s y m m e tr^ ...(3 )


Now if the lines of section of the plane lx+ m y+ nz= 0 and
the cones ax*+ by*+ cz*^0,fyz+ gzx+ hxy—0 be coincident, then
we should have

Pt Ρ* qiq» V i
bn*+cm* _ c/2-i-an*_ am*+ bl*
Jjl “ g/m “ h/n
Multiplying the denominators throughout by lmn, the required
conditions are given by
bn*+cm* cl*+an* am*+bl*
fm n β gnt ~ him ·
Ex. 11. Show that {he planes which cut ax*+ by*+ cz*—0 In
perpendicular generators iouch the cone
x*l(b+c)+y*l(c+a)+z*l(a+b)=o: (M.U. 1990)
Sol. The equation of the given cone is
ax2+^y2H-CZ1=O. ..( I )
Let the equation of any plane through the vertex (0, 0, 0) of
the cone (I) be
Kjf+vy+H>z=0. .. (2)
Suppose the plane (2) 4puts the cone (I) in perpendicular
generators. Then proceeding as in Ex, 3 above, we have
(h+c) u*+(c+a) v*+(a+h) W2=O. -(3 )
The Cone 367

From (3) we observe that the normal to the plane (2) through
the origin i.e. the line x/u= y/v= zlw lies on the cone
(6+c) x*+(c+a) y*+(a+b) z*=0. ...(4)
Now the plane (2) is a tangent plane to the cone which is
reciprocal of the cone (4). The equation of the cone reciprocal
to the cone (4) is
**/( b + c) +y*l(c+a)+ z*j[a+ b)= 0 ... (5)
[See Ex. 4 page 347].
Hence if the plane (2) cuts the cone (I) in perpendicular
generators then it touches the cone (5).
Ex. 12. Show that the locus of the line of Intersection o f tangent
planes to the cone ax*+by*+Czt =O which touch along perpendicular
generators Is the cone
a* (b+c) x*+b* (c+a) j^+c* (a + b )r* = 0 .
(Kanpur 1977, 81)
Sol. The equation of the given cone is
ax*+by*+cz*=0. ...(I)
Let x//= yIOi=Zjn · ...(2)
be the line of intersection of the two tangent planes to the cone
(I) which touch the cone along two generators O Fand OQ where
P and Q are the points («,, βι„ yx) and (a*, )3*, y*) respectively.
The tangent plane to (I) at (Ot1, βu yt ) is
aoti x + bfiiy+ CytZ= 0.
Since it contains the line (2), therefore
a*1l+bfi,m+cyln=0. . ...(3)
Similarly since the tangent plane to (I) at the point (a2, /3*, y*)
also contains the line (2), therefore
OixiI+bfitm + Cyjt=O. —(4)
From (3) and (4) it is clear that both the points («*, JS11 yt)
and (α*. β·2, y*) lie on the plane
alx + bmy+cnz= 0 (5)
through the origin. Hence (S) is the equation of the plane OPQ
containing the two generators OP and OQ thetangent planes
along which intersect in the line (2).
If we put al=u, bm=v, cn=w, the equation of the plane (5)
becomes
ux+ vy+ w z= 0. ..(6)
The plane (6) cuts the cone (I) inperpendiculargenerators
OP and OQ. Proceeding as in Ex. 3 above, the condition that the
plane (6) cuts the cone (I) in perpendicular generators is given by
368 Analytical Geometry 3-D

(b+c) tP+(c+a) v* + (a+ 6 ) Wf=O


or (b+c) (a/)*+(c+a) (bm)*+{a+b) (cn)*=0
or a* (b+c) l*+b* (c+a) ni2+c* (a + b) n*=0.
.·. The locus of the line of intersection xll=ylm=*z]n is
a* (b+c) x*+b* (c+a) y*+c* (a+b) z*=0,
which is the equation of a cone.
Ex. 13. Prove that the locus o f the line o f intersection o f two
perpendicular tangent planes to ax*+by*+ cz*= 0 is
a (b+c) x*+b (c+a) j J^-c (a+b) z* —0. (Rohilkhand 1982)
Sol. The equation of fi»e given cone is
•ax*+ by*+cz*=0. ···(!)
Let x//=^//w*=z/n --(2)
be the equations of the line of intersection of two perpendicular
tangent planes to the cone (I).
Let the equation of any tangent plane to the cone (I) be
ux+ vy+ w z—0. .(3 )
Then the normal to the plane (3) through the vertex (0, O9 0)
of the cone (I) namely the line x ju ^ y /v ^ z /w will be a generator
of the reciprocal cone x*la+y*lb+z*lc=*0, so that we have
u*/a+ VtIb + 0 or bcu%+ cava + abw2= 0. .. (4)
If the plane (3) contains the line (2)9 then
I//+ Vtft + W7!=»0. ..(5)
The equations (4) and (5) give the d r.'s of the normals to
those tangent planes to (I) which pass through the line (2). Since
(4) is a quadratic in u, v, W9 therefore (4) and (5) will give two
pairs of values of u, v, w. Thus two tangent planes to (I) will
pass through the line (2;. Let u,, vlf W1 and u2, v2, w2 be the d.r.’s
of the normals to these two tangent planes. Since the tangent
planes are given to be perpendicular, therefore we have
Uiiz1H-ViV2H-W1Wtt=O. ...(6)
Now eliminating w between (4) and (3), we get
bcu2+cav%+ ab {—(u/H-vm)/ZiJ2--O
or (bcn*+abl%) tf+lablm uv+icarf+abm 2) v*=0
or (bcn2+obl2) (u/v)2+2ablm (u/v) + (can2 +(Jbmt)=^O.
T* « . . Ui U2 carp + abm*
Itsproductof the roots·=— . — = — rrr'
V1 v2 benr + obr
Uj U1 Vj V2
(by symmetry).
Cant -^ubmt α abi1+ ben2 e bcm* ^ca lt
Putting these proportionate values in (6), we get
The Cone 369

(can*+abmt) +·(abP+bcn*)+(6cwi*+fe/*)«=0
or a(b+ c) l9+b ( c + q) m*+c (o-\-b) n*=*Q.
.·. The required locus of the line of intersection (2) is given
by
a (b + c )x * + b (b+a) y*+c (a+b) 2*=0
which being a homogeneous equation of second degree represents
a cone with the vertex at the origin.
Ex. 14. A line OP Is such that the two planes through OP, each
o f which cuts the cone aXt +byt +czt = 0 in perpendicular generators
are perpendicular. Prove that the locus o f OP is the cone
(2a+b+c) xt +(a+2b+c) y*~+(a+b+2e) Zt =O.
(Jodhpur 1976)
Sol. Let xll= ylm = zln -(1)
be the equations of the line OP.
The equation of any plane through OP may be given by
ux+vy+wz=Q -*(2)
where ul+vm+wn=0. -(3)
Now proceeding as in Ex. 3 above, the condition that the
plane (2) cuts the cone axt +by*+czt = 0 in perpendicular genera­
tors is given by
(b+c) 11*+(<·-(a) +6) We==O- —(4)
The d.r.’s 11, v, w of the normal to the plane (2) are given by
the relations (3) and (4). Also the equation (4) being a quadratic
in 11, v, w shows that there will be two planes like ux+ vy+ wz= 0.
Let Hi, Vj, Wi and ut , v2, w* be the d.r.’s of the normals to the two
planes. Since the two planes are given to be perpendicular, there­
fore we have
Ui Ut + ViVi -I-WiWi =O. .(5)
Now eliminating w between (3) and (4), we have
(b+c) u*+(c+a) v*+(a+b) {—(ii/+vm)/n}*—0
or {(b+c) n»+(a+b) /*} (u/v)*+2 (a+ b) Im (u/v)
+ {(c+a)«*+(a+b) m*}=0,
which is a quadratic in u/v.
Its prodnct of roots' u%_ Μ* _ (c+a) n*+(a+b) iw*
ν ,· ν 2 - (b+c) n '+ le + b ) /*
. ___________ U l Ut _____________________________ V 1V 2 ___________
(c+o) «*+(a+b) m* “ (a j-b) P +(b + c) n*
______ w,wt ___
(by symmetry).
“ (b+c) m*+(c+a)P
370 A nilytkal Geometry 3-D

Putting these proportionate values in (5), we get


{(e+a) rp+(a+b) ml}+{(a+b) l*+(b+c) n1}
+ {<*+c) m *+(c+a)l') = 0
or (2a+b+c) P+ (a+ 2b+c) m2+ (a+ 6 + 2 c) n*=0.
The locus of OP Le. of the line (I) is given by
(2a+b+c) x*+(a+2b + c) y*+ (a+b+2c) Z2=O
which being a homogeneous equation of second degree represents
a cone with the vertex at the origin.
§ II. Three antnallj perpendicular generators.
7b find the condition for the cone
F (X9y %z)==ax2+6y* + a?+ 2fyz+ 2gzx+ 2hxy=0
to hawe three mutually perpendicular generators.
(Agra 1974; Rohilkhand 80; Madras 76)
The equation of the given cone is
F (x 9y 9 z)sax*+by*+cz%+2fyz+2gzx+2hxy=0. ...(I)
Let jr/u=y/r=z/w be the equations of one of the generators
of the cone (I) so that its d.r.’s will satisfy the equation (I) of the
cone and, therefore, we have
F( U9 V9 W) =O . .(2)
The equation of the plane through the vertex (0, Ov 0) of the
cone (I) and perpendicular to the generator xlu ^yjv= zjw is
ux+vy+wz=0. .(3)
Now from ‘Deduction I to § 10* the plane (3)will cut the
cone (I) in two perpendicular lines (f.e. generators) if
(o+ 6+ c) (u2+ v, + ivJ) = F (u t v9 w)
or (a+b+c) (1^ + ^ + ^ ) = 0. using (2)
or o + ft+ c = 0 [7 u^ + ^ + h^^O being the sum of squares of real
numbers not all zero]
The condition a+b+c*= 0 being independent of u, v, w shows
that the plane through the vertex of the cone and perpendicular
to any generator cuts the cone in two mutually perpendicular
generators and each one of these two generators is also perpendi­
cular to the first generator Hence we conclude that the cone (I) has
a set o f three mutually perpendicular generators if
a + b+ c= 0 (4)
Le. if the coefficient of x* + the coefficient o f y*
+ the coefficient of z2= 0.
We also see that the condition a+ b+ c= 0 is . independent of
u, v9 w and hence a cone, in general, has an infinite number of
sets of three mutually perpendicular generators.
The Cone 371

Remark. If it is required to prove that a plane ux+vy+wz=0


cuts the cone ( I ) in two perpendicular Iioesr and if We show that
the cone (I) has a set of three mutually perpendicular generators
iii ο+6+<·=0, then the normal to the plane ux+ vy+ tvz= O
through the vertex must be a generator of the.cone (I) and hence
the d.r.’s u,v, w of the normal must satisfy the equation (I) of the
cone t.e. F (u, v, w) must be equal to zero.
§ 12. Three mutually perpendicular tangent planes.
To find the condition that the cone
F (x, y, z)= axt +by*+cz, + 2fyz+ 2gzx+2hxy=0
may have three mutually perpendicular tangent planes.
The equation of tbe cone reciprocal to the given cone is.
[See § 9 (A)
Ax2+By2+Cz*+2Fyz+2Gzx+2Hxy=0. ...( 2 )
Tbe given cone will have three mutually perpendicular tangent
planes if its reciprocal cone (I) has three mutually, perpendicular
generators and the condition for tbe same is
A+B+C= O
or (b e-f* )+ (ca -g * )+ (Ah-A1)=O
or bc+ca + ab^=f*+g*+ht.
SOLVED EXAMPLES (F)
Ex. I. Show that the plane lx-\-my+nz=0 cuts the cone
(b—c) x*+(c—a) y 3+ (a —b )z* + Ifyz+ 2gzx+ 2hxy=0 in perpendi­
cular lines if
(b—c) /2-f-(c—a) m2+ (a -b ) z2-+2fmn+2gnl+2hlm=0.
(Meerut 1983S)
Sol. The equation of tbe given cone is
(b -c) x*-(-(c—a) y*+(a—b) z*+2fyz+2gzx+2hxy=0 ...(I)
Here we see that tbe sum of the coefficients of x*. and z*
in the equation of.the given cone = ( b - c)+ (c—a)+(a—b)—0.
Hence tbe given cone (I) has an infinite number of sets of
three mutually perpendicular generators . [See § 11]
Now if tbe plane lx+ my-Ynz=O cuts the given cone (I) in two
perpendicular lines (l.e. generators) then the third mutually per­
pendicular generator is the normal to the plane lx+ m y+ nz= 0
through the vertex (0, 0, 0) of the cone (I) i.e. the line x/l= y/m
=*i!n is the third generator of the cone (I) and hence -its d.r’s I,
m, n will satisfy the equation (I) of the cone. Therefore, the
required condition is given by
372 Analytical Geometry 3-D

(b-r-c) P + (c—a) m * + (a -b) n2+2fmn+ 2gnl+ 2hlm=0.


Ex. 2. Find the locus o f the point from which three mutually
perpendicular lines can be drawn to intersect the conic
2=0, ax*+by*= I. (Rajasthan 1974)
Sol. Let the point whose locus is to be determined by
P (*· β· y )· Now first we shall find the equation of the cone whose
vertex is the point P (α, β, y) and the base curve is the conic
Z=0, ax*+by*= I.
The equation of the above cone is [See Ex. 3 on page 340]
a ( a z -y X )* + b ( fiz -Y y )* = ( z -y ) * . ...(I)
The cone (I) will have three mutually perpendicular genera­
tors if
the coeff. of x*+the coeff of y’+ th c cocff. of z*=0
or ey*+6y*+(ea*+fi/3*—1)=0
or a**+bfi*+(a+b) y *= l.
Hence the required locus of P (*, β, y) is ax*+by*+(a+b) z*= I.
Ex. 3. Prove that the locus' o f the points from which three
mutually perpendicular lines can be drawn to intersect a given circle
x*+y*=a*, z= 0 is a surface of revolution.
Sol. Let the point whose locus is to be determined be
P (a, β , y). Now first we shall find the equation of the cone whose
vertex is the point P (α, β , y) and the base curve is the circle
x*+ y*= a*,z= 0. ...(I)
The equations of any line through P (α, β, y) are
(x —*)ll= (y—fi)lm = (z—y)/n.' ...(2)
It meets the plane z= 0 at tbe point
)'
This point will lie on the circle (I) if
(a—/y/n)*+(/J - my/n)*=a*. ...(3)
Eliminating I, m, n between (2) and (3), tbe equation of tbe
cone with tbe vertex at P (α, β, y) is _

or (az-yx)» +(βz-yy)*= a* (z-y)*. ...(4)


Tbe cone (4) will have three mutually perpendicular genera­
tors if
the cocff. of x*+the coeff. of y*+the coeff. of z*=0
t.e., y, +y*+(**+/3*-a*)=0, or **+β*+2y*=a*.
tbe required locus of P (α, β, y) is x*+y*+2z*=a*
The Cone 373

which is the equation of an ellipsoid (a surface of revolution)


obtained by revolving the ellipse
y= 0, χ*+2ζ*«·ο* or x= 0 ,y* + 2 ζ*=α*
about the z-axis.
Ex. 4. Find the locus of the points from which three mutually
perpendicular tangent planes can be drawn to touch the ellipse
XtIat -^yiIbt = I, z= 0.
Sol. Let the point whose locus is to be found be P (a, β, y).
The equation of the cone with the vertex at P (a, β, γ) and the
base curve the ellipse XiItP-^ytIbi = I f z = 0 is [See Ex. 2 (a), set
(C), page 339]
b* («z—yx)*+fl* (fiz+yy)i = aibt (z—y)*
or b*y*Jt*+ a*y*.y*+ (6*«*+ α*β*—α*!Ρ) Zi - I a i B yyz-IlP ctyzx
+ 2a*b*y Z -^ fl, />*y* =* 0 ...(I)
The cone (I) will have three mutually perpendicular tangent
planes if
“bc+ ca+ ab= fi +gt + hi ” [See § 12J
i.e. aty'i.(b,xi y a1β*—atbi)-\-(b*at -}-atfit —aibt)b tyt -\-albiyt
= ( —e*^y)*+(—h*ay)*+(0)*.
Dividing throughout by y*, we get
QiIPat + ο ψ - α * Ρ + Ρ Ρ + α ψ β * - α * Ρ + α ^ Υ = α ψ + IPat
or u2i* (**+j3*+y*)—e*i* (e*6*)“ Q
or α*+β »+y*=fl*+b*.
Λ The locus of P (a, β, y) is χ*+^, + ζ , = β , +Λ*
which is a sphere with centre at (0, 0, 0) and radius y/iaP+b*).
Ex. 5. Show that a cone whose vertex Is at the origin and which
passes through the curve o f intersection o f the sphere x i -ty i + zi =3ai
and any plane at a distance V from the origin has three mutually
perpendicular generators.
Sol. The equation of the given sphere is
**+>*+z*<=3a* ..( I )
Let the equation of a plane at a distance ‘e’ from the origin
be (normal form of the equation of a plane)
lx+ m y+ nz= a ...(2)
where I, m, n are actual direction cosines.
Now making (I) homogeneous with the help of (2), the equa­
tion of the cone with the vertex at the origin and the base curve
given by ( I) and (2), is given by
**+.y*+z*=3e* {(lx+ m y+ ηζ)/α}*
or x i + yt + zi -3 (lx + m y + n z )t = 0. ...(3)
374 Analytical Geometry 3-Z)

The cone (3) will have three mutually perpendicular genera·


tors if
the coefif. of Xa- H b e coeff. of y*+the coeff. of za= 0 .
i.e. (I —3/*)+( I —3m*) + ( I —3«*) = 0
or 3 (/a+ m a+ n a)= 3 or /a+m *+na—I
which is true. Hence proved.
Ex. 6. Three points P, Q, R are taken on the ellipsoid
XtIcP+ y*lb*+ ZtIct = I,
so that the lines joining P, Qt R to the origin are mutually perpendi­
cular. Prove that the plane PQR touches a fixed sphere.
Sol. The equation of the given ellipsoid is
xa/ea+JttIbt + ZtIct = I. ...(I)
Let the equation of the plane PQR be taken as
/x+ m y+ nz= I. ...(2)
Now making (I) homogeneous with the help of (2), the equa­
tion of the cone with the vertex at the origin and the base curve
given by (I) and (2), is given by
XtJat + y*/h*+ za/ca= (/x+ my+ nz)* ... (3)
If the cone (3) has three mutually perpendicular generators
then we have '
the coeff. of xa-f the coeff. of ya+ tbe coeff. of za= 0
or (I /a*—/*)+ ( I /ba—ma) + ( I/c*—na)= 0
or /* + ma+ /»*= I/a* + 1/b* + 1/ca= I/Rt (say) ...(4)
where R is coostant since a, b, c are given numbers.
Now consider a sphere whose equation is
*a+ y a+ z a=J?a. .(5)
If the plane (2) touches the sphere (5) then the length of the
perpendicular from the centre (0, 0, 0) of the sphere (5) to the
plane (2) will be equal to the radius of the sphere, i.e. we have

V i l ' + ^ + n * r ± R or /i+ m a+", “ 1/jR‘


which is true by virtue of the relation (4). Hence the plane (2)
touches a fixed sphere (5).
Ex. 7. I f x=y=*z/2 be one o f a set of three mutually perpendi­
cular generators of the cone 3yz—2zx—2xy—0, find the equations of
the other two generators. (Kanpur 1980)
Sol. The equation of the given cone is
3yz—2zx—2xy=*0. -(I)
The Come 375

In the equation of the cone (I), the sum of the coefficient! of


x*. y* and z* is zero, hence the cone (I) has an infinite set of
three mutually perpendicular generators.
Thus if jr/1 = //1 = ^ 2 is one of a set of three mutually
perpendicular generators then the other two mutually perpendi­
cular generators will be the lines of intersection of the given
cone (I) by the plane through the vertex (0,0, 0) and. perpendi­
cular to the given generator namely x/1 <=y/l«=z/2 l.e. by the
plane l .x + l.y + 2 .z = 0 . Let x//=y/m =z/n be a line of inter­
section, so that we have
Zm n-Znl-Ilm =Q ...(2) and /+n»+2n=0 ...(3)
Eliminating m between (2) and (3), we get
3 ( - / - 2 a ) n—Znl-Tl ( - / - 2»)=0
or Ilt - I n - M = Q or (l-2 n ) (2/+3n)=0
or l=2n, I= —\n.
When l=2n, from (3) we have m +4n=0 or «ι——4·.
Λ 2 l= -m = 4 n or //2 ~ m /(-4 )= « /l. ...(4)
When /= —f a, from (3) we have m + Jn=O or «=· —Jn-
I a lm = —Je or //3 = « ι/1 = ·/(—2). ...(5)
Therefore, from (4) and (5), the equations of the other two
generators are
x/2 - y li —4 )= z/l and x /3 = y /l = 2 /(-2 ) ...(6)
Clearly both these generators are perpendicular since
2 .3 + ( - 4 ) .( 1 ) + 1 .( - 2 ) = 0 .
Also each of these two generators is perpendicular to the
given generator x/1 =j?/l=z/2 since
2 .1+ ( - 4 ) .( 1 ) + 1.2 = 0 and 3.1 + 1 .1 + (-2 ).2 = 0 .
Hence the required two generators are given by the equations
(6) above.
Ex. 8. If x/1 =y/2=z/3 represents one of a set o f three
mutually perpendicular generators of the cone Syz—S z x - 3 x /= 0 ,
fin d the equations of the other two.
(Meerut 197«. 83 P; RaJasIhau 77; Kanpur 78)
Sol. The equation of the given cone is
Syz—S z x - 3xy=0. —(I)
In the equation of the cone (I), the sum of the coefficients of
x*, y* and z* is zero, hence the cone (I) has an infinite set of
three mutually perpendicular generators.
Thus if x /l= y /2 = z /3 is one of a set of three mutually
376 Analytical Geometry 3-D

perpendicular generators then the other two generators will be


the Iioesof intersection of the given cone (I) by the plane through
the vertex (0, 0, 0) and perpendicular to the given generator
namely X / 1=>»/2=2/3 i.e. by theplane l.x + 2 y + 3 r= 0 . Let
xjl= y Im= z/n be a line of intersection, eo that we have
5mn—$nl—3lm—0 —(2)
and /+ 2 m + 3 n = 0 . ...(3)
Eliminating / between (2) and (3), we get
Smn—in ( —2m—3n)—3 ( —2»i—3«) ffi=0
or 6m*+30mn+24#i, = 0 or m*+5mn+4n*=0
or (m+4n) (m + /t)= 0 or m = —4n,m = —n.
When ιη«=-4 β, from (3) we have /— 5/x=0 or /=5n.
Λ 41=—5m=20n or //5 = m /(-4 )= n /l. ...(4)
When m ——n, from (3) we have /+ « = 0 or /= —n.
:. I= m = —n or //1= m/1 = « /(—I). ...(5)
Therefore, from (4) and (S)t the equations of the other two
generators are
x/5= y/(—4)=2/1 and x /l= > » /l= 2/(—I). ...(6)
Clearly both these generators are perpendicular since
5 .1 + ( - 4 ) .1 + ( 1 ) .( - 1 ) = 0 .
Also each of these two generators is perpendicular to the
given generator x /1=^/2=2/3 since
5 .1+ ( - 4 ) . 2 + 1 .3 = 0 and 1.1 + 1 .2 + (-1 ).3 = 0
Hence the required two generators are given by the equations
(6) above.
Ex. 9. I f x= y /2 = 2 represents a set of three mutually per­
pendicular generators o f the cone U yz+ 6zx—I4xy=0, find the
equations o f the other two.
Sol. Proceeding exactly as in Ex. 7 or Ex. 8 above the
equations of the required generators are
x/(—11)=3^/2=3/7 and x /2 = ^/(—3)=2/4.
Ex. 10. Prorethatthe angle between the lines In which the
plane x + ) + r » 0 cuts the cone ayz+bzx+cxy=>0 will be Jw if
a + fi+ c = 0 .
Sol. The equation of the cone is
ey2 + b zx + cx y = 0, ...(I)
S iac e in tb e e q u a tio n o fth e c o n e (I) the sum of the coeffi­
cients of Xs, y* and 2* is zero, therefore the cone (I) has an infinite
set of three mutually perpendicular generators. If two perpendi­
cular generators are the sections of the cone (I) by the plane
The Cone 377

x + y + z= 0, then the third mutually perpendicular generator will


be the normal to the plane x + y f z —0 passing through the vertex
(O9 O9 0) of the cone i.e.%the line x /i~ y /l= z /l will be the third
generator. Hence the d.r.’s of the third generator x/is=y/le=z/i
will satisfy the equation (I) of the cone. Hence the required
condition is given by
a . 1 .1 + 6 .1 .1 + c .1 .1=0, or a + 6 + c = 0 .
Note. One way for solving Ex. 10 above has already been
given in Ex; 5 (a) in the Solved Examples set (E) on page 359.
§ 13. Right circular cone.
Definition. A right circular cone is a surface generated by a
line which moves in such a way that it passes through a fixed point
{called the vertex) and it . makes a constant angle Θ with a fixed
straight line through the vertex.
The constant angle Θ is called the semi-vertical angle of the
cone and the fixed straight line through the vertex is called the axis
of the cone.
(A) The section of a right circular cone by a plane perpendi­
cular to its axis is a circle.
Let A be the vertex, AO the axis and Θ the semi-vertical
angle of the right circular cone. Now draw any plane perpendi­
cular to the axis AO meeting it at the point O.
Consider any point P on the
section of the cone by this plane A
and join PA Now OP is a line
lying on the plane section of the
cone which is perpendicular to
the axis AO of the cone. Hei^ce
OP is perpendicular to the axis
AO and thus the triangle POA is
a right angled triangle with
L P A O ^e.
Λ OP=AO tan 0. But the P
semi-vertical angle 0 is constant
and the axis AO infixed and hence
the length OP is constant for all positions of the point P on the
section. Hence the locus of the point P (i e. the section of the
right ciicular cone by a plane perpendicular to its axis) is a circle
with the centre O and radius OP
378 Analytical Geometry 3-D

On account of this fact the cone is termed as right circular


cone. Thus the right circular cone can also be defined as :
A cone which has a plane circular section whose normal passes
through the vertex is a called a right circular cone.
(B) To find the equation o f a right circular cone.
(Kanpar 197¾ Punjab 81; Lndtnnw 79)
Let A (α, β9 γ) be the vertex of the cone and the equations of
the axis AO be
(x -<*)//= (y - /3 )/m = (z-y)/*,
where I9to, n are the d.r.*s of the axis. Let the co-ordinates of
any point R on the surface of the cone be (x, y9 z) so that the
d.r.’s of the line AR are x —a, y —β9 2—y. If B is the semi-
vertical angle of the cone i.e. the angle between the lines AO and
AR is S9 then it is given by
n I (x - * ) + m (y -fi)+ n ( z - y)________
V O i i- m* + n*)y/{(x - a)1+< J—f i f + ( z - y)1}
Squaring and cross-multiplying, the required equation of the
right circular cone is given by
{/ ( x - a ) + m (y—β)+η (z-y)}1
c=(/*+m* + nf) ((X-O)*-f (y_0)*+ (z-y)·} cos1 B. ...(I)
Particular cases. Case I. If the vertex is at origin. If the
vertex A be taken at (0, 0, 0) then putting a«=/3=y=0 in the
equation (I) above, the equation of the right circular cone with
the vertex at the origin and semi-vertical angle B is given by
(/x+m.y+wz)t c=(/, +i»ia+»i1) (x*+jP+z?) cos1 B
or (Ix + m y + n zf^ P A -n t+ n * ) (x1+ > i + z 1) (I - s in 1 B)
or (/2+*!1+ !!1) (x1+ > 1+ z 1) sin1 0 = (/1+m 1+ » 1) (X1-Ky1H-Ze)
—(/x+my+nz)1
or (i1+™*+»1) (X1-Ky1H-Ze) sin1 Q
=(*wz—i»y)e-K(**—lz)*+ {ly-m x)9 ..(2)
[By Lagrange’s Identity]
Case 11. I f t h e v e r t e x i s a t t h e O r i g i n 9 t h e a x i s o f the cone i s
t h e z-axis and the s e m i - v e r t i c a l angle is B.
(Kanpur 1980; Meerut 79, 82, 83, 86 P, 89 S)
Let OX9 OY9 OZ be the co-ordinate axes. The vertex of the
cone is at the origin O9 the axis of the cone is along the z-axis
OZ and the semi-vertical angle is 0. Let It (x, y 9 z) be any
current point on the surface of the cone so that OR is a generator
of the cone.
The Cene 379

The d.c.’s of OZ are 0, 0, I and the d.r.’s of OR are x—0,


y —0, z —0 i.e.9 x, y 9 z. The angle between the lines OR and OZ
is equal to the semi-vertical angle Θ of the cone. Therefore

_____ x .O + y .O + z.l_____ e ______ z


cos e V(02+ o a+ i 2)v/(x*+j>l + z 1)e v'(*1H-.>'e+ z e)
or z1 sec1 e*=*xt +y*+zi or ^ + / a z 1 (sec1 Θ—I)
or ^ + / - Z 1 tan1 Θ. ...(3)
which is the required equation of the cone.
Case III. I f the vertex is at the origin, axis the y-axls and
semi-vertical angle Is Θ. (Burdwan 1980)
The d.c/s of the v-axis are 0 ,1 ,0 . Hence putting /= /!« 0
and /η » I in the equation (2) above, the required equation of the
cone is given by
(x1+ ^ 1+^*) sin1 A=Z1+ * 2, or (za+ x 2) (1 —sin* 0 ) - y1 sin1 0
ot zt + x %—yi sin2 0. - (4)
Case IV. Ifth evertexisa tth eo rig in 9 axis the x-axis and
semi-vertical angle is 0. (Agra 1979, 81)
The d.c.’s of the x-axis are I, 0, 0. Hence putting w=?»®0
and 7 « I in the equation (2) above, the required equation of the
cone is given by
(JC1^-Jj1H-Z1) sin1 O ^ z '+ y 2, or (y2+ z2) ( l —sin1 0)=»xf sin1 0
or y 2+ z2= x 2 tan1 0. ---(5)
380 Analytical Geometry 3-D

Note. The stodeots should write complete proofs of cases III


and IV as given in the proof of case II.
SOLVED EXAMPLES (G)
Ex. I. Find the equation to the right circular cone whose vertex
is {2, —3. 5), axis makes equal angles with the co-ordinate axes and
semi-vertical angle is 30°. (Agra 1977)
Sol. The co-ordinates of the vertex A of the cone are given
as (2, —3, 5;. If I, m, n are the d c.’s of the axis of the cone then
since the axis makes equal angles with the co-ordinate axes, we
have
I m _ n _ Ii + « * + « * )_ I
T T “ I '=V(I»+*1‘+ 1,)- V3'
Consider ageneral point R[x, y, z) on the cone so that the
d.r.’s of the generator AR a r e x —2, y + 3 , z—5.
Now the semi-vertical angle of the cone is 30° i.e. the angle
between the axis of the cone and the generator AR is 30°, there­
fore, the required equation of the right circular cone is given by
, ΛΟ < l/V 3 ).U -2 > -W lV 3 ) fy-H3>-H(l/V3)(z-5)
3 0 --------------V t l - < - ^ + 0 + 3 ) - + l * - V )
V 3 ________x + y + z —4_________
or 2 “ v 3 V t( * - 2 ) * + 0 '+ 3 ) * - H r - 5 |!t}'
Squaring and cross-multiplying, we get
9 (x*—4x+4-J-y*+6y+9+z*—10z+25}=<4 (Xi -Hy2-Hz2
+ 16-H2xy-H2xz+2yz—8x—8y—$z)
or 5 (X2-Hy1-I-Zi) - S (yz+ zx+xy)—4x+86y—58z+278=0.
Ex. 2. A right circular cone is passing through the point
(1,1, I) and its vertex is the point (1,0, I). The axis of the cone Ijs
equally inclined to the co-ordinate axes. Find the equation o f the
cone.
Sol. ,The co-ordinates of the vertex A of the cone are
(I. 0, I).
Also the cone is passing through the point (i, I, I) and hence
the d.r.’s of the generator passing through (I, 0, I) and (I, I, I)
are 1 - 1 , 1 - 0 , 1 -1 i.c.,0, I, 0.
The axis of the cone ie equally inclined to the co-ordinate
axes and so its d.c.’s are 1/V3, 1/V3, I/V3. [See Ex. I above].
Thus if Θ be the semi-vertical angle of the cone, then
cos 0*
O.Q/V-i)+ I -(1 ν /3 )+ 0 .(1 /ν 3 )_ I
VIO2+ Ii -HO2I v3‘ .(1)
The Cone 381

Now let R (x, y, z) be a general point on the cone and so the


d.r.’s of the generator AR are x—I, y —0, r —I. The angle bet­
ween the axis of the cone and the generator AR is also Θ and so
we have
cos e i* -I M l/\/3 )+ y .(l/V 3 )+ (z -l).( l/\/3 )
—I)*+O')*+ U—I )*}
or J __________ x + y + z - 2
using (I).
V i V i V(x* +y*+z*—2x—2z+ i f
Squaring and cross-multiplying, we get
(x* + y * + 2*—2x—2z+ 2) ^ (x + y 4- z—2)*
or yz-\-zx+ xy—x —2y—z+l**0.
This is the required equation of the right circular cone.
Ex. 3 (a). The axis o f a right circular cone, with vertex ar origin
0 , makes equal angles with the co-ordinate axes and the cone passes
through the line drawn from O with direction cosines proportional to
1, —2, 2. Find the equation. (Aradh 1978)
Sol. The vertex of the cone is the origin O (0, 0,0). The
axis of the cone is equally inclined to the co-ordinate axis and so
its d.c.’s are 1/\/3, 11V i , I I V i [See Ex. I above]. Since the cone
is passing through a line with d.r.’s I, —2, 2 and so this line is a
generator of the cone. Let Θ be the semi-vertical angle of the cone
and so we have
_ a_ ( l/V 3 ) .l+ ( l/V 3 ) .( - 2 ) - l- ( l/V 3 ) .2 _ I
V {l, +(-2)*+(2)·} iV 3 -(I)
Now consider a general point R (x, y, z) on the cone and,
therefore, the d.r.’s of the generator OR are x —0, y —0, z —0 l.e.
x, y, z. Hence Θ is also the angle between the axis of the cone
and the generator OR. Thus
( I / V 3 ) · x + ( l / V 3 ) .y + ί I / V 3 ) . z
cos 0= y-(JC*+ j,2 + zi)
I x+ y+ z
or
3v/3°
or ix’ + y ' + z 'i—9 (x+y+z)*
o: 4JC1+ 4y%+ 4z2+ 9y z+ 9zx+ 9xy 0.
This is the required equation of the cone.
Ex. 3 (b). Showthatthe equation of the right circular cone
which passes through the line 2x*=*3y*=* —5z and has x ^ y ^ z as its
axis also passes through the axes oj co-ordinates.
382 Analytical Geometry 3-D

Sol. The equations of a generator of the cone are


2 x= 3 y= —5z or x/l5=>>/10=»z/(—6) . ( 1)
The axis of the cone is
x /l= y /l= z /l. ...(2)
If Θ be the semi-vertical angle of the cone, then
15.1 + 10.1 + ( - 6)! I 19 =
cos 9=
V { 1 5 * + 101+ ( - 0)»}V( 1’ + 1*+T*)—V(361)V3 y / y
...(3)
Consider a general point R(x, y, z) on the cone. The vertex
of the cone is the origin O, which is the point of intersection of
the lines (I) and (2). Therefore, the d.r.’s of the generator OR are
x —0, y —0, z —0 i.e. x, y, z. We have
x .l+ y .l+ z .l
cos 0=
V (^ + ^ + z * )V (i* + i* + i* )
I χ + -y+
or --------- ■ z ,,.
using (3)
λ/3 V W W + y '+ z ·) '
i>r x %+ y%+ zz= (x+ y+ z)*
or yz+ zx+ xy= 0 .
' This is the required equation of the cone which clearly passes
through the co-ordinate axes-because I, 0, 0; 0, I, 0; and 0, 0, I
all satisfy the equation of the cone.
Ex. 4. Find the equation of the right circular cone whose vertex
is the origin and whose axis is the line x —t, y=>2t, z= 3t and which
has a vertical angle of 60°. (Punjab 1982 (S))
Sol. The vertex of the cone is 0(0, 0, 0) and the equations
of its axis are
Xe=It y= 2t, z= 3 f i.e. x/I =>>/2—z/3=i. ...(I)
the d.r.’s of the axis of the cone are 1,.2, 3.
Consider a general point R f kX, y, z ) on the cone. The d.r.’s of
the generator OR are x —0, >»—0, z—0 I.e. x ,y , z. The vertical
angle of the cone is 60° arid so its semi-vertical angle 30° is given
by
cos 30°=·______I .x + 2 .y + ,3 .z _____
V ( l * + 2 1 + 3»)Vl**+^+**)
or x+2.y+3z
2 VCMlVi*·+*·+*»)
or 21 (x * + /+ z * )= 2 (*+2y+3z)*
or 19x*+ 13>2+ 3z*—2Ayz— 12zx —8jty«= 0.
This is the required equation of the right circular cone.
T h e Com e 383

Ex. 5. Find the equation of the right circular cone whose axis
is x = y = z , vertex is the origin and whose semi-vertical angle Is 45°.
(Beriwin 1974)
SM. Proceeding as in Ex. 4 above the required equation of
the cone is given by
x?+y*+z*—4 (yz+zx+xy)*=*0.
Ex. 6. Find the equation of the right circular cone with vertex
ut (I, —2, —I), semi-vertical angle 60® and the axis
(*=* I)/3** —O'+2)/4 =*(z+1)/5. (Gorakhpur 1980)
SoL The vertex of the cone is A(I9 —2, —I). The equations
of the axis of the cone are (jc—1)/3=0^+2)/(—4)=»(z+1)/5.
The d.r.*s of the axis of the cone are 3* —4, 5. The semi­
vertical angle of the cone is 60°.
Consider a general point R{x9y 9 z) on the cone and so the
d.r.’s of the generator AR are x - 1 , j + 2 , z+ 1 . Hence the requi­
red equation of the right cone is given by
cos 60 3 . ( x - 1 ) + ( - 4).(y+ 2 )+ 5A z+ 1)
Λ/{(3>*-h ( —4>*+ ( 5)*}
. 3x—4 j+ 5 z —6
or
V W W ( x * + y * + z %-2 x + 4 y + 2 z + b j
Squaring and cross-multiplying, we get
25 (x*+.y, + z f -2 x + 4 y + 2 z + 6 )= 2 (3x 4y+5z-6)*
or 7x* -T y t - 25z*+ 80yz—60zx+ 48xy+ 22x+ 4y-|-170z+78** 0.
Ex. 7. Find the equation of the cone formed by rotating the line
2x+3y=6, Z=O about the y-axts.
Sol. A generator of the cone is given by
2x+3y«=»6. z= 0 or 2 (x—3 ) = —3.y, z = 0
or (x —3r/3=j^/(—2) =z/0. ...(I)
The axis of the cone is. y-axis whose d.c.’s are 0 ,1 ,0 . The
vertex A of the cone is the point of intersection of (I) with the
y-axis (i.e. x = 0 , z =0) and is given by (0, 2, 0).
If 0 be the semi-vertical angle of the cone then it being the
angle between the generator (I) and the y-axis (t.e, the axis of the
cone) is given by
_________ 3 .0 + (-2 ).1 + 0 .0 _________ j —2
V{(3)t + ( - 2 ) t +(O)t}V{Ot + l t +0*) “ V(13V ...(2)
Consider a general point ϋ (χ 9y 9 z) on the cone and so the
d.r.’s of the generator AR are x —0, y —2, z —0 i.e. x, ^ - 2 , z.
Also the angle between the generator AR and the axis of the cone
(i.e. y-axis) is 0. Hence we have
384 A n a lytica l G eom etry 3 -D

χ.Ο+Ο»—2).1 + z 0
cos
V{*’ + iy 2)»+2*}V(0*+ i *+ o»>
_2 y _2
or V(T3)“ V ( ^ + ^ + z * - ^ + 4 ) * ue,Dg (2)
Squaring and cross-multiplying, we have
4 (jc*+ j * + z*-4 ^ + 4 )= 13 (>·-4^+4)
or 4 x e— 9 y * + 4 z f + 3 6 y — 3 6 = 0 .
This is the equation of the required cone.
Ex. 9. Find the equation o f the cone generated by rotating the
line x ll^ y /m ^ z /n about the line xla=*ylb~z/c as axis.
Sol. The equations of the axis of the cone are
xfa=ylb=zlc. ...(I)
The equations of a generator of the cone are
xll—y lm ^ z/n . ...(2)
Let Θ be the semi-vertical angle of the cone then it is the
angle between the lines (I) and (2; and so we have
al+bm +cn
cos 0=
..(3)
Consider a general point R (x9y, z) on the cone. Nowthe
vertex A of the cone being the point of intersection of (I) and (2)
is given by (0, O9 0). Thus the d.r.’s of the generator AR are
x —O9y —0, z - 0 i.e. x, y %z. Also the angle between the axis (I)
and the generator AR is Θand hence we have
c o s 9 e _______ ax+ by+ cz_______
y/{a*+6*+ £*)-*/(** +y*+Z)* (4)
Equating the two values of cos 0 given by (3) and (4), the
required equation of the cone is given by
ax+ b y+cz al+bm+cn
Vixt+ jP + ViP+nP+n*)
or (ax+by+cz)* (l9+mt + rf)= (xi + y%+ z*) (al+bm+cn)*.
Ex. 9 (a). Ifa right circular cone has three mutually perpendi­
cular generatorst then show that the semi-vertical angle is tan"1
tPonjab 1982 S; Madras 76)
Sol. Let us consider the equation of the right circular cone
with the vertex at the origin, axis the z-axis and the semi-vertical
angle 0. Its equation is given by ISee § 13 (B) case II]
x*+y*+z* tan* 0 = 0; ...(I)
Ifthe cone (I) has three mutually perpendicular generators,
The Cone 385

then the sum of the coefficients of x%,y 2 and z* in the equation (I)
must be zero.
.·. 1+ 1+ ( (an2 0)--=0; or tan2 0=2, or ta n 0 = -* A
or 0 = tan "V 2 ·
Ex. 9 (b). I f a right circular cone has three mutually perpendi­
cular tangent planes, then show that the semi-vertical angle of the
cone is given by cot_I \/2.
Sol. Let us consider the equation of the right circular cone
with the vertex at the origin, axis the z-axis and the semi-vertical
angle 0. Its equation is given by [See § 13 (B) case 11]
**+y2- z 2 tan* 0=0. ...(I)
The cone (I) will have three mutually perpendicular tangent
planes if
,bc+ca+ab=f*+g*+hv [See § 12]
or. I . (-ta n * 0 ) + ( - tan* 0).1 + 1 .1 = 0 + 0 + 0
or 2 tan2 0=1 or cot* 0 = 2 or 0=θοΙ-1·\/2.
Ex. 10. Lines are drawn from O with direction cosines propor­
tional to ( 1,2, 2); (2, 3, 6); (3, 4, 12). Show that the axis o f the
right circular cone through them has direction cosines —I/V 3. I/V^*
l/\/3 and that the semi-vertical angle of the cone is cos'1 (l/y/3 ).
(Meerat 1973)
Sol. Clearly the vertex of the cone is at the origin 0(0, 0, 0).
Let the direction cosines of the axis of the right circular cone
be I, m, n. Let 0 be the semi-vertical angle.'
The cone is passing through the given lines drawn from O
and so these given lines are generators of the cone. Hence each
of these given lines with d.r.’s (I, 2, 2); (2, 3, 6); (3, 4, 12) i.e.
d.c.’s (1/3, 2/3, 2/3); (2/7, 3/7, 6/7); (3/13, 4/13, 12/13) is inclined
at an angle 0 to the axis of the cone whose d.c.’s are /, m, n.
We have
cos 0=(1/3) /+(2/3) »!+(2/3) n ...(I)
cos 0=(2/7) /+(3/7) m + (6/7) n ...(2)
and cos 0= (3 /1 3 )/+(4/13) m+(l2/13) n. ...(3)
Subtracting (2) from (I), we get
0= (1/21)/+ (5/21)01-(4/21)0 or /+ 5 0 i-4 « = O . ...(4)
Again subtracting (3) from (I), we get
4/+1401-100= 0. -(5 )
Solving (4) and (5), we get
386 Analytical Geometry 3-D

I m n vW*+»»*+»*) I
—1 1 = I _ V{(-D*+i*+'*} " V 3 ‘
Λ The d.c.*s of the axis of the cone are
- I I V h I I V h 1/V3.
Patting the values of I, m, n in (I), we get

CM " - ^ 7 3 + 3 ^ 3 + 3 ^ 3 = 3 ¾ = ^ or ψ ί
§14. The Enveloping Cone.
Definition. The enveloping cone o f a given surface is the locus
o f the tangent lines drawn from a given point to the given surface.
The enveloping cone Is also called the tangent cone to the surface
with the given point at Its vertex.
(A) The equation of the enveloping cone. To find the equa­
tion o f the enveloping cone of the surface (conlcold) ax*+by*+ cz*=*I
with the vertex at the point (xu y u Z1)'. (Kanpur 1976, 78)
The equation of the given surface is
ex*+6y*+cz*=l. --(I)
The equation of any line through (X^y1, zt) are
( x - x 1)//= (y -y ,)/m = -(z -z ,)//i= r (say). . (2)
The co-ordinates of any point on the line (2) are
{lr+xu m r+ yu nr+z,). . (3)
Let the line (2) meet the given surface (I) at the point given
by (3). Then the co-ordinates of the point given by (3) will
satisfy the equation (I) and so we have
a. (Ir+ X1)*+ b (mr+ y, )*+ c (nr+ Z1)* = I
or (al*+bm*+cn*) r*+2 {alxi+bmyx+cnzx) r
+(ox,*+by,* + C z 1* - 1 ) = 0 . ...(4)
If the line (2) is a tangent to the given surface (I), then it
will meet the surface ( I) at two coincident point and hence the
two values of r obtained from (4) should be equal. The condition
for which is iBt =-AAC'
or 4 (alxx+bmyx.+ cnzx)*=4 (al*+bm*+cn*) (Ox1*+by,*+Cz1* - 1)
• or (alxx+bmyi+cnzx)*—(al*+bm*+cn*)(axx*+byx*+czx*—l) ...(5)
The enveloping cone of the surface (I) i.e., the locus of the
tangent line (2) is obtained by eliminating I, m, n between the
equations (2) and (5) and is therefore, given by
{a ( X - X 1) X1 + 6 ( y - y ,) yt + c (Z -Z 1) Z1)*
= { a ( X - X 1)1 + 6 ( y - y j ’ + c (Z -Z 1)*) (Ox1*+ ^y1*+ Cz1*—I )
The Cone 387
or {(axxx+ byyx+ Czz1- 1 ) - (axx*+byt*+ czx*- 1)}*
**{(ax*+by*+cz* —1 ) - 2 (fljcxj + f t ^ + c z z i — I)
+ (α*χ*+ byi*+czx*—I )} X (axx*+byi*+czi*—l). .. .(6)
For convenience let us set
Saax*+by* + cz* —I, S1=OX1H h y 1H i z 1* - Il
and T = axxx+byyx+czzx— I. J -·;(7)
Using the above notations, the equation (6) of the enveloping
cone becomes
( T - S i)* = (S -2 T + S 1).S 1
or T * -2 T S x+ S x*=SSx- 2 T S 1+ S l*
or SS1=T*
where S, Si and T are given by equations (7).
SOLVED EXAMPLES (H)
Ex. I. Find the equation o f the enveloping cone o f the sphere
\ x H y H z 2= a 2 with the vertex at the point ( x , , J11, Zi).
(Punjab 1975; Allahabad 75)
Sol. Proceeding as in § 14 (A), the enveloping cone is given
by
(X2+y* + z*—a*) (X iH y ,2+ Z1* - a*)=(xxx+ yyx+ zz,- a*)*.
Ex. 2. Find the equation o f the enveloping cone o f the ellipsoid
x*/a*+y*lb*+z*lc*=‘ I with the vertex at the point (xt, y x, zt).
Sol. Proceeding as in § 14 (A), the equation of the enve­
loping cone is given by

Ex. 3 (a). Find the enveloping cone o f the sphere


x H y M 2*—2y + 6z + 2 = 0 with its vertex at (I, I, I).
Sol. We have S=x*+y*+z*—2y + 6 z+ 2 = 0 .
The vertex of the enveloping cone is (I, I, I).
.·. S1= I*+12+ 12—2.1 + 6 .1 + 2 = 9 ,
T = x . l + y . l + z . l - ( y + l ) + 3 ( z + l) + 2 = x + 4 z + 4 .
The required equation of the enveloping cone is given by
-SS1= T 2*
or (x*+y*+z2—2 y + 6 z+ 2 ).9 = (x + 4 z+ 4 )2
or 9x2+ 9 y H 9 z 2—18y+54z+ I8 = x 2+16z2+ 16 + 8 zx + 8 x + 3 2 z
or 8xH 9y2- 7 z 2- 8 z x - 8 x - 1 8 y + 2 2 z + 2 = 0 .
Ex. 3 (b). Find the enveloping cone o f the sphere x*+y*+z*
—2x+ 4z= I with its vertex at (I, I, I).
Sol. Proceeding as in Ex. 3 (a) above, the required equa­
tion of the enveloping cone is given by
388 AnalytlcalGeomeiry 3-D

4x2+3y2—Sz2—6yz— 8x+ 16z—4=0.


Ex. 4. Show that the lines drawn from the origin so as to touch
the sphere x*+y*+z*+2ux ±2vy+2wz+ d= 0 lie on the cone
d {x*+y*+z*)-=(ux+vy+wz)i. (Kanpur 1976)
Sol. Here it is required to find the enveloping cone of the
given sphere with the vertex at the origin.
The equation of the given sphere is
S= x*+ yt +z*+2ux+2vy+2wz+d=0. -(1 )
We have (?ι*=04·0·|·0Ί·0“|·0“1“0-|·ίΙ3 ΐ(ι
T = x .0 + y .0 + z.0 + u (x+0)-|-v O'+OJ+v»' (z+0) + d
= ux+ vy+ w z+ d.
Therefore the equation of the enveloping cone of the sphere
(I) with the vertex at the origin is given by
tSSl= Tv
or (x*+y*+z*+2ux+2vy+2wz+d).d={ux+vy+wz+d)*
or (x*+y*+z*) d+2 (ux+vy+wz) d+d*
=(ux+vy+wz)*+2 (ux+vy+wz).d+d2
or d (x*+y*+Zi) = (ux+vy+wz)*.
Ex. 5. Prove that the plane z=Q cuts the enveloping cone of
the sphere Xi +y*+Zi = 11 which has its vertex at (2 ,4 ,1 ) in a
rectangular hyperbola.
Sol. Here
S==x*+y2+ z 2- 11 = 0 . 5 ,= (2 )2+ (4)2+ ( l) 2- l 1= 10,
T = x .2 + y .4 + z.l —1I= 2 x + 4 y + z — 11.
Therefore by the formula SSi =T*, the equation of the
enveloping cone of the sphere 5 = 0 with the vertex at (2, 4, I) is
given by
(x’+ ^ + z 2- 11). 1 0 = (2 x + 4 y + z - 11 )2. ...(I)
The plane z= 0 cuts the envelopingcone (I) in the conic given
by (x * + y * - ll) .1 0 = ( 2 x + 4 y - ll) 2, z= 0
or 6x2—6y2—16xy+44x—88y-231 = 0, z=0. ...(2)
Now in the first of the equations (2),
the coefficient of x*+the coefficient of y2= 6 + ( —6)=0.
Hence the conic (2) represents a rectangular hyperbola.
Ex. 6. Find the locus of a luminous point which moves so that
the sphere x*+y2+ z 2—2az=0 casts a parabolic shadow on the
plane z = 0 .
Sol. Let the luminous point be P (xi, y t, *i).
The equation of the given sphere is
The Cone 389

S = X i + y2+ z2—2az= 0.
Λ Sx= χ χ*+ γλ*+Zxi - I a z i and T=xxx+yyx+zzx—a {ζ+ ζχΧ
Tbe equation of the enveloping cone of the sphere S= 0 with
the vertex at P (x lt ylt zt) is given by SS1= T i
l e. (χ2+ γ*+ ζ%~2αζ) (Xxi +yxi +zxi—2azi)
= {xxx+ y y i+ z z i-a (z+ zt)}2 ...(I)
The plane z = 0 cuts the cone (I) in the conic given by
(*'+y*) (Xxi +yx2+z12-2azi)=(xxx+yyi-a z i)i, z= 0
or Xi {yxi + z x i — 2 a z x ) — 2 x x y l x y + y 2 ( χ ^ + Ζ ι 2— 2 α ζ χ)
+ 2 αΧ χ Ζ χ Χ + ^ a y j Z i y - O i Zx2= O t Z = 0. ...(2)
Now we know that the conic
ax2+2hxy+by2+ 2gx+ 2fy+ c= 0, z= 0
represents a parabola if h2=ab.
Applying this condition the conic (2) will represent a para­
bola if (—x Jy i)t =(yxi +ZLi -2 a z 1) {Χχ2+Ζχ*—2αζχ)
or Xtiyti r=xJiyxi +yxiZxi -2aziyxi +ZtiXii +Zi*—2az19- 2 a z iXx2
—2αΖχ*+4α2Ζχ*
or Z12 (Xx2+ y t i + z i i — 4 a z i + 4 a 2) — 2 a zi (Xi*+ ) ^ ) = 0
or Z1 (Xxi + y x i + z Ji - 4 a z i + 4 a 2) - 2 a (Xii - ^ i i)=O.
The locus of the point P (X1, y Xt Zi) is
z (x2+ y2+ z2—4az+4o2) —2a ( X 1 - J - ^ 2) = O .
Ex. 7. The section o f the enveloping cone o f the ellipsoid
x i/a2+ y2!bt = zilci = I whose vertex is P (x Jt y u Z i ) by the plane
Z=Ois(I) a parabola, (it) a rectangular hyperbola, (Ui) a circle.
Find the locus of P in the above three cases. (Meerut 1984R)
Sol. The equation of the given ellipsoid is
S s x iIa2+Ji2Ib2+ZiIci - 1 = 0 .
Therefore S i= Xi*/a2+yx2/b2+ Zx2Ic2- 1,
and T=xxxla2+yyx/b2+zzxlc2—l.
The equation of the enveloping cone with the vertex at
P (X uyuZ i) ^ SS1= T 2
or l ^ + y l + t .. i \ /½!+*%?!? _ i \ = i H i + y j i + 2Si- I Y
\ a 2^ b 2 Ci M o* b2 T c * J \ a2 V ^ ci }
...(I)
The section of the enveloping cone (I) by the plane z= 0 is

or
390 Analytical Geometry 3-D

_ . Case (i). If the conic (2) represents a parabola, then apply­


ing the condition 'h*=ab\ we have
X| / V i V _ , \ L / £ i !4-£L*_ i \
a*b* a*\b* c* ) ’ b* \a* T c8 /
-Xr1V i2_ACtV1* i yi*zi*_ y ii Zij8-X18 . ,
or
a*b* a*b* b*c* b% c2n2 + c * c* a* Ct ^

ΟΓ
...(3)
But XitIat +yS/b*+ZltIc2— I O since thp point P (xu Vi. Z1)
does not lie on the given ellipsoid.
Hence (3) gives Zi2Ict - 1=0 or z4*=»c* or zj = ± c.
.·. The locus of the point P (Xi, y u Zi) is z==±c.
Case (ii). If the conic (2) represents a rectangular hyperbola,
then we should have
th e c o e ffic ie n t O f x 1- H b e c o e ffic ie n t o f V2= O

or Ι / Λ ' + Ξ Λ Λ + 1 /*!+ £»?_


Ot Xbt c2 ) b* \o* T C*
or Xi*+Vi* , Zia
a*bA c2 \a 2 b2J a2
or c2 (Xia+Via) + ( e a+ b a) Zia=U2
Λ The locus of the point P (xlt Vu z0 is
c2 (xt + yt ) + (at + bt) z2= a 2-f-6a·
Case (Hi). If the conic (2) represents a circle then we should
have the coefficient of x2= th e coefficient of v*. and the coefficient
of xv=0.

··· (3)
and Xi1Vi=O i.e. X1=O or Vi=O.
If Xj=O then from (3), we have

or Z i l + 5l / L - U = L - L
a2b2 c* \a a b2J a2 b2
or c*yi*+(b*—a2) Z12^ i b 2-Q 2) ca.
in this case the locus of P (xle y u zx) is
c2y 2± (b 2—a2) 2*«=t2 (b2—a2),
The Cone 391

If 7 i <=>0 then from (3), we have

or
ΜΜ-ρβ,+*-')
c2Xi*+(a2—b*) 2,2= c2 (a* - b2).
in this case the locus of P (x,, y„ z t) is
c*x®+(a* -ft2) Zt - C t (a*—b*)t y = 0.
Ex. 8. Find the locus o f a luminous point which moves so that
the ellipsoidXtJa2-^y2Ibt +ZtIct =] casts a circular shadow on the
plane 2 = 0.
Sol. Let P (x,, yu 2,) be the luminous point. It is required
to find the locus of P when the section of the enveloping cone
of the given ellipsoid with the vertex at P (x,, y it r,) by the
plane z —0 is a circle. Now proceed as in the case (iii) of Ex. 7
above.
Ex. 9. Show that three mutually perpendicular tangent lines
can be drawn to the sphere x t +y*+z2= ri from any point on the
sphere χ 2+>»2+ 2*=(3/2) r2.
Sol The equations of the given spheres are
x 2+ y2+ z2= r2 ...(I) and x t + y2+ zt =(3l2) r*. ...(2)
Let P (x,, y,, z,) be any point on the sphere (2), then
*ι, + Λ , +*ι*-(3/2) r·. .. (3)
Now three mutually perpendicular tangent lines can be drawn
to the sphere (I) from the point P if the enveloping cone of the
sphere (I) with the vertex at P has three mutually perpendicular
generators.
For the sphere (I), we have
S = x t + yt + zt - r t, Sl= x 1i + yl2+ zl3- r 2
and T = X X 1 + yyt + 22, —r*.
The enveloping cone of the sphere (I) with the vertex at
P ( x i , y u Zi) is tSS i = T 2'
or ( x 2 + y 2 + z 2 - r 2) (Xi* +.V,2+ z *—r2) =(xx, + y y x + 22, —r *)*.

It will have three mutually perpendicular generators if


t h e c o e f f . o f X 2 - H h e c o e f f . of > * + t h e c o e f f . o f 2 * = 0
or ( x , 2+ ^ i 2+ Z i * - r 2— X i 2) + ( x , a+ . y i 2+ z , 2 — r 2—^ 12)
+ (*18+ f l 2+ 2l* — r* —2,2= U
or 2 (Χι 2+ ^ ,2+Ζ ι 2) = 3 γ2 or 2 (3/2) r2= r2t usiog (3)
or r2= r2, w h i c h is t r u e . H ence Proved.
392 Analytical Geometry 3-D

Ex. 10. Find the locus o f points from which three mutually
perpendicular tangent lines can be drawn to the paraboloid
ax*+by*=2cz.
Sol.' Let P (xi. y u Zi) be any point whose locus is required.
Here S=ax*+by*—2cz, S ^ a x S + b y S —2czlt
and T = axxi+ byyt —c ( z + z t).
A The enveloping cone of the given paraboloid with the
vertex at P (Xu yu Zi) is tSSi = Tv ,
i.e. (ax*+by*—2cz) (axS+ byt*—2czt)
= {axxt +byyt - c (z+ z,)}*. ...(I)
Now three mutually perpendicular tangent lines can be drawn
from P (X|, P1, Z1) to S=O if the enveloping cone (I) has three
mutually perpendicular generators, the condition for which is that
in the equation (I) the sum of the coefficients of x2, p2 and z2
should be equal to zero
i.e. a (byS —2czt) + b (axS—2czt) —c2=0
o r. ob (Xi2+Pi*) —2c (α+h) Zi =C*.
:. The locus of P (Xi, yit Zi) is
. ab (X2-Hp*)-2c (a+b) Z=c*.
Ex. II. Find the locus o f the points from which three mutually
perpendicular tangent lines can be drawn to the conicoid
ax*+by*+CZ2=»i.
Sal. Proceeding as in Ex. 10 above the equation of the
required locus is
a (h-Hc) x*+h (c+a) y*+c (β+h) z*=a+b+c.

Exercises
1. Find the equation to the cone whose vertex is the origin
and whose base curve is the circle z=3, X2-Hp2-=9.
Ans. X2-Hp2- Z 2=O.
2. Find the equation of the cone with the vertex at the
origin and which passes through the curve
Ux2-Hhp2-Hcz2= I , ax2+/3p2=2z. (Kanpur 1981)
Ans. 4z2 (ax*+by*+cz*)=(c^x*+βy*)*.
3. Find the equation ot the cone with vertex at the origin
and direction cosines of its generators satisfying the relation
3/2—4m2-Hin2=O.
Ans. 3x2—4p2-Hiz2=O.
The Cone 393

4. Prove that tbe equation of a cone whose- vertex is the


point (α, β, γ) and base curve as z*=46y, x«=0 is
(yx—az)*'=4b (a—x) (ay—βχ),
5. Prove that the equation
X 2—2y *+ 3 z2— 4 x y 4 r Syz —6 z x + 8x—19 y —2z— 2 0 = 0
represents a cone. Show that the coordinates of its vertex are
( 1 , - 2 , 3). (Lucknow 1982; Meerut 86 S)
6. Show that the vertex of the cone
7X2+ 2y 2+ 2z*—I Ozx + 1Oxy+ 26*—2 ^ + 2z —17 =>0
is (I, - 2 , 2).
Central Conicoids

§1. Introduction Intbe chapter on lThePlane' we have


shown that the general equation of first degree in X9 y and z
represents a plane surface. In a similar way the general equation
of second degree in x \ y and z namely
ax2+ by2+ CZ2-f- 2fy z + 2gzx+ 2hxy+ lu x + Ivy+ 2wz+ d= O ... (I)
always represents a conicoid or a quadratic surface.
The equation (I) by a proper transformation of axes can be
reduced to the following standard form
ax2+fry2+CZa= I. ...(2)
The standard form (2) is calked the central conicoid.
Now we shall discuss below some important properties of the
central conicoid given by (2).
(a) The origin is the centre of the central conicoid
0X2+ 6y2+ cz2= l .
Let P(xi, yu Z\) be any point on the central conicoid (2), so
that we have
a x f+ b y t2+Czi2=,1
which can also be written as
a ( - X i )2+ b ( - ^ 1)2+ c ( - Z1)2- 1. .. .(3)
The relation (3) shows that the point £?(—x u —yu -Z 1) will
also lie on (2) if P(x,, ^ l j Z1) lies on (2). The middle point of the
chord PQ is (0, 0, 0). This shows that allchords of the surface
(2) which pass through the origin are bisected at the origin.
Therefore the surface (2) has a centre and it is at the origin. Thus
origin is the centre of the central conicoid (2) and due to this
property the surface (2) is called a central conicoid.
(b) To prove that the co-ordinate planes bisect all chords per­
pendicular to them.
Let P(xu }u 2i) be a point on the central conicoid (2)9 then
clearly the point Q(— yu z\) also lies on it. The ,middle point
of the chord PQ is (Of yu Z1) which clearly lies on the YOZ plane
Centrral Conlcolds 395

Ue. the plane The d.r.’s of the chord PQ are 2xu 0, 0 Ue.
I, 0, 0 showing that the chord PQ is perpendicular to the plane
YOZ. Hence we conclude that the co-ordinate plane YOZ bisects
all· chords perpendicular to it. Similarly we can prove that the
co-ordinate planes ZO X and JfOFalso bisect all chords perpendi­
cular to them respectively.
Hence the central conicoid (2) is symmetrical with respect to
all the three co-ordinate planes and these planes are called the
principal planes of the central conicoid.
The three principal planes (f.e. the co-ordinate planes in this
case) taken in pairs intersect in three lines (ie. the co-ordinate
axes in this case) which are called the principal axes of the central
conicoid.
(c) The standard form (2) of the central conicoid can take
the following'three forms each of which represents a surface with
a definite name assigned to i t :
x* y* 2* ,
(i) 3 Ϊ+ 5 5 + 3 Ϊ"= 1 (Ellipsoid)
* ' y2 2* ,
(ii) flV (Hyperboloid of one sheet)
X* y % Zi
(iil) (Hyperboloid of two sheets)
a1 ~ b*~ c- “ -
Now in the following articles we shall discuss the above three
surfaces (i.e. central conicoids) in details.
§ 2. The I7Ilipsoid.
The standard equation ot the ellipsoid is given by
X2Ia9+ y 2/b2+ z%jc2--1. .. (I)
(i) The Origin is the centre of the ellipsoid given by (I). Pro­
ceeding exactly as in § I (a) we see that all the chords passing
through the origin are bisected at the origin. Therefore the surface
(I) has the origin as it centre.
(ii) The co-ordinate planes bisect all chords perpendicular to
them. Proceeding exactly as in § 1(b), we see that the co-ordinate
planes bisect all chords perpendicular to them. Hence the ellipsoid
(I) is symmetrical " ith respect to all the three cc-ordinate planes
and these planes arc called the principal planes of the ellipsoid.
T hethreeprincipalplanestakenin pairs intersect in three
lines (i.e. the co-ordinate axes in this case) which are called the
principal axes of the ellipsoid.
396 Analytical Geometry 3-Z>

(iii) The ellipsoid is a closed surface. The equation (I) may


be rewritten as
y %Ibi ^ z i Ici = I - X i Ia t . ...(2)
If x is numerically greater than a, then from the equation (2)
we find that y i/bi + zilci =a - ve quantity i e. the sum of two
perfect squares is a negative quantity which shows that the least
one of y and z should be imaginary. Hence it follows that x can­
not be numerically greater than a and therefore, the surface exists
between the two parallel planes x= a and x = —a. In a similar
manner it follows that the surface (I) lies between the parallel
planes y = b t y= —b aud z=c, z= —c.
Thus the ellipsoid (I) is a closed surface.
(iv) The Intercepts on the co-ordinate axes,. The equations of
the x-axis are y = 0 , z- 0. It meets the surface (I) in the points
Ala, 0, 0) and A'( —a, 0, 0). Therefore, the surface (I) intercepts
a length 2a on the x-axis. In a similar manner the surface (I)
intercepts lengths 2b and 2c on the y and z axes respectively,
These intercepts 2a, 2b and 2c are called the lengths of the axes of
the ellipsoid.
(v) The sections of the ellipsoid by the planes parallel to the
co-ordinate planes. Consider a plane z=A parallel to the XOY
plane.
The section of the ellipsoid (I) by the plane z=A is the ellipse
given by
χ 2/α2+ > 2/6?= 1—A2/c*, z=»A. ...(3)
The lengths of the semi-axes of the ellipse (3) are
a V (I —A2/c2) and ^ y f( I - X iIci).
Now the following cases arise :
Case I I f Xt s numerically greater than c. Wp have proved
in (iii) of this article that the ellipsoid is a closed surface bounded
by the planes x = a , x = —a;y=*b9y = —b and z=c, z=» —c. Hence
A cannot be numerically greater than c otherwise the section will
not be real t.e., the plane z=A will not cut the ellipsoid.
Case II. if A=c or A= —c. In this case (3) becomes
Xi Iai J r y i I b * = 0,
showing that x = 0 , ^ = 0 . Therefore, the section is an infinite-
sinally small ellipse or in other words the section reduces to the
points (0, 0 , c) and (0, 0 , —c) and both the planes ?= c and z= —c
are the tangent planes to the ellipsoid (I) at these points.
Central Conleotds 397

Case III. IfX**-0. In this case (3) becomes


XtIat + yt Ibt = I, z = 0
which is an ellipse in the XOY plane.
Case IV. I f λ increases from 0 to c. When λ increases from
0 to c, the quantity ( I - X iIct ) becomes smaller and smaller and
vanishes at X=c. Hence the lengths O yf(I-X tIct) and b y /(l—XtIct)
of the semi-axes of the ellipse (3) become smaller and smaller and
vanish at X=c. Hence the ellipse (3) diminish in size as we go
away from the XOY plane.
Now if we consider the section of the ellipsoid (I) by the
plane z= —X, all the above arguments hold good.
Similarly the sections of the ellipsoid (I) by the planes para·
Ilel to the planes YOZ and ZOX can be considered.
(vi) Sketch. From the above discussion it is possible to draw
a rough sketch of the ellipsoid ( I) and is given below :§*

§ 3. The Hyperboloid of one sheet.


The standard equation of the hyperboloid of one sheet is
given by
X t Ia t + y * / b * + Z t Ict = I . ...(I)
(i) Proceeding exactly as in § 1(a) we find that all the chords
passing through the origin are bisected at the origin. Therefore,
the surface (I) has the origin as its centre.
(ii) Proceeding exactly as in § 1(b) we observe that the
co-ordinate planes bisect all chords perpendicular to them. Hence
we conclude that the hyperboloid of one sheet (I) is symmetrical
398 Analytical Geometry 3-D

with respect to all the three co-ordinate plaoes and these planes
are called the principal planes of the surface (I).
The three principal planes taken in pairs intersect in three
lines (/.^. the co-ordinate axes in the present case) which are called
the principal axes of the surface.
(iii) The intercepts on the co-ordinate axes. The equations of
the x-axis are .V=O, z=0. It meets the surface (I) in the points
A(a, 0, 0) and A \ —af 0, 0). Therefore, the surface (I) intercepts
a length 2a on the x-axis. In a similar manner the surface (I)
intercepts a length 2b on the .y-axis. The z-axis i.e. x = 0 , y= 0
meets the surface (I) in imaginary points [ z ^ ± V ( - " c)] *-e· l^e
z-axis does not meet the surface (I) in real points.
(iv) The sections of the hyperboloid of one sheet by the planes
parallel to the co-ordinate planes. The equation of any plane
parallel to AOir plane is given by 2 ^=»λ. Thesection of the sur­
face (I) by the plane z=*A is the ellipse given by
x 2/a2+ y 2lb2= I + X 2Ic29 z=A. ...(2)
The lengths of the semi-axes of the ellipse (2) are
a ^ (\+ X 2tc2) and by/(l+ X2j'c2)
and its centre is (0, 0, A) i.e. centre lies on the z-axis at a distance
A from the origin. Since the quantity VO + A2/c2) remains real
for all real values of A, we get an infinite number of elliptical
sections as A increases numerically from 0 to oo. Also the sizes
of these ellipses increase as the value of A increases i.e. as we go
away from the XOY plane.
In a similar way the sections of the surface (I) by the planes
Z= —Aare the ellipses on the other !sides of the plane XOY.
The section of the surface (I) by the plane jc=A (i.e. plane
parallel to YOZ plane) is the hyperbola given by
y2/b2—Z2Ic2= I - X2Ia29 x=X.
Similarlythesectionof the surface (I) by the plane ^=A
(i.e. a plane parallel ZOX plane) is the hyperbola given by
X2Ia2- Z 2Ic*= I - A 2Ib29y = A.
<v) Sketch. From the above discussion it is possible to draw
a rough sketch of the hyperboloid of one sheet and is given
below :
Central Contcotds 399

§ 4. The Hyperboloid of two sheets. Its standard equation


is given by
XiIat - y '/ b · - 2 * / c2 = 1 . ...(I)

(i) Proceeding exactly as in § 1(a), we find that all the chords


passing through the origin are bisected at the origin. Therefore,
the surface (I) has the origin as its centre.

(ii) Proceeding exactly as in § 1(b) we observe that the co­


ordinate planes bisect all chords perpendicular to them. Hence
we conclude that the hyperboloid of two sheets given by the equa­
tion (I) is symmetrical with respect to all the three co-ordinate
planes and these planes are called the principal planes of the
surface (I).
400 Analytical Geometry 3-D

The three principal planes taken in pairs intersect in three


lines (/ e. the co-ordinate axes in the present case) which are called
the principal axes of the surface (I).
(iii) The Intercepts on the co-ordinate axes. The x-axis (y=0,
z=»0) meets, the surface (I) in the points A (a, 0, 0) and
A' ( —a, 0, 0). Therefore, the surface (I) intercepts a length 2a on
the x-axis. But the y and z axes do not meet the surface (I) in
real points.
(iv) . The sections o f the hyperboloid o f two sheets by the planes
parallel to the co-ordinate planes. The equation of any plane para­
llel to YOZ plane is given by x=A. The section of the surface (I)
by the plane x=A is the ellipse given by
y tlbt + zilct =Xt jat — I, x=A. - (2)
The lengths of the semi-axes of the ellipse (2) are
b y / M a * - 1) and cy/(XtIai -X )
and its centre is (X, 0, 0) l.e. the centre lies on the x-axis at a
distance λ from the origin. The ellipses (2) are real when λ is
numerically greater than a and increase in size as λ increases
beyond a. If A is numerically less than a, then the ellipses given
by (2) become imaginary and hence no portion of the surface lies
between the planes x —a and x = —a.
The sections of the surface (I) by the planes y=A and z=A
are hyperbolas given by the equations
X i I a t - Z t J c t = I + XtIbt, y=X

and xilat - y tlbt = I +XtIctf z=X


respectively.
(v) Sketch. From the above discussion it is possible to draw
a rough sketch of the hyperbolid of. two sheets and is given
ahead.
Remark. From the articles I to 4 discussed above it follows
that the standard equation Axi - F ^ i -Fcz2- I of the central coni-
coid represents
(i) an ellipsoid if a, b, c are all positive,
(ii) a hyperboloid of one sheet if any two of a, b, c are posi­
tive and the remaining third is negative, and
(iii) a hyperboloid of two sheets if any two of a, b and c are
negative and the remaining third is positive.
Central Conicolds 401

§ 5. The tangent plane.


(A) To find the equation o f the tangent plane to the central
eonicoid ax*+Ay*+cz®= I at the point Ix1, y lf Z1).
(Kanpur 1979, 81; Lncknow 78, 80)
The equation of the central eonicoid is
ax*+hy*+cz*=l. --(I)
Let (xlf y„ Z1) be any given point on the surface (I).
The equations of any line through (X 1, ylt Z1) and having
d.c.’s /, m, n are given by
(x -X i)ll= (y -y i)lm = (z —Z|)/n=r (say). - (2)
The co-ordinates of any point on the line (2) are (lr+ xt,
m r+ yu nr+Zi). Let the line (2) meet the eonicoid (I) at this point,
then this point will satisfy the equation (I) and so we have
a (Ir+ x tf+ b ( m r + y ^ + c (nr+Z1)*= I
or r* (a/*+bm*+cn*)+2r (Olx1-^bmy1+ Cnz1)
+(ax^+byS+czf—1)=0. ...(3)
Since the point (X 1, ylf Z1) lies on the eonicoid (I), we have
<*x*+byi*+Cz1*= I. ...(4)
Using (4), the equation (3) becomes
r* (a/*+h/n*+ei?*)+2r (a/x1+6my1+c«z,)=0 -(5)
The equation (S) is a quadratic equation in r and hence gives
two values of r i.e. the line (2) meets the surface (I) in two points.
402 Analytical Geometry -3D

Ifthe line (2) is a tangent lin etoth e conicoid (I) at the point
(*ι> Λ . zi) then the two values of r should be coincident. Clearly
one value of r given by (5) is zero and hence in order that the
line (2) is a tangent line to the conicoid at (Jt1, y 1% Z1), the other
value of r should also be zero the condition for which is
alxi + bmyx+ cnzt = 0. ... (6)
The tangent plane to the conicoid at ( X 1 , y l9 Z1) is the locus
of such lines through (xl9 y l9 Z1) that satisfy the condition (6) and
so its equation is obtained by eliminating /, m, n between the
equations (2) and (6). Hence the required equation of the tangent
plane to the conicoid (I) at the point (X1, ^1, Z1) is given by
a (X -X 1) X1*4"b ( y - y x) y x+ c (Z -Z 1) Z1=O
or axxx+byyx+ Czz1= axxf + b y f + c z f
or O x x ^ b y y 1+CZZ1= I . using (4).
Rule. To write the tangent plane to the conicoid at the point
(*i, Z 1) replace x8 by XX1 y 2 by y y x and z2 by z z v

(B) To find the equation o f the tangent plane to the ellipsoid


x*/a*+y*lb*+z*/c2= l at the point (xlf y u Z1).
Proceeding exactly as in § 5 (A) and replacing a by !/a2, b by
1/62 and c by I/c2 everywhere, the required equation of the tan­
gent plane is given by
XXxIaf + y y jb 2+ ZZ1Icf = I.
§ 6. The condition of tangency.
(A) To find the condition that the plane lx+ m y+ nz= p may
touch the central conicoid axf + byf + czf = I.
(Agra 1980, 82 ; Allahabad 78; Lucknow 82; Kanpur 78;
Meerut 83S, 85; Rohilkhand 77; Punjab 78; Rajasthan 71/
The equation of the given plane is
Ix + m y + n z—p --(I)
and the equation 6f the given conicoid is
axf +by1+ czf = I. · ·(2)
Let the plane (I) touch the conicoid (2) at the point (X1, ^1, Z1).
The equation of the tangent plane to the conicoid (2) at the point
(*i. Λ . zO jS
Axx1+ byyx+ C z z 1 = 1.. ...(3)
If the plane (I) touches the conicoid (2) at the point (X1, yu
Z1), then the equations (I) and (3) represent the same plane and
hence comparing their coefficients, we have
m n
Central Conicoids 403

Again since (x1# V1, Z1) lies on (2), we have


a xf+ b yS + czS = I m .
or a (l/ap)2+b (m/bpY+c (<njcpY—I, using (4)
or Il/a+rit1/b+n*lc=p*. --(5)
This is the required condition that the plane (I) touches the
conicoid (2).
The co-ordinates of the point of contact of the plane (I) with
the conicoid (2) are given by (4) Le. are (//up, mlbp, njcp) where
p is given by (5).
Note. Substituting the value of p from (5) in (I), it immedi­
ately follows that the planes

always touch the central conicoid (2).


(B) Tofindthe condition that the plane lx+ m y+ nz= p may
touch the ellipsoid x1lai +y^/bt + z%lc1= l.
(RajaslhaR 1974,76, 78; K arakhetra 77; Boriwaa 76, 80;
Laekaow 86)
Proceeding exactly as in § 6 (A) above and replacing a by
l/a*i b by I/6* and c by l/c* everywhere, the required condition of
tangency is given by
α»/*+6»ιη·+Αι*=/Α ~ 0)
Note. From (I), we get
P = ± ^ / ( α Ρ Ρ + P m ^ + c^ n P ). '
Using these values of p, we observe that the planes
' i* + m y + K = +
always touch the ellipsoid x*/a*+y*/b*+ z*/c* = I.
§ 7. The Director Sphere.
Definition. The director sphere is the locus o f the point o f inter­
section o f three mutually perpendicular tangent planes to a central
conicoid. (RoMlkhaad‘977; Allahabad 79).
The eqaation of the director sphere. To find the equation o f the
director sphere o f the central conicoid eje*+by*+cz*=l.
(Rohilkhaad 1981; Lacknow 79; Kaapar 76; Meerat 84 S, 89 S)
The equation of the central conicoid is
ax*+by*+cz*=l. ...(I)
Let the equations of three tangent planes to (I) be
I1X+mtf+R1Z= VUi la+mflb+n^/c} . .(2)
liX+tthy+ntz= y/ {If/a+ m fi'b+ nflc} ...(3)
404 Analytical Geometry J-D

and 4x+i»iay + ii* z = V { V /« + 'V /* + V M ...(4)


where I1, mu u,; /,, n, and /„ o, are the actual direction
cosines of the normals to the above three tangent planes respecti­
vely. If these tangent planes are mutually perpendicular, then
we have
W1H m 9*+!*,*= I, »i*+«,Hu**= I - (5)
(ι*ι+ /|ΐ»ι+/|ΐΒ ,=0, m,n1+mti>9+m an9= 0 , V 1+ i»*/*+V*=0.
.(6)
The director sphere of the surface (I), being the locus of the
point Of intersection of the three tangent planes (2), (3) and (4) is
obtained by eliminating I1, m1%B1; /„ Ittl3 Kt and /„ 11½, n9 between
the equations (2), (3) and (4) with the help of the relations (5)
and (6). Squaring and adding (2), (3) and (4|, we get
(I1X +m1y+»iz)H{/tx+/nty+iitz)H(4je+ m«y+«»V
=(1/β) Σ /,*+(1/6) Σ Bt1HOAr) Σ It1*
or x* r/j*-| y* Z m f+ z* Σ *,*+2yz Σ m ^ + l z x Σ K1I1
+ 2 xy Σ Z1ZW1= ( I/a) +- (I/A) +- ( I /c), using (5)
or * H y H z * = (l/a )+ (l/6 )+ (l/e ), using (5) and (6).
This is the required equation of the director sphere of the
central conicoid (I).
Corollary. If the central conicoid be the ellipsoid
**/e*+y,/6Hz*/c, = l,
then proceeding exactly as above, the equation of its director
sphere is given by
Jt*+y*+z*=e*+6*+c*.
(Pasjab 1977; Rajasthan 77; Gnraaaaakdev 75)
SOLVED EXAMPLES (A)
Ex. I. Find the equation o f the tangent plane to the central
conicoid 3x*—5y*+z*+2=0 at the point (1,1, 0).
SoL The equation of the given conicoid is
3x*-5y*+z*+2=0. * -0 )
The equation of the tangent plane to the conicoid (I) at the
point (I, I, 0) [See Rule to $ 5] is given by
3jc.1 —5y.l + z .0 + 2 = 0 or 3x—5y+ 2= 0. Aas.
Ex. 2 (a). Find the equations to the tangent planes to the ellip­
s o id ^ jef+ y3/63+ z*/c?= I which are parallel to the plane
lx+ m y+ nz= 0. (Ponjab 1972; Rohilkhaod 80)
SoL The equation of the given ellipsoid is
x*/o*+y*/6*+z*/c*= I. -(0
Central Conicoids 405

Let the equation of a tangent plane parallel to the plane


lx+my+nz=Q be given by
lx+ m y+ nz= p. ...(2)
If the plane (2) couches the ellipsoid (I) then we have [See
§ 6, (B) give complete proof.]
a*/*+ /P fli^cW = P * or ρ= £·\/(αΨ + Ρ η?+ 6*ηΡ ). .. (3)
Putting the values ofp from (3) in (2), the required equations
of the tangent planes are
lx+ m y+ nz= ±^(α*1*+ΡηΡ+<?η?).
Ex. 2 (b). Find the equations o f two tangents planes o f the
surface ax* by*-^czt = I which are parallel to the plane
lx+ m y+ nz= 0. (M.U. 1990)
Sol. Proceeding as in Ex 2 (a) above, the tangent planes are
lx+ m y+ nz= ± y/(l*la+m*/4+n*/c)
Ex. 2 (c). Show that the plane x + 2 p + 3 z = 2 touches the coni·
add Xa—2 /+ 3 r* = 2 . (GwaaanakdeT 1976, Meerut 83)
Sol. The equation of the given conicoid is
x· -2 k*+3 z*=*2 or (1/2) x*-p*+(3/2) z*= l. ...(I)
The equation of the given plane is
■*+2p+3z=2. ·*·(2)
If the plane (2) touches the conicoid (I), then applying the
condition /*/a+m*/I>4 n*jc—pl· [See § 6 (A)], we have

TH+ - ^ i + W r 4 " 2 - 4 + 6 - 4 or 4 - 4 .
which is true. Hence the given plane (2) touches the given coni­
coid.
Ex. 3. Find the equations to the tangent planes to the hyper­
boloid 2x*—6y*+3z*==5 which pass through the line
3 x -3 y + 6 z -5 = 0 = x + 9 p -3 z .
(Agra 1976; Rajasthan 74; M eant 86)
Sol. The equation of the given hyperboloid is
2x*—6y»+3z*=5 or (2/5) Xs- (6/5) y*+(3/5) z* ~ l. ...(I)
The equations of the given line are
3 x -3 p + 6 z —5=0, x+ 9 y—3z=0. ...(2)
The equation of any plane through the line (2) is
3x—3p+6z—5+A (x+ 9p—3z)=0
or (3+A) x + ( —3 + 9 λ )ρ + (6 —3λ) z= 5. ...(3)
If the plane (3) touches the hyperboloid (I), then applying
J f rthe condition /*/a-|-m*/6+/i*/c=p* [See § 6 (A)], we have
(5/2) (3+A)*+(—5/6) (—3+9A)*+(5/3)(6—3A)*=(5)*
or 15 (9+6Α+Λ*)—5 (9-54A+81A*)+10 (3 6 - 3fA+9A*)= 150
406 Analytical Geometry 3-D

or 3 (9+6λ4-λ2)~ (9 -5 4 λ + 8 1 λ 2) + 2 (36-36A+9A2)= 3 0
or —60A2-f 60—0 or A2= I or A= ± I.
When A= I, from (3) the equation of the tangent plane is
4jc+6y+3z = 5.
When A= - I , from (3) the equation of the tangent plane is
2 x -1 2 y + 9 z = 5 .
Ex. 4. Atangentplanetothe ellipsoid x^ja^+y^lb2+ Z2Ic2= I
meets the co-ordinate axes in points Pj Qand R. Prove that the
centroid o f the triangle PQR lies on the surface
flVjc2+Aa/>2+ c 2/z2= 9.
(Rohilkhand 1978; Kanpur 77; Agra 75; Meerut 85S, 89)
Sol. The equation of the given ellipsoid is
jrlaP+y^/P+z^lc2= I. ···G )
The equation of any tangent plane to the ellipsoid (I) is
[See Note to § c(B)]
lx+ m y+ nz= V (a*P + b2mz+cin?). .(2)
The plane (2) meets the co-ordinate axes in the points given
by
P {y/irr+ P m t+ c'n*)/!, 0, 0); Q {0, y/(a*F+b*m*+cV)//w, 0}
and R {0, 0, + b2m2+ c2/i2)/n}.
Let (α, β , γ ) be the co-ordinates of the centroid of the triangle
PQRj then
«- 0+0 I V i f P + M + c< n*)t

A= y/U P P + b fin P + c-n 2), Ύ = · ^ ^ i a i P + P m i +Al*).


Thus 3Ia= y /(<Pl*+IPnii +cb?) or 9/V=a*/®+6*m*+Ar.
. a* 9Pa* 9a»/* ]
** a* “ 9/¾ *- a*/*+An»+A t* ' /
.. . 6* Wmt c* 9c*/t* I
uni ar y » y* ~ a2/*+An*+c*n* J ...( 3)
Adding the relations in (3), we get
a* A* A 9 (β*/*+Αη*+Αι*)
α* β*. y*— e*/*+An*+Ai*
Λ The locus of the centroid («, p, y) is
a8/x, +ft*/>*+ci/z*=9. Proved.
Ex. 5 (a). Tangent planes are drawn to the ellipsoid
X i I c r + +1Ib1+ Z 1I c t = I through the point (α, β, γ). Prove that the
perpendietdars to them from the origin generate the ewe
(*χ+ β^+ γζ)*=(<?&+P y t +<τζ*). (Allahabad 1980)
Central Conicoids 407

Sol. The equation of the ellipsoid is


x 1la*+yi/bi +z*/cl= l. ...(I)
The equation of any plane through the point ( α , β, y) is
I (x - a ) + m (ν—β)+ η (z -y )= 0
or lx+ m y+ nz= U + m fi+ ny. ---(2)
If the plane (2) touches the ellipsoid (I), then we know that
tPP+b2m2+c2n2=(la+ mfi+ny)2 [See § 6 (B)]. -- (3)
Also the equations of the perpendicular from the origin to
the plane (2) are given by x]l=y!m = zjn. :-.(4)
Eliminating /, m, n between (3) and (4), the required equation
of the cone generated by the line (4) is given by
OiX1+ b2y 2+ C1Z2= (lx+my+nz)2. Proved.
Ex. 5 (b). Tangent planes are drawn to the ellipsoid
x 2/a2+ y2lb2+ z2/c2= l through the point (α ,β ,γ ). Provethatthe
perpendiculars to them through (α, β, γ) generate the cone
{* (x -* )+ j? (y - β )+ γ (z -y )} 2= a2(x - a)2+b2 ( y - fi) 2+ c2 ( ζ - γ ) 2.
Sol. Rewrite all steps of Ex. 5(a) upto the equation.(3).
Also the equations of the perpendicular to the plane (2) from
the point (α, β, γ) are given by
(X-Ci)Il= (y—fi)/m =(z ~ y)jn. .. .(4)
Eliminating l,m , n b e tw e e n (3) and (4), the required equa­
tion of the cone generated by (4) is given by
a1 (x ~a)2+ b2 (y—fifi+c2 ( z - y )*+{<* (χ—α)+β ( y - P ) + y ( ζ - γ ) } 2
Proved.
Ex. 5 (c), Tangent planes are drawn to the conicoid
ax2+by2+cz2= I through the point (α, β, γ). Prove that the per­
pendiculars to them from the origin generate the cone
(xx+ fiy+ yz)2= (x 2/a)+ (y2lb)+ ‘z2lc). (Kanpur 1976, 81)
Also show that the reciprocal cone of the above cone is
(ax2+by2+cz2) (aet2+bfi2-f-cy2—I) -(aa.x+bfiy+cyz)2=0.
and hence show that the tangent planes envelope tie cone
(ax2+by2+cz2—\) (a*2+bfi2+ cy2—l)=(aox+bfiy+cyz—I)*,
Solution I Part. Proceeding exactly as in Ex.. 5 (a) above and
replacing a2 by 1/a, b2 by I Ib and c2 by l/c, the equation of the
required cone is given by
(xx+fiy+ yz)2= (x2/a)+ (y2!b)+(z2/c). ... ( I)
Proved.
II Part. Tofind the equation o f ti e Reciprocal cone to (I).
The equation (I) may be rewritten as
x2 («*—I/a)+y* (β2- \ /b)+ z2 (y2- 1 f i ) +2fiyyz+2yazx+Iafixy=O.
- ( 2)
408 Analytical Geometry 3-D

Let the equation of the cone reciprocal to the cone (2) be


Ax*+By*+Cz*+2Fyz+ 2Gzx+2Hxy=0. ...(3)
Then ^ = f c - / * = ( / ? * - - ^ ) |y * - ± ) - ( / l y ) *
Ifit y1 , I \ _ a(fr?»+cy»-l)
\ c c be J abc
. j ax* (bfi*+cy*—I) ax* (a**+bfi*+cy*— I ) -a*a*x*
•· * “ abc ---------------------- a5c '
with similar expressions for Jfy1 and Czt.
Also F—g* - a/=(jtt).(a/?)-(a*—Ι/a) (Py)=AW*.
C=yat/b, H ^afilc.
Putting these values in (3), the required equation of the
reciprocal cone is given by
[(ax* (aa*+ty?*-f-cy*- 1) -a*».*x*}
+{Ay* (ea*+ 6/)*+cy*—1 ) - A*0*.y*}
+{cz* (a»*+ 6^*+<7 *-l)-c*y*z*)}]
2flyyz 2yazx 2*)? _
+ ^ + V +T ^ -°
or (ax*+Ay*+ez*) (a«*+Ap*-f cy*—I) - (a*a*x*+A*A*,y*+c*y*z*
—Ibcfiyyz-2caydzx—2ab&fixy}—0
or (ax*+b)P+cz*) (aa*+AA*+O'*—I )= (* * * + AAy+<7 Z)*· Proved.
Ill Part. To find the equation of the enveloping cone.
The equation of the enveloping cone is given by SSi -T * .
Hence S=ax*+Ay*+cz* —I. The point is (a, fi, y).
S i—avi+bfi*+ cy* —I
and T*=actx+bfiy+cyz— 1.,
Hence the required equation of the enveloping cone is
(ax* +by*+cz*— I) (a<x*+bfi*+cy* - I) = (a«x+ bfiy+ cyz— I)s.
Ex. 5 (d). Tangent planes are drawn to the central conicoid
ax*+ 6y*+cr*= I through the point (a, fi, y). Show that the per-
pendiculars from the centre o f the conicoid to these planes generate
the cone (ax+fiy+yz)*=x*/a+y*lb+z*lc.
(Rohilkhand 1979; Kanpur 81; Madras 77)
Sol. The centre of the conicoid ax*+by*+cz*= I is the
origin. Hence it is the same problem as Ex. 5 (a) above.
Ex. 6 . Find the locus o f the foot o f the centre perpendicular
on varying tangent planes to the ellipsoid
x*la*+y*/b*+z*lc*= I.
(Meerut 1984; Rajas’han 75; Kanpur 81)
Central Conicoids 409

Sol. The equation of the given ellipsoid is


X2Iat + y 2lb%+ ZtIct = I. ... ( I)
The equation of any tangent plane to the ellipsoid (I) is
lx + m y+ nz= y/(a2l2+ Pmt +C1Ht). .. .(2)
Thecentre of the ellipsoid (I) is the origin and hence the
equations of the normal to the plane (2) from the centre (0, 0, 0)
of (I) are xll= ylm = zln. - (3)
The foot of the perpendicular from the centre on the tangent
plane (2) is the point where the line (3) meets the plane (2) and
hence its locus is obtained by eliminating /, m, n between (2) and
(3) and, therefore, is given by
x.x + y.y + z.z= y/(a2x2+ P y2+CtZt).
Squaring, the required locus is
(JCf + y2+ z2)2= a2X2+ b2y24 c2z2.
Ex. 7. Find the locus o f the perpendiculars from the origin to
the tangent planes to the surface x 2la2+ y2lb2+ Z2Ict = I which cut off
from its axes intercepts, the sum o f whose reciprocals is equal to a
constant k.
Sol. The equation of the given ellipsoid is
x t/at + y tJl)t 4- Z2Ic2= I. ...(I)
The equation of any tangent plane to the ellipsoid (I) is
lx+ m y+ nz = i/(a 2l2+b2m2+c2n*). ... (2)
The intercepts made by the plane (2) on the co-ordinate axes
are y/(a2l2+b2m2+c2n2)lt9 x/(a2l2+b2m2+c2n2)/m
and y/(a2l2+ b2m2+ c2n2)/n respectively.
The sum of the reciprocals of these intercepts is given to be k.
l+ m + n ,

or ((l+ m + n)2= k2 (n2/?4-bPm2+c2n2). ...(3)


Now the equations of the perpendicular from the origin to
the tangent plane (2) are
xll= ylm = zln. -.(4)
Eliminating /, /?i, n between (3) and (4), the locus of the per­
pendicular (4) is given by
(x+ y + z)l = k 2 (a2x2+b2y 2+c2z2). Ads.
Ex. 8. Prove that the equation to the two tangent plhnes to
the central conicoid ax2+ by2+cz2=*\ which pass through the line
κ αΞ / ι * 4 - m xy 4 - /I1Z u ^ l%
x + m%
y +ntz - p$=0 is
410 Analytical Geometry 3-D

* (¥ + ? + ¥ -* ) ■ ( $ + T + ?? - λ * )

Sol. The equation of the given conicoid is


a x ^ frj^ + c z 2= ! . ...(I)
T h e eq u ation o f any plane through the given line M 1=O=Mt is
M 1-I-Am8=O ...(2)
or ( /Ix + » i 1.y-f W1Z- / h l + A (/jx + w jty + n tz —pt) = 0
or ( I i A X l i ) x + (W7j+Awj2) ^+(M1+A n8) Z=(^1-I-Ap8). -(3 )
I f t h e plane (3) touches the co n ico id (I ) th en ap p ly in g the
co n d itio n “ Z2/a + w j2/fr I w2/ c = p 2” , wc have
(IiA-Xli)tIa -I (m, + XmiYjb+ (w,+ XniYjc= O i + XptY- ...(4)
Now from (2), we have A= -U lIui. Putting this value of A in
(4), the required equation of the two tangent planes is given by
(l1—ullt!uiY/a+{mt—u1mi/uiY!b-\-(nl—ulnijui)ilc=(pl—ulpi/uiY
or (Z1M8 - U1IiYla + (WJ1M2 - M 1WJ2) V f r + (W1M2 - M1W2)S/C = O lM t - W|ft) 2
„ ( L 2 WJ22 η.2 „\ / LI. WJ1JJJ.2 W1Wt \
or
“■*U +7 +v -*·) U+/T +ir - H
, , ZZ12 mI2 , Wi2 P12ϋ \J^O. Proved.
+Ut' \ a + b + c ' Pl I
Ex. 9. I f l r is the distance between two parallel tangent planes
to the ellipsoid x2/a2-\-y2/b2-[-zi/c2= ], prove that a line through the
origin perpendicular to the planes lies on the cone
X2 (a2- r 2)+ y2 ( b2 - r2) + z2 (c2- r2) = 0. (Kanpur 1977)
Sol. The equation of the given ellipsoid is
x2/a2+ y 2/b2+ z2/c2= l. ..(I)
The equations of the two parallel tangent planes to the ellip­
soid (I) are
lx + m y+ n z= ± \/(a2l2+l}2m2+ c2n2). ...(2)
The distance between the two parallel tangent planes given
by (2)
=»The difference of the perpendicular distances from
the origin to the planes given by (2)
j __2 <f(a2l2+b2m2+c2n2)
V (/2+.W22+fl2)
or a2/2-)-A2m2-(-c2/i2= r 2 (l2+m2+ n2) [on squaring]
or I2 (a2—r2)+ m 2 (b2- r 2) + n2 (c2—r2) = 0. • .(3)
CentralConicoids 411

Now the equations of the lines through the origin and perpen­
dicular to the planes (2) are
xll=y!m=z/n. ...(4)
Eliminating I9 m9 n between (3) and (4), the required locus of
the perpendicular (4) is given by
xr (a2—r*)+y* (b*-r*)+z* (c * -r2)= 0 . Proved.
Ex. 10. Show that the tangent planes at the extremities o f any
diameter o f an ellipsoid are parallel. (Rajasthan 1978)
Sol Let the equation of an ellipsoid be
x*!cfi+y*lb*+z*/c*= I. ...(I)
W eknow that the centre of the ellipsoid (I) is the origin.
Hence every line passing through (0, 0, 0) and intersecting the
ellipsiod (I) is a diameter of (I). Thus let the equations of a
diameter of (I) be
x/l= y Irn=^z/n. ...(2)
Any point on (2) is (Zr, wr, nr). If it lies on (I), then
r* (/Vn8+ m2/6*+ ;i2/c2) = I
or r= ± \/y/iP /tP + m P /P + P Ic9)= ±A (say).
Now the extremities of a diameter are
(/Af m \9 ηλ) and ( —/A, —wA, —ηλ).
The equations of the tangent planes to (I) at these points are
/Ax m \y ηλζ —I i lx my nz_ I
a~ΊΓ + P ' + C ,.2* · * &a* +5*"
‘ Λ* +■ 7i~
c* -(3)
—/Ax mky wAz . lx my nz__ _1_
and
a* “ 6* Ci = .......*
i e" a*
a* + b3 + o*~ -----λλ ..,(4)
The equations (3) and (4) are the equations of two parallel
planes.
Ex. 11. I f P be the point of contact of a tangent plane to the
ellipsoid which meets the co-ordinate axes in A9 B and C and PL9
PM9 PN are the perpendiculars from P on the axes9prove that
OL.OA=a\ OM .OB=P9ONOC=C2.
Sol. Let the equation of the ellipsoid be
jp/a*+y*lb*+z*lc*= I. ... ( I)
Let the co-ordinates of P be (a, /?, ;>), so that the equation of
the tangent plane to the ellipsoid (I) at P (a, β 9 y) is
a x/a'+ fiy/P+yz/c*= I. ... (2)
The plane (2) meets the co-ordinate axes in A9Bi and C and
so OA9 OB and OC are the intercepts made by (2) on the co-ordi­
nate axes hence we have
OA=QiIoL9 O B = P t f9 OC=C2Iy.
412 Analytical Geometry 3-D

Again it is given that PL is the perpendicular from the point


P (α, β, γ ) on the x-axis so that OL=*.
Similarly ΟΜ —β and Ο Ν = γ.
Thus 0L.CM=ot.(o*/a)=o3 ; ΟΜ .ΟΒ=β.(ΡΙβ)=Ρ
and ON. OC= y.(c*/y)= c*.
Ex. 11. I f the line o f intersection o f perpendicular tangent
planes to the ellipsoid whose equation referred to rectangular axes is
x'la'+ yt/lP + zfc*= I passes through the fixed point (0, 0, k), show
that it lies on the cone
x * (£*+c*-**)+y* (c*+o*-Jfc*)+(z -Jfc)* (a*+b2)= 0.
(Meerut 90 ; Kurufcshetra 71)
Sol. The equation of the given ellipsoid is
x*/o*+)-»/**+**/<*= I. ...(I)
The equation of any plane through the point (0, 0, k) is
/(x -0 )+ iw (y —0 )+ n (z—Jfc)=0 or lx+ m y+ nz= nk. ...(2)
If the plane (2) touches the ellipsoid (I), then we have
e*/*+0*/ii*+c*n*=n*Jfc* [See § 6 (B)]
or α*/*+6*ιw*+(c*—Jfc*) w*=0. --.(3)
Now the equations of any line through the point (0, 0, k) are
x —0 _ y —0 _ z —k x_ y z —Jfc
λ — μ ~ v λ μ v -(4 )
If the line (4) lies in the plane (2), we have
IX+mp+nv=0. ...(5)
The direction cosines l,m ,n of the normal to the tanget
plane (2) are related by the relations (3) and (S) and since (3) is a
quadratic relation in /, m, n, it shows that there are two sets of
values of /, m, n and hence for every line (4) through the fixed
point (0, 0, k) there are two tangent, planes to the ellipsoid (I)
which intersect in the line (4).
Eliminating n between (3) and (S)1 we have
a2/*+c*iM*+(c*-Jt*) ^ ' ^ )= 0
or (o*v*+A*c*—AtJfc*) P f-Ilm λμ (c*—Jfc*)
+ ( 6 V + ,A 2- P 2Jfc2) iM*=0
or (oV+A*c*-A*Jfc*) (//m)*+2Ap (c*-Jfc*) (Iim)
+ (A V + p2c*-p*Jfc*)=0.
Let its roots be IlIml, IJmt.
Then the product of the roots= —. — , ~}~ft iL ~ !tci ’
K IW1 ii», oV+A*c2—A2Jfc*
Central Conicoids 413

. _ IiU _ Ot1Tnt U1Ttt


·· b»v*+(c* -Jt*) μ* -(C t k*) λ*+<Λ*_ ο·|**+**λ*
[by symmetry]
Since the tangent planes are to be perpendicular, we have
V i+W i^i+K A =®
or {6*v«+(c*—**) **}+'{(«*—**) λ»+Λ *}+ f a V + W - Q
or (h*+c*-jfc*) A*+(e*+c*-**) |^+(e*+6*) v*=0. ...(6)
Eliminating λ, μ, v between (4) and (6), the equation of the
required cone generated by (4) is given by
(£*+<:*-**) x*+(as+ c * - Jt*) /+ (a* + * * ) (z-k)* = 0 .
Ex. 13. Through a fixed point (Ar, 0, 0) pairs o f perpendicular
tangent lines are drawn to the conicoidax*-(-by*+Czt= I. Show that
the plane through any pair touches the cone
(x —k)*’ y* Zt
(b+c) (Ofci - I ) ^ c («£*—! ) - α^~b* (ak*—1 ) -
(Allahabad 1979)
Sol. The equation of the given conicoid is
ax*+by*+cz8= l . ...(I)
The equations of any line through the given point (k , 0, 0)
are
(x-k)/l= ylm = zln**r (say). ...(2)
The co-ordinates of any point on the line (2) are (lr-\-k, mr,
nr). If the line (2) meets the conicoid (I), then this point will
lie on (I) and so we have
a (lr+k)*+b (m rf+ c (Itrf= I
or r* (aP+bm'+cn*) 4-2Are/r+(eifc*—1)=0. . (3)
If the line (2) touches the conicoid (I), then the two roots of
(3) must be equal and hence applying the condition tB2 - 4 A C - 0 ’,
we have
(Vealf - 4 (al'+bnfi+cn*) (dfc*-l)=0
or (al'-^bnf+en2) (Okt- X ) ^ k iCPlK ...(4)
Thus for any tangent line (2) to (I) through the point (k , 0, 0),
the values of I, m, n should satisfy the relation (4).
Foranytw o perpendicular tangent lines to (I) through the
point (k, 0, 0), let the two sets of values of I , m, n be I u mu H1
and Ii, Tnt , nt so that the equations of these perpendicular tangent
lines through (k , 0, 0) are given by
x —k y z , x —k y z
414 Analytical Geometry 3-D

Therefore, in view of (4), we have the following two relations


(a li+ b m f+ CZi12) (α/c2— II=A^a2Z12
and (alf+bm 2*+ cn9*) (ak2—I) = Zrn2Z12.
Adding, we get
{a (Zi2+ V )+ ^(ZH12+ /Wa2H Cfn11-f-rt*2)} (ek- — (Zi2+Zr)
v.(6>
Let the line ( x - k)llz=yjm9=z/nz through the point (k> 0, 0)
be the normal to the plane containing the two tangent lines given
by (5).
Thus we have three mutually perpendicular lines. Hence we
have the relations
V W + " , 4= ! = /,* + /.3+/,* or /^+V=O t ,2+"!*:
Similarly W12+ m*2= n ,2+ Z32, and B12+ nt2= Z32+ ms2.
Using these relations, (6) becomes
{a (ms2+w,2)+ 6 (η,*+/,*)+c (/,*+«,*)} (ak*--1)
=fc*o* (/»,*+«,*)
or Z32 (Z>-|-c) (ak2—l)+ w 3* {c(nfc2—1)—o}+n*2 {b (ak2—1)—a}=0
•(7)
The equation of the cone generated by the normal to the
plane containing the perpendicular tangent lines given by (S) is
obtained by eliminating I3, m3, n3 between the equations (7) and
(x —k)ll3=ylm3=zln3 and is therefore given by
(*-*)* (Z>+c) (ak2— I)+y* {c (efc*-1)-o}
+2* {b (afc2—1 ) - a}=0. ...(8)
The cone reciprocal to the cone (8) is clearly given by
___ f a - * ) 8 __ £ ____ i - =0
(b+c) (Afce- I ) T c (Ofc2- I ) - a (afc2- l ) - a
which is the required equation of the cone which is touched by
the plane through the lines given by ( )
Ex. 14. Prove that the locus o f points from which three mutu­
ally perpendicular planes can be drawn to touch the ellipse
W + J * / * 1= !· 2=0
is the sphere x2+ y l+z*=ai+ b2. (Punjab 1973; Luckuow 86)
Sol. The equations of the given ellipse are
x2/a2+y*lb*=\9 2= 0. ...(I)
L etthe equationof one of the three mutually perpendicular
planes be'
Ux+n ttf+ Wizep1. ...(2)
The plane (2) meets the plane z= 0 in the line given by
Central Conicoids 415

h * + Wtf=Pu * = 0. -(3 )
Now if the plane ( 2) touches the ellipse (I) then the line (3)
will touch the ellipse (I) in the xy-plane and the condition for
the same is
O2I12+ Fnti2=Pl2 or Pi=zV ia0I t+ P m l2).
Putting this value of pv in (2), the equation (2) becomes
i1x+ m 1y-\-n1z = V ( a2li2+ b'ni12). ...(4)
Similarly the equations of the other two tangent planes to
the ellipse (I) are
l2x+rn2y-\- n2z = V ia 2U2+ b2m% 2) .. .(5)
and Z3* + m9y + w3z= V ia 2U2+ b2ma2). .. -(6)
Squaring (4), (5) and (6) and adding, we get
*2 TZ12+ / 1 Σ πί^ + ζ- Σηχ2+ 2γζ Σ η ι^ + ΐζ χ Tzi1Z1
+ 2 x y TZ1ZW1=A2 TZ1H h 2 Στηχ2
or x2A + y l.\+z-.\-\-2yz.0+2zx.0 + ?xy.0=a2.\ +Zr.l
or *H pH *W +h8 t v TZi2= I i TZ1W1=-Oetc.]
which is the required locus.
§ 8. The Polar Plane. (Kanpur 1982, Meerut 84 R)
Definition. Let ax2+by2+cz2= I .(1)
be the equation o f a conicoid and Afa9β9 γ) be any point. Draw lines
APQ to meet the conicoid (I) in the points P and Q. The locusof the
point R such that AR is t h e harmonic mean of AP and AQ (i e.9 AP9
AR and AQ are in harmonic progression) is called the polar plane of
the point A w ith respect to the given conicoid.
(A) The equation of the polar plane.
The equations of any line through the point Α(<χ, β9γ) are
( χ - α )/l= iy ~fi)/m =iz - y)/zi=z* (say), ..(2)
where Z, m, n are the actual d c.’s of the line.
The co-ordinates of any point on (2) distant r from Λ(α, β 9 γ)
are Qr+α, mr+β, nr+ γ). Therefore, the distances of the points
P and Q where the line (2) meets the given conicoid (I) are given
by the quadratic in r2
a (Zr+«)2+ 6 im r+fi)'+ c (nr+y)*= I
or r2 (al2+bm2+cn2) + 2r ial&+bm$+cny)
+(A aH tyJH cy2- 1)=0. ...(3)
L etthetw o values of r be T1 and ra and let Z1= ^ P a n d
T9=sAQ. Now AP9 AR9 AQ are in harmonic progression.
416 Analytical Geometry 3-D

2 —2 (ala 4 bmfi 4 ctry)


OT Α ΪΓ m fi+ b p + c f — X
or a».2+bfii + cyi —l = ~{a% <I.AR)+bfi (m.AR)A-cy (n.AR)}:
...(4)
Now let (x, y, z) be the· co-ordinates of the point R on the
line (I) whose distance from the point A (a, β, γ ) is AR and so
we have
(x —a)ll= (y—fi)/m=(z—y)ln=AR.
I A R ^ x —a, m A R = y—fi, n.A R = z—y. - (5)
Now in order to find the locus of R, we are to eliminate /,
m, n between (4) and (5) and so it is given by
aat+bfP A-cy1' - 1= —{a* (je-ct )+bfi ( ν - β) Α cy ( z- y) }
or a*x+bfiy+cyz=l. - (b)
This is the required equation of the polar plane of the point
4 (α, β, γ ) with respect to the conicoid (I).
Note. The equation (6) of the polar plane of the point A (a,
β, y) w.r.t. the conicoid (I) is of the same form as that of the
tangent plane of the conicoid (I) at the point A (a, β, γ) on the
conicoid.
(B) The pole of a given plane.
Tofind the pole o f the plane lx-\-my-\-nz=p w.r.t. the conicoid
ax2+by*+czt= I. ( Meerat 1983)
The equation of the given conicoid is
ax2+by2A-Cz2= I. ...(I)
Let the pole of the plane
IxA-my+nz=p ...(2)
w.r,t. the conicoid (I) be the point /*(*,/», γ). But the polar
plane of P (a, β, γ) w.r.t. to (I) is
aax-A-b$y+cyz=\. ...(3)
Therefore the equations (2) and (3) represent the same plane
and so comparing their coefficients, we get
a* bB cy I . I n m n
—r = — = ~ = — i.e. a = —, β=-τ- , y = ----
I m n p ap bp cp
:. The required pole is (l/(ap), m/(bp), n/(cp)).
§ 9. Properties of the polar planes and the polar lines.
(A) Let P («!, 0j, ;·,) and Q(a2, fit, yt) be any two points. It
clearly follows that if the polar plane of the point P with respect
to a given conicoid passes through the point Q then the polar
plane of Q w.r.t. the same conicoid will pass through P. The
Central Conkoids 417

points P and Q satisfying this property are called the Conjagate


points.
(B) Let U1=SZ1X+/W1JH- ηλζ - P 1=O
and Ut= ItX+Ott V+n^z—pt =Q
be the equations of the two planes. It clearly follows that if the
pole of the plane H1=O lies on the plane Ut = Othen the pole of the
plane W2=O will lie on the plane W1=O. The two-such planes W1= O
and W 2=O are called the conjugate planes.
(C) The polar lines. The polar line or simply polar o f a given
line AB is another line PQ such that the polar planes o f all points on
AB pass through PQ.
Let the equation of the conicoid be
ax^+bjP+cz*=I. ...(I)
Let the equations of the line AB be
(* - * )/l= (y -fi)lm = (z-y )ln = r (say). -(2)
Any point on (2) is R (Ir+a, mr+ β, nr+γ). The equation of
the polar plane of the point R with respect to the conicoid (I) ·*
ax (lr+a)+by (mr+fi)+cz (n r+ y )= l
or (aax+bfiy+cyz— l ) + r (alx+bmy+cnz)=0.
This plane for all values of r clearly passes through the line
~ of intersection of the fixed planes
axx+bfiy+cyz —I = 0, alx+bmy+cnz=0.
Let this line be the line PQ.
Thus we observe that the polar plane of any point R on AB
passes through every point of the line PQ and hence the polar
plane of any point on the line PQ must pass through R. But R is
an arbitrary point on AB and so the polar plane of every point on
PQ will pass through every point of the line AB.
Thus we conclude that the lines AB and PQ are such that the
polar plane of every point on AB passes through PQ and vice
versa and the two such lines are called the polar lines.
(D) To findtheconditionihat the two given lines be the polar
lines. *
Let the equation of the conicoid be
ax*+by*+Czli= I. ···(!)
Let the two given lines be denoted by AB and PQ and let
their equations be given by
x———a - Sy—-
- β = z——yc .and, x— —

x' y —β' z - y'
I m n I m n
respectively.
418 Analytical Geometry 3-D

Proceeding as in § 9 (C) above the polar line of AB with


respect to the conicoid (I) is the line of intersection of the two
planes given by -
aaix-\-b£y-i-cyz— 1= 0 ....(2)
and alxA -bm y+ cnz= 0. ...(3)
Now if the polar line of AB be the given line PQ then PQ will
lie on the planes (2) and (3) both. The conditions for which are
axa'+bfi&’+ c y y ' —1= 0 I
aal'+ bfim '+ eyn '=0 J -.(4)
and a la '+ bm p'+ crry'= Q I
all'^rbm nt’-Vcnn’—Q J - (5)
Hence the required conditions are given by (4) and (S).
(E) Conjagate lines. Two Bnes A B m d PQarecaUed the con­
jugate lines i f when the line PQ intersects the polar line o f the line
A B then the line AB also intersects the polar line o f the line PQ.
To fin d the condition fo r the two given lines to be conjugate tines.
First rewrite the steps upto equation ( 3 ) of $ 9 (D) above.
Any point on the line PQ is (/'r+a', m ’r +β". m’r+ γ '). If the
line PQ intersects the polar line of A B [given by (2) and (3)) then
for some valde of r this point will satisfy both the equations of
the planes (2) and (3) and so we get
an (P r+ x'i+ b fi (m 'r + D + c y (*> + / )- 1= 0
and at (/'r+«')+im (#n'r+/T)-fcir (n'r+y')= 0
or («ta'+ftSp'+eyy'—1)= —r {auT+ bfim .+ cytf) ...(4)
and {a<Tl+b$'m+cy'n)= —r (aU'+bmm'A-crtn'). -(5)
Eliminating r between (4) and (5), we get
(a em '+ b fifi'+ cyy'— I) (air+ bnan'+ cnn')
M a n l'+ b p n ’+ c y if) (alx'+ bm £'+ a r f). „.(6)
This is the condition that the line PQ intersects the polar
line of AB. The symmetry of the result (6) shows that the line AB
will also intersect the polar line of PQ. Hence (6) is the required
condition.
SOLVED EXAMPLES (B)
Es. I. Prove that the locus o f the poles o f the tangent planes
o f a x ? b y 1+Czt = I with respect to a'jr + b ’jp+ c'z?= i is the conl·
cold (a 'x f/a + (b 'y flb + (e’z flc = I. (Allahabad 1982)
Sol. The equations of the given conicoids are
a x'+ b jf+ cz* = I, -(1) e'x*+b'y*+c';r*= I. -.(2)
Caaral Conieoids 419

Let the equation of a tangent plane of the conicoid (I) be


lx+my+nz=>p. ...(3)
Then (/*/e)+(mVh)+(«*/e)=p*. -W
Now let the pole of the plane (3) w.r.t. the conicoid (2) be
(xt, J1, zt) so that (3) is the same plane as
O1XX1+ V y y i+C1ZZ1= I. ...(5)
Comparingthe coefficients of the equation (3) and (S)t
we get
I m n p
a'X1 ~ b ’y t ~ CtZl ~ I
or I=OtXtP, O i= VyxP, n = c’ztp.
Putting these values of /, m , n in (4)· wc have
(V x ip r io + iV y ip y ib + ic 'z ip r ic = ?
or («%)*/«+ W y ir ib + (c’z jflc = L
Λ The locos of the pole (xlt Jlt Z1) of the plane (3) w.r.t. (2)
is given by
(V x)tIa + (V y)V b+ (c'zY lc = I. Proved.
Ex. 2. Prove that the locus o f the poles o f the tangent planes
o f O1X*+bry* c'-r= I with respect to «V+/!®?*+J 8Z1=I *■* the
hyperboloid o f one sheet. (Kurnkshetra 1974)
SoL Proceeding exactly as in Ex. I above and replacing
a by e*t b by A*, c by —t*, a ' by «*, V by β*, c ' by y8, the required
locus is given by
e*jt* fPyj_ γ*ζ* _ .
a* b* c8
which is the equation of the hyperboloid of one sheet.
Ex. 3. Show that the locus o f the pole o f the plane
lx+ m y+ n z= p with respect to the system o f coracoids
Σ ixV(o*+ifc)]=lt is a straight line perpendicular to the given plane
where k is a parameter. (Kanpur 1976 ; Bunielkhand 72)
SoL The equation of the given system of conicoids is
x* y* Zt
e8+*+*8+Jfc^c8+Jfc ...(I)
The equation of the given plane is
lx+ m y+ n z= p . <·...(2)
Let ( cl, β , γ ) be the pole of (2) w.r.t. the conicoid (I). The
polar plane of (α, β, γ) w.r.t. the conicoid (I) is

«r+Jfc+ O8+ Jt+ c*+/fc . ( 3)


420 Amdytioat Geometry 3-D

The equations (2) and (3) should represent the same plane
hence comparing their coefficients, we have
« /(* + k )J l / ( P + t o _ y l(c*+k) I
I m a H p
·*♦ « = * {<d+k)jp, β = « ι (**+*)//>, j = n {<r+k)jp
OtI Ik m il· m k ml· ok
W a ----=—, ft------ ——, y ----- = ----
* P P P P P P
Therefore, we clearly we have
a —QtIjp β —mb'fp j —mPlp t
I = ' m η p
•V The locns of the pole («* β , j) is
-T- OtIfp^ y -B ttitIp Z-HtPjp
/ “ m ~ η
This is a straight line with its direction cosines proportional
to /, in,« which are also the direction ratios of the normal to the
plane (2). Hence the locos is a straight line perpendicular to the
given plane (2).
Εκ. 4. Find the loots o f straight lines drawn through a fixed
point (m. β, γ) a t rigjht angles to their polar Hith respect to the
conicoid at*+ byh+cz *= I.
(Lnrknew 1976, 80; Kanpnr S2; Meerat 89)
SoL The equation of the given conicoid is
ajp+byr+cz*'= I. -.-(I)
The equations of any straight line through the point (a, /I, y)
are (x —x )!I= iy —fi)!m = {z -y);n = r (say). --(2)
The co-ordinates of any point on (2) are (lr+ α, ra rjA n r 'r j).
The equation of the polar plane of this point w.r.t. the conicoid
(I) is ax {lr+ x)+ b y {m r+ fi)+ cz {nr+ 7)= I
or (aax+ b$y+ cyz— l)+ r (aix+ bm y+ cnz)= 0.
This plane for all values of r will pass through the line
a xx + b fiy+ c jz— I=0, alx+ bm y+ cnz= 0. ...(3)
This is the polar of the line (2) w.rX the conicoid (I).
Let I, ft, ·» be the direction ratios of the polar line (3) so that
we have
axh+ bflp+ cj*= 0, aIX vbm p+ cn*= 0.

b c {α β — m y) ca \1 γ -n a ) ~ a b ( m x —Ιβ) ---(5 )
Since the lines (2) and (3) are given to be perpendicular, we
have lk+ m p+ ny= 0
or Ibc (ιφ—my)-J-mai (fy - na)+nab (met—//?)—0.
Dividing bf abcimn throughout, we get
Central Comeoids 421

p _ _ i _ . y __ _°L. JL A =0
am an*bn bl ' d cm
or
-(5 )
The locus of the line (2) is obtained by eliminating I, m ,»
between (2) and (5) and hence is given Iqr

^ (K b 4 (K )^ (y 4 )-*
Ea. 5. I f P (Xu y i, Zi) and Q ix tx y » z j a n any two points
then fin d the equations o f the polar o f PQ with respect to the coni-
coid OJCi-J-Ajs-T-Cz2= I.
Sol. The equation of the given conkoid is
<uc*+bj*4 Czi = I. .:.(1)
Now the equations of the line joining P (xu J 1, r j and
¢ ( ¾ , I n Z1) *re
JC-
- —-Jr1 = -J——
- J 1 = ----
z—z, . .
L=r (say).
Jtf-Jfi y t —yi z t - * i
The co-ordinates of any point on this line PQ are
(*i+r (Xt - X i), y t+ r (J1- J 1)vZl-Hr (Zt - Z 1)).
The polar plane of this point w.r.t. the conkoid (I) is
a x (JC1-Hr (Xt - X1IJ-Hhj (J1-Hr (J1- J 1)Hcz (z,+r (Z1-Z 1)J=I
or (axx1+ byyi + czz1— \)+ r {ax (Xt X1) + by (J1- J 1)
-Hcz (Z1-Z 1)J=O.
This plane for values of r will pass through the line given by
th e p la n e s
O XX14 £>JJi+ C ZZ1 = I .(2 )
and a x ( X t - X 1) ^ b y (J 1- J 1)-H c z (Z 1- Z 1)= -O
i,e . «MTJTf+b y y t + C zz2= a x x t + b y y ^ c z z 1 -

IJ t„ a x x t + b y y t + C zzt = I. [u s in g ( 2 ) ] . -(3 )
The equations (2) and (3) are the required equations o. the
polar of the line PQ.
Ex. 6. I f P (xlr J 1, Z1) and Q (Jc1, J 1, Z t ) be any two points on
a conkoid axt+ by*+ cz*= I, show that the polar o f the line PQ is
the line o f intersection o f the tangent planes at P and Q. (AM. 1978)
SoL Proceeding as in Ex. 5 above the polar of the line PQ
is the line of intersection of the planes given by the equations (2)
and (3). But (2) and (3) are the equations of the tangent planes
to the conicoid (I) at the points P (X1, J 1, Z1) and Q (Xt, J 1, zt).
Hence the polar of the line PQ is the line of intersection of the
tangent planes at P and Q.
422 Analytical Geometry 3-D

Ex. 7. Find the locus' o f straight lines through a fix e d p u n t


(*, β, γ ) whose polar lines with respect to the cotiicoids ajV+b]p
+cz*=I and V x*+ V y*+ c’z*= I are coplanar.
SoL The equations of the two cbnicoids are
ax*+ byi+ cz*= l ...( I ) , V jV + V j P + V z * = I . ...( 2 )
The equations of any line through the point (z, β , γ ) are
(x —d )!l= (y y),'n—r (say). ...(3)
The equations of the polar line of the line (3) w.r.t. the coni-
coid (I) [See § 9 (C)J are
aax+ bfiy+ cyz= I, alx+ bm y+ cnz= 0. (4)
Also the equations of the polar line of the line (3) w.r.t. the
conicoid (2) are
a'e x + V fiy + V y z= I, a’lx + Vm y+VnZ=O . ...(5)
The two polar lines given by (4) and (5) are given to be
coplanar and hence we have
aa bfi cy I » 0 ( a -V )a (b -b ')fi ( c c ') y C I=O
or

O
r
0
δ.

al bm cn S
Vm V fi c'y I Vu V fi Vj I
Vl Vm c?n 0 Vl Vm Vn I
or (a -«') a (*- -V ) β (c—c') y =0, expanding
determinant
td bm cn along the fourth
column
a’l Vm c'n
or ( a - V ) « (b c '- b ’c) m n + (b -V ) β (<ca’- c ’a) nl
+(c—c') y (ab'—a’b) Im=O.
Dividing by Imn throughout, we get
(a—a’) (be'—Vc) (a//)+(b—V) (ca'—Va) {β/m)
+(c--c') (ab'—a'b) (y/n)=0. —(6)
The locus of the straight line (3) is obtained by eliminating
Umt n between (3) and (6), and therefore, it is given by
ifl- a ') (b c '-b 'c ) [ ~ ^ { b ~ b ' )

+ (C -O ( e b '- e 'b ) ^ ^ ) = 0 .

Ex. I Prore that the lines through (α, β , γ ) at right angles to


C e n tra l C o m co id s 423

their polars with respect to x*/(a+6)+y*/(2fl)+z*/(2ft)=l generate


the cone
(y —β) ( « - ?* )+ (* - r ) (<*y-fix)=0,
Sal. The equation of the given conicoid is
x*/(a+ b) +JftJ(Ta)+z*/(2£) = I.
For convenience Ictus put e '= l/( e + i) , b '= l/(2 a), e '= 1 /(2 b).
Using these values the given equation of the conicoid becomes
o V + iy + d ^ = .! . ...(i)
Now proceeding exactly as in Ex. 4 above the equation of the
required locus is

x —a \ b ‘ c ' J + y —β χά" t f ) ^ z —y \ a ' b’J


or ( 2 a - 7 b ) + ^ (2b —a —b ) + ~ —y (a + b -2 a )= 0
(Potting the values of a ', b ‘, c']
- b ) β (a —b) y ( e - i ) 0 o f 2 « £ _ _ JL e o
or
a y - β z - y x — a

or ( « β V, ( * y \ i—I= Oυ
\ x - m H * —*
y - β ) z~ y /
(e y - β χ ) M Z -y x
or -O
vI
(y -fi)^ (x -* ) fr -r )
< *-« )
or ( y - β ) (ez—γ χ ) + ( ζ — y) (* y - β χ ) = 0 .
§ M. Locns of chords bisected at a given poiat. (PnnJab 1976)
Let the equation of the conicoid be
axt+ bjp+ c& = l. --(I)
Let f z , β , y) be the middle point of a chord of (I) so that its
equations may be given by
(x—a)JI= (y-fi)lm = (z-y)]n= r(s*y), ...(2)
where /, m , n are the actual d.c.’s o f the line.
Now the co-ordinates of any point on (2) at a distance r
from (α, β , y) are (ir - f t, m r+ β , n r+ γ ). Let this point lie on (I)
so that the distances of the points of intersection of the chord (2)
with the conicoid (I) from the point (η, β , γ ) are given by the
quadratic in r*
a (lr + * f + b (m r + fif+ c (n r+ y )* = I
or rr (e/*+ftmH-e»*)+2r (ala+bmfi+crry)
+(aa*+bfit + cy1-1 )= 0 . ...(3)
But («, β , γ) being the middle point of the chord (2), th e
distances of the points of intersection from (*, β, y) should be
4J4 Analytical Geometry 3-D

equal in magnitude but opposite in sign i.e. the sura of the two
-Wtltict of r should be zero and so we should have
. , __ —2 (altt+bmfi+crry) n
r*+ r *------- al’+bm'+cn*-------0
or al*+bmfi+cny—Q. ...(4)
Therefore, the locus of the chord (2) for varying values of
/, m, n is obtained by eliminating /, m, n between (2) and (4) and
is thus given by
act (λ - a)+h? ( y -fi)+ c y ( z ~ y )—0
Or aax+bfiy+ cyz= ao? 4 bfi*+ cy V
of' aajc+bfiy+cyz —\ —aa*+δβ*+ cy*—I
or T= S1 .(5)
where T and Si have their usual meanings.
Property. The section o f the conicoid by the plane (5) is a
conic with its centre at (α, β ,γ ).
We know that the section of a conicoid by a plane is a conic.
Here the given plane (5) is the locus of the chords of the coni­
coid with their middle point as (α, β, y). Therefore, all the
chords of the conic section of the conicoid by the plane (5) and
passing through the point (α, β, y) are bisected at the point
(«, β, y) and hence by definition of centre the centre of the conic
is («, β >y)·
SOLVED EXAMPLES (C)
Ex. I. Find the equation to the plane which cuts 3jc*-|-2y*
—15z*=4 in a conic whose centre is at the point ( —2, 3, —I).
Sol. The equation of the conicoid is
3x* y* 15z*
S= • + 2 - ' -1 = 0
and the centre of the conic is ( - 2 , 3, —I).
We have
3 (-2 )* (3)* 1 5 (-1 )* 9 1_
*1— 4 — I-— 3--------1 = 3+ Λ 2’
and Τ = \ χ { - 2)+*τ ( 3 ) - i | l ( - l ) - - l » - | . x + ^ + ^ - i .
Λ The required equation of the plane is T = Sx
3 3 15
or or 2x - I y - 5 z+ 2 = 0 .
^ H Ans.
GZ· 2# Find the centre of the ■conic
**/9+yV16+z*/4-l, 2 x + 2 y -z = 3 .
Central Conicoids 425

Sol. The given conic is the section of the conicoid


χ*/94-γ*/16+ ζψ = \ ...(I)
by the plane 2x+ 2y—z=3* ...(2)
Let (α, 0, y) be the centre of the given conic. Then the given
conic is the section of the conicoid by the plane tT = S i'
y*
i.e. OU
-Q β± + yz_
+77Γ T · —
a* 0*
-I
~9 16 4 9 +Γ6 +
or ?£ ,§y_ , 2 2 . . £ 1 j . t -
9 + 16 + 4 _ 9 + l 6 i ' 4 -( 3 )
The equations (2) and (3) must represent the same planes and
hence comparing their coefficients, we have
(a/9) (0 /1 6 ) (y/4) . («79)+(0V16)+(y»/4>
2 " 2 = -l 3 —Λ (say).
.;. « = 18 ) t, 0 = 3 2 J fc, y =· — ...(4)
and
9 + 16 + 4 ...(5)
Putting for a, β , y from (4) in (5), we get
36A*+64jfc2-M**=3Jfc or fc=3/l(J4 (since * # 0 ).
Putting for Λ in (4), we get
«=27/52, 0=12/13, y = -3 /2 6 .
The required centre is (27/52, 12/13, —3/26). Ans.
Ex. 3. Prove that the centre of the conic
lx+ m y+ nz= p, axi + byi -,-cz%= I « the point
I Ip mp nP \
WPoi ’ bPoi ' CPot Y
where /2+m 2+ /j2= l and (/*/a)+(m2/h)+(n2/c)= pe*.
Sol. The given conic is the section of the conicoid
ojca+by2-|-cza= l ---(I)
by the plane lx+ m y+ nz= p. ---(2)
Let («, 0, γ) be the centre of the given conic. Then the given
conic is the section of the conicoid by the plane tT = S1'
i.e. ay.x+bfiy+cyz=a3.--{-bfit + cyt. ---(3)
The equations (2) and (3) should represent the same planes
and hence comparing their coefficients, we have
M j 3I = cI - J r f ± b g + c £ = k . ,
I m n p y }>·
:. o.=kl/a, fi=kmjb, y=kn/c ---(4),
and ae?+bfi*-\-cy2=pk
or a.[k2lila2]+b.[kim2lb'1] + c.[k2n2/c2]=pk, using the relations (4)
426 3-D

or Ffm + a^h+ tP [e= gk or « *=*W -


P ottin -Iori m »4), wc get i = ^ 1, P="ViPm*, T=mPfPtt-
Tie required centre b BWfV, BPrV J-
El 4 Ftare Amt A e cemtrr t f A e seetimm t f the eSpsoid
jc“/o*+j*^»+ 2*/r*=I h r Aeplmae ABC whose M ptw m is
xfm + jfh - f-x/e=I is A t cemtrmii o f the U im jte ABC.
SaL Let (x, β Ύγ) be tie centre of the section of the eDipsoid
flV '+ jW + tM t* = I ..( I )
by tie plane ^ + JiL rZ ff= I- -( 2)
Now the coBK sectio· of the ellipsoid (I) having its centre as
the pout (<■,£, j>) is g im by the plane cT = S J
Lr. ^ ^ j_ y ,7*
·* ■ί» V —«* ‘ 1» + e* ...(3)
The (2) aad {3) icpitstai the sae and hence
pc h a re
(■/■*) <#£=> ϋ Α Α + w y H - f r T M
M »y)
(M rt' "(*?*> (l/e)~ I
=<*,* =Uc3T = A • rW
m * i* + IF p P + ft* = k
A* A* A*
+ ^ + k , BSIBe the (4)
3 i* = k or k = i IV
Putting this w i* of L ia (4% wc get i = K i = t t r= k
Λ The ceatic of the section is ( ¾ , $ 6, J cJl ...(S )
Assin the eqnaticn of the plane ABC is given by (2) and so
the vertices of the IriaBgIe ABC mse A im , Ο, Οχ B (0, h , 0) and C
(D , Ol r). Keace the co-ordinates of the centroid of the triangle
ABC are (Je, *A, Jc) which are same as the co-ordinates of the
centre green by (:)- Proved.
Pv S- Prose AtU A e eem ucstf A e seethm s t f m iF+hjP+cz*=!
by A c plmmes w kkh ore mt a camstmmt AsttBKe p fro m A e origin tie
om the om fiK t
( m s * + b ? ± t ( ^ + 6^»+Α«χ
SaL The cqnathm of the green cookotd is
e r f i j !- r ir = l. ...(I)
Let (a. 8*y) be the ceaiie of a plane section of the comcoid
(ΐχ Then the equation of the plane givnig this section of the
cqnicoid is 'T = S J
/ r, max J b fiy +e?= = K T + b fP + cT - ...(2)
Central Camcoids 427

If the plane (2) is at a constant distance p from the origin,


then
:.o*4*A3*+cy*

Λ (®**+ft^*+cy*)*=p*(o*«*+i*^*+cV)-
Hence the locus of (a, 0, y) is the surface
(aj?+by*+cz*r=p9 (u*x*+*V +A *)-
Ex. 4. Shnr that the locus o f the middle pom is o f the chords
o f the canieoid as?+by*-rC 2?= I which passthrough a fix e d porri
(X09y 9 Z0) is ax ( x - x 0)+ by ( y - y ') + c z (Z -Z 0)=O.
(LaCknaw 1979, 82 ; Kaapm 79 ; IlKIwM 81)
SoL The equations of the given conicoid is
ax*+byi + cz* = \. —(I)
Let (a, 0 9 7) be the middle point of a chord of the corac oid
(I). Then this chord lies in the plane tT = S 19 Le.9
a o x+ b p y + c y z —I=aa*+bfP+cy*— I
or aax-\-bpy+cyz=aa*-\-bfi*+cy*. —( 2)
Now if the chords of the conicoid (I) having (ο, β 9 τ) as then-
middle point pass through the fixed point (X09J f9 z^, then the
point (X09 / , Z0) should lie on the plane (2).' Therefore, we have
a*x0+ b $ y'+ c/ ζ '= ao2+ b p * + cy
or asL ( * - x 0)+ b p (fi—y ')+ c y (y—zT)=0.
Hence the equation of the required locus of the middle point
(α, β 9γ ) of the chords of the conicoid (I) which pass through the
fixed point ( x \ / , z ) is
a x ( x —x 0)+ by (y —y ')+ c z (Z -Z 0)=Om
Ex. 7. Prove that the section o f the ellipsoid
x7 fl*+y*/h*+z*/c*=l
whose centre is at the point (in, £6, £c) passes through the extrem i­
ties o f the axes . (Bahflhhaad M I)
Sol. The equation of the given ellipsoid is
s ^ /a ^ + y /ip + z - ic 2- 1 = o
-- The centre of the section is (Ja, \b 9 £c) and hence the equa­
tion of the plane giving this section is 4T=S/
X jfl V j* Z jc ,
*· · a* + b* + c* l~ «» + -& r + ~*— 1
or xja+ yfb-t- z;c= I. _„(lj
The extremities of the axes of the given ellipsoid are (ο, 0, 0>,
(0, b, 0) and (0, 0, c) and the plane (I) clearly passes throagh these
three points. Hence the section whose centre is at the pnim
428 Amilytical Geometry 3-D

Htt9 ib 9 if) passes through the extremities of the axes.


Ex. 8. Prove that a line joining a point P to the centre o f a
conicoid passes through the centre o f the section o f the conicoid by
the polar plane o f P.
Sol. Let the equation of the conicoid be
ax2+by2+cz2= I. --0 )
Let the co-ordinates of the point P be zf) so that the
polar plane of P ( jc\ y \ z9) w.r.t. the conicoid (I) is
axx'+ byy9+ czz9= I. —(¾
Let (a, /?, γ) be the centre of the section of the conicoid (I) by
the plane (2) so that the equation of the plane giving this section
is iT= Sl9
Le., axn+byfi+czy — I =a&2+bfi2+cy*— I
or a&x+bpy+cyz^acP+bfP+cy2. -.(3)
The equations (2) and (3) represent the same planes and hence
comparing their coefficients, we get
a« β _cy a β γ
a x '^ b y '^ c ? OT x 9=/ z=z r ---(4)
Againthe centre of the conicoid (I) is (0, 0, 0) so that the
equations of the line joining P (X99y ', z9) and (0, 0, 0) are
jc—0 y —0 z —0 ^ x y z
X ^ b ^ y ' - 0 = z’- 0 ° r x ,= y ^ z ~ ' ...(5)
ff the line (5j is to pass through the point (α, β> y) then the
co-ordinates of the point (α, β , γ) should satisfy (5) and so we
should have —, I j r which is true by virtue of (4). Hence
JC y
proved.
Ex. 9. Triads of tangent planes at right angles are drawn to the
ellipsoid X2Ia2+ y2/b2+ z2/c2= I. Show that the locus o f the centre o f
section of the surface by the plane through the points o f contact is
(*2+>’2+ ζ 2)=<α2+ * Η c2) (x*/fl2+ y*lb*+ Z2itC*)2.
Sol. The equation of the given ellipsoid is
x2la2+y2lb2+ z2/c2= I. ...(I)
Let (*', y 9, z') be the co-ordinates of the point from which
three tangent planes at right angles are drawn to the given ellip­
soid (I). Then by definition of the director sphere the point
(*', y 9, zf) will lie on the director sphere of the ellipsoid (I) Le< on
x*+y2+ z2=a2+b*+c*. [See § 7]
Λ x '2+ y '2+ z ’2= (fi+ b2+<*. (2)
CentraI Comeoids 429

Now the plane through the points of contact is the plane of


contact of the point ( x ',y ',z ') with respect to the' ellipsoidal)
and its equation is given by
x'x lJ + y 'y /P + z'zIc *= I. .. .(3)
Again let (α, β, γ ) be the centre of the section of the ellipsoid
(I) by the plane (3), so that the equation of the plane giving this
section is T = S ,’
Le. axloP+fiylP+yz/c· - 1=a*lat +fi*/b*+yi!ct— I
or axla*-\-fiylbi + yz/ct =a.ilat + fltl^ + y tlci. ...(4)
The equations (3) and (4) represent the same plane and hence
comparing their coefficients, we have
Xr v' z' I
« “ β - γ -(« * Ι°* )+ Φ ΙΡ )+ (γ* Ι< ? ) -( 5 )
Putting the values of x ’, y ’, z' from (5) in (2), we get
**+fit + f _____ t 4 bz , 2
[(*;<*)+(β*ΙΡ)+(γ*!<*)]* +
Λ The locus of the centre (α, β, γ) is given by
(χ1+ ?*+ *1) = ^ + ^ + C2M(X1Zai) H y1Ib-1)+(ZtIct)]*.
Ex. 10. Show that the centres o f the sections o f a central
eonicoid that are (i) parallel to a given line lie on a fixed plane and
(ii) that pass through a given line He on a conic.
Sol. Let the equation of the central eonicoid be
ax*+by*+cz·= I. - ( 1)
Let (*, β. y) be the centre of a section of the given central
eonicoid (I). Then the equation of the plane giving this section
is T - S 1*
ι-c., aax+bfiy -f-cyz— l= aat +bfii +cy2 - I
or aax+bfiy-\-CYZ=axi +bfi*+cyi. ■ -(2)
Again let the equations of a given line be
(X X t)Il= (Y -Y i)Im = (Z -Z i)In- 0)
(i) . If the plane (2) is parallel to the line (3) then we have
(aa).l-\-(bfi).m+(cy).n—0.
The locus of the centre (a, β, γ) is
alx+ bmy+ enz= 0
which is the equation of a fixed plane since a, b, c, I, m, n are all
constants being given numbers.
(ii) . If the plane (2) passes through the line (3), we have
Oixv^bfiyl -\-cyzt = acf+bfi* cy*,
and (tn).l +- (bfi).m -|-(cy).n= 0.
430 Anafyiiad Geometry 3-D

Λ Tbe locus of (a, /?, 7 ) is the curve of intersection of the

aaP+typ+cz*—axxx - by^y-C z lZ=0 I


and atx+ bm y-\-cnz= 0 J -W
Tbe two equations given by (4) together represent a conic.
Hence tbe locus of the centre is a conic.
Es. 11. F m dtkelacus o f the centre o f the section o f the com-
addaaP+byP+c 2* = l which touches A jp + B jr + C r — L·
(Mains IfTI; Alahabni 74)
SaL Let (*, f ty ) be the co-ordinates of the centre of a
plane section of the conicoid as?+ bjp+ cz*= I.
Then the equation of the plane giving this section is
Le, aax+ b fiy+ cyz - 1 1
or aax-\-bP y+ cjz= aat*+ bfP + ef. —( 1)
NonrifthepIane(I) touches the conicoid itf+ V + ® *1=!»
Acn w c have
^ + ^ + (^ = (« * + W + c y V - [using § 6(A)]
Λ Tfce locos of (α, β , /) is
A*M + 6V/«+«*z*/C=(a**+i!rs+a*)'.
Bk. 12. Prowe that the noddle points o f the chords o f
ajp-\-bjp+ cz?= I
o d a eh a n p a ro M to x =0 and touch x*+y*4-z* Iieo u th e surfi.ee
h p tk* + b ? + ez* --b r* )± cz* ic3 i* + b i* + cz* cr*)=0.
(Kaapv 1912)
SaL The equation of the conicoid is
«*+Ay*+C2*--1. ..(I)
Let (q, β, γ ) be the middle point of a chord. The equations
of any straight line passing through the point (ο, β , γ ) and para­
llel to the plane x=0 (Le. YO Z plane) are given by
(x «)/O=ij-/0/m=(z-7)/ii=A (say), .. (2)
where 0, m, *vare the actual dx.’s of the line.
Any point on the line v(2) at a distance Λ from (c, β, γ) is
(*» n l+ A Λ+Τ). Ifit lies on the conicoid (S) then the distances
lo llh e pointsof intersection of (I) and (2) from (■, I, 7) are

aaP+b Im X + fif+ c (11A+7)*= I


«■ I? tbaP+cn*)+ 2A (b m filc n y) \a z * + b ? + c f— 1)=0.
S tace (x. fim7 ) is th e m id d le p o in t o f th e c h o rd in tercep ted by
Central 431

(2) on (I), therefore the sun of the roots {*>- of the above
equation should be zero and for it ire have
-Η*7 =θ or W ·)= - { c j i t f t . —PI
Now according to the given question if the chord 0 ) toadies
the sphere x*+jr+ 2*-r*t then we fare
the length of the perpendicular Arm the centre (CI9O9O) of the
sphere to Ac line (2 ) = ik radius r of the sphere
U. I °/ f+ | ° e* u °_ ' [ .H

or (my—flw}»+A F -H A ^= T e ( * * + ^
i v α*-Η**-ί-··=ΐ]
o im j jtm P-H "? Tn*| *?=** (**+**)
or (-r*+**) -?***=«
or ( -»*+**> J5
or t—
<-'·+**> {,+»£Μ **ίτ 'Γ- ®—*■·«
or (-* * + * ¾ w r - H V i - K i r + n v = · ·
The Iopb of (■ , /I, 7) is
(-1 --+jr*) (**r+A *)+?£y*+r?*J*=0
or -r V / A V + ^ j y + A V + r y + r z · f2 f c F ¥ = 0
or b jr i t o P + t f + c z · — f c ^ + c z * ί β ^ Η Η Ι ^ + β 5 - o r * ) = € L
py.
§11. N a B d to a C W M i.
(A) 7o /arf rfce qaainiB of lt f D rw f t» a UJtlral camira iJ Hf
the point (a, β , /). (Agra UQ; t i l i W · ! » , IHfcai 7·)
Let the cqnatioa of the central ooweoid be
e * + ^ * + e z * = l. —(11
The equation of the taagrm obne at ( c f , 7) of the cooicokl
(I) is
β χ χ + A flf T e y r = I - - .( 2 )

The MHatal to the comcoid (I) at (a, ft, 7) k Ac straight Bac


perpendicnhr to the tangent plane (2)and passing through
(α, β , 7) andhence the required cqnalians of Acnormal are
given by
( 1 - ^ · ) - - IF— r*A*y) --13)
where dz, bf^ c j are the direction radios o f the novnal (3).
Now let p be the length of the perpendicular from the origin
to the tangent phiac (2) so that
432 Analytical Geometry 3-D

P = V I a V + b*fi*+ c2y2j ΟΓ A· * )
In view of (4) the actual direction cosines of the normal (3)
are a%p9 b$p9 cyp and hence the equations (3) of the normal to the
conicoid (I) at (a, 3, γ) in terms of actual direction cosines are
given by
*—q y - β ^ ζ - γ
cap αβρ ~~ayp .- (5)
(B) To find the equations of the normal to the ellipsoid
x2/a2+ y2lb2+ z2/c**=l at the point (α, β9 γ).
(Kanpur 1980; Locknow 81; Kornshetra 73)
Proceeding exactly as in § 11 (A), the equatipns of the normal
to the given ellipsoid at (α, β 9γ) are
χ — λ y — β ζ — γ

apia1 ~ βρ/b2 ~ yp lcr ...0 )


I α2 β
and a — j -T-rv~
^Γ=^Γ+ΛΓ-;%Γ
p2 a4 ^ b 4 c* and
where X pla29 ^ p lb 29 y p \ c 2 are direction cosines
X- QL y —β ζ —γ
or
OLla2 ~ ^ b 2 ” y/c2 9
where aja2y fi/b29 γ /c2 are direction .ratios.
SOLVED EXAMPLES (D)
Ex. I. The normal at P(α, β 9 γ) o f a central conicoid meets the
three principal planes at G19 Gt, G3 ; show that PG19 PGt9 PGt are in
a constant ratio. (Agra 1980; Meerut 84S; Kanpur 78)
Again if PG12+ PG22+ PGt2=Ic29 then find the locus of P.
(Rohilkhand 1981)
Sol. Let the equation of the central conicoid be
ax2+by2+cz2= I.
T heequationso fth en o rm alto th e conicoid (I) at the point
(α, β, γ) are
x-cc y - β z —y ^
aap bfip cyp -.(2)
wherep = \j^/(a2oi2+b2fi2+c2y2) and where r denotes the actual
distance of any point on the normal (2) from P (α, β 9 γ).
We know that the principal planes of the central conicoid (I)
are the co-ordinate planes. The normal (2) meets the plane x = 0
in G1. So we have r=PGv
Λ from (2), °— =PGt or PGt= - J /(ap).
Central Conicolds 433

Similarly if the normal (2) meets the plane y = 0 and Z= O in


the points G9 and G9 respectively, we have
PGt ==-1/( bp) and PG9=3— I Hep).
:. PG1: PG9 : PG9= - 1l(ap) : - 1l(bp) : - 1Hep)
=HIa ) : (1/6): (I/c)
which are in a constant ratio since a, b, c are fixed for the given
conicoid.
Second Part.
Now it is given that
PGt*+PG91+PG9'·«=»*·

B H -W +B )-*
w j» (? + P + ? )"* ·
or ( α Φ + Ρ /Ρ + c V ) ( I/o*+ 1/6*+ 1/«*)=**
[V l/p*=a,<x, +6*/i*+cV>
.'. the point P(x, β, y) also lies on the surface
( a V + f c y + Λ » ) ( I/a* + 1/6*+ I /c*)= &*. -(3)
But the point P(x, β, y) lies on the given central conicoid (I)
and, therefore, the locus of P is the curve of intersection of the
conicoid (I) and the surface (3).
Ex. 2. Find the distance o f the points o f Intersection o f the
normal at P(x, β, y) to a central conicoid with the co-ordinate planes.
Sol. It is the same as the first part of Ex. I above.
Particular case when the given conicoid is an ellipsoid. Let the
central conicoid be, the ellipsoid
x*Ia*+y*Ibt ^ z tIct = I. --(I)
The equations of the normal to (I) at P(a, β, y) are
X *^ y - β _ z - y
apia» βρ/b* yplc* { y) ...(2)
where l/p*=«*/fl1+/3,/64+y*/c4 and where r denotes the actual
distance of any point on the normal (2) from P(«, β, y). If the
normal (2) meets the plane x = 0 in Gi then putting r=PGt in (2),
we get
,re ,— ·/*
Similarly if the normal (2) meets the co-ordinate planes y = 0
and z= 0 in G9 and G9 respectively then
PG9= —b*lp and PG9= -C tIp.
434 Analytical Geometry 3-D

Ex. 3. I f Q Is any point on the normal at P to the ellipsoid


x*la*+y*/b*+z*/c*<=l such that 3PQ=PG1+PGi +PGi where Gi,
Gu Gs are the points where the normal at P meets the co-ordinate
planes respectively, then the locus o f the point Q Is
Qi Xt ■ b*y* , c‘z* _ I
(2a*-b*-c*)* "^(2h '- c '- a * ) * * (ic*—a*—b*)* 9
(Kanpur 1977; Bnndelkhand 79)
Sol. The equation of the ellipsoid is
xV a * + y W + z* lc 2= l . ...(I)
Let the co-ordinates of a point P on (I) be («, β, y) so that
the equations of the normal at P («, β, y) are
* -« = y -0 _ r - y
p&fa* ρβ/b* py/c* ' ...(2)
where L = i ‘+ ^ + ^ .
p* a* b* c* ...(3)
Let Q be the point (Jc1, ylt 7,). Since Q lies on (2), we have
J f i- « _ j,,- j3 _ 7 ,- y
pafa* ρβ/b* py/c* = r '
A Χι=*+(ρ*ήα*), y ^ f i + ^ r l b * ) , Z1=Y-Hpyrlc*). ...(4)
Now P(ci, β, y) lies on the ellipsoid (I) so that
«'/e*+j8Vh*+y*/c2= l -(5 )
Again it is given that 3PQ=PGi+PGt + PG».

= P P P [V and putting
the values of PGi etc. as
found in Ex. 2 above]
or r>= —(a8-f-b*+c*)/(3p).
Putting this value of r in (4), we get
a ( A 8- K b i -I-C i ) 12a * -b * -c* \
- 3a8----- = a ( 3a — ) ’
«_ 3ax,
so that
a ~ 2 a *-b *-c * ’
y __
Similarly β,
3b y, 3cz,
b 2 b 8 — c 8 — a8’ c ° 2 c 8 — A1—b2*
Putting these values in (3), we get
9a8JC,8 , 9b8y,8 . 9c87,8
( 2 A * - b , , - c 1,) 1 , + ( 2 b * - c 1,- A , ) 8 "t ’ ( 2 c 8 - A 8 - b i ) i

.·. the locus of Q (x,, ylt Z1) is


435

A* . **p* , A e _1_
(2 ^ -4 ^ -0 ^ ^ (2 * * - e*-«*j* + (2 e ·-··-* ·)* 9 Unmet
Ex. 4. Wimitke Iemgtkef tkemarmot ekard ikramgk Pef tke
ellipsoid χ*/«*+ρ*/»*+2*/«*= I amdpme Aatff it is efmol to 4PG*
where Gt is Ike potmt wkere tke mmmal ckord Jhramgk P meets tke
piame z=0, themP lies amtke came
x* (2c*-e*>K +J* (2c*-**>/ft*+a*A^=0.
SxL The equation o f the effipsoid is
J*/·*+**/*5+ **/**=!- --4*)
Let the co-ordinates of · point P oa (I) be (c, β. y \ so that
•*/«•+/»■/*»+,*/«*=1. <2)
The equations o f the n o n a l to the cflipsoid (I) at P (ο, ft. r)
are

p*A*· #»** PtKm i Jh -(3)

where
p* a* ** c* (4)
The co-ordinates o f any point g (say) oa the Bbreal (3) are
« + (p u r/A ftHpflr/i^L r+tpyrft·) where r*=PQ-
VPQ n the nonaal chord of the ellipsoid (I) Ibea Q will lie
on (I) and so we get

or
** (?+£ +?)+M ?+£ +$M ii+S+? - 1H
or r ·' ■ * * « -< « )
__ —2
or
r e P* I**l+ )+ iF IIP i+ tfn ·)}
**PQ, the length of the noresal chord. ...(5)
Now if PQ=APG, (as given) then PQ= —4c?Ip
(See Ex. 2 above for the value of PGH
or —2 4e*

[Patting the value of PQ from (5)1


or
* ■ <4 >

or
436 A n a fy tfe a i G eo m etry 3-D

Λ Tbe locus of P (λ , fi, y) Is


*■ (2c*-e*>/a·+^ (2c*-ft*)/6*+z*/c« =0. which is a cone.
Ex. 5. i f a length PQ be taken on the normal at any point P o f
the ellipsoid XtIait +PtIbt +z*/c*= I such that PQ=XtIp where X is
constant and p is the length o f the perpendicular from the origin to
the tangent plane at P, the locus o f Q is
e*x* IPyt A*
(fl*+AV+(6* +AV+ (c*+A*>*
Sul. The equation of Ihe given ellipsoid is
x*/e*+y/6»+z*/c*=l. . ( 1)
Let the co-ordinates of a point P on (I) be (α, β, y), so that
οΡ/βΡ+βΡ/ΙΡ Y y1Ict — I. ..-(2)
The equations of the normal to tbe ellipsoid (I) at P (α, β, y)
are
* -« y - β - z~y - r fsav)
paid» Pfilbt Pylct {yU -(3)
where p u /a \ pfijtP, pyftP are the actual d.c.’s of the normal.
Let (xi, y ,. Z|) be the co-ordinates of a point Q on the normal
(3) such that P Q - XtIp (as given).
Then putting x=x,. y y t , z=z, and T=XtIp in (3), we get
, αλ1 . , fiX* , yX*
*· a* ♦ 3'i“= P + t zie y + ^ ■
e*jri IP Vi _ Ai
Λ *'
e*+A** P 6»+A*· y c*+A*
Putting these values of (a. fi, y; in (2), we get
A i* , A l* . A 1* .
(fl*+A*f (A*+A*)* <c*+A*j*
The locus of the point Q (X1, y u Z1) is given by
Ot X t j_ IPyt Ct Zt
(n*+ A*)* (6*+A*)1 (c*+A*)* '·Proved.
Ex. 6. The normal at a variable point P o f tbe ellipsoid
jt*/a*+y*/6*+z*/c*=l m eets the plane z - 0 (t.e. the x y plane) in <7*
and G*Q is drawn parallel to z-axis and equal to G»P. Show that
the locus c f Q is given by
x*/(e*—c*)+3^/(6*—c*)+z*/e*= I.
Also fin d the locus o f R i f OR is drawn from the centre equal
and parallel to G3P.
Sol. The equation of the ellipsoid is
χ*{(Ρ - T y t I I P + Z 2I t P = I . -(1)
Central ConktMs 437

Let (*, β, γ ) be the co-ordinate* of a point P on the ellipsoid


(I), so that
« • /« •+ W + y ·/* * = » . —(2)
The equations of the normal to the ellipsoid (I) at P («, β , γ)
are
= . (say)
p o l* ρβ/b * p y l* ’ ( y)’ ...(3)
where p o l* , pfif4*, py/c* are the actual d.c.’s of the normal so that
r denotes the actual distance of any point on the normal (3) from
the point P («, β . γ). If the normal (3) meets the plane z= 0 in
G9 then putting z—0 and r= PG a in (3), we get
O-Y
-PGa, so that PGta —-—
PYtc
Putting r=/Gs= —c*/p in (3), the co-ordinates of Cs are
(<x-c**/a\ β ~ * β Ι * , 0).
Now Cs is a point in the xy-plane and GaQ is a- line drawn
parallel to z-axis such that GaQ=GaPc=—CtIp. If z,) are
the co-ordinates of Q, then
Xi=x-co-ordinate of Cs= * - (c**/a*>,
y x=J^co-Ordinate of Ga- β —(c*/l/A*),
end Z1=GaQ =-CPIp.
IPy1 c
a —· * x , . β -
tP -c* ‘b*— C1 P -(4)
From Zt Iea -C lp , weghave
V Ci Zi*
C1 Pf or ?
[Putting for 1/p* from § 11 (B)]
or y* _ v
Ca c* 1 -* + *^ 1y ...(5)
Putting the values of «, β and y from (4) and (5) in (2),
we get
fpyt*
W .,/*»» ^ / * 1 , P U - .1
(a· - c2)* (P-
• - O ’ lc* ))
or AlljcI1
,· . b*yx* Z1* <*xt* _ c*>i*
(«*-
‘ -c * ) * (* * -« * )» ^ c * (O1 -C * )* (b * — c*)*
[Again using (4) for * and β \
xt* (a*-c*) y,> OP- * ) , Z1*
or (a*—c1)* (b1—c*)* "r C* “
*»» ΡΓ
or o*-c* ’ b»-c* l’-c*
5 r» l.
43S Analytical Geometry 3-D

. The locos of Q (x,, y ,. z,) is

Proved.
Seeooi part. The equations of the line Q R passing through
O (0. 0, 0) and drawn parallel to the normal at P are
x _ J _ z
= k (say).
CWe*) Iρβΐ**) Ipylct)
Taking 4= O R=G 3P - -C tIp. the co-ordinates of R are

Also let the co-ordinates of A be (x*, y*, z*L so that we have


Xs=-OC2,a2, y»= —fit^jtP, Zt = - γ
or « =— **■
Pnttiog these values of *, β, y in (2), we get
n * x ,V e * + W /c ·+ ^ /^ = !-
The locus of R (x», y%, Zt ) is Ρι*+Ρ^+Λ*=ί*· Aas.
Ex. 7. The normals at P <md F1, points o f the ellipsoid
tPjtP +**/£*= 1» m eet the plane z —0 in Gt and C t and make
aqglrr 0, & with P P '. Show that
PGt eos B^-PwGwt cos Bt=O. (Agra i n i )
Sal. Let (ο, β. γ) and (*', β", y ’) be the co-ordinates of the
points P and P' respectively. Now proceeding exactly as in Ex. 2
above, we have
PGt----- CtIp. F G t = - C tIp',
where p and p ' have their meanings as given in § 11 .
The d.c.’s of the normal at P are p a l·1, pfiJIP, pylt? and the
d c.’s of the normal at P ' arc pV/a*. ρ'β/ΙΡ , p 'y'h ^- The direction
ratios of PPr are β ' —β, y —y and so the d.c.’s of PPr are
(a -* )J P P ', \β ’ —βίΙΡ Ρ \ iy - yVPP ’.
Since 9 is the angle between the normal at P and the line P F ,
we have
Central Contcoids 439

c* [ax' β β ', γ γ ' I


— PP ^ ^
since P (α, β, γ) lies on the given ellipsoid.

Similarly/>'<!·, cos [ ' - ( ^ + J T + ' t )]·

·’· ρ 'σ · ™ ° ' - Μ £ +β£ + £ ) - ' ]

C=-PGi cos Θ
or PGi cos θ+P'G’t cos ¢ '= 0 . Proveo.
Ex. 8. I f the normals at P and Q, points on the ellipsoid
XtIat -^ytIbt +z*/c*=l> intersect then prove that PQ is at right angles
to Its polar with respect to the ellipsoid.
Sol. The equation of the given ellipsoid is
x 2/a* -^y2Ibt +ZiIct = I. ...(I)
Let the co-ordinates of the points P and Q on (I) be
(«i. fiu yi) and (a*, fit, y») respectively.
The equations of the normals to the ellipsoid (I) at the points
P and Q are respectively given by
yi
«i Iat β,/b2 Yilct -(2 )
x - * i _ y - ^ i_ z - Y i
atf a2 β%Ib2 YiIc2 -0 )
If the normals (2) and (3) intersect, then we have
« ι-« * fii-fi* Y t-Y t

a t Ia* f i l l b* Y ilc t

Xi I a t fit/b \ Y tlc t

+ < » -" » ( s ^ g P fc)-* - ,4,


Now the equations of the line PQ are
* - « i _ y -j3 1 ^ z - Y i
« i—a* f i t - f i t y i — Yi --(5 )
The polar of PQ i e. the polar of the line (5) [See § 9 (C)]
with respect to the ellipsoid (I) is given by
^iX/^+fity/b* + yjz/c*—1=0 ...(6)
440 Analytical Geometry 3-D

and (« ,-« ,) (χΙα')-Η βι-β*) (*/«')--0.


Subtracting (7) from (6), we get

orb% Ct ...(8)
Let Iu mu /I1 be the d.c.’s of the polar line of PQ t-e. of the
line of intersection of the plane (6) and (8). Then we have
h — mi — "t
βι y»—ft»n *ιβ»—
b*c* c*a* a*b*
Therefore, if the line PQ [given by (5)] and its polar [whose
d.c/s are given by (9)] are to be perpendicular then applying the
condition for perpendicularity, we have

<*. - « .) ( rjW ii)

+ ( « —>■·) ( 11¾ . ! ^ 1) - 0
which is true by virtue of (4). Hence the result.
§ 12. Number of normals.
To prove that six normals can be drawn to an ellipsoid from a
given point (xj, y u Zi).
(Allahabad 1975, 78; Lucknow 71, 74, 77; Madras 76;
Meerut 86; Rohilkband 78; Punjab 72)
Let the equation of the ellipsoid be
x*lo*-\-y*/b*+z*lc*“ l. ...(I)
The equations of the normal to the ellipsoid (I) at P (α, β, y)
are
x —a _ y —β _ 2—y
λ (say).
«/α* β jb* γ /c* ( 2)
If the normal (2) passes through the given point (xu y u zt),
then this point will satisfy (2) aud hence we have
Xx~a _ y i— λ.
ala* β/b* γ /c*
γλ
Λ - 0 + fr. y+ Ct

or αχi β _ byx >1« « ι_ β


β β*+Λ* b Z>a4-A* c ca-j-A ...(3)
But the point P (α, β9 γ) lies on the ellipsoid (I).
A aVfl*+j3*/6*+rVc2= l
Central Conicotds 441

or . b*yi* . Λ ,*
(a*+A)* + (b*+A)* + tc»+A)* “ ...(4)
[Putting the values from (3)]
This equation is of sixth degree in A and hence gives six
values of A. Puttiug these six values of A one by one in (3), we
get six values of α, β, y i e. we get six points on the ellipsoid (I)
the normals at which pass through a given point (Xi,yi, Zi).
Hencefrom a given point six normals can be drawn to an ellipsoid.
Corollary. Proceeding exactly as above, we can prove that
from a given point (Xl t ^1, Z1) six normals can be drawn to any
central conicoid ex*+by*+«·*=»I.
§ 13. Cubic curve through the feet of the normals.
To prove that the six feet of normals (drawn from a given point
to an ellipsoid) are the intersection o f a cubic curve with the given
ellipsoid.
Let the ,equation of the ellipsoid be
x*/<J, +y*/^i +z*/c*ea 1· ···(!)
Let ( X j , y„ Z1) be the given point. Now if the normal at
P { a , /3, y) to the ellipsoid (I) passes through (Xj, ylf Z1) then
proceeding as in § 12, we get
_ _ P Xi_ o P yi __
C1Z1
e*+A * bb*'V+ kA'· yr ~c*+ A"
_ c*-f A ...(2)
Now consider the curve whose parametric equations are
3, r j α*χ ι V J Pyi - , Pzl
fl*+A ’ y h*-|-A’ c*+A ...(3)
where Ais a parameter. We have proved in§12 that A has six
yhlues [given by equation (4) of § 12] and hence corresponding to
ieach of these six values of A, we shall get a point on the curve (3).
Again consider a plane A x+ B y+Cz+Dc=O. ...(4)
The points of intersection of the curve (3) with the plane (4)
are given by

.:.(5)
This equation being a cubic in Λ gives three values of λ and
shows that the curve (3) meets the plane (4) in three points and
hence the curve (3) is a cubic curve.
Hence we conclude that the six feet of the normals that are
442 Analytical Geometry 3-D

d ra w n fro m a g iv e n p o in t {xu z,) to th e e llip s o id ( I ) a re th e


in te rs e c tio n o f th e e llip s o id ( I ) w it h th e c u b ic c u rv e (3 ).
§ 4 . To f in d the equation o f the cone through s ix concurrent
norm als ( th e s ix norm als drawn fro m a p o in t to an ellipsoid).
(L u c k n o w 1 9 7 1 ; 8 1 ; R o h ilk h a n d 7 9 ; M e e ru t 8 4 )
L e t th e e q u a tio n o f th e e llip s o id be
XtIat -^yt Iba+ z * / c * = l . ...(I)
L e t (X 1, > u Z1) be a n y p o in t ( n o t o n th e e llip s o id ) . W e k n o w
th a t f r o m ( x 1( yit Z1) six n o rm a ls c a n be d ra w n to th e e llip s o id ( I )
[S ee § 1 2 ] .
N o w th e e q u a tio n s o f a n y lin e th r o u g h th e p o in t (X 1, yu Z i)
a re
x — x, y -y t z ,-z ,
/ 7 " = n ...(2)
I f th e lin e (2 ) is th e n o rm a ! to th e e llip s o id ( I ) a t th e p o in t
(α , β , y ) , th e n w e h a v e
l= p x j a 2, m = P fiIbt , O = P y lc t . ... (3 )
N o w i f th e n o rm a l a t /*(<*, fi, y ) t o th e e llip s o id ( I ) passes
th r o u g h ( x i, > i, Z1) th e n p ro c e e d in g as in § 12, w e g e t
= fl_ b% yt CtZl
a * + A ' P" b * + \ ’ y “ c » + A ...( 4 )
P u ttin g th e va lu e s o f α, β, γ fr o m (4 ) in (3 ), w e g e t
IJ L °*XL· m P b*yi n - p - 2'
Ut ' ο*+λ "V h*+ A ’ ~ c*‘ Ct + A
or Ρ1'α α * + λ , ? £ = Ρ + λ , ^ = * + λ .
i m n
F ro m th e a b o v e re la tio n s , w e see th a t

P lU b t - C t) + ^ - 1 (Ct -O t) + ^ ( a * - ? )
= ( « * + * ) ((>*— c * )+ (/> 2+ A ) ( c * _ o 2) + ( c * + A ) ( f l * - 0 ‘ )
«=0. ...(5)
E lim in a tin g I, m , n b e tw e e n (2 ) a n d (5 ), th e e q u a tio n o f th e
c o n e o n w h ic h th e n o rm a l (2 ) lie s is g iv e n b y ,
p x t (b 2+ 2 c 2) Pyl (Ct -Q t) DZ1 (as —b%)
x —x, y —y , ' +
+ Z -Z 1 ~
Λ, x< Ib2- C t) , y, ( C 2 - Q i ) , Z1 (a2—b2) Λ
·*-*. + y-y i + 2- Zi ...(6)
T h is is th e re q u ire d e q u a tio n o f th e c o n e o o w h ic h lie a ll th e
s ix n o rm a ls d ra w n fro m (X1, yt, Z1) to th e e llip s o id ( I ) .
Omiral Omicoids 443

Corollary. To prove that the curve through the six feet of the
normals drawn from a point {xu y u zi) 1° tin ellipsoid lies on the
cone through the six concurrent normals.
The parametric equations of the cubic curve through the feet
of six normals are given by [See § 13]
A r1 v- » y ±
β*+λ· y ~ p + \ ' ...(7)
where λ is a parameter.
If this curve is to lie on the cone (6) then the values of x %y, z
from (7) should satisfy the equation (6). Putting the values of
x 9y9 z from (7) in (6), we get
Σ (α2+λ) (fc2- c 2)= 0

or (e*+A) (A*-c*)+(**+λ) (c2-fl*)+(c*+A) (a*-fr*)~0


or O=Ov which is true. Hence the required result is proved.
SOLVED EXAMPLES (E)
Ex. I. Prove that the feet of the six normals drawn to the
ellipsoid X9Iag-^y9Ib*+z?/c9= I from any point (xu y u Z1) lie on the ,
curve o f intersection o f the ellipsoid and the cone
a1 (A9^ c t) X 1 b* ( c 9 — a9y yx c 9 {a9—b2) ζ Λ j
x ' ^y z “ ·
[Rohilkhand 1980; Madras 77; Gurunanakdev 75;
Lucknow 74; Meerut 85, 89, 90 (P)]
Sol. The equation of the ellipsoid is
X 9I a 9 + y 9l b l + z 9 l c 2 *=* I ...(I)
Proceed as in § 12v the co-ordinates (α, β 9 γ ) of the six feet
of six normals drawn to the ellipsoid (I) from the given point
(¾. yu Zi) are given by
A r i g. b - y i C9 Z 1 . . . . .
e = - r — , /3=—,—. Y - - T - where λ is a parameter and is
e*+A r b*+A ' C1-I-A
given by an equation of sixth degree.
Solving each of the above relations for Λ, we get

* P Y
Now we clearly have
(--* * ) {c-- a * ) + c*) (a2-A J)
=A ( b * - c sf + X (C t - O i ) + A (ο*-**)=,0
444 Analytical Geometry 3-D

O9X i (If1 - C t ) b 9y i ( c 9 — q9) C9Z 1 (g*—b*)


or « H β + — Y
=e* (**-c*)+6* (c*-a*)+c* (a·-* ·)
= 0.
The six feet ία, β, y) of the normals to (I) drawn from
(*i, yu z.) He on the surface
a * X \ Cha-C 1) b*yx ( C t - O t ) . c*zt ( a*—b* ) n
* + y + z ---(2 )
The equation (2) is a homogeneous equation of second degree
and hence it represents a cone. But since the six feet (α, β , γ ) of
these normals lie on the given ellipsoid (I), therefore they Iieon
the curve of intersection of the ellipsoid (I) and the cone (2).
Ex. 2. I f A, B, C; A', B', C are the feet o f the six normals
from a given point to the ellipsoid x*/a*+.y*/(,* + 2Vc*= l aru^ t^e
plane ABC Is given by lx-\-my+nz=>p, prove that the plane A' IP C
Is given by (xlatl)+(ylbtm)+(zlctn)-\-(\lp)—0. (F n ja b 1977J
Sol. The equation of the given ellipsoid is
x*la*+y%lb*+ZtJct = I ... (I)
Let the co-ordinates of the given point be (Xi, y u zi)· Now
the co-ordinates (α, β, γ) of the feet of six normals irom (*i, y u zi)
to (I) are given by^
α = ο*Χι/(ο*+λ), β'=‘b^y^|(bt +λ), y=e*Zi/(c*+A) ...(2)
where Λ is a parameter and its six values are given by the
equation
Ott X t * b*yt* CtZt3 ,
(a* + A)2 + (b2+A)2 + (c2-|-A)2 ~ ...(3)
Now the equation of the plane ABC is given to be
Ix+my \ - n z = - - p
and three of the six feet of normals given by (2) lie on this plane,
so that we have
Iai X l m b 2y i n c zz t ft
aa+ A +^+ A + c*i-A P ...(4)
rI hisbeingacubicequationiuA gives us three values ot A.
Again let the equation of the plane Af Bt Ct be
ϊ χ + m 'y + r iz - p '^ 0.
According to the equation the remaining three of the six feet
of normals given by (2) be on this plane and so we have
I'UiXi m'b'yi n’c*zx
(e 2+ A ) + b- + Λ + C2-I-A p ...(S)
Central Cenicoids 445

This equation is of third degree in Λ and hence gives us the


remaining three values of A.
Clearly the product of the equations (4) and (5) must give the
same equation as is equation (3) Hence comparing the like
terms, we get
ΙΙ'α * χ ,* O1Jti* ., I
(ti’ + A ) * “ (O1H-A)* 0Γ ' “ a1/
Similarly, we have
m i .___ I
IjtPt * C1ΛII„' · ' P — P.
Pnttingthesevalues in /'jt+ m 'y+ n'z—p’· 0, the required
equation of the plane Ar Br C is given by
Λ y L· i Λ
P z + P m + C1Ji+ / ^ ‘ Proved.
Ex. 3. I f th e f e e t o f the three norm als fr o m P to the e llip so id
tP ftP + jP /lP + Z t Ict = I lie In th e plane x / a + y / b + z / c = I, prove th a t
the f e e t o f th e other three lie in the plan e x j a + y / b + Z j C +- I=U and
P lie s on th e line
a (IP—c*) x ^ -b (C1-O 1) v —r fa* —Λ1) z.
(Allahabad 1979)
Sol. The equation of the ellipsoid is
x*lat + y t lbt + zt lct -=l ...(I)
The first part is exactly similar to Ex. 2 above.
Now clearly the six feet of the normals lie on
( x / a + y / b + z / c — I) ( x / a + y l b + z / c ■1-1)=0
or (jt/o+ y ! b -\- r/c)1—1=0
or ( x t la t + y * lb t + z t lct — I )+2 {(y zjb c ) + ( z x f c a ) + ( x y /a b ) ) = ‘0 ...(2)
Since these six feet lie on tne ellipsoid (I), therefore, using (I)
the equation (2) becomes
Γ^-+ΖΧ+ - ^ = ϋ or a yz+ bzx+ exy= 0 (3)
DC aD
Now if the n rmals be drawn from P(xi* y u *i) t0 t^e ellip­
soid (I) then these six feet of the normals lie on the cone.
[See Ex. I (a) above]
Q2JC1 (b*-c*) P yl (Ct -U 2) C2Zi (a1—
jc · y ‘ z
or A2Xi (fe2—c2) y z + b 2y x (cl —a2) zx + c2zx (a%—b2) xy*=>0 ...(4)
Comparing (3) and (4), we get
A2Xi (b2—c2)ja-=tWyl (C2-O 2)Ib=C2Zl (a2—b%)lc
446 Analytical Geometry 3-D

or a c * ) X 1 *= b (c*—o*) y x = c ( a * — 6 * ) Z 1.
(b * —
.·. The locus of P(xt, y t. zt) is the line given by
a (ft2—c*) x —b (c2—a*) y = c (a2—ft*) z.
Ex. 4. Prove that the lines drawn from the origin parallel to
the normals to ax*-\-by*-^czt = I at its points o f intersection with the
plane lx+ m y+ nz= p generate the cone
, lx* y* z*\ Ilx my nz \*
Η « +£ + γ Μ γ + 7 + Τ Γ ( 1977)
Sol. The equation of the given conicoid is
ax*+6y*+czf= l . • •(I)
The equation of the given plane is
lx+ m y+ nz= p. .( 2)
Let (α9β, γ) be a point of intersection of the conicoid (I) with
the plane (2), so that we have
α<χ*+ δβ2+ογ**=1 and la+mfi+ny=>p. —(3)
Now the equations of the normal to the conicoid (I) at
/ O Λ X-OL y - β ζ —γ
_ _ _ _ _ _ „ .,4 )

The d.r.’s of this normal are αχ9 δβ9cy.


Λ The equations of the line from the origin parallel to the
x___ y_ z
normal (4) are -.(5)
α&~~δβ
From the relations (3), we have
aa*2+A|3>+ cy*= | /«+ w P+ nyj*
or p* {aΛ*+bβ*+cy*}=‘{lx+mβ+ny f
or -. /<«*)*. Φ β ) * . (Cy)iI // («*). m Φ β) . n (cy) I*
p \ - r + - r +— γ ί ~ + -* -+ -—) .( 6}
Eliminating ax, ft/9, cy between (S) and (6), we see that the
line (5) generates the cone
, fx* y* Z2 I {lx my nz \*
' ’Ή + τ Η ί α + Τ + τ ) ·
Ex. 5. Prove that the points on the ellipsoid the normals at
which intersect a given straight line lie on the curve o f intersection
o f the ellipsoid and a conicoid.
Sol. Let the equation of the ellipsoid be
x ilai + y2/bt + ztlct^ I. ...(I)
Let (oc, β, γ) be a point on the ellipsoid (I). The equations
of the normal at (α, β9 γ) to the ellipsoid (I) arc
Central Conlcoids 447

JC -α y —β ζ —γ
α/fl* “” /3/6a “ We* -« (2 )
Let the equations of the given line be
(*—Xi)lt= (y—yi)lm ~ (z—z1)ln ...(3)
If the normal (2) intersects the given line (3), then the lines
(2) and (3) are coplanar so that we have

Λ the locus of (α, β, y ) is

· (4>
This is the equation of a conicoid. But the point (α, β, γ)
lies on the ellipsoid (I) as well. H encetbe locus of (α, β, y) is
the curve of intersection of the ellipsoid (I) and the conicoid (4).
Ex. 6. Two planes are drawn through the six feet o f the
normals drawn to the ellipsoid χ*Ια*+γ*Ιδ*+ζ*Ιο*·=1 from a given
point, each plane containing three. . Prove that if At and At be the
poles o f these planes with respect to the ellipsoid then
A1Att - O A 1* -O A tt (e*+h*+c*).
Sol. The equation of the ellipsoid is
xVa’ + y 1/*2+ ZiVci = I - -(1)
Let V (x,, y u zt) be the given point. Proceeding as in § 12,
the values of A giving the feet of six normals drawn from
P (*i, y u zt) to the ellipsoid (I) are given by
OtX1* CtZ1*
- 1= 0 .
(α * + Α )* 'Γ (6*+ ^ + λ )·
(ί»β+ Α )2 ------ ...(2)
Let the co-ordinates of the poles A1 and A3 of the two planes
be (xa, yt, Zi) and (xa, y3, z3) respectively, so that the equations of
the polar planes of A1 and A% are respectively ^jven by
XXt * * i , m i zz* m o
a* + b2 + c* ^ ...( 3 )
The feet of the six normals, namely
/ a*x, F y1 CtZt \
\at + h , b * + \ t c*+X J
where A has six values, lie on the two planes given by (3) and so
we have
448 Analytical Geometry 3-V

0**1 X})I - I = O and Σ / a * x ,


α*+Λ * a*} \a 2-M • 5 ) - -
ot X\Xi y*yt . Z 1Zt
1= 0
a 2 + \ + A2+A 1 C2-M -.(4)
and *1X3 ,_.y>y* j . 21¾ - 1 = 0 .
h Vt -I-A h1C2-I-A -(5 )
Obviously (2) is tbe same as !he product of (4) and (5), so
comparing them, we have
XiXs y»y» ZtZ3 I
Oi “ b* " c* “ - Γ
A **Xs=»—ο*. Λ Λ = —6*. ZiZ8= -C * . - (6)
Now A1A J - O A 1* -O A t*={(xs-Xs)1+ 0 ¾ - * ) 1+ ( 2!-¾)*}
—(Xa*+ y j + z*1)—(x»2+^a1+ 2**)
= —2 (X*X3+^*J’»+Z*Zs)
= —2 ( —a*—b*—c*), using (6)
= 2 (o2+ 6 2+c*). Proved.
Ex. 7. A Is a fixed point and P a variable point such that its
polar plane Is at right angles to AP. Show that the locus of P is a
cubic curve through the feet o f the normals from A to an ellipsoid. ·
Soi. Let the equation of the ellipsoid be
x*la*+y*lb*+z*lc*=l. ..( I )
Let the co-ordinates of the points A and P be (xj, y lt Z1) and
(*, β, y) respectively.
The equation of the polar plane of P (α. β, γ) w.r.t. the
ellipsoid (I) is
ax./a*+ βγΙ b* + yr/c5= I . ...(2)
Now the d.r.’s of the line AP arc X1- a, y t —j3, Z1- γ.
If the polar plane d ) of P is at right angles to AP then the
normal to the plane (i) is parallel to the line AP and so we have
X i - « , y i- β
φ * + β/b* V “ A <Say)·
αλ Ot X 1
X‘—OC=^i- or CL --
O2-M -
Vi C1Z 1
Similarly β = “χ ~-(-Λ
Λ and
C2-I-A
Tbe4ocus of P (α, β, γ) is
O 2X i b*yt Ci Z1
'α 2+λ y = b 4 a · z= c2+A
which are the parametric equations of the cubic curve through the
feet of the six normals drawn from A (x,, y u Z1) to the ellipsoid
(I). [See § 13J.
Central Conicolds 449

§ 15. Diametral plane.


Definition. For a central Conicoid9 a diametral plane is the
locus o f the middle points o f a system o f parallel chords.
(A) To find the equation of a diametral plane.
Let the equation of a central conicoid be
cxt +byt +czt = I. —(I)
Let the equations of a system of parallel chords be
( x - * ) ll ~ { y - i8)!m = (z-y)/n= r (say), ...(2)
where I9 m9 n are the direction cosines of each chord and so are
fixed, and r is the actual distance of any point (* + /r#β +m r9γ +nr)
on (2) from (oc, β9 γ). If (2) meets (I) then the distances of the
points of intersection of (I) and (2) from (α ,β ,γ ) are the roots
of the quadratic in r given by
* a (a+lr)2+ b (fi+mr)2+c (y+nr)2= !
or r 1 (al2+bm2+ cri*)+ Ir (ala+bmfi+cny)
+(a*2+ b p + c y2- 1)=0 ...(3)
If (α, β9 γ) be the middle point of the chord, than its distances
from the two points of intersection should be equal in magnitude
and opposite in sign and hence the sum of the two values of r
should be zero, so that we have alcL+bmfi+cny=0
The locus of the middle points (α, β, γ) of the system of
parallel chords with d c.’s I9 m, n i.e. the equation of a diametral
plane for the conicoid (I) is
alx+bmy+cnz=0:, -(4)
Clearly the diametral plane (4) passes through the centre
(0, 0, 0) of the conicoid (I).
If /, m, n are the d.r.’s of a direction, even then the equation
of the corresponding diametral plane is given by (4).
Corollary. In case of the ellipsoid x2la2+ y2llp + Z 2Ict = I,
proceeding as above the equation of a diametral plane is
Ixlat +- my IIP 4 nz/c2= 0. —(5)
This plane bisects a system of parallel chords of the ellipsoid
with direction cosines I9 m, n.
(B) To prove that every plane through the centre is a diametral
plane o f the central conicoid corresponding to some direction.
Let the equation of the central conicoid be
axt+by*+cz*^ I. - - (I )
Consider a plane A x+ B y+ C z= 0 ...(2)
through the centre (0, 0, 0) of the conicoid (I).
450 AmUytIeaI Geometry 3-D

We are to prove that the plane (2) is a diametral plane of (I)


corresponding to some direction I9 m, «1 (say).
Now the equation of the diametral plane of (I) corresponding
to the direction I9 Ui9 u is
a b c+ b m y+ a tz= 0. -.(3 )
The planes (2) and (3) should be the same and hence com­
paring them, we get
at bm_cn I m __ n
θ ΟΓ A jiT * B fb ~ C jc .(4)
Hcoce tbe plane (2) is a diametral plane of the conkoid (I)
corresponding to the direction I, m, a given- by (4).

Let the equation o f the ellipsoid be


x»/a*^y*/fc*+zVc«=l ...(I)
with its centre at O (0 .0 ,0 ).
Let P i i |, / i . z,) be any point on the ellipsoid (I) so that
d.r.*s o f the line OP are xu y,, z. The equation of the diametral
plane [See § IS (A)] of the line OP is
xxJ e*+W i /6*+ zz,/e*= 0 . ...(2)
Now take a point Q (x*. yt . Z1) on the section o f the ellipsoid
(I) by the plane (2). This point Q win satisfy both the equations
of the ellipsoid (I) and the plane (2).
Λ ·χ*χ ι JtPjTytyi /^ + Z 1Z1Zc1==O. ...(3)
Thns the relation (3) is the condition that the diametral plane
of the line OP passes through another point Q on the ellipsoid
Cl). But the result (3) is symmetrical in xt. Ti, z, and X1, τ·. z*
and hence it shows that tbe diametral plane of the line OQ will
also pass through tbe point P.
Th us we eoadade th a t if the Jkam etrnl plane e f a Um OP passes
thnm gh th e potat Q them Me diam etral plane o f the line OQ will also
pass tkrongh th e potat P.
Now let the line o f intersection of the diametral planes of OP
and OQ cut the surface of the given ellipsoid in the point
AfxS. Th Zj), so that R lies on both the diametral planes of OP
and OQ and consequently the points.!* and Q lie on the diametral
plane of the line OR i.e . on the plane
XXsjst + y y ^ lP + O j iP = O . ...(4)
Hence it s£ovs that the diametral plane of OR is the plane
POQ. In a similar way the diametral planes of OP and OQ are
the planes QOR and ROP respectively.
Central Contcoids 451
Now we shall define conjugate semi-diameters and conjugate
planes.
Conjngate semi-diameters. The three semi-diameters OP, OQ
and OR o f an ellipsoid which are such that the plane containing any
two is the diametral plane o f the third, are called the conjugate semi-
diameters.
Conjugate planes. The three planes POQ, QOR and ROP which
are such that each is the diametral plane o f the line o f intersection of
the other two, are called the conjugate planes or conjugate diametral
planes.
§ 17. The relationship between the co-ordinates of the points P f Q,
R where OPf OQ and OR are the conjugate semi-diameters
of an ellipsoid.
Let the equation of the ellipsoid be
XtIat -^ytIbt +ZtIct - 1. -(1 )
Letthe co-ordinates of the points P, Q, R be (xlf Pu Z1),
(*«, y %, zt), (x3, y 3, z,) respectively and OP, OQ, OR be the conju­
gate semi-diameters of the ellipsoid (I) with 0(0. Of 0) as centre.
Since the points P, Q, R lie on the ellipsoid (I), we have
Xi*la*+y i*lb* + ZliIci- I Ί
I
xSIcP+yJItP+zJlc* - 1 v ...(2)
I
x S la '+ y fltfi+ z J lc * -) J
We know that the diametral plane of any one of OP, OQ and
OR passes through the extremities of the other two and therefore,
we have
*i*»/a*+.y, y t/lP+Z1Zi Ict -O ')
I
XtXtlrf+ytytlb* + ZiZiIct - O ...(3)
I
x»xi/at + yiy 1lbt + ZiZlIct - O J
If I, m, n be the direction cosines- of any line j^ien we know
that Zi -Hmi -M*= I. Inview of this relation, tfifc relations (2)
imply that X1Ia, yi/b, Z1Ic-, x t/a, yt !b, ZiIc and x ja , yt/b, zt/c are
the direction cosines of some three lines. 1
Also it I1, O
t 1, H1 and Ii, mt, n3 be the direction cosines of two
perpendicular lines then we know that /j/a-l-/WiOT*-|-WiWa= 0. In
view of this relation, the relations (3) imply thht the above three
lines are mutually perpendicular to each other.
452 Analytical Geometry 3-D

Again if I1, mu nlt lt, m%, nt ; lt , ms, n, are direction cosines of


three mutually perpendicular lines then we know that I1, U, Itl
rrti mt, m*; nt, nt, n, are also the direction cosines of three lines
which are mutually perpendicular so that
/», +/», +/»*·=1 etc. and /imi+/»m»+/*»»s=*0 etc.
In view of these relations, we have
X i'/a'+ xS/a'+ xS/o*— I
'·«· „ Xis+X1*+Xtt ^a*. )
Similarly > ...(4)
and z S + z J + z J ^ c ·. )
Also 5 Λ + 5 + Λ + 5 Λ .0
Q b a b a b
l-e- Xtyi+ x*y»+ xtyt^O . )
Similarly yx2i+ ytz,+ ytZt~0, > ...(5)
and ZtX1+ ?tx t+ ZtXt *=0. )
Solving the first and third equations of (3) for X1Ia, yrfb, Z1Ict
we get
x i Ia _ yt/b _ Ztlc
ytZ t-ytZ t ZtXt-ZtXt x ty t - x t y t
be ea ab ,
. V (x itlo*+yi*lb, + z1*lc*) I _, i t
“ ^ ν ΐ Σ {(y,z, - y tzt)lbc)*\ sin 90o=
. X1 , - ^ » 2» yx ZtXt-ZtXt Zl ___ xtyt— Xtyt
*' a sa^ be 'b ·*" ca ' c ■ ab
- ( 6)
Note. The results (2), (3), (4) (5) and (6) given above should
be committed to memory.
§ 18. Properties of conjugate semi-diameters of an ellipsoid.
Here we shall give some properties of conjugate semi-dia­
meters of an ellipsoid. The meanings of OP, OQ and OR are the
same as already defined in § 16 and § 17 above.
I. The sum of the squares of any three conjugate semi-diameters
of an ellipsoid is constant.
(Allahabad 1978, 80; Agra 82; Madras 76; Kuruksbetra 77)
We have OPt +OQt + O Rt
= ( V + V + Zl*) + !*!* +Λ* + 2,*) + (X,*+ys*+ 2»*)
= (Λΐ, + *** + *8*) + ( ί Ί * + Λ ' + Λ ' ) + (21»+ 2,» + 23*)
«=e*-|-6*+c* [using relations (4) of § 17]
which is a constant. Hence proved.
Central Conicolds 453

II. The volume of the parallelopiped having three conjugate


semi-diameters o f an ellipsoid as coterminous edges Is constant.
(Rohilkhand 1981)'
Let OP, OQ, OR be the three conjugate semi-diameters of an
ellipsoid. The volume V of the parallelopiped having OP, OQ and
OR as three coterminous edges=6 X (the volume of the tetrahe­
dron OPQR)
le . V = 6x\ Xt Ti
y%

X t y% Ζ»

:. v »- Xi yi zi Xi y\ Zi

x t y t zt χ% y* Zi

x t y t zt Xi y* zz
I
= Xi X t X t X\ X% Xz

y i P t y t yi y% yz

Zi Zt Zt Zi Zz Z9

*1*+ *«*+ *«* *1 y i+ x ty t+ x ty t x iz i+ x tzt + x tz t

Xiyi+x%yt+x$yt y 2+ y% + y t y i z i + y tzt + y tzt


ZiXt +ZtXt+Zt Xt y iz i+ y tz t+ y tz t zi*+ zt2+ zt*
[using relations (4) and (5) of § 17]
0 , IP 0
0 0 c*
ot V*=a9b*<P or Fsaabc which is coustant.
III. The sum o f the squares of the projections o f three conju­
gate semi-diameters on a line is constant.
Let the direction cosines of a giveo line be I, m, n. Let OP,
OQ and OR be the conjugate semi-diameters of an ellipsoid.
The projection of OP on the line whose d.c.’s are I, m ,n
=(X1-O ) /+(.yi—0) Wi-Hz1-O ) U=Ixl +Otyl + nzu
454 Analytical Geometry 3-D

Similarly the projection of OQ and OR in the given line are


lx2+ m 2y+ n z2 and lx»+my,+nz, respectively.
Λ The sum of the squares of the projections of OP, OQ and
OR on the given line
— (/*1 +myx+nzx)*+(lx2+my2+nz2)*+(lx2+my»+nz2)*
=/*(*»*+***+* 3*)+»»* Z yJ + n* Zzx*+2lm Z xly t
4·2μβ ZyiZi+2nl ΖζχΧχ
=/*e*+m*&*+n*c*+0 + 0 + 0
[using the relations (4) and (5) of § 17 above]
=fl,/*4m t2>*+c*n,t which is constant.
IV. The sum o f the squares o f projections of three conjugate
semi-diameters (of an ellipsoid) on any plane is constant.
Let the direction cosines of the normal to a given plane be
I, m, n, so that the square of the projection of OP on the given
plane is equal to
OP*—(/* !+ myx+ HZ1)*.
Similarly the squares of the projections of OQ and OR on the
given plane are
OQ*—{lxt+myt+nzt)* or OR*—(lx2+my2+nz2)*
respectively.
A The sum of the squares of projections of OP, OQ and OR
on the given plane
= OP*+ OQ*+ OR*—[(Ix1+ my x+ BZ1)2+ (lx2+my2+/«,)*
+ {ixt+ m yt+ nztf}
= (a*+ 6*+ c2)—(a*/*+ b*m*+ c*n*)
[Putting the values from properties I and 111 above]
=o* ( l —/*)+/>* (1—m*)+c2 ( I - η 2)
^constant, since f, m, n are constants for a given plane.
V. I f OP, OQ and OR are the conjugate semi-diameters of an
ellipsoid, then the sum of the squares of the areas o f the faces QOR,
ROP and POQ of the parallelopiped having OP, OQ, OR as coter­
minous edges is constant. . (Rohilkband 1979)
Let I1, mi, /I1; I2, m2, n2 and I2, m2, n2 be the direction cosines
of the normals to the planes QOR, ROP and POQ respectively.
Also let Αχ, A2 and A2 be the areas of the faces QOR, ROP and
POQ respectively.
Let (xlt y u zi), (x2, y 2, z2) and (x8, ^8, z2) be the co-ordinates
of the points P, Q and R respectively. Now if we project the
area QOR on the yz-plane i.e. on the plane x = 0 , then we get a
triangle whose vertices are (0, y2, z2), (0, 0, 0) and (0, y2, z2) and
the area of this triangle.
Central Cameoidt 455

= Ι (Μ -Λ % )= ±2a^ [using relations (6) of § 17 above].


But the projection of the area At on the js-plaoea^/,.
Λ Λ * ι= ± (* « ι)/(2 α ).
Similarly if we prefect the area QOR I jc. A1 on the planes
ZO X and X O Y. we get
4««=±(<wFi)/(24) and AlOi ^ ± (a b z^l(l£ ).
Squaring and adding these three relations, we get
A 1* (/1*+m,, +Wi,)=l(Acr1)*/(2a)*+(ca/,)*/t2A), +(a6zJ*/(2c)*J
4*e* . . A»1 . . « V
or iV ijji ** +4^« Λ*Ί" 2r * ...(I)
In a similar way, the areas At and At are given by
HtCa . eV e*h*
»*+ 4* * * + * Γ * β ...(2)
and. .. . . A P p f«+. Α
—» z f>.
-(3)
Adding (I), (2) and (3), we get
Λ ·+ Λ ,* + * * = » £ γ ^ . ’+ ^ Σ ζ'
4V ^ . A * .,,<»*4* ^
β 4β* '^+44* +4e*
[See relations (4) of § 17 above]
= J (iV + A P + aV ) which is constant.
SOLVED EXAMPLES (P)
Nate. In all the exercises given below we shall denote OP;
OQ and O R as conjugate semi-diameters of the ellipsoid
x*/u*+ J*/4*+2*/c*= I
and (X i, Jt1, Z r), (x*. 7 * . Zt ) and (xfc / „ zt)
as co-ordinates of the points P , Q and R respectively unless other­
wise stated.
Ex. I. Let OP, OQ, OR be the conjugate semi-diameters o f
3x*+Ay*+r*= I. Find Ai f OPt t OQi + O JP= 0.
(AUahahad 1919)
Sal. The equation of the given ellipsoid is
3x*-M7 *+z*=l.
.-. e * = J.4 » = l/A ,e ·= !.
We know that OP*-f-OC*+OA*=e*+4*-|-e*.
[See property I, § 18 above]
.. 2=J+1/A+1 or A=3/2.
456 Analytical Geometry 3-D

Ex. 2. Show that the sum of the squares o f the projections of


three conjugate diameters o f an ellipsoid on the x-axis is constant.
(Allahabad 1979)
Sol. The direction cosines of x-axis are 1 ,0 ,0 . From
property III of § 18 above, putting /= I, /» = 0 , n = 0 , we get that
the required sum—a*.(I)* '+ ^ .(0^ + ^ . ( 0^ = 0* which is constant.
Ex. 3. Find the equation o f the plane PQR·
(Allahabad 1975, 81; Agra 75. 79. 81; Bundelkhaod 78;
Madras 76; Rohilkband 77, 80; Ravishanker 70.71)
SoL Let the equation of the plane PQR be
lx+ m y+ nz—p. —(1)
Since the plane vl) passes through P(Xi,yi,Zi), Q(xi>.V*· z*)
and R(xt, y,, z,), w e have
Ixi+ m yx+ n zi^p , ·· (2)
/Jt1-Hmy,+HZ1=P, (3)
and /jc* + my,+Hz,=p. -(4)
Multiplying the equations (2), (3) and (4) by Xj, x, and x,
respectively and adding, we have
/(x,*+ x,*+ x,*)+ m ( x ,y ,+ x ,y ,+ * y ,)+ n (χ, ζ, + χ, ζ, + χ λ )
=P (* i+*»+**)
or /.o*+m .0+n.0=*p ( χ ,+ χ ,+ χ ,) ,
using the relations (4) and (5) of § 17
or /-- p (xi+x,-H *·)/«*.
Similarly we have
ot=p (yi+y,+y,)/b*, n=>p (zj+z,+z»)/c*.
Putting the above values of /, m, n in (I), the required equa­
tion of the plane PQR is given by
(x/o*)(x,+x,-f x,)+(y/b*l (y ,+ y , f y3)+(z/e*) (zj+ζ,-Η ζ,)= I.
Ex. 4. Prove that If the pla^e lx + m y + n z -p passes through
the extremities P, Q, R of the three conjugate semi-diameters, then
<Pl*+ IPnP +(Pnt —3p*. (Kanpur 1980; Lackaow 75)
Sol. Rewritting the steps of Ex. 3 above upto equation (4)
and then squaring (2), (3) and (4) adding, we get
(/x1+my,+Hz1)*+(/x,+my,+HZ,)*+(/x,+my,+HZ,)*=3p*
or P Σ x,*+m* Σ y^+ n* Σ z1i+2mn 2?y1z1+2o/ Σ Z1X1
+2/m 2? Xjy1= 3p*
or /*e*+m*b, +H,c, =3p*
[using relations (4) and (5) of § 17 above].
Ex. 5. Prove that the plane PQR touches the ellipsoid
C en tra l C o n teo id s 457

x*fa* +y*lb*+ 2*/c*«=»$


at the centroid of the triangle PQR. (Knmkshetra 1977)
Sol. Proceeding as in Ex. 3 above, the equation of the plane
PQR is
(x/a*) U i+ *»+*»)+07**) (Λ + Λ + Λ )+ (*/«*) (21+ ¾ + ¾ ) = 1
...(D
Now clearly the co-ordinates of the centroid G of the triangle
PQA tu t ( *■+*■+*·, * + » + » .
It can be easily seen that this point lies on the ellipsoid
XtIat + Jft Ibt +Zi ,Ict = I
The equation of the tangent plane to the ellipsoid
x*/fl*+y*/6*+ z*/c*= J
at the point G is
* .(x ,+X14- x3) ^ y .( y \ +.v*+y,) ^ 2 .( 2, + 2, + z,
3e* ^ 3b* ^ 3Ct 1
or * ( * ,+ * , +x#)_,_y (yt+y»+y*)J_z (Zt+Zt+Z3)
------ ---------+ -----------------+ ------ -j--------- I .
...(2)
The equation (2) comes out to be the same as the equation (I)
and hence it shows that the plane PQR touches the ellipsoid
Σ (XtIat ) - 1/3 at the centroid of the &PQR.
Ex. 6. Prove that the pole o f the plane PQR lies on the ellipsoid
x*ItP+y*Ibt + ZtIct =Z. (Agra 1978; Rohilkband 88)
OR
Prove that the locus of the pole of the plane PQR Is
XtIat +y* Ibt +z* I Ct = 3.
Sol. Proceeding as in Ex. 3 above the equation of the
plane PQR is
(x/a*) (x i+ x t +x3)+(y/bi) (yi+y*+y»)+(zlct) (z i+ z ,+ 2#)=»l
...(I)
Let the pole of the plane (I) with respect to the given ellip-
SOid xVa*+>*/<>*+z*/<JWi be (α, β , γ).
The equation of the polar plane of (α, β, γ) with respect to
the ellipsoid Xt Iat +jp/b*+ZtIct= I is
axliP+fiylb'+yzjc*=) ...(2)
Comparing ( I ) and (2), we get
. g ___= ___ β ___ γ
*i+ *»+*s >, T>'a+>’3 Zi+Zt+Zt I
458 Analytical Geometry 3-D

·’.· *— +·**+·*»« 0=3^+3^+3¾* )'=*¾+¾+¾-


Now, we have
α* , β* , +*»+**)* , (Λ +Λ +Λ )* . (¾+¾+¾)*
a* b* c* a* h* c*
£ * ι · . £ Λ 1 i ^ Zi
b* + 7. + 2 {(¾?+¾?+*?)
+ ( ^ - + ^ + 7 - ) + ( ¾ 1+ ^ ' + ^ ) }
β* £ . ?
[Using relations (4) and (3) of § 17 above]
'ο»+ />* c*
=3.
/. The locus of the pole (α, β, γ) is XtIa2
Ex. 7. Prove that the locus o f the centre o f the section o f the
ellipsoid x tJa2+ y2lb2+ ztic2'=*\ by the plane PQR is the ellipsoid
Xt /a2+ y 2/ b 2+ z2j C2— Provefurther that this is also the Iocusof
the centroid of the triangle PQR.
Sol. The equation of the given ellipsoid is
S = x tlat + ytlbt + ztlct - 1 - 0 . ...(I)
Proceeding exactly as in Ex. 3 above the equation of the plane
PQR is given by
(x/a% ) (xi+ xt+ x*)+ (y/bt) (y i+ y .i+ y 3)+(zlc*) (z i+ z ,+ z ,)= l.
. .( 2)
Let (α, β, y) be the co-ordinates of the centre of the section
of the ellipsoid (I) by the plane PQR so that the equation of the
plane PQR is given by tT - S i'
ie , ^ , > ΐ = ί! . ,
a* + b*+ c» = o*+ b»+ c* ...(3)
The equations (2) and (3) represent the same plane and hence
comparing them, we get
___ a __ ' β___ e y _ «Va»+p»/fr*+r*/c»
Xi + ac»+.*» .yi+3¾+3¾ Zi+z*+z» I
. * _ ίΧ ι + Χ*+Χ»\ί**>β*4 Υΐ \ ^ _ /Λ + Λ + Λ \/« * .^ * .
·· o - \ — a----- / U 4+ ^ + ? /* 5“ l ----- b ------J U + Si+ ? ;
and (™ )£ 4 :+£)
Squaring and adding the above three relations, we get
* !£ + £ + ZY Jt>)4_i_C3?i~l~3^1+ 3¾)4
? + ? c* U 4 + ^4 + W I «4 ‘ + 6*
(Z l + * » + Z » )M
+ f« I
Central Conicoids 459

0Γ ( ί + ^ + ? ) = ( ^ + ^ + ? ) 2·3 IproceedinS as in Εχ· 6 ab0Ve]


or CtiIat + β*/δ* + y*/c*= J .
Λ The locus of (α, β, y ) is XiIai +γ*Ιβ*+ζ*Ι<Ρ*=\. ...(4)
Second part. Let the co-ordinates of the centroid of the
triangle PQR be (*, β, y), so that we have
*“ i (* i+ x 2+*s), β = \ (yi+y*+y*), y = i ( z i + r 2+z»).
. «* . £* , V * _ l ['<Xi + X 2 + * 3 ) i!J_ 0 'i+ .> '2 + .> '3 ), _L (Zi + Z « + Z s)* l
" B*T b* c* 9 [ a* b2 Ti J
= £ .3 (Proceeding as in Ex. 6 above]
= έ·
Λ The locus of (α, β, γ ) is X i I a i + y * / b 1 + Z i I c i = J which is
the same as (4). Proved.
Es. 8. Find the equation of plane through the points (X i, xt, Xg),
OiIr JfU y») and (Z1, zt , z3) where (x,, y„ zr), r = l , 2, 3 are the extre­
mities o f the three conjugate semi-diameters and show that it touches
a fixed sphere.
Sol. Let the equation of the required plane be
lx+ m y+ nz= p. -...(I)
The plane (I) passes through the points (xi( xf> x*), (yi, y t, y*)
and (zg, zt, zt), so we have
lxi+ m xt + nxt =p, ...(2)
Iyi+ my*+By3=P, ...(3)
and IZi+mZi+nz3—p. —(4)
Now multiplying the equations (2), (3) and (4) by Xijai, yi/b*
and ZiIci respectively and adding, we get

' ( £ +£ +? > " (¾?+¾*+¾)+- ( ψ +ψ +ψ )

or l= p (Xilat+yillP+zdc*) [using relations (2), (3) of § 17].


Similarly m = p (XtIai +yt/b*+ZtIci), n=p (X g/a* + yt/b2+ Zg/c*).
Putting the values of I, m, n in (I), the equation of the requi­
red plane is

(W )+ ' (W )-· ···<»


Again we are to show that the plane (5) touches a fixed
sphere.
460 Analytical Geometry 3-D

The length of the perpendicular from the origin to the plane


(5)
<=>1
I ZfDxl*j _D^,*j _Dz, * 2ZXiyl j_IEylZl ^ l E z lXl]
=>1
/« /[ β * *· Cj t eV 6*c* ^ c»a* J
= 1
/VfS+S^+H
[using the relations (4) and (S) of § 17 above]
= 1/ / ( 4 - + γΪ-·1~·4γ) ,=' - ;/ ι·. » .a* f , .
l*J\a* b1 e8 / V(b ^ + c ’a*+fl*o*/. which is constant.
Hence the plane (5) touches the fixed sphere
Q1AVi
x*+^*+z*«
i V + c V + f l ’i·
Ex. 9. Prove that the locus o f the foot o f the perpendicular
from the centre of the ellipsoid x8/e*+.y8/68+ z 8/c8= l to the plane
through the extremities o f three conjugate diameters is
a*x* 4 AV*+ c*z*= 3 (x* +y* + z*)8.
(Allahabad 1976; Meerat 83S)
Sol. The equation of the given ellipsoid is
XiIot +y*lb*+z*lc*=· I. -..(I)
Proceeding exactly as in Ex. 3 above, the equation of the
plane PQR is
( X l a t ) ( X i + X a + x » ) + ( y l b * ) (^1 +^ 2 +^ 3)+ ( 2 /0 1) (z,+ z,+ z,)= I.
- ( 2)
Let N (α, β, γ) be the foot of the perpendicular from the
centre O (0, 0, 0) of the ellipsoid (I) to the plane PQR.
Let p be the length of the perpendicular ON so that
p=OAf--the distance between O (0, 0, 0) and N (a, β, γ)
= V(«*+/** + / ) . -(3 )
The d.c.’s of ON, the normal to the plane PQR, are
a M a ’ + ^ + y * ) , β ΐ ν ( * * + β * + γ \ y /V (« * + / 3* + y * )
Le., a/p, β/p, γ / ρ , using (3).
Hence the equation of the plane PQR is also given by
( Φ ) Χ+ {βΙ ρ) y + ( v l p ) Z = P or ax+£y+yz=>p*. ...(4)
Comparing the equations (2) and (4), we get
Xi+ * 3+*3 j'l+ J ’t + j ’s Zi+Za+Zg 1_
a* IP c* p*
Central Conleotds 461

X1+ x 2+ x t ax y i+ y t + yt h/3 z i+ z t + z 8. cy
a* “V* b* “ p*· ' c* “V
Squaring and adding, we get
(*!+*»+*»)* O', , (Zi +Z js+ z,)* , a*a*+h*/3*+cV
a* ' + b* + c* + p*
Ex11 S jl*
or
a· + b* ·* Μ “ ( τ )+“ ( 7 )+ ( μ τγ )]
α*«*+ 6*j8*+ e*y*
Pi
or (1 + 1 + 1 +0+ 0+0)T ?«=a,a*+6,^ + c V
[using (4) and (3) of § 17]
or 3 (a*+/3»+y*)»=(a*a»+h*/9*+<?V), from (3).
The locus of («, β, γ) is
3 (x*+yt + zt)t saa*x, + bty*+cizt. Proved.
Ex. 10. I f λ, μ, v are the angles between a set o f equal conju­
gate diameters of the ellipsoid x*la*+y*/b*+z*jc*=\, then show
that
, , . . , 3E (b * -c a)*
cos* A+«w* μ +cos* ν- 2 (0 , + 6»+ ^ ϊ ·
Sol. The equation of the given ellipsoid is
x*la*+y*/b*+2*/c*<=^l. -..(I)
Let OPt OQt OR be a set of equal conjugate semi-diameters
of the ellipsoid (I) so that
OP=- Ogc= OR= r (say).
We know that OP*+OQ*+OR*^a*+b*+c*.
Λ 3r*=a*+b*+c*. .v.(2)
If the co-ordinates of Pt Qt R are (xlt y u z,), (xt, y 2t zt ),
(xt, y t, Z3) respectively, then the d.r.’s of OQ and OR are
x tt y2t zt and x8, y3. z8 respectively. If λ be the angle between
OQ and OR, then
„„„ i _ x*x*+y*y*+ztza XiX9+ yty»+Z2Z2
V W + y S + z ^ V W + y S + z t ' ) ------------ Γ*
Λ cos· λ—(x2*t +jW s+*s*t)e/ '4· ...(3)
From Lagrange’s identity, we have
(xJ+ yS+ z*2) (X9*+yzt +Zs2) - ( x 2x3+ y2ys+ZiZ9)t c=Z {yizt - y t zt)%.
Putting from (3) on the L.H.S. and from the relations (6) of
§ 17 on the R.H.S. of this identity, we have
2 4 cos*. A, = P—ciXl' c'a'yi2 a*bW
.r2—r4 — - 4-, +-
a2
462 Analytical Geometry 3-D

Dividing by r* and transposing, we get


. , OtP ct ( Xit y t* z,*\
cos* A= '"4 ( a r + V c - r ) · ...(4)
(x t* ■y»* ζ223
OtPc1 \
Similarly cos* w= 1-
~ \ a * + P + c* )' ...(5)
and OtPct /£«!+ λ * V J
COS* V = I —
\.e* + p + c4 / ...(6)
Adding (4), (5) and (6), we have
cos* Λ+cos* μ + cos* v
, OtPct TExl* Z V Zz11I , o*6*c* fa* , P c tI
“ 3— T * ~ y p •“"λ·- + J= 3 |o i + 5i + c<]
= 3 - J ; (6*c*+c*a*+a»6*)=3-9 ( ^ >y ^ f f >) [See (2)]
3 f(a»+6*+c*)«_3 (Pct + c*a*+ a*6*)l
“ ■ (a*+P+P)*
= 3 [a«+P + ct - P c t - c tat -a*P]l(at + P + c t)t
3 12o*+2P+2c* - 2Pct - 2c*a*- 2a*6»]
~ 2(a*+6*+c*)*
3 Γίi>4- 26*c*+ c*) + (c‘ - 2c*a*+ a*) + (a« - 2o*Z>*+ MP
“ 2 (a*+6*+c*)*
3 [(6*-c»)»+(c*-a*)*+(a* -ft*)*]
” 2 (a*+6*+c*)* Proved.
Ex. 11. Find the locus o f the equal conjugate diameters o f the.
ellipsoid XtIat + y* /P +ZtIct = I . (Meerot 1985)
Sol. The equation of the given ellipsoid is
XtIat +JitI P +z*/c*= I . ...(I)
Let OP, OQ and OR be the equalconjugate semi*diameters
of the ellipsoid (I), so that O P --O Q -O R = r, (say).
Let (X1, y u Zi). (* 2, y%. zt) and (*3, y3, z3) be the coordinates
of the points P, Q and R. We have ·
O P=i-»OPt = rt =>x1t + yit + zlt = rt. .. (2)
Similarly Xtt + y tt +z»t = r t ...(3)
and V + V + Z 3*=»r*. ...(4)
Adding (2), (3) and (4), we get
a*+h*+c*=3r*. ..(5)
[ V Z V = a * , Z Jttt = P f Z Zit =Ct]
Now let I, m, n be the d.c.’s of OP so that the coordinates of
the point P are (Ir, mr, nr). Since P (Ir, mr, nr) lies on the ellipsoid
(I), we have
F r t Iat + i P r ' I P + P P / P = I
CetOrol Cmtkotds 463

_ P t m* . «· I /* , to2 , η* 3 i/»+m*+n·)
β·+ 4» + c*” r« ΟΓ α1 + 6* + c» " e*+6»+c» ...(6)
[Patting the value of T1Trom (S) and using the fact that
/*4·ιη1·4-«, =Β1]·
The relation (6) shows that the semi-diameter OP generates
the cone f.e., the locos of OP is the cone
Jt1 jp za 3 (X1-Kp1-I-Zs)
(P+ IP+ Ct a*+tP+c*
or (Xt ItP) (2( P - P - C t) M y aIb*) (2V - C t -(P )
+(ZtIct) (2c1—a1—b*)=0.

Exercises
1. Show that the plane x + 2 p + 3 z= 2 touches the conicoid
Jt1- 2jP+3z*=2 and find tbe point Of contact. (Meerut 1983)
Ana. ( 1 ,- 1 , 1 ) .
2. Find the equations of the tangent planes to the ellipsoid
7 ^ + 5 ^ + 3z*=60 which pass through the line
5y—3 z = 0 = 7 x + I Op—30. (Kanpur 1974).
Ana. 7χ+ 5ρ+ 3 ζ= 3 0 , 14x+10p+9z=50.
3. A tangent plane to tbe conicoid ax*+Ky2-f-cz, = l meets the
coordinate axes in P, Q and R. Prove that the centroid of
tbe triangle PQR lies on the surface
-L J - _!__g
axt+ ayt+ az*~
4. Show that the locus of the point of intersection of three
mutually perpendicular tangent planes to a central conicoid
is a sphere concentric with the conoicid.
(Meerut 1984 S)
[Hint. Refer § 7.]
5. Show that the locus of the centres of all sections of the
conicoid ex1+bp1+CZ1- I by planes which pass through a
fixed point (x', y', z') is
o x ( x - x ') + b y (y - y ') + c z (z -z ')= 0 .
6. Show that the centres of the sections of tbe ellipsoid
χ*1(Ρ+γ*ΙΙΡ+z*l(P==l that are (i) parallel to a given line
lie on a fixed plane and (ii) that pass through a given line lie
on a conic.
464 Analytteal Geometry 3-D

7. Prove that the six normals drawn from any point to a central
conicoid are the generators of a quadric cone.
(Meerat 1984,89 S)
[Hint. Proceed as in § 14.]
8. P r o v e that the feet of the six normals drawn to the central
conicoidax*+by*+ c z* = I from any point (*ι, Λ, *i) l*e 0,1
the curve of intersection of the given ellipsoid and the cone
a (b—c) Xi b (c—a) yt c (a—b)
x + y + z
(RoUlkhnnl 1977)

You might also like